1ygm3hwjpgu090312060334 tài liệu bồi dưỡng giáo viên trường thpt chuyên mon hóa...

275
Ph n 1 Ç HãA HäC §¹I C¦¥NG CÂN BẰNG HÓA HỌC – CƠ SỞ GIẢNG DẠY DẠY VÀ HỌC HÓA HỌC: QUY LUẬT, ĐỊNH LƯỢNG Trần Thành Huế Giảng viên cao cấp, Phó Giáo sư, Tiến sĩ Bộ môn Hóa học lí thuyết và Hóa lí, khoa Hóa học, trường ĐHSP Hà Nội MỞ ĐẦU Cân bằng hóa học (cbhh) là một nội dung có tầm quan trọng lớn trong giảng dạy, nghiên cứu Hóa học (kể cả Hóa học cơ bản lẫn Hóa học công nghệ và Hóa học ứng dụng nói chung).Trong phạm vi kiến thức Hóa học, cbhh là một trong những kiến thức nền tảng cung cấp quy luật và định lượng cho các lĩnh vực khoa học mô tả như Hóa học Vô cơ (Hóa học nguyên tô), Hóa học Hữu cơ,... Hệ Hóa học, cụ thể là phản ứng Hóa học, là một hệ nhiết động. Do đó, kiến thức cơ sở của cbhh là các kiến thức Toán học, Vật lí học. Nhà Hóa học vận dụng kiến thức cơ sở đó để hiểu các mối quan hệ định tính, định lượng cùng với quy luật tồn tại trong hệ Hóa học. Chỉ có như vậy nhà Hóa học mới giảng dạy, nghiên cứu có kết quả như mong đợi. Chúng tôi trình bày tài liệu này theo tinh thần khoa học đó cùng với sự chú trọng đúng mức tính sư phạm bằng các dẫn dắt, gợi ý cần thiết trong lí thuyết cùng sự phân tích tóm tắt và có thể có mục Trao đổi về giảng dạy trong một số dạng bài tập. Phần cuối tài liệu có một số bài tập để các bạn đồng nghiệp tham khảo. Hi vọng các bạn sẽ tìm thấy đôi điều bổ ích cho công việc từ tài liệu này. 7

Upload: tam-kisu

Post on 05-Jul-2015

2.484 views

Category:

Documents


22 download

TRANSCRIPT

Page 1: 1ygm3hwjpgu090312060334 tài liệu bồi dưỡng giáo viên trường thpt chuyên   mon hóa học - phan 1- năm 2012

Ph n 1ÇHãA HäC §¹I C¦¥NG

CÂN BẰNG HÓA HỌC – CƠ SỞ GIẢNG DẠY

DẠY VÀ HỌC HÓA HỌC: QUY LUẬT, ĐỊNH LƯỢNG

Trần Thành Huế

Giảng viên cao cấp, Phó Giáo sư, Tiến sĩ

Bộ môn Hóa học lí thuyết và Hóa lí, khoa Hóa học, trường ĐHSP Hà Nội

MỞ ĐẦU

Cân bằng hóa học (cbhh) là một nội dung có tầm quan trọng lớn trong

giảng dạy, nghiên cứu Hóa học (kể cả Hóa học cơ bản lẫn Hóa học công

nghệ và Hóa học ứng dụng nói chung).Trong phạm vi kiến thức Hóa học,

cbhh là một trong những kiến thức nền tảng cung cấp quy luật và định lượng

cho các lĩnh vực khoa học mô tả như Hóa học Vô cơ (Hóa học nguyên tô),

Hóa học Hữu cơ,... Hệ Hóa học, cụ thể là phản ứng Hóa học, là một hệ nhiết

động. Do đó, kiến thức cơ sở của cbhh là các kiến thức Toán học, Vật lí học.

Nhà Hóa học vận dụng kiến thức cơ sở đó để hiểu các mối quan hệ định tính,

định lượng cùng với quy luật tồn tại trong hệ Hóa học. Chỉ có như vậy nhà

Hóa học mới giảng dạy, nghiên cứu có kết quả như mong đợi.

Chúng tôi trình bày tài liệu này theo tinh thần khoa học đó cùng với sự

chú trọng đúng mức tính sư phạm bằng các dẫn dắt, gợi ý cần thiết trong lí

thuyết cùng sự phân tích tóm tắt và có thể có mục Trao đổi về giảng dạy

trong một số dạng bài tập. Phần cuối tài liệu có một số bài tập để các bạn

đồng nghiệp tham khảo. Hi vọng các bạn sẽ tìm thấy đôi điều bổ ích cho

công việc từ tài liệu này.

7

Page 2: 1ygm3hwjpgu090312060334 tài liệu bồi dưỡng giáo viên trường thpt chuyên   mon hóa học - phan 1- năm 2012

(Xin được thưa thêm với quý bạn đọc: Cân bằng hóa học là một trong các cân bằng vốn có của thế giới vật chất, của vũ trụ. Thế giới vật chất tồn tại ở trạng thái cân bằng thích hợp. Vì một nguyên cớ nào đó, do thiên nhiên hay con người gây ra, cân bằng này bị xâm phạm hay bị phá vỡ, sớm hay muộn chúng ta đều phải trả giá. Một loạt thực trạng, từ lỗ thủng tầng ôzôn tới sự tan băng trên các cực và nước biển dâng,… là những bằng chứng “biết nói”).

ĐỀ CƯƠNG

I. Khái quát về: Hệ nhiệt động. Các hàm trạng thái / thế nhiệt động

I.1. Hệ nhiệt động

1. Hệ nhiệt động là gì?

2. Trạng thái, quá trình; tham số

3. Thuận nghịch, bất thuận nghịch nhiệt động

I.2. Các hàm nhiệt động

A. Khái niệm

1. Hàm số trạng thái

2. Thế nhiệt động

B. Các hàm nhiệt động

1. Nội năng U(S,V)

2. Entanpi H(S,P)

3. Năng lượng tự do Hemhon (Hemholtz) F(T,V)

4. Năng lượng tự do Gipxơ (Gibbs) G(T,P)

Tiêu chuẩn về cân bằng nhiệt động hoặc quá trình tự hay không tự xảy ra (kể cả phản ứng hóa học).

II. Cân bằng hoá học

II.2.1. Phần định tính

A. Khái niệm: 1. Nội dung; 2. Nhận xét

B. Chuyển dời/dịch cbhh: 1. Nguyên lí Lơ Satơlie; 2. Các yếu tố

8

Page 3: 1ygm3hwjpgu090312060334 tài liệu bồi dưỡng giáo viên trường thpt chuyên   mon hóa học - phan 1- năm 2012

II.2.2. Phần định lượng

A. Bổ sung một số vấn đề NĐLHH: 1. Thế hoá học; 2. Hệ số hoạt độ

B. Tỉ số (hay tỉ lệ) Q: 1. Pha khí; 2. Dung dịch

C. Hằng số cbhh: 1. Pha khí; 2. Dung dịch

Bài tập áp dụng

II.2.3. Tính cân bằng hoá học

A. Dựa vào pha của các chất trong phản ứng

1. cbhh trong pha lỏng (dung dịch hoặc nguyên chất); 2. cbhh có pha khí.

3. cbhh có cả pha lỏng với pha khí,...

B. Dựa vào đặc điểm hoá học của phản ứng thuận nghịch

Có 4 trường hợp điển hình: 1. Cân bằng axit, bazơ; 2. Cân bằng oxi hoá khử;

3. Cân bằng tạo phức; 4. Cân bằng kết tủa, hoà tan kết tủa.

C. Dựa vào liên hệ giữa hằng số cbhh với các đại lượng nhiệt động

1. Từ p­ G 0

pRT lnK∆ = − hay ∆Gopư = – RTlnKC

2. Liên hệ ∆Gopư = – nFEo

3. Tính sự phụ thuộc nhiệt độ của hằng số cbhh.

Bài tập áp dụng

II.2.4. Một số lưu ý

A. Phân biệt sự hằng định của trị số bằng cbhh với sự chuyển dịch vị trí cbhh.

B. Trở lại câu hỏi: "Hằng số cbhh có đơn vị hay không?"

C. Liên hệ giữa cbhh với động hóa học

III. Một số bài tập tham khảo

IV. Một số tài liệu tham khảo chính

9

Page 4: 1ygm3hwjpgu090312060334 tài liệu bồi dưỡng giáo viên trường thpt chuyên   mon hóa học - phan 1- năm 2012

NỘI DUNG CHI TIẾT

I. Khái quát về: Hệ nhiệt động. Các hàm trạng thái / thế nhiệt động

I.1. Hệ nhiệt động

1. Hệ nhiệt động là gì?

Hệ vật chất được khảo sát trong Nhiệt động lực học được gọi là hệ nhiệt động.

(Nhiệt động lực học là lĩnh vực của Vật lí học. Nội dung Nhiệt động lực học áp dụng trong Hóa học tạo ra lĩnh vực Nhiệt động lực Hóa học (qui ước viết tắt là NĐLHH). Các nội dung được đề cập trong tài liệu này thuộc lĩnh vực NĐLHH).

VD1: Nước đá đang tan (tại 4oC, P= 1 bar);

VD2: Phản ứng N2(k) + 3H2 (k) ⇌ 2NH3 (k).

Nói chung, trong hệ nhiệt động xảy ra/ có sự biến đổi chất và năng lượng.

Hệ nhiệt động ngăn cách với môi trường bởi ranh giới phân chia.

2. Trạng thái, quá trình; tham số

a) Trạng thái của một hệ nhiệt động được qui định bởi chất (trạng thái, thành phần định tính, định lượng) và các yếu tố/ điều kiện tồn tại, chủ yếu là nhiệt độ T và áp suất P.

b) Sự chuyển hệ nhiệt động từ trạng thái này tới trạng thái khác, ta nói: đã có một quá trình xảy ra.

c) Các tham số là các đại lượng liên hệ với trạng thái hay quá trình biến đổi hệ.

Có một số căn cứ để phân chia các tham số, thường xét như sau.

*) Tham số quá trình là các đại lượng gắn liền với quá trình, khi quá trình kết thúc các đại lượng này không còn được xét. Hai tham số quá trình điển hình của NĐLHH là nhiệt Q, công A (hay W). Gắn liền với tham số quá trình là hàm số trạng thái (xem phần sau đây).

*) Hai tham số vật lí là nhiệt độ T và áp suất P.

3. Thuận nghịch, bất thuận nghịch nhiệt động.

a) Thuận nghịch nhiệt động là quá trình chuyển hệ từ trạng thái đầu tới trạng thái cuối rồi trở về trạng thái đầu theo đúng con đường đã đi qua mà không để lại bất cứ một biến đổi nào trong trong hệ và môi trường.

10

Page 5: 1ygm3hwjpgu090312060334 tài liệu bồi dưỡng giáo viên trường thpt chuyên   mon hóa học - phan 1- năm 2012

VD3: Thực hiện sự nén – giãn khí trong xi lanh có pit tông không ma sát bằng cách thêm/lấy dần từng lượng (hạt) cát vô cùng nhỏ.

Nhận xét (viết tắt là NX): Đây là quá trình (khái niệm) giả định, không có thật.

Bất thuận nghịch nhiệt động là quá trình không đạt được ít nhất một trong các yêu cầu trên.

NX: Đây là quá trình có thật, thường xảy ra.

I.2. Các hàm nhiệt động

A. Khái niệm

1. Hàm số trạng thái: Một đại lượng, chẳng hạn hàm số f = f(x,y), được gọi là hàm số trạng thái, nếu:

*) Vi phân của nó là một vi phân toàn phần đúng

df = Mdx + Ndy (với M = (f

x

∂∂

)y, còn N = (f

y

∂∂ )x) (I.1a);

Vậy df = (f

x

∂∂

)y dx + (f

y

∂∂ )x dy (I.1b).

*) Biến thiên của nó khi hệ chuyển từ trạng thái 1 tới trạng thái 2 được tính theo:

∆f = f2 – f1 = 2

1

df∫ (∆f được gọi là biến thiên của f) (I.2)#.

(# Nếu viết: f1 – f2 = – ∆f, ta có độ giảm của f).

*) Biến thiên của nó khi hệ thực hiện chu trình (quá trình kín) là:

∆f = df∫Ñ = 0 (I.3).

2. Thế nhiệt động là các đại lượng thông qua nó và các đạo hàm riêng các cấp của nó, tính được các đại lượng đặc trưng cho hệ.

Từ sự thống nhất biểu thức vi phân của nguyên lí I và II, có 4 thế nhiệt động được xác lập (xin xem ngay sau đây).

11

Page 6: 1ygm3hwjpgu090312060334 tài liệu bồi dưỡng giáo viên trường thpt chuyên   mon hóa học - phan 1- năm 2012

B. Các hàm nhiệt động

Cả 4 thế nhiệt động trên cũng đều là hàm số trạng thái. Do đó, chúng còn được gọi là hàm nhiệt động.

1. Nội năng U = U(S,V) (I.4);

2. Entanpi H = H (S,P) (I.5);

3. Năng lượng tự do Hemhon (Hemholtz) F = F (T,V) (I.6);

4. Năng lượng tự do Gipxơ (Gibbs) G = G (T,P) (I.7).

Do cặp biến số qui định, nên phạm vi và mức độ được sử dụng mỗi hàm nhiệt động có khác nhau.

Nội năng U được tính chủ yếu khi áp dụng nguyên lí I, thường là trong bài toán khí lí tưởng hay sự chuyển pha.

Entanpi H có biến thiên ∆H là nhiệt đẳng áp (Qp) của quá trình hay của phản ứng nên được sử dụng khá rộng rãi.

Năng lượng tự do Hemhon (Hemholtz) F = F (T,V) được sử dụng trong các quá trình hay phản ứng xảy ra phù hợp điều kiện về T, V.

Năng lượng tự do Gipxơ (Gibbs) G mà G = G (T,P), được sử dụng rộng rãi nhất.

∆G (chứ không phải ∆Go) được dùng làm tiêu chuẩn về quá trình tự hay không tự xảy ra hoặc đạt tới trạng thái cân bằng nhiệt động (kể cả phản ứng hóa học).

∆G > 0: quá trình không tự xảy ra (I.8);

∆G = 0: quá trình đạt tới trạng thái cân bằng nhiệt động (I.9);

(phản ứng hóa học thuận nghịch đạt tới cân bằng: cân bằng hóa học)

∆G < 0: quá trình tự xảy ra (I.10).

NX: Cơ sở Toán học, Vật lí học đảm bảo tính định lượng, logic cho các kết luận Hóa học. Do đó, dù ở mức độ sử dụng nào, các thầy cô giáo dạy Hóa học cũng cần coi trọng đúng mức cơ sở này.

II. Cân bằng hóa học

II.2.1 – Phần định tính

A. Định nghĩa

12

Page 7: 1ygm3hwjpgu090312060334 tài liệu bồi dưỡng giáo viên trường thpt chuyên   mon hóa học - phan 1- năm 2012

1. Nội dung: Cân bằng hoá học là trạng thái của một phản ứng thuận nghịch, tại đó:

a) trong một đơn vị thời gian, một đơn vị thể tích có bao nhiêu phân tử chất đầu chuyển thành sản phẩm thì cũng có bấy nhiêu phân tử chất đầu đó được tạo ra từ sản phẩm.

b) hoặc tốc độ phản ứng thuận bằng tốc độ phản ứng nghịch.

c) hoặc nồng độ của các chất (của phản ứng) không thay đổi theo thời gian.

2. NX:

Cân bằng hoá học (viết tắt là cbhh) là một trạng thái động, tại đó phản ứng thuận, phản ứng nghịch vẫn xảy ra.

Trong ba cách định nghĩa cbhh ở trên, hai cách đầu chỉ rõ được đặc điểm trạng thái động (hay cân bằng động) nên thường được dùng, trong đó b) phổ biến nhất.

B. Sự chuyển dời vị trí cân bằng hoá học

1. Nguyên lí Lơ Satơlie

Từ thực nghiệm, nhà hoá học người Pháp là Lơ Satơlie (Henri LeChaterlie) đã đưa ra kết luận, ngày nay được gọi là nguyên lí Lơ Satơlie:

"Nếu tác động vào một trong các yếu tố quy định vị trí cbhh, vị trí đó sẽ chuyển dời về phía chống lại ảnh hưởng do tác động này gây ra".

2. Các yếu tố

Trong hoá học xúc tác có vai trò quan trọng (xem, chẳng hạn, sách: Trần Thành Huế, Tư liệu Hóa học 10, NXB Giáo dục, Hà Nội 2006, 2008 mục 6.2 trang 198 – 227). Tuy nhiên xúc tác không gây ảnh hưởng tới sự chuyển dời vị trí cbhh. Còn lại ba yếu tố sau đây được xét kĩ:

a) Lượng chất:

Ta cần lưu ý đề cập khái niệm rộng là lượng chất, bao gồm cả nồng độ, khối lượng, số mol,số thể tích,..., thậm chí cả số nguyên tử hay số hạt nhân.

Yếu tố lượng chất này có vai trò đối với cả phản ứng thuận nghịch pha khí, pha lỏng, pha rắn (xem Bài tập áp dụng II.1 sau đây).

b) Nhiệt độ:

Yếu tố này cũng có ảnh hưởng đến các phản ứng ở các pha khí, lỏng, rắn như yếu tố lượng chất ở trên.

13

Page 8: 1ygm3hwjpgu090312060334 tài liệu bồi dưỡng giáo viên trường thpt chuyên   mon hóa học - phan 1- năm 2012

Áp dụng nguyên lí Lơ Satơlie, ta có kết luận:

Nếu phản ứng thuận thu nhiệt (∆Hpư thuận > 0), sự tăng nhiệt độ dẫn tới tạo

thêm nhiều phân tử/ lượng sản phẩm.

Nếu phản ứng thuận toả nhiệt (∆Hpư thuận < 0), sự tăng nhiệt độ dẫn tới sự tái

tạo nhiều phân tử/ lượng chất đầu.

Về mặt định lượng, nội dung đó được biểu thị bằng phương trình Van Hop (Vant Hoff, nhà hoá học Hà Lan):

∆∂ =∂

p­o

P 2

HlnK( )

T RT (II.1)

Trong đó: K là hằng số cbhh (xem mục II.2.2 sau đây):

∆Hopư là nhiệt của phản ứng được xét

R là hằng số khí

T là nhiệt độ của phản ứng (bắt buộc theo thang Kenvin).

c) Áp suất

Trước hết cần chú ý phân biệt áp suất riêng(phần) pi của từng chất trong hệ với áp suất hệ P hay còn gọi là áp suất toàn phần hay áp suất chung.

Với các khí lí tưởng các pi liên hệ với P theo định luật Đantơn

P = ii

p∑ (II.2)

Lưu ý tiếp theo là yếu tố áp suất chỉ có vai trò đối với chất khí.

Ta xét một phản ứng thuận nghịch tổng quát:

λ1A1 + λ2A2 +... + λlAl ⇌ ν1B1 + ν2B2 +... + νmBm (II.3)

Kí hiệu

∆n = ∑j νj – ∑i λi (II.4)

Với nội dung đang xét, ta giả thiết các chất trong (II.3) đều là khí lí tưởng, theo nguyên lí Lơ Satơlie:

− Nếu ∆n > 0, sự tăng áp suất P thuận lợi cho sự tái tạo các chất đầu.

− Nếu ∆n < 0, sự tăng áp suất P thuận lợi cho sự tạo ra sản phẩm.

14

Page 9: 1ygm3hwjpgu090312060334 tài liệu bồi dưỡng giáo viên trường thpt chuyên   mon hóa học - phan 1- năm 2012

Bài tập áp dụng II.1

Áp dụng nguyên lí Lơ Satơlie, hãy xét sự chuyển dời vị trí cbhh của mỗi phản ứng hoá học thuận nghịch sau (đề nghị bạn đọc thêm điều kiện cần thiết (nếu có) của từng phản ứng):

1. RCOOH + R'OH ⇌ RCOOR' + H2O (pha lỏng)

Cụ thể CH3COOH + C2H5OH ⇌ CH3COOC2H5 + H2O

2. N2 (k) + 3H2 (k) ⇌ 2NH3 (k)

3. 2SO2 (k) + O2 (k) ⇌ 2SO3 (k)

4. CaCO3 (r) ⇌ CaO (r) + CO2 (k)

5. NH4HS (r) ⇌ NH3 (k) + H2S (k)

Quy ước viết tắt: k là khí; r là rắn.

Đây là các phản ứng cơ bản được xét trong SGK, xin bạn đọc vui lòng cho câu trả lời.

Bài tập áp dụng II.2

Tại sao người ta nói (II.1) là biểu thức định lượng của nguyên lí Lơ Satơlie khi xét ảnh hưởng của nhiệt độ tới sự chuyển dời vị trí cbhh?

Trả lời tóm tắt: Trong (II.1) R > 0; T > 0;

− Nếu ∆Hopư < 0,

dlnKdT

< 0, lnK (hoặc K) nghịch biến với nhiệt độ T. Vậy

khi phản ứng toả nhiệt, cbhh chuyển dời sang trái (sự tái tạo chất đầu thuận lợi), nếu T tăng.

Kết quả đó phù hợp với kết luận được rút ra từ nguyên lí Lơ Satơlie.

− Đề nghị bạn cho câu trả lời chi tiết nếu ∆Hopư > 0.

II.2.2. Phần định lượng

A. Bổ sung một số vấn đề NĐLHH

Một phản ứng hoá học thuận nghịch nói chung, ở trạng thái cân bằng nói riêng, là một hệ nhiệt động.

15

Page 10: 1ygm3hwjpgu090312060334 tài liệu bồi dưỡng giáo viên trường thpt chuyên   mon hóa học - phan 1- năm 2012

1. Thế hoá học

Với mỗi cấu tử i (khái niệm "cấu tử" mới phù hợp về mặt nhiệt động, ta dùng gần đúng là "chất"), đều có

j

i ii T,P,n

GG

n∂ = = µ ÷∂ (II.5)

iG là năng lượng Gipxơ (hay thế đẳng áp) mol riêng (phần) của chất i.

iµ là thế hoá học của chất i (trước đây vẫn gọi là hoá thế).

Kí hiệu j

i T,P,n

Gn

∂ ÷∂ cho biết: Lấy đạo hàm của G theo số mol chất i khi giữ

không đổi T, P và số mol các cấu tử j khác trong hệ.

Với mỗi hàm nhiệt động U, H, F ta cũng có biểu thức tương tự (II.5).

Từ các liên hệ nhiệt động, biểu thức chi tiết của iG (hay µi) là:

i

0 0i iG G RT ln(f / p )= + ; i là khí thực (II.6a)

hay i

0 0i iRT ln(f / p )µ = µ + (II.6b)

i

0 0i iG G RT ln(p / p )= + ; i là khí lí tưởng (II.6c)

hay i

0 0i iRT ln(p / p )µ = µ + (II.6d).

2. Hệ số hoạt độ/ hoạt áp

Hoạt áp fi của khí thực liên hệ với áp suất riêng phần pi nếu đó là khí lí tưởng

(hay được xét ở điều kiện tiêu chuẩn):

fi = γpi (II.7a)

γ là hệ số hoạt áp; điều kiện tiêu chuẩn được chọn sao cho γ = 1 → fi = pi

(II.7b).

i

0µ là thế hoá học ở điều kiện tiêu chuẩn, tỉ lệ với i

0G .

Đối với các khí, thường được coi là khí lí tưởng, người ta chọn

p0 = 1 bar (hay 1atm) là áp suất ở điều kiện tiêu chuẩn (II.8)

16

Page 11: 1ygm3hwjpgu090312060334 tài liệu bồi dưỡng giáo viên trường thpt chuyên   mon hóa học - phan 1- năm 2012

Áp dụng (II.8) thì, (II.6c) và (II.6d) được rút lại gọn hơn:

iG = i

0G + RTlnpi = µi = i

0µ + RTlnpi (II.6e)

Trong (II.6e) pi là một lượng không thứ nguyên (không đơn vị) theo quy ước (II.8), phù hợp với phép toán logarit là lnX hay lgX, X chỉ là một số (một lượng) không thứ nguyên.

Tương tự trên, với mỗi chất i có nồng độ CM:

iG = i

0G + RTln (ai /C0) II.9a) hay µi = i

0µ + RTln (ai / C0) (II.9b) (dd thực).

iG = i

0G + RTln (Ci /C0)(II.9c) hay µi = i

0µ + RTln (Ci / C0) (II.9d) (dd loãng/lt).

Hoạt độ ai của chất; trong dung dịch thực liên hệ với nồng độ Ci trong dung dịch đủ loãng (hay dung dịch lí tưởng):

ai = γCi (II.10a).

Khi điều kiện tiêu chuẩn được chọn, có kết quả gần đúng được chấp nhận:

γ = 1 → ai = Ci (II.10b)

Còn có Quy ước C0 = 1 M (II.11)

Áp dụng quy ước này và chú ý tới (II.10b), ta viết lại (II.9c) và (II.9d) như sau:

iG = i

0G + RTln Ci = µi = i

0µ + RTln Ci (II. 9e)

Cũng tương tự pi trong (II.6e), Ci trong (II.9e) là lượng không thứ nguyên. Đây chỉ là quy ước !

Cùng với các quy ước trên, từ việc chọn điều kiện tiêu chuẩn cho các cấu tử (chất) trong dung dịch, còn có quy ước

Chất rắn nguyên chất có hoạt độ hằng định, thường lấy bằng 1. (II.12)

Theo quy ước này, hoạt độ hay nồng độ chất rắn nguyên chất không có mặt trong biểu thức của tỉ số Q hay bằng số cbhh K (vì bằng 1 chứ không phải bằng 0).

B. Tỉ số (hay tỉ lệ) Q

Ta trở lại phản ứng tổng quát (II.3) trên:

17

Page 12: 1ygm3hwjpgu090312060334 tài liệu bồi dưỡng giáo viên trường thpt chuyên   mon hóa học - phan 1- năm 2012

λ1A1 + λ2A2 +... + λlAl ⇌ ν1B1 + ν2B2

+... + νmBm

Các kết quả trên có thể thu được từ việc xét hàm: G = G (T, P, n, ξ); (trong

đó ξ là mức độ (xảy ra) của phản ứng).

Từ NĐLHH, ta có dG = – SdT + VdP + i ii

dnµ∑ (II.13a)

Giả thiết ta xét phản ứng xảy ra khi T = const, P = const (đẳng nhiệt, đẳng áp), biến thiên vô cùng nhỏ của năng lượng tự do Gipxơ của phản ứng, dGpư, được tính theo

dG = i ii

dnµ∑ (II.13b).

Mà dni = νidξ (II.14).

νi là hệ số chất i bất kỳ, chất đầu hay sản phẩm, trong (II.3).

Đưa (II.14) vào (II.13b), ta có: dG = i

∑ μiνidξ (II.13c).

Từ NĐLHH, ta có: ,( )T P i i pui

GGν µ

ξ∂ = = ∆∂ ∑ (II.14)

Chú ý (II.5) trên, biến thiên năng lượng tự do Gipxơ của phản ứng, ∆Gpư,

được tính theo: ∆Gpư = m l

j ij ij 1 i 1

G G= =

ν − λ∑ ∑ (II.15).

1. Nếu phản ứng pha khí, ta đưa (II.6e) vào (II.15), sẽ được:

ν λ ∆ ∆ + π π p­p­ = j i

j i

0B A

j iG G RT ln (p ) / (p ) (II.16)

Kí hiệu j 1 2 m

1 2 mj B B Bj(p ) p p ...p

ν ν ν νπ = (II.17a)

1 2 li1 2 li A A A(p ) p p ...pλ λ λλπ = (II.17b).

Với pi là áp suất riêng phần của khí i ở vị trí bất kì được xét của phản ứng.

Người ta quy ước dùng kí hiệu:

18

Page 13: 1ygm3hwjpgu090312060334 tài liệu bồi dưỡng giáo viên trường thpt chuyên   mon hóa học - phan 1- năm 2012

1 2 mj1 2 m i

j i1 2 l1 2 l

B B Bp B A

j iA A A

p p ...pQ (p ) / (p )

p p ...p

ν ν νν λ

λ λ λ = = π π

(II.18)

Qp (hay Q) là tỉ số hay tỉ lệ của phản ứng (II.3) ở vị trí bất kì được xét của phản ứng thuận nghịch.

Khi đưa (II.18) vào (II.16), ta có:

∆Gpư = ∆G0 + RTlnQ (II.19)

Ghi chú: Nếu dùng (II.6e), ta có Qp.[(po)∆n], nghĩa là: Lượng (bằng số) của tỉ lệ Q nhân với đơn vị thích hợp (bar) ∆n.

2. Nếu phản ứng trong dung dịch, đưa (II.9e) vào (II.15), ta có:

λ

ν ∆ = ∆ + π

π i

i

opu pu Aj i

jG G RTln / CBj

C (II.20)

Kí hiệu π đã quen thuộc. Lúc đó, ta có:

1 2 mj1 2 m i

B A1 2 l j i1 2 l

B B Bp

j iA A A

C C ...CQ C / C

C C ...C

ν ν νν λ

λ λ λ = = π π

(II.21)

Qc hay Q là tỉ số hay tỉ lệ của phản ứng thuận nghịch (II.3) ở vị trí bất kì của phản ứng đó.

Thế (II.21) vào (II.20), ta có:

∆ ∆ +0G RT lnQp­p­ G = (II.22).

Ghi chú:

*) Về nguyên tắc, đặc biệt với phản ứng pha khí, phải phân biệt Qp với QC hay Kp với KC vì biến số hàm Gipxơ G là T, P khác biến số hàm Hem hon F là T, V.

Có liên hệ: Nếu phản ứng pha khí có ∆n ≠ 0, ∆G0 = ∆F0 + RT∆n. Tuy nhiên với

dung dịch lỏng biến thiên thể tích do phản ứng không đáng kể, ∆V ≈ 0, nên có thể dùng ∆G như trên.

*) Cũng như Qp ở trên, với QC, nếu chú ý tới thứ nguyên, ta sẽ có:

19

Page 14: 1ygm3hwjpgu090312060334 tài liệu bồi dưỡng giáo viên trường thpt chuyên   mon hóa học - phan 1- năm 2012

QC.[(Co)∆n )] = QC. ( ) ∆− n1mol.L (II.23).

(Giá trị bằng số của tỉ lệ Q nhân với đơn vị thích hợp).

*) Với trường hợp dung dịch, có một điểm xin lưu ý bạn đọc: Việc chọn điều kiện tiêu chuẩn cho chất tan và dung môi có khác nhau. Tuy nhiên do chấp nhận

gần đúng γ → 1 → Ci → ai nên sự việc đỡ phức tạp hơn. Dù vậy, với các dung

dịch loãng chất điện li cũng khá phức tạp vì số tiểu phân trong dung dịch tăng lên (so với số tiểu phân chất ban đầu). Trong trường hợp này phải tính đến tương tác giữa các tiểu phân thông qua hoạt độ trung bình, hệ số hoạt độ trung bình.

*) Ngoài phản ứng trong dung dịch, một số trường hợp phản ứng pha khí cũng có thành phần hệ được biểu thị theo nồng độ C (mol. L_1) như vừa xét ở trên.

C. Hằng số cân bằng hoá học

Áp dụng điều kiện (I.9) cho phản ứng (II.3): Khi phản ứng này đạt tới cbhh ta có:

∆Gpư = 0 (II.24)

1. Với phản ứng pha khí: Đưa (II.24) này vào (II.21), ta có

∆ ∆ +0G RT lnQp­p­ G = 0 = (II.25a)

Vậy: ∆Gpư o = – RTlnQ = – RTlnK (II.25b)

Vậy, tại cbhh ta có:

jj

ii

Bj

pAi

(p )K

(p )

ν

λ

π=

π

(II.26)

Trong đó pi là áp suất riêng phần chất i tại cbhh.

Khi T = const; với một phản ứng thuận nghịch cụ thể, trị số Kp hằng định nêu được gọi là hằng số cân bằng nhiệt động (hay gọi tắt là hằng số cbhh).

(Cụm từ tiếng Anh của thuật ngữ trên là: thermodynamic equilibrilium constant; chẳng hạn, xem sách: Atkins, Physical Chemistry, Oxford Uni. Press.,

1998, trang 220, dòng 2↓).

Đưa (II.26) vào (II.25b), ta có:

20

Page 15: 1ygm3hwjpgu090312060334 tài liệu bồi dưỡng giáo viên trường thpt chuyên   mon hóa học - phan 1- năm 2012

p­ G0

pRT lnK∆ = − (II.25c)

Đây là một trong những phương trình có vai trò quan trọng của NĐLHH.

Ghi chú: Cũng tương tự với Q, ở đây ta cũng có Kp[(po)∆n ].

Vậy giá trị bằng số của K nhân với đơn vị thích hợp

[(po)∆n ] = (bar)∆n (II.27).

2. Với phản ứng trong dung dịch (hay pha khí có thành phần theo nồng độ C).

Xét tương tự trên, ta có:

∆Gpư = ∆Gpưo + RTlnQ (II.28)

Vậy

j

j

i

i

Bj

c

Ai

K

ν

λ

π =

π

(II.29)

Kí hiệu [ ] chỉ nồng độ cân bằng.

KC là hằng số cân bằng nhiệt động cho phản ứng thuận nghịch trong dung dịch hay pha khí có thành phần biểu thị theo nồng độ mol.L –1.

Ở đây ta cũng có biểu thức quan trọng: p­ G 0

cRT lnK∆ = − (II.30)

Ghi chú: Đơn vị của Kp hay KC được giải quyết tương tự trên.

Vậy: Với một phản ứng hóa học thuận nghịch xác định, tại nhiệt độ xác định (T=Const), Kp hay KC là hằng số. Đó là hằng số cân bằng hóa học.

Hằng số cân bằng hóa học là một đại lượng nhiệt động (hằng số cân bằng nhiệt động).

Nói chung, hằng số cân bằng hóa học có thứ nguyên/đơn vị thích hợp:

Kp[(po)∆n ] = Kp. [(bar hay atm)∆n];

KC.[(Co) ∆n] = KC.[(mol.L –1 hay M) ∆n].

(Xin xem thêm Bài tập áp dụng II.3 và điểm B. mục II.2.4 sau đây).

Ta xét tiếp một nội dung lí thú đáng chú ý về hằng số cân bằng hóa học.

Bài tập áp dụng II.3

21

Page 16: 1ygm3hwjpgu090312060334 tài liệu bồi dưỡng giáo viên trường thpt chuyên   mon hóa học - phan 1- năm 2012

1. Trị số của hằng số cbhh K có giá trị trong khoảng nào (của trục số thực)? Trong khoảng đó, đoạn hay phần nào tương ứng với sự chiếm ưu thế của phản ứng thuận? Phản ứng nghịch? Tại sao?

2. Hệ số các chất trong (II.3) đều được nhân với thừa số x, phương trình hoá học mới có hằng số cbhh kí hiệu là K'.

a) K' liên hệ với K như thế nào?

b) Sự thay đổi của trị số hằng số cbhh có ý nghĩa hoá học không? Tại sao?

c) Thực tế quy ước như thế nào về hệ số các chất trong phương trình phản ứng?

Trả lời tóm tắt:

1. Trị số của K trong khoảng: 1 < K< ∞ (II.31)

Giải thích: Dựa vào biểu thức của K (cũng có thể dựa vào ∆Go).

Có 2 đoạn / phần: 1 < K < ∞: Phản ứng thuận chiếm ưu thế.

0 < K < 1: Phản ứng nghịch chiếm ưu thế.

Nếu K = 1. Mời bạn cho câu trả lời (hoặc xem Bài tập IV.1 dưới đây).

2. a) Liên hệ K' = Kx (II.32).

Chú ý: x là số nguyên hoặc phân số.

b) Sự thay đổi K (cả trị số và đơn vị) đó không có ý nghĩa hoá học, chỉ thuần tuý do làm toán.

c) Quy ước: Dùng bộ hệ số là số nguyên tối giản nhất.

Trao đổi về giảng dạy: Sẽ còn một số câu hỏi phát sinh từ nội dung trên. Có thể là:

*) Trong các nội dung trên, có phải đề cập chỉ với 1 (một) phản ứng cụ thể các phản ứng?

*) Sự thay đổi hệ số (các) chất trong ptpư làm thay đổi K (cả trị số và đơn vị), nhưng không có ý nghĩa hoá học. Vậy khi làm toán, cần xử lí vấn đề này như thế nào?

Bài tập áp dụng II.4

Có những trường hợp nào về liên hệ giữa Q với K của một phản ứng? Ở mỗi trường hợp đó, phản ứng thuận nghịch xảy ra như thế nào? (T = const).

Trả lời tóm tắt:

Có 3 trường hợp liên hệ:

22

Page 17: 1ygm3hwjpgu090312060334 tài liệu bồi dưỡng giáo viên trường thpt chuyên   mon hóa học - phan 1- năm 2012

a) Q > K (II.32);

b) Q < K (II.33);

c) Q = K. (II.34)

Đề nghị bạn cho ý kiến tiếp theo về việc xảy ra phản ứng thuận, nghịch ở mỗi trường hợp và nếu được, hãy nêu ví dụ minh họa.

Bài tập áp dụng II.5

Giả thiết có cbhh trong pha lỏng

2FeCl2 (aq) + Cl2 (aq) ⇌ 2FeCl3 (aq) (*)

Vị trí cbhh đó biến đổi như thế nào trong mỗi trường hợp sau đây? Giải thích cụ thể.

1. Pha loãng dung dịch bằng một lượng thích hợp nước nguyên chất.

2. Hoặc thêm vào hệ (*) một lượng thích hợp dung dịch KMnO4 có H2SO4

loãng.

Giả thiết chỉ xét (*) trong dung dịch; bỏ qua các quá trình phụ khác liên quan tới hệ (*); lượng thích hợp là lượng vừa đủ gây ra một biến đổi trực tiếp đối với hệ (*), không xét biến đổi tiếp theo.

Trả lời tóm tắt:

1. Có thể xét định tính hoặc liên hệ Q với K (xem Bài tập áp dụng II.4 trên).

2. Bạn nên dùng thêm trị số thế khử tiêu chuẩn E0OX/Red để trả lời câu hỏi này.

Trao đổi về giảng dạy: Nảy ra một câu hỏi thú vị từ nội dung trên:

Cần hiểu cụ thể như thế nào về yếu tố “lượng chất” khi xét nguyên lí Lơ Satơlie?

Sau các bài tập trên, chắc chắn bạn sẽ hỏi: Kx có phải là hằng số cbhh không? Nếu có quan tâm như vậy, bạn thực sự sẽ có cơ hội hiểu sâu hơn, áp dụng tốt hơn bài toán cbhh. Ta sẽ xét tóm tắt vấn đề thú vị này.

Với một dung dịch hay hỗn hợp khí hay rắn, ta có

= ∑i

ij

j

nx

n (được gọi là phần mol hay tỉ lệ mol của i) (II.35).

Trong đó i là một chất (khí) xác định; j bao gồm tất cả các chất của hệ, kể cả i.

23

Page 18: 1ygm3hwjpgu090312060334 tài liệu bồi dưỡng giáo viên trường thpt chuyên   mon hóa học - phan 1- năm 2012

NX: xi là lượng không thứ nguyên;

có: 0 < xi < 1 (II.36) và ii

x∑ = 1 (II.37).

Từ đó, ta cũng có biểu thức cho Qx và Kx tương tự QC (hay Qp) và KC (hay Kp).

Tất nhiên cũng có Qx và Kx đều là các lượng không thứ nguyên.

Chỉ trong phản ứng pha khí (Tại sao? Mời bạn cho câu trả lời) mới thiết lập được liên hệ giữa Kx với KC (hay Kp) của một phản ứng.

Lưu ý: Dù có hay không có liên hệ với KC (hay Kp), nói chung, Kx vẫn chưa phải là hằng số cbhh! (Nó chỉ là một tỉ số hay một tỉ lệ). Vậy khi nào Kx là hằng số cbhh?

Mời bạn cho câu trả lời. Phần sau sẽ có bài toán áp dụng Kx.

II.2.3. Tính cân bằng hoá học

Đây là một nội dung rất quan trọng khi xét cân bằng hoá học. Có một số căn cứ khác nhau để phân loại bài toán này.

A. Dựa vào pha của các chất trong phản ứng

Theo căn cứ này, có 2 trường hợp chính:

1. cbhh trong pha lỏng (dung dịch hoặc nguyên chất).

2. cbhh có pha khí.

3. cbhh có cả pha lỏng với pha khí,...

B. Dựa vào đặc điểm hoá học của phản ứng thuận nghịch

Theo căn cứ này, ta có 4 trường hợp điển hình:

1. Cân bằng axit, bazơ.

2. Cân bằng oxi hoá khử.

3. Cân bằng tạo phức.

4. Cân bằng kết tủa, hoà tan kết tủa.

(Trong mỗi bài toán, có thể xét riêng rẽ từng cân bằng hoặc kết hợp từ hai trở lên trong bốn cân bằng trên).

Một số người vẫn cho rằng đây là lĩnh vực hoá học phân tích. Xét về bản chất, đây là các bài toán cbhh áp dụng cụ thể với từng đối tượng, hoặc phối hợp một số đối tượng trong cùng một bài toán.

24

Page 19: 1ygm3hwjpgu090312060334 tài liệu bồi dưỡng giáo viên trường thpt chuyên   mon hóa học - phan 1- năm 2012

Trong hai dạng bài toán cbhh trên, nội dung chung là: Tính K và các đại lượng trong biểu thức của K (áp suất (riêng pi hoặc của hệ P); nồng độ hay thành phần hệ;…) tại cbhh.

C. Dựa vào liên hệ giữa hằng số cbhh với các đại lượng nhiệt động

Có một số trường hợp chủ yếu sau đây:

1. Từ p­ G0

pRT lnK∆ = − (II.38a) hay ∆Gopư = – RTlnKC (II.38b)

Biểu thức (II.38a) được dùng nhiều hơn (Liên hệ ∆Gopư với KC đề cập phần trên).

Mặt khác, có liên hệ: ∆Gpư = ∆Hpư – T∆Spư (II.39a)

Và ∆Gopư = ∆Ho

pư – T∆Sopư (II.39b).

∆H và ∆H0 được tính theo một trong các cách xác định nhiệt phản ứng;

∆S0 được tính dựa vào nguyên lí III của NĐLHH (hay định lí nhiệt của Nernst).

(Sơ lược về nguyên lí III hay định lí nhiệt của Nernst: Tại 0K, chất nguyên chất có entropi bằng zero: So

0K = 0; do đó ∆So = SoT – So

0K = SoT (#)).

2. Liên hệ ∆Gopư = – nFEo (II.40).

Phản ứng được xét có thể tại mỗi cực của pin (bán phản ứng) hoặc phản ứng toàn mạch (tổng cộng).

Kết hợp (II.39b) với (II.40), ta có:

RTlnK = nFE0 → K = exp[nFE0/RT] (II.40).

Đây là biểu thức cơ sở để tính cân bằng oxi hoá khử ta đã đề cập ở trên.

3. Tính sự phụ thuộc nhiệt độ của hằng số cbhh.

Cơ sở cho dạng bài toán này là phương trình Kiêchôp đã đề cập ở trên:

∆∂ =

∂p­

o

P 2

HlnK( )

T RT (II.1).

a) Dùng (II.1) này để biện luận (xem Bài tập áp dụng (II.2)) trên.

b) Tích phân không xác định (kxđ) phương trình (II.1), ta được:

lnK = –[(∆Hopư)/RT] + const (II.41).

Trong đó: const là hằng số tích phân kxđ; các đại lượng khác đã quen thuộc. Phương pháp đồ thị thường được sử dụng cho (II.41).

25

Page 20: 1ygm3hwjpgu090312060334 tài liệu bồi dưỡng giáo viên trường thpt chuyên   mon hóa học - phan 1- năm 2012

Tích phân xác định (xđ) giữa hai nhiệt độ T1 và T2 phương trình (II.1), ta được:

ln(K2/ K1) = (∆Hopư/R)[(T2 – T1)/ T1. T2] (II.42).

Phương pháp tính thường được sử dụng cho (II.42).

Tất nhiên, sự phân loại trên chỉ có tính quy ước, trong thực tế có những bài toán cbhh đòi hỏi phải vận dụng phối hợp các kiến thức và biểu thức khác nhau.

Tính cân bằng hoá học là một vấn đề rất thú vị. Với các thầy cô dạy Hóa học trường THPT chuyên, những tổng kết tóm tắt và đôi điều gợi mở về kiến thức cùng tư duy về cơ sở của cbhh như vậy, có lẽ tạm đủ. Dưới đây chúng tôi giới thiệu tiếp một số bài tập áp dụng để quý vị tham khảo cả về nội dung và cách suy xét vấn đề khi dạy và học.

Bài tập áp dụng II.5

Tại 25oC, phản ứng CH3COOH + C2H5OH ⇌ CH3COOC2H5 + H2O

có K = 4. Ban đầu người ta trộn 0,6 mol C2H5OH với 0,5 mol CH3COOH.

1. Tại cbhh thu được bao nhiêu mol este?

2. Theo bạn, có những điểm nào cần chú ý khi thực hiện lời giải trên (để hiểu đầy đủ và sâu sắc nội dung của vấn đề được đặt ra)?

Trả lời tóm tắt:

1. Kí hiệu số mol este được tạo thành là x; điều kiện: 0 < x < 0,5 (a)

Liên hệ CH3COOH + C2H5OH ⇌ CH3COOC2H5 + H2O (*)

Ban đầu 0,5 0,6 0 0

Phản ứng x x x x

Tại cbhh (0,5 − x) (0,6 − x) x x

Vậy: x.x

K 4(0,5 x(0,6 x)

= =− − (b)

Giải phương trình bậc 2 là (b), thu được 2 nghiệm:

x1 = 0,483 và x2 > 1.

Theo điều kiện (a), chỉ có x1 = x = 0,483 mol.

2. Chúng tôi nêu một số vấn đề, mời bạn cho ý kiến.

26

Page 21: 1ygm3hwjpgu090312060334 tài liệu bồi dưỡng giáo viên trường thpt chuyên   mon hóa học - phan 1- năm 2012

− Nếu đặt điều kiện (a) là 0 ≤ x ≤ 0,5 có được không?

− Có bạn cho rằng (b) phải là x

4(0,5 x)(0,6 x)

=− − (ở tử số là x chứ không

phải x2 vì H2O là dung môi). Đúng hay sai?

− Có cần xét nồng độ cụ thể là mol.L –1 hay phần mol? Tại sao?

Ghi chú: Cùng với các nội dung trên nhưng có tầm bao quát hơn, mời các bạn tham khảo kỹ Bài tập IV.2 trong mục IV sau đây.

Bài tập áp dụng II.6

Tại nhiệt độ T = const, phản ứng PCl3(k) + Cl2(k) ⇌ PCl5(k) có Kc = 1,9.

Hỗn hợp cân bằng trên được chứa trong bình thể tích 2,0 lít có 0,50 mol PCl5

(k); 0,32 mol PCl3 (k).

1. Tính nồng độ cân bằng của Cl2 (k).

2. Bài toán đơn vị được xử lí như thế nào trong trường hợp này?

Trả lời tóm tắt:

1. Kí hiệu [ ] được dùng để chỉ nồng độ cân bằng. Ta có:

[ ][ ] [ ]

5c

2 3

PClK

Cl PCl= → [ ] [ ]

[ ]5

2c 3

PClCl

K PCl= (a)

Tính nồng độ cân bằng của các chất đã biết, đưa vào (a), ta có:

[ ]20,25

Cl1,9 0,16

=× → [Cl2] = 0,82 M (b)

2. Bài toán đơn vị: [Cl2] = M/(M –1M) = M

Kí hiệu [Cl2 ] ở vế trái để chỉ bài toán đơn vị nồng độ cb của Cl2.

Vậy ta có thể xét đơn vị kèm theo trị số Kc = 1,9 theo:

+) Ta có: ( ) n0cK C

∆ (c). Với phản ứng đã cho ∆n = −1; C0 = 1M

Vậy ( ) n0cK C

∆= 1,9 M –1 (d)

+) Hoặc dựa vào phương trình và số liệu đã cho trong đề bài, ta cũng có được trị số Kc kèm theo đơn vị thích hợp là 1,9 M –1.

27

Page 22: 1ygm3hwjpgu090312060334 tài liệu bồi dưỡng giáo viên trường thpt chuyên   mon hóa học - phan 1- năm 2012

Vậy kết luận rút ra là nếu không chú ý tới đơn vị kèm theo trị số 1,9 ta sẽ thiếu căn cứ để làm bài toán đơn vị nồng độ cb của Cl2.

Bài tập áp dụng II.7

1. Hãy viết biểu thức Kp, Kc cho phản ứng (II.3) ở pha khí, giả thiết đều là khí lí tưởng.

2. Thiết lập biểu thức liên hệ Kp với Kc; Kp với Kx.

3. Kx có phải là hằng số cbhh không? Tại sao?

Trả lời tóm tắt:

1. Ta sẽ có biểu thức Kp là (II. 26), Kc là (II. 29). Để có biểu thức Kx, ta chú ý

= ∑

ii

jj

nx

n (II.36)

Tương tự với Kp, Kc ta có: Kx = [πj(xj)λj]/[πi(xi)ν

i] (II.37)

2. Liên hệ: Khí lí tưởng tuân theo phương trình trạng thái

PV = nRT (II.38)

Từ đó lấy điểm liên hệ:

Kp = Kc (RT)∆n (II.39) và Kp = Kx. P∆n (II.40a)

3. Trước khi trả lời câu hỏi này, ta cần nhớ: Hằng số cbhh, Kp, Kc, không phụ

thuộc áp suất, tức là TK

( ) 0∂ =∂ρ (II.41)

Để xét Kx, từ (II.40) ta có Kx= Kp. P −∆n (II.40b).

Vậy Kx chỉ hằng số khi phản ứng có ∆n = 0 hay P = 1bar (hoặc 1atm).

Bài tập áp dụng II.8

Tại T = const; P = 10,00 atm phản ứng N2(k) + 3H2(k) ⇌ 2NH3 (k) (*)

đạt tới cbhh có Kp = 1,64.10−4.

1. Biết rằng lượng khí ban đầu được lấy theo tỉ lệ mol H2 với N2 theo hệ số phương trình phản ứng, hãy tính tỉ lệ khí NH3 trong hỗn hợp cân bằng. (Có những cách tính nào?)

2. Bài toán đơn vị ở đây được giải quyết như thế nào?

28

Page 23: 1ygm3hwjpgu090312060334 tài liệu bồi dưỡng giáo viên trường thpt chuyên   mon hóa học - phan 1- năm 2012

3. Nếu phương trình (*) được viết 2 2 31 3

N (k) H (k) NH (k)2 2

+ = (**)

có dùng Kp = 1,64.10−4 được hay không?

Trả lời tóm tắt:

1. Có 2 cách giải:

Cách 1: Dựa vào Kp: Kí hiệu áp suất riêng phần của NH3 trong hỗn hợp cbhh là p; điều kiện: 0 < p < 10 (a);

(áp suất của N2, H2, tương ứng là pN2, pH2). Có các liên hệ:

pN2 + pH2 + p = P = 10 atm → (pN2 + pH2) = (10 − p).

Tổng số áp suất (pN2 + pH2) có 4 phần, pN2 chiếm 14

; pH2 chiếm 34

(b).

Có Kp = (p)2/[(pN2)(pH2)3] (c); thay liên hệ ở (b) vào (c), được:

24

3

p1,64.10

1 3(10 p). (10 p)

4 4

−= − −

(d).

Trong (d), p là pNH3. Đây là phương trình bậc 4 đối với p, sẽ được xuống bậc 2 (cả hai vế). Đáp số: NH3 chiếm 3,85%.

Cách 2: Kí hiệu phần mol NH3 trong hỗn hợp cân bằng là x; điều kiện: 0 < x < 1. Lập luận tương tự trên, ta cũng có phương trình bậc 4 đối với x (chú ý dùng Kx; có liên hệ với Kp theo (II.40b)).

Kết quả: NH3 chiếm 3,85%.

NX: Hai cách giải trên tương đương nhau; cách 2 giải được ngay đáp số.

Lưu ý: Thầy cô cần có yêu cầu cho HS, đặc biệt là HS trường chuyên “tìm các cách khác nhau có thể để giải một bài toán Hóa học. Nhận xét/so sánh, nếu có, giữa các cách đó”.

2. Bài toán đơn vị: Cũng lập luận như ở Bài tập áp dụng II.6 trên, ở đây trị số

1,64.10−4 phải có đơn vị thích hợp là: Kp = 1,64.10−4(atm)−2.

3. Nếu nhân hệ số các chất trong phương trình hoá học ban đầu với 12

thì

Kp (**) = (Kp (*) )1/2

29

Page 24: 1ygm3hwjpgu090312060334 tài liệu bồi dưỡng giáo viên trường thpt chuyên   mon hóa học - phan 1- năm 2012

Bài tập áp dụng II.9

1. Khảo sát phản ứng thuận nghịch pha khí: NH4Cl ⇌ NH3 + HCl (*).

Tại nhiệt độ T thích hợp phản ứng đạt tới cbhh, áp suất chung của hệ là P, độ

phân li của NH4Cl là α.

* Hãy thiết lập biểu thức tính hằng số cbhh Kp của phản ứng.

* Viết biểu thức liên hệ Kc với Kp, Kx với Kp của phản ứng. Trong trường hợp này Kx có phải là một lượng hằng định hay không? (Vẫn xét ở cùng nhiệt độ T). Hãy giải thích cụ thể.

2. Có hai thí nghiệm về phản ứng được tiến hành độc lập trong hai bình có thể tích cố định, bằng nhau; giả thiết ban đầu hai bình đều chân không.

Thí nghiệm 1: Ban đầu người ta cho 4 mol NH4Cl vào bình 1. Khi phản ứng đạt tới cbhh ở nhiệt độ T, áp suất chung của hệ là P1 = 1,15atm, độ phân li của NH4Cl là α1 = 0,60.

Thí nghiệm 2: Ban đầu người ta cho 16 mol NH4Cl vào bình 2. Khi phản ứng đạt tới cbhh ở nhiệt độ T, áp suất chung của hệ là P2 bằng bao nhiêu atm?

3. Kết quả tính độ phân li của NH4Cl trong hai thí nghiệm trên có phù hợp với nguyên lí Lơ Satơlie hay không? Hãy trình bày cụ thể.

(Đề thi chính thức chọn học sinh giỏi Quốc gia Hoá học năm 2006; Bảng B; Câu IV.2)

Trả lời tóm tắt:

Ta xét phản ứng NH4Cl ⇌ NH3 + HCl

Số mol khí ban đầu n 0 0

Đã phân li nα

Số mol khí ở cbhh n − nα nα nα.

Tại cbhh: Tổng số mol các khí là n(1 + α)

NH4Cl là 3(1 )

, NH( )

− α1 + α và HCl đều là (1

α+ α)

* Theo định luật Đantơn, ta có biểu thức tính áp suất riêng phần mỗi khí:

30

Page 25: 1ygm3hwjpgu090312060334 tài liệu bồi dưỡng giáo viên trường thpt chuyên   mon hóa học - phan 1- năm 2012

4 3NH Cl NH HCl(1

p P ; p p P− α) α = = = (1 + α) (1 + α)

(a)

* Theo định luật tác dụng khối lượng, ta có biểu thức:

34

HCLp NH

NH Cl

pK p

p= (b)

31

Page 26: 1ygm3hwjpgu090312060334 tài liệu bồi dưỡng giáo viên trường thpt chuyên   mon hóa học - phan 1- năm 2012

Thay (a) vào (b) và thực hiện các biến đổi thích hợp, ta được:

pK P .(1 )

2

2

α= − α

(c).

* Các khí đều được coi là khí lí tưởng và phản ứng (2) có ∆n = 2 − 1 = 1 nên:

KC = Kp(RT)− ∆n = Kp(RT)−1 và Kx = KpP−∆n = Kp.P−1

Kx phụ thuộc áp suất P chung của hệ, mà P lại phụ thuộc vào số mol khí tức là

phụ thuộc vào độ phân li α. Vậy đối với phản ứng (*) Kx không phải là một lượng

hằng định mặc dù cố định nhiệt độ T.

2. Thay các số liệu thu được từ thí nghiệm 1 vào biểu thức (c), ta có:

2

p 2

0,6K 1,5 0,84375 0,844

(1 0,6 )

= =

− : (d)

Ta có thể tính áp suất P2 theo một số cách khác nhau. Ta dùng cách sau đây.

Kí hiệu độ phân li của NH4Cl trong thí nghiệm 2 là α2; điều kiện 0 < α2 < 1 (**).

Tổng số mol các khí tại cbhh trong bình 2 là n2 = 16(1 + α2).

Tổng số mol các khí tại cbhh trong bình 1 là n1 = 4(1 + α1) = 4(1 + 0,6) =

6,4(mol).

Với điều kiện V = const và T = const, ta có liên hệ:

2 2 22 1

1 1 1

P n n (1 )P P 16 .1,5 3,75(1 )

P n n 6, 42

2+ α = → = = = + α

(e)

Mặt khác, khi thay α2 cho α trong biểu thức (c) và chú ý tới (d), ta có:

2 2P 0,844

(1 )

22

2

α = − α

Kết hợp (e) với (f) và biến đổi thích hợp, ta có:

3,75 0,844 0,844 0.22 2α + α − =

Giải phương trình này và chú ý tới điều kiện 0 < α2 < 1, ta được α2 = 0,38.

Đưa kết quả này vào (e), ta có: P2 = 3,75(1 + 0,38) → P2 = 5,175 ≈ 5,18 (atm)

32

Page 27: 1ygm3hwjpgu090312060334 tài liệu bồi dưỡng giáo viên trường thpt chuyên   mon hóa học - phan 1- năm 2012

3. Đã có: thí nghiệm 1 dùng n1 = 4 mol NH4Cl, độ phân li α1 = 0,60;

thí nghiệm 2 dùng n2 = 16 mol NH4Cl, độ phân li α2 = 0,38.

Kết quả đó hoàn toàn phù hợp với nguyên lí Lơ Satơlie.

Hai thí nghiệm được tiến hành trong điều kiện V = const và T = const nên khi tăng số mol khí, cbhh sẽ chuyển biến dời về phía chống lại ảnh hưởng sự tăng số

mol đó. Phản ứng (*) có ∆n = 2 − 1 = 1 > 0, cbhh chuyển dời sang trái, nghĩa là

giảm sự phân li của NH4Cl, hay α2 = 0,38 < α1 = 0,60 khi n2 = 16 mol > n1 = 4

mol.

4. Câu hỏi bổ sung: Nếu trong thí nghiệm 3, ban đầu người ta cho vào bình 1 4 mol NH4Cl và 4 mol khí trơ, chẳng hạn Ne, số liệu thu được từ thí nghiệm 1 có bị ảnh hưởng không? Tại sao?

II.2.4. Một số lưu ý

A. Phân biệt sự hằng định của trị số bằng cbhh với sự chuyển dịch vị trí cbhh

Có ý kiến cho rằng "Khi cbhh CaCO3 (r) ⇌ CaO(r) + CO2 (k) được thiết lập,

việc thêm CaCO3 (r) vào hệ không làm chuyển dịch cb đó vì Kp = pCO2 = const"

Bạn có đồng ý như vậy không?

Theo tôi, không nên kết luận "cả gói" như vậy.

*) Khi cbhh đó được thiết lập, Kp = pCO2 = const tại T = const và áp suất ngoài

P = const.

*) Nếu thêm CaCO3 (r) vào hệ, theo nguyên lí Lơ Satơlie, vị trí cbhh đó bị chuyển dời do sự phân huỷ CaCO3 (được đưa vào) tăng lên. Có 2 lí do dẫn đến kết quả đó:

− CO2 được tạo ra sẽ thoát khỏi hệ để vẫn duy trì pCO2 = const (bằng cách tạo

sự thông thoáng trên bề mặt, hoặc có thể tăng thể tích hệ).

− Chất rắn CaCO3 có các trạng thái kết tinh khác nhau, lúc đó trong hệ còn có

sự dịch chuyển vị trí cân bằng giữa các dạng tinh thể đó. Cụ thể CaCO3 ban đầu là tinh thể, khối còn CaCO3 mới được tạo thành do CaO(r) + CO2(k) trong phản ứng sẽ ở dạng bột là chủ yếu. CaO(r) cũng có các dạng tinh thể khác nhau.

Vậy không nên kết luận như ban đầu.

33

Page 28: 1ygm3hwjpgu090312060334 tài liệu bồi dưỡng giáo viên trường thpt chuyên   mon hóa học - phan 1- năm 2012

Còn đối với cbhh pha lỏng, pha khí không có sự đồng nhất giữa K = const với sự không chuyển dời vị trí cbhh mặc dù hệ được thêm lượng chất đầu (hoặc giảm lượng chất cuối).

B. Trở lại câu hỏi: "Hằng số cbhh có đơn vị hay không?"

1. Trước hết ta khẳng định một số nội dung.

a) Khái niệm "Thứ nguyên" có ý nghĩa rộng hơn "đơn vị".

Thứ nguyên là để chỉ một loại đối tượng, đơn vị chỉ cụ thể.

Chẳng hạn: Thứ nguyên độ dài;

Đơn vị độ dài có thể là mét hoặc ước số của mét (dm, cm, mm,...) hoặc bội số của mét (km).

Hoặc: Thứ nguyên thời gian;

Đơn vị thời gian có thể là giờ (h), ngày, tháng, năm,...

Như vậy, khi ta nói "đơn vị" là đề cập cái cụ thể; để khái quát, câu hỏi đặt ra ở mục này phải là "Hằng số cbhh có thứ nguyên hay không".

b) Về nguyên tắc, một đại lượng vật lí bao giờ cũng gồm hai phần: trị số (giá trị) và thứ nguyên.

Đại lượng Vật lý gồm: Trị số và thứ nguyên (II.42).

c) Phép lấy logarit (cơ số 10, lg, hay cơ số e, ln) chỉ thực hiện với một trị số, không có thứ nguyên.

lgX hay lnx: X là lượng không thứ nguyên (II.43).

(xin xem M.IA. Vưgotxki: Sổ tay toán học, NXB Tiến bộ, Matxcơva, trang 254, dòng cuối).

2. Hằng số cbhh

a) Theo NĐLHH, biểu thức của hằng số cbhh K được thiết lập dựa vào điều

kiện cb nhiệt động theo tiêu chuẩn hàm Gipxơ mol riêng (phần) iG hay thế hoá

học µi.

Biểu thức của iG hay µi là (II.6a) và (II.6c) hoặc (II.6b) và (II.6d).

Để phù hợp quy ước toán học (II.43), đã đưa vào lượng o

pln( )

p hay o

aln( )

c

để dưới dấu logarit chỉ là trị số. Từ quy ước đó, ta có biểu thức của hằng số cbhh Kp (II.26), Kc là (II.25). Từ đó có kết luận:

34

Page 29: 1ygm3hwjpgu090312060334 tài liệu bồi dưỡng giáo viên trường thpt chuyên   mon hóa học - phan 1- năm 2012

Khi xét lượng logarit của K, lnK hay lgK; K chỉ là một trị số không có thứ nguyên.

(Trong tài liệu 2** có câu bằng tiếng Anh: Numerial values for Kc can come

from experiments, trang 704, dòng 1↓.)

b) Vì K là hằng số cân bằng nhiệt động (là đại lượng nhiệt động hoặc là một đại lượng vật lí), phải tuân theo (II.42)

Mặt khác trong bài toán thực tế, khi phản ứng có ∆n ≠ 0 buộc chúng ta phải

làm bài toán đơn vị cho trị số K.

(xem Bài tập áp dụng II.6, Bài tập áp dụng II.7 phần trên).

Có 2 cách để giải quyết bài toán đơn vị.

Cách thứ nhất: Luôn chú ý liên hệ K với đơn vị:

Kp. (po) ∆n hoặc KC. (Co) ∆n (II.44)

Với quy ước thông thường theo hệ số SI

[po] = [1 bar] (hay gần đúng là 1 atm); [Co] = [1 mol.L−1] = [1M] (II.45)

Cách thứ hai: Căn cứ vào từng bài tập cụ thể để ghi thêm đơn vị thích hợp cho trị số của K.

Cách làm thứ nhất "chính quy, hiện đại" hơn.

Dù thực hiện cách nào ta cũng luôn nhớ

Khi phản ứng có ∆n ≠ 0, trị số của hằng số cbhh K cần kèm theo thứ

nguyên thích hợp.

C. Liên hệ giữa cbhh với động hóa học

1. Lập biểu thức hằng số cbhh:

Trên đây ta xét theo cơ sở chính thống của NĐLHH: ∆Gpư = 0 và dung thế hóa học µi.

Một cách rất gần đúng (hoặc gần đúng rất thô), ta dựa vào định nghĩa “… tốc độ phản ứng thuận bằng tốc độ phản ứng nghịch”, vt = vn.

Đề nghị bạn đọc so sánh hai cách này và cho lời khuyến cáo cần thiết.

2. Yếu tố nhiệt động, yếu tố động học

Ta xét phản ứng quen thuộc: 3H2 (k) + N2 (k) ⇌ 2NH3; ∆H < 0.

35

Page 30: 1ygm3hwjpgu090312060334 tài liệu bồi dưỡng giáo viên trường thpt chuyên   mon hóa học - phan 1- năm 2012

Về nhiệt động: Để cbhh chuyển dời sang phải, tạo ra nhiều NH3, cần giảm nhiệt độ xuống càng thấp càng tốt.

Về động học: Phản ứng chỉ có thể xảy ra khi tốc độ chuyển động của các phân tử (chất tham gia) đủ lớn. Về nguyên tắc: Nhiệt độ cao thuận lợi.

Với phản ứng này, hai yếu tố nhiệt động, động học bị nhiệt độ chi phối ngược chiều nhau. Do đó phải chọn một nhiệt độ trung gian để thoả mãn đồng thời hai yếu tố, vào khoảng 500oC.

Cũng có phản ứng hai yếu tố đó có biến đổi cùng chiều dưới tác dụng của nhiệt độ. Đó là các phản ứng thu nhiệt.

3. Kiểm soát nhiệt động và kiểm soát động học một phản ứng hh thường được đề cập khi khảo sát động học phản ứng. Khái niệm này xuất hiện khi xét phản ứng phức tạp, vừa song song, vừa thuận nghịch. Sơ lược vấn đề này như sau.

Có hai phản ứng song song (còn gọi là phản ứng cạnh tranh) đều bậc nhất:

A 1k→ C và A

2k→ D (a)

Từ việc thiết lập biểu thức định luật tốc độ cho từng phản ứng và cả hệ (dạng vi phân, dạng tích phân), cuối cùng thu được liên hệ

[C] / [D] = k1 / k2 (b)

Ở đây dùng kí hiệu [ ] để thuận lợi cho việc liên hệ với cbhh ở đoạn sau.

Thực tế, dù mức độ (tốc độ) thấp, phải chú ý phản ứng nghịch:

C 1k−→ A và D

2k−→ A (c)

Tất nhiên, cũng phải xét đến liên hệ: C ⇌ D (d)

Nếu ta xét với thời gian đủ lâu, tại t = ∞, để toàn hệ đạt tới cbhh giả định (lí

tưởng). Ta có K1 = [C] / [A]; K2 = [D] / [A] → [C] / [D] = K1 / K2 (e)

Tình huống này được nói là có sự kiểm soát nhiệt động đối với sản phẩm, lúc đó sản phẩm có biến thiên thế đẳng áp âm nhất, ∆G < 0.

Nếu bỏ qua (c), (d) mà dùng biểu thức (b), tình huống đó được nói là có sự kiểm soát động học đối với sản phẩm.

Nếu các hằng số tốc độ k−1 và k−2 lớn hơn rất nhiều so với các hằng số tốc độ

k1, k2; tình huống này được nói là có sự kiểm soát động học đối với các sản phẩm ngay cả khi lượng A bị tiêu tốn gần hết.

36

Page 31: 1ygm3hwjpgu090312060334 tài liệu bồi dưỡng giáo viên trường thpt chuyên   mon hóa học - phan 1- năm 2012

Nếu xảy ra k1 / k2>> 1 còn K1 / K2 << 1, lúc đó có sự kiểm soát động học đối với C, có sự kiểm soát nhiệt động đối với D. Hiệu suất tương đối của sản phẩm phụ thuộc vào việc có kiểm soát động học hay nhiệt động.

(Mời xem Problem 22: Kinetics and Thermodynamics, 37th International Chemistry Olympiad, 2005, Tapei, Taiwan Preparatory problems and Worked Solutions (Bài tập chuẩn bị và lời giải, Olympic Hoá học quốc tế lần thứ 37 tại Tapei, Taiwan, 2005; mục D trang 217 sách: Trần Thành Huế, Tư liệu Hóa học 10, NXB Giáo dục Hà Nội 2006, 2008).

IV. Một số bài tập tham khảo

Dưới đây là một số bài tập để quý bạn đọc tham khảo. Tất cả các bài đều có gợi ý trả lời. Một số bài xét thấy cần sẽ có thêm gợi ý trao đổi bằng câu hỏi. Hi vọng các bài tập này, cũng như cả tài liệu này, bổ ích cho công việc khó khăn các bạn đang đảm nhận.

Bài tập IV.1

Có phản ứng thuận nghịch pha khí: a1A1 + a2A2 ⇌ b1B1 + b2B2 (1).

1. Hãy thiết lập biểu thức Kp cho (1) dựa vào: a) Thế hóa học µi;

b) Hoặc định nghĩa: cân bằng hóa học là trạng thái có vt = vn.

Chỉ rõ điểm mạnh, điểm yếu của mỗi căn cứ thiết lập biểu thức Kp trên.

2. Hãy thiết lập biểu thức liên hệ Kp với Kc; Kp với Kx. Giả thiết các khí đều là khí lí tưởng.

3. Các hằng số cân bằng hóa học (viết tắt là hs cbhh) đó có thứ nguyên không?

Tại sao? Xét với ví dụ phản ứng pha khí: SO2 + Cl2 ⇌ SO2Cl2 (2).

4. Trị số của hs cbhh K:

a) Trong khoảng giá trị nào? Tại sao?

b) Trong khoảng giá trị đó, đoạn/ phần nào ứng với sự chiếm ưu thế của phản ứng thuận hay phản ứng nghịch? Tại sao?

c) Nếu K =1 ta có kết luận nào?

d) Hãy trình bày (dạng đề cương) ý kiến của anh/chị sử dụng các nội dung từ a) đến c) trên đây vào giảng dạy bậc phổ thông (ban nâng cao hay chuyên).

37

Page 32: 1ygm3hwjpgu090312060334 tài liệu bồi dưỡng giáo viên trường thpt chuyên   mon hóa học - phan 1- năm 2012

Bài tập IV.2

Phản ứng đồng thể pha khí E + F ⇌ G + H (2) có hằng số cân bằng hóa

học K = 30 tại nhiệt độ T, áp suất chung P, đều hằng định.

1. Ban đầu người ta trộn 0,5 mol E với 0,3 mol F. Hãy tính lượng mỗi chất (theo các cách biểu thị khác nhau có thể) tại cbhh trong mỗi trường hợp sau đây:

a) Cho thể tích bình phản ứng 5,0 lít và hằng định.

b) Không biết thể tích bình phản ứng.

2. Nhận xét và giải thích kết quả tính trên.

3. Nếu đề bài không cho biết "pha khí", trong nội dung trả lời câu hỏi 1. có chi tiết nào cần được lưu ý?

Bài tập IV.3

Phản ứng Cgr(r) + CO2 ⇌ 2CO ở nhiệt độ T = 1000K có Kp = 1,862.

1. Xác định áp suất riêng phần của các khí CO và CO2 khi có cân bằng của phản ứng dưới áp suất chung 1 atm.

2. Ở áp suất chung bằng bao nhiêu thì hỗn hợp chỉ chứa 1% CO2?

3. Ở nhiệt độ và áp suất không đổi 1 atm như trên nếu trộn CO và CO2 theo tỉ lệ mol 4:1 khi grafit dư thì phản ứng sẽ xảy ra theo chiều nào?

Bài tập IV.4

I) Phản ứng 2 SO2 (k)+ O2 (k) ⇌ 2 SO3 (k) (1)

có vai trò quan trọng trong HHPT.

1. Hãy viết biểu thức Kp, KC, Kx và lập liên hệ giữa Kp với KC; Kp với Kx. Theo hệ SI, mỗi đại lượng đó có đơn vị nào? Tại sao? Kx có phải là hằng số cân bằng hóa học (cbhh) không? Tại sao?

2. Sau khi bổ sung các chi tiết cần thiết còn thiếu vào (1), anh/chị hãy chỉ rõ yếu tố nhiệt động, động học có vai trò như thế nào đối với việc chọn điều kiện thích hợp cho (1).

II) Phản ứng CaCO3(r) ⇌ CaO(r) + CO2 (k) (2)

cũng có vai trò quan trọng trong HHPT.

38

Page 33: 1ygm3hwjpgu090312060334 tài liệu bồi dưỡng giáo viên trường thpt chuyên   mon hóa học - phan 1- năm 2012

1. Hãy viết biểu thức Kp, KC, Kx (nếu có) và lập liên hệ giữa Kp với KC; Kp

với Kx. Theo hệ SI, mỗi đại lượng đó có đơn vị nào? Tại sao? Kx có phải là hằng số cân bằng hóa học (cbhh) không? Tại sao?

2. Sau khi bổ sung các chi tiết cần thiết còn thiếu vào (2), anh/chị hãy chỉ rõ yếu tố nhiệt động, động học có vai trò như thế nào đối với việc chọn điều kiện thích hợp cho (2).

3. Hãy trình bày ý kiến của anh/chị sử dụng nội dung trên trong bài dạy HHPT (gạch đầu dòng).

Bài tập IV.5

Đối với phản ứng 2KClO3 (rắn) ⇌ 2KCl (rắn) + 3O2(khí) (*);

Thực nghiệm cho biết liên hệ giữa nhiệt độ T (theo Kenvin) với hằng số cân bằng hóa học K như sau đây: T (Kenvin) 773 973

K 27 45.

1. Hằng số cân bằng hóa học K là KC, hay Kp, hay Kx? Tại sao?

2. Chưa tính, hãy chỉ rõ ảnh hưởng của nhiệt độ T đến cân bằng hóa học (*).

3. Tính nhiệt phản ứng, ∆H0pu , của (*).

4. Tính biến thiên năng lượng tự do Gipxơ tiêu chuẩn, ∆G0, của (*).

5. Nhận xét sự biến đổi theo nhiệt độ T của giá trị ∆H0pu

, ∆G0 phản ứng (*).

Cho R= 8,314 J/mol.K; các đại lượng trong bài đều có đơn vị theo hệ quốc tế (SI).

GỢI Ý TRẢ LỜI

Bài tập IV.1

1. a) Dựa vào thế hóa học µi: Viết b/t µi; Viết b/t ∆Gpư; Liên hệ 2 b/t đó và

xét đk cbhh: ∆Gpư=0, từ đó có ∆G0 = – RTlnKp với Kp = 1 2

1 2

1 2

21

b bB B

a aA A

p p

p p (1a);

Khái quát: Kp = 1

1

j

j

i

i

nb

Bj

maAi

p

p

=

=

Π

Π(1b); (i là chất đầu; j là chất cuối).

39

Page 34: 1ygm3hwjpgu090312060334 tài liệu bồi dưỡng giáo viên trường thpt chuyên   mon hóa học - phan 1- năm 2012

b) Dựa vào đ/n cb hh: vt = vn; Từ đó Kp =t

n

k

k =

1 2

1 2

1 2

1 2

B B

A A

p p

p p

β β

α α (2) trong đó các lũy

thừa là bậc pư, nói chung khác hệ số trong (1).

NX: Cách dựa vào thế hóa học µi chặt chẽ hơn; b) chỉ là gần đúng (rất thô),

đặc biệt về bậc pư, nói chung, bậc pư không bằng/không phải là hệ số trong (1) mà do thực nghiệm cung cấp.

2. Lập b/t liên hệ:

Kp với KC: Khí lí tưởng có piV=niRT; từ đó có Kp = KC(RT) n∆ (3).

Kp với Kx: Khí lí tưởng có pi=xiP; từ đó có Kp = Kx(P) n∆ (4).

3. K có đơn vị thích hợp là Kp.(bar) n∆ (5); KC.(M) n∆ (6) (g/t);

Chỉ khi ∆n =0 mới không có đ/v.

4. Trị số của K: (Ghi chú: Xem câu trả lời ở Bài tập áp dụng II.3 trên);

Nếu K=1, là một trường hợp đặc biệt của cbhh khi nồng độ (hoặc áp suất riêng) tất cả các chất bằng nhau và bằng 1; hoặc do sự liên hệ ngẫu nhiên giữa lũy thừa với n/đ (hoặc áp suất riêng); hoặc do sự liên hệ ngẫu nhiên mà có ∆Go = 0.

(Chú ý: Nếu nói "khi đó pư thuận nghịch mới đạt tới cbhh", đúng hay sai?)

Giảng dạy: Cần chỉ rõ cho h/s biết nội dung trên để áp dụng đúng; Có thể cho VD để học sinh phân tích.

Bài tập IV.2

NX: Phản ứng có ∆n = 0 nên Kp = KC = Kx (1*); Giải pt dạng ax2 + bx +c =0, cần đặt điều kiện cho ẩn x phù hợp từng trường hợp (2*).

1. Đáp số:

a) Vbình =5,0 lít:

Theo CM chất E là 0,043; chất F là 0,003; chất H và G đều là 0,057 (a1);

Theo số mol: chất E là 0,21; chất F là 0,01; chất H và G đều là 0,29 (a2);

Theo tỉ số mol: chất E là 0,263; chất F là 0,001; chất H và G đều là 0,363 (a3)

b) Không có số liệu Vbình: Chỉ tính được đáp số (a2) và (a3).

2. Mặc dù cùng trị số của K nhưng khác cách biểu thị thành phần nên

(a1) ≠ (a1) ≠ (a3).

(Chú ý: Có tính được áp suất riêng pi của mỗi khí hay không? Tại sao?)

40

Page 35: 1ygm3hwjpgu090312060334 tài liệu bồi dưỡng giáo viên trường thpt chuyên   mon hóa học - phan 1- năm 2012

3. Nếu đề bài không cho biết "pha khí", trong nội dung trả lời câu hỏi 1. chỉ rõ: kết quả tính ở trên không cần phân biệt "pha".

Bài tập IV.3

1. KP = P2CO/PCO2 ; Vậy: 1,862 = P2

CO/(1 – PCO) và giải ra được PCO = 0,721 atm;

PCO2 = 1,000 – 0,721 = 0,279 atm.

2. Hỗn hợp chứa 5% CO2 thì CO là 95%. Ta có PCO2 = 0,05P và PCO = 0,95P

(P: áp suất chung). Vậy: 1,862 = (0,95P)2/0,05P; suy ra P = 0,098 atm.

3. ∆G = ∆Go + RTln (Q) (Q = P2CO/PCO2 chưa cân bằng); PCO = 1 x 4/5 = 0,8atm;

PCO2 = 1 x 1/5 = 0,2 atm.

∆G = – 8,314.1000.ln(1,862) + 8,314.1000.ln(0,82/0,2)

= + 8,314. 1000.(ln(0,82/0,2/1,862))= 4502 J/mol > 0

Vậy phản ứng xảy theo chiều nghịch.

Bài tập IV.4

I)

1. Viết b/t của từng đại lượng Kp, KC, Kx; để lập liên hệ giữa Kp với KC; Kp với Kx cần giả định các khí đều là lí tưởng; Chỉ ra đơn vị của Kp, KC; Kx không có đơn vị và nó không phải là hằng số cbhh vì có ∆n khác 0.

2. Thêm: to C; ∆H < 0; xt, phân tích sự ngược nhau của yếu tố nhiệt động ∆H <0, giảm to C, động học cần tăng to C → chọn to C thích hợp

II)

1. Với pư này, chỉ có đại lượng Kp = pCO2; không có KC, Kx;

(chú ý: ∆n =1> 0).

2. Thêm: to C; ∆H> 0. Phân tích sự đồng biến của yếu tố nhiệt (∆H >0), với nhiệt độ to C; cũng phù hợp với yêu cầu động học; Chú ý: cần tăng to C, nhưng phải chọn to C thích hợp để kích thước hạt phù hợp cho sự thoát khí (tại sao?)

3. Dạy HHPT: *) Nên dạy cho tất cả HS; *) H /s giỏi: Phân biệt sự hằng định của hs cbhh khác với sự cd trạng thái pư theo nguyên lí Lơ Satơlie.

Bài tập IV.5

1. Chỉ có thể là Kp vì chỉ có duy nhất 1 chất khí, không thể tính nồng độ 1 chất nguyên chất.

41

Page 36: 1ygm3hwjpgu090312060334 tài liệu bồi dưỡng giáo viên trường thpt chuyên   mon hóa học - phan 1- năm 2012

2. Cân bằng hóa học (*) chuyển dời → khi T tăng (phản ứng thu nhiệt, 0puH∆ > 0).

3. Từ pt Kiêchôp, có0puH∆ = [(RT1T2)ln(KT2/ KT1)]/(T2 – T1) = 15,974 kJ/mol;

4. Từ pt 0puG∆ = – RTlnK; Thay lần lượt trị số T, tính được tại T=773 K

có 0puG∆ = – 21,181 kJ/mol; T= 973K có

0puG∆ = – 30,794 kJ/mol.

5. Coi 0puH∆ không phụ thuộc T; còn mỗi trị số T, có tương ứng một trị số

0puG∆ .

IV. Một số tài liệu tham khảo chính

[1] Nguyễn Đình Huề . Giáo trình Hóa lí I, II. NXB Giáo dục, 1982; 2001.

[2] Trần Thành Huế. Tư liệu Hóa học 10, NXB Giáo dục 2006, 2008.

[3] P.W. Atkins. Physical Chemistry, Sixth edition, Oxford Univ. Press,

1998.

[4] Ira N. Levine. Physical Chemistry, Third edition, McGraw – Hill Book

Comp, 1988.

[5] A.I. Geraximov. Nhiệt động lực Hóa học, NXB Hóa học, Maxcơva,

1975 (tiếng Nga)

[6] Hans Kuhn and Horst –Dieter Försterling. Principles of Physical Chemistr, John Wiley & Sons Ltd, 2000.

[7] Robert A. Alberty, Robert J. Silbey. Physical Chemistry, John Wiley & Sons Ltd, 1992.

[8] K.W. Whitten, R. E. Davis, M. L. Peck, G. G. Staley. General Chemistry, Thomson Brooks/Cole, 2004.

42

Page 37: 1ygm3hwjpgu090312060334 tài liệu bồi dưỡng giáo viên trường thpt chuyên   mon hóa học - phan 1- năm 2012

Ph n 2ÇHãA HäC PH¢N TÝCH

CHUẨN ĐỘ AXIT – BAZƠ

PGS.TS Đào Thị Phương Diệp

Khoa Hóa học – Trường Đại học Sư phạm Hà Nội

Trong các đề thi vòng 1 (thi học sinh giỏi Quốc gia – HSGQG) và vòng 2 (thi chọn đội tuyển Quốc gia dự thi Olympic Hóa học Quốc tế) Hóa học phân tích chiếm một vị trí khá quan trọng, tuy nhiên nội dung thi thường tập trung về tính toán cân bằng ion trong dung dịch, mà chưa đề cập nhiều đến phân tích thể tích – là một trong những nội dung có trong chương trình thi học sinh giỏi Quốc gia và Quốc tế. Chưa kể đến các đề thi vòng loại của nhiều quốc gia, hay trong các bài tập chuẩn bị hàng năm của các nước, trong nhiều đề thi Olympic Hóa học Quốc tế (International Chemistry Olympiad (ICho)) ngay từ những năm đầu tiên, nội dung chuẩn độ thể tích nói chung và chuẩn độ axit –bazơ nói riêng đã được đưa vào nội dung thi chính thức. Ví dụ trong đề thi lý thuyết lần thứ 3 tại Hungary, năm 1970 (3rd ICho Budapest 1970, Hungary) đã có câu: Cân 1,287 g một mẫu Na2CO3.xH2O (mẫu A) và cho phản ứng với dung dịch HCl dư, thu được 100,8 cm3 một chất khí (đo ở điều kiện tiêu chuẩn). Mặt khác hòa tan 0,715 g một mẫu muối Na2CO3.yH2O khác (mẫu B) trong 50 cm3 dung dịch H2SO4 0,2 N rồi chuẩn độ lượng H2SO4 dư bằng dung dịch NaOH 0,1 M với chất chỉ thị metyl da cam, hết 50 cm3 NaOH. Tính giá trị x và cho biết A và B có cùng thành phần không? Ở đây phép chuẩn độ được đề cập đến chính là phép chuẩn độ hỗn hợp axit mạnh và đơn axit yếu. Hoặc trong đề thi lý thuyết lần thứ 7 cũng tại Hungary, năm 1975 (7th ICho Veszprém 1975, Hungary) đã đề cập tới phép chuẩn độ đa bazơ. Hoặc trong các đề thi thực hành lần thứ 9 tại Czechoslovakia năm 1977 (9th ICho Bratislava 1977,

43

Page 38: 1ygm3hwjpgu090312060334 tài liệu bồi dưỡng giáo viên trường thpt chuyên   mon hóa học - phan 1- năm 2012

Czechoslovakia) và lần thứ 10 tại Balan (10th ICho Torun 1978, Poland) đã có các bài thi về chuẩn độ đơn axit, đơn bazơ, v.v…

Hiện nay ở Việt Nam nội dung chuẩn độ dung dịch bao gồm chuẩn độ axit –bazơ và chuẩn độ oxi hóa–khử bằng phương pháp pemanganat đã được đưa vào trong chương trình sách giáo khoa (SGK) 12 nâng cao, tuy nhiên thời lượng dành cho nôi dung nay quá ít (2 tiết lý thuyết và 1 tiết thực hành) và nội dung kiến thức cũng hết sức đơn giản (một số khái niệm, một số dụng cụ đo thể tích; nguyên tắc chung của phép chuẩn độ axit –bazơ, chuẩn độ oxi hóa –khử; chuẩn độ HCl bằng NaOH; chuẩn độ chất khử bằng KMnO4), chủ yếu sư dung đinh luât hơp thưc tính nồng độ hoặc thể tích của dung dịch chất phản ứng, chưa hương dẫn cho học sinh biêt cách xác định thể tích tương đương, cach chon chất chỉ thị thich hơp cho phép chuẩn độ va cach phác họa đường cong chuẩn độ,... ma những vấn đề này rất hay gặp trong đề thi vòng 2, trong các đề thi vòng loại của các nước trên thế giới và trong các đê thi Olympic Quốc tế.

Điều này khó đảm bảo để giáo viên (GV) và học sinh (HS) các trường chuyên có thể giải quyết trọn vẹn được các bài toán về định lượng hóa học được ra dưới các dạng khác nhau trong các đề thi học sinh giỏi quốc gia, quốc tế.

Để rút ngắn khoảng cách giữa nội dung kiến thức được học ở các trường chuyên và nội dung thi Olympic quốc gia, quốc tế, ở đây chúng tôi sẽ trình bày các vấn đề về chuẩn độ axit –bazơ, nhằm mục đích trang bị cho giáo viên và học sinh những kiến thức nâng cao ngang tầm quốc tế, nhưng vẫn đảm bảo mức độ hợp lý, phù hợp với trình độ học sinh phổ thông.

A – ĐỊNH NGHĨA VÀ CÁC KHÁI NIỆM CƠ BẢN [3], [5]

– Phương pháp chuẩn độ axit – bazơ là phương pháp chuẩn độ thể tích dựa trên sự tương tác giữa các axit, bazơ, tức là dựa trên sự đo thể tích dung dịch thuốc thử là axit (hoặc bazơ) có nồng độ đã biết phản ứng với một thể tích xác định dung dịch bazơ (hoặc axit) có nồng độ chưa biết cần xác định. Hay nói cách khác phản ứng chuẩn độ axit –bazơ chính là phản ứng trung hòa.

– Dung dịch cần chuẩn (dung dịch phân tích) là dung dịch cần xác định nồng độ.

– Dung dịch đã biết nồng độ chính xác và được dùng để xác định nồng độ các dung dịch khác gọi là dung dịch chuẩn (dung dịch thuốc thử).

44

Page 39: 1ygm3hwjpgu090312060334 tài liệu bồi dưỡng giáo viên trường thpt chuyên   mon hóa học - phan 1- năm 2012

– Quá trình chuẩn độ là quá trình thêm dần dung dịch chuẩn vào dung dịch cần chuẩn.

– Điểm tương đương (ĐTĐ) là thời điểm tại đó lượng chất chuẩn cho vào phản ứng vừa hết với lượng chất cần chuẩn.

– Trong quá trình chuẩn độ cần thiết phải dựa vào một tín hiệu nào đó để dừng phép chuẩn độ. Chất có khả năng thay đổi tín hiệu khi chuẩn độ gọi là chất chỉ thị.

– Thời điểm tại đó chất chỉ thị thay đổi tín hiệu gọi là điểm kết thúc chuẩn độ (điểm dừng chuẩn độ hay điểm cuối chuẩn độ (ĐCCĐ)).

– Khoảng pH tại đó chất chỉ thị đổi màu gọi là khoảng chuyển màu của chất chỉ thị.

– Chỉ số chuẩn độ pT của chất chỉ thị là giá trị pH (thuộc khoảng pH chuyển màu) tại đó màu chi thi thay đổi rõ nhất. Chỉ số chuẩn độ phụ thuộc vào chất chỉ thị và thứ tự chuẩn độ, phụ thuộc vao nhiệt độ, dung môi, lực ion và môi trường của dung dịch.

– Đường chuẩn độ trong phép chuẩn độ axit –bazơ là đường biểu diễn sự phụ thuộc của pH theo lượng thuốc thử cho vào.

– Sự biến thiên đột ngột pH tương ứng với sự biến thiên một lượng không đáng kể chất chuẩn (trong phạm vi sai số cho phép) ở gần điểm tương đương tạo thành bước nhảy chuẩn độ (BNCĐ) trên đường cong chuẩn độ.

– Nguyên tắc phải chọn chất chỉ thị có chỉ số chuẩn độ pT gần với pH tại điểm tương đương của phép chuẩn độ (pT ≈ pHTĐ). Hoặc có thể chọn bất kì chất chỉ thị nào có chỉ số chuẩn độ nằm trong bước nhảy chuẩn độ tương ứng với sai số cho phép.

– Sự sai lệch giữa điểm cuối chuẩn độ và điểm tương đương gây nên sai số chuẩn độ. Sai số chuẩn độ chính là tỉ số % lượng thuốc thử đã cho dư hoặc còn thiếu so với lượng cần thiết để chuẩn độ đến điểm tương đương.

B – CÁC PHÉP CHUẨN ĐỘ AXIT –BAZƠ

1/ Chuẩn độ axit mạnh, bazơ mạnh.

2/ Chuẩn độ axit yếu, bazơ yếu.

3/ Chuẩn độ hỗn hợp axit mạnh và đơn axit yếu; hỗn hợp bazơ mạnh và đơn bazơ yếu.

4/ Chuẩn độ hỗn hợp 2 đơn axit yếu; hỗn hợp 2 đơn bazơ yếu.

45

Page 40: 1ygm3hwjpgu090312060334 tài liệu bồi dưỡng giáo viên trường thpt chuyên   mon hóa học - phan 1- năm 2012

5/ Chuẩn độ đa axit, đa bazơ.

6/ Chuẩn độ hỗn hợp axit mạnh và đa axit; hỗn hợp bazơ mạnh và đa bazơ.

C – CÁC DẠNG BÀI TẬP VỀ CHUẨN ĐỘ AXIT –BAZƠ

1/ Tính pH của dung dịch tại các thời điểm dừng chuẩn độ.

2/ Tính nồng độ, thể tích,… của các chất tham gia phản ứng.

3/ Chọn chỉ thị thích hợp cho phép chuẩn độ.

4/ Xác định bước nhảy chuẩn độ.

5/ Tính sai số chuẩn độ

6/ Vẽ dạng đường cong chuẩn độ.

7/ Chọn hỗn hợp chuẩn độ.

CÁC VÍ DỤ ÁP DỤNG

I. Dạng bài toán tính pH tại các thời điểm chuẩn độ

Ví dụ 1[4]

Chuẩn độ 10,00 ml dung dịch axit fomic HCOOH bằng dung dịch NaOH 0,100 M, hết 12,50 ml NaOH. Tính pH của dung dịch HCOOH trước khi chuẩn độ và sau khi thêm NaOH vơi thê tich: 1/ 10,00 ml; 2/ 12,45 ml; 3/ 12,50 ml và 4/ 13,00 ml

Phân tích:

Trước hết, từ phương trình phản ứng chuẩn độ, HS cần phải tính được nồng độ gốc Co cua HCOOH. Ở đây cần lưu ý rằng: bài ra không đề cập đến chất chỉ thị phải dùng trong phép chuẩn độ, nên chúng ta phải ngầm hiểu rằng phép chuẩn độ dừng đúng điểm tương đương, nghĩa là thể tích thuốc thử tiêu thụ chính là thể tích tương đương (VTĐ).

HCOOH + OH – → HCOO – + H2O

Co = 0,100.12,50

10,00o

CV

V= = 0,125 (M).

Để tính được các giá trị pH ứng với từng lượng thể tích thuốc thử cho vào, nếu người học đã được trang bị về kiến thức Hóa học Phân tích, khi đó có thể sử

46

Page 41: 1ygm3hwjpgu090312060334 tài liệu bồi dưỡng giáo viên trường thpt chuyên   mon hóa học - phan 1- năm 2012

dụng phương trình đường chuẩn độ: P = )]([

)]([

0

0

00

h

KhCC

h

Kh

hK

KCC

VC

CV

w

w

a

a

−+

−−+

= , hoặc

phương trình sai số:

q = 1−ooVC

CV = – (h – Kw/h)

o

o a

C C h

CC K h

+ −+

để tính h = [H+], từ đó tính được pH tương ứng. Tuy nhiên với cách giải này không phù hợp với học sinh phổ thông (HSPT), vì các em chưa được trang bị đầy đủ kiến thức trên và nếu chỉ áp dụng thuần túy các công thức, các phương trình, các em sẽ không hình dung được các quá trình xảy ra trong dung dịch.

Chính vì vậy GV cần hướng dẫn HS so sánh thể tích thuốc thử đã dùng trong từng trường hợp với VTĐ, để các em sẽ xác định được điểm dừng chuẩn độ, từ đó xác định được thành phần của hệ và dễ dàng tính được pH của hệ.

Ví dụ: tại các thời điểm kết thúc chuẩn độ trước ĐTĐ (Vc < VTĐ), thành phần thu được gồm axit dư và bazơ liên hợp tạo thành. Bài toán đưa về dạng bài tính pH của dung dịch đệm. Ngược lại nếu Vc > VTĐ, hệ thu được gồm bazơ mạnh và đơn bazơ yếu. Nếu dừng chuẩn độ đúng ĐTĐ, bài toán đưa về dạng tính pH của dung dịch đơn bazơ.

Cụ thể với trường hợp thể tích NaOH thêm vào là 10,00 ml, ta có: V1 = 10,00 ml < VTĐ = 12,50 ml, dư axit:

HCOOH + OH – → HCOO – + H2O

(mmol) 0,125.10,00 0,100.10,00

(mmol) 0,250 – 1,00

Thành phần giới hạn (TPGH) của hệ thu được: HCOOH 0,125 M và HCOO –

0,500 M

pH = 3,75 + log(0,500/0,125) = 4,35 (điều kiện áp dụng thỏa mãn)

Tương tự, các em có thể tính pH cho các trường hợp còn lại.

Ví dụ 2.

47

Page 42: 1ygm3hwjpgu090312060334 tài liệu bồi dưỡng giáo viên trường thpt chuyên   mon hóa học - phan 1- năm 2012

Chuẩn độ 50 ml dung dịch KCN 0,100 M và HCOONa 0,090 M bằng dung dịch HCl 0,100 M. Tính pH tại điểm tương đương. Cho biết pKa (HCN) = 9,35; pKa(HCOOH) = 3,75.

Phân tích

Đây là phép chuẩn độ hỗn hợp đơn bazơ yếu bằng axit mạnh vì thế GV hướng

dẫn HS xét tỉ số hằng số phân li của axit liên hợp 4

35,9

75,3

)(

)( 1010

10 >= −

HCNa

HCOOHa

K

K → có

thể chuẩn độ riêng từng bazơ. Đầu tiên KCN bị chuẩn độ trước. Đến khi gần như hết KCN thì HCOONa mới bị chuẩn độ và như vậy phép chuẩn độ này có hai ĐTĐ và cần tính pH tại hai thời điểm này.

Tại ĐTĐ1 thành phần gồm có HCN và HCOO –. Bài toán đưa về dạng tính cân bằng của dung dịch gồm một axit yếu và một bazơ yếu. Sử dụng ĐKP các em sẽ tính được [H+]TĐ1.

Trong trường hợp này, tuy vào trinh đô cua HS ma GV yêu câu HS tinh gân đung (châp nhân [HCN] = CHCN, [HCOO –] = CHCOO

–) hoăc phai tính chinh xac băng cach tinh lăp. Nhưng nêu cân tinh lăp, GV phai lưa chon bai toan chi cân lăp môt lân, không đi qua sâu vê tinh toan.

– Tại ĐTĐ2: lúc này cả hai bazơ đã bị chuẩn độ hết, thành phần gồm có hai axit HCN và HCOOH. Việc tính pH tại ĐTĐ2 chính là tính pH của dung dịch gồm hai đơn axit yếu có hằng số cân bằng chênh lệch nhau.

Ví dụ 3 (Bài tập chuẩn bị Olympic Quốc tế lần thứ 31 – Thái Lan 1999)[1]

Axit photphoric, H3PO4, là một axit ba chức. Chuẩn độ dung dịch H3PO4

0,100 M bằng NaOH. Hãy đánh giá pH tại các thời điểm sau:

1/ Giữa điểm bắt đầu và điểm tương đương thứ nhất.

2/ Tại điểm tương đương thứ hai.

3/ Tại sao rất khó xác định đường cong chuẩn độ sau điểm tương đương thứ hai?

Cho Ka1 = 7,1.10 –3; Ka2 = 6,2.10 –8; Ka3 = 4,4.10 –13.

Phân tích:

48

Page 43: 1ygm3hwjpgu090312060334 tài liệu bồi dưỡng giáo viên trường thpt chuyên   mon hóa học - phan 1- năm 2012

Đây là phép chuẩn độ đa axit bằng bazơ mạnh. H3PO4 là axit có tỉ số các hằng

số phân li từng nấc 4

3

24

2

1 10,10 >>a

a

a

a

K

K

K

K do đó có thể chuẩn độ riêng từng nấc.

Phương trình phản ứng chuẩn độ:

Nâc 1: H3PO4 + OH – → H2PO4 – + H2O

Nâc 2: H2PO4 – + OH – → HPO4

2 – + H2O

Nấc 3: HPO42 – + OH – → PO4

3 – + H2O

Để tính pH của dung dịch tại thời điểm giữa điểm bắt đầu và điểm tương đương thứ nhất, nghĩa là mới chuẩn độ được 50% nấc 1 của axit H3PO4. Dung dịch thu được là một hỗn hợp đệm gôm H3PO4 dư va H2PO4

– tao thanh, có nồng độ bằng nhau, do đó pH1 = pKa1.

pH tại điểm tương đương thứ hai chính là pH của dung dịch muối axit HPO42 –,

nên pH2 = 77,92

21 =+ aa pKpK

Vì hăng sô phân li nâc 3 rât nho (Ka3 = 4,4.10 –13), do đo không co kha năng chuân đô đươc nâc 3, trên đương cong không co BNCĐ, nên không thê chuân đô chinh xac đươc.

II. Dạng bài toán tính nồng độ, thể tích,… của các chất tham gia phản ứng

Ví dụ 4

Chuẩn độ 100,00 ml dung dịch NaOH đến mất màu chỉ thị phenolphtalein thì tiêu thụ hết 48,00 ml dung dịch HCl 5,00.10 –3M. Tính chính xác nồng độ dung dịch NaOH.

Phân tích:

Đây là phép chuẩn độ bazơ mạnh bằng axit mạnh, nên pHTĐ = 7,00

So với ví dụ 1 thì bài toán này phức tạp hơn ở chỗ: VHCl = 48,00 ml không phải là thể tích tương đương (VTĐ). Do dừng chuẩn độ tại thời điểm mất màu phenolphtalein, tức là phép chuẩn độ kết thúc ơ pH = pT = 8,00 > pHTĐ = 7,00 (dừng trước điểm TĐ), lượng axit cho vào chưa trung hòa hết 100,00 ml NaOH. Nếu có kiến thức sâu về chuẩn độ axit – bazơ thì chúng ta có thể tính chính xác CNaOH một cách dễ dàng thông qua sai số của phép chuẩn độ hoặc theo phương trình ĐKP hay phương trình bảo toàn điện tích:

49

Page 44: 1ygm3hwjpgu090312060334 tài liệu bồi dưỡng giáo viên trường thpt chuyên   mon hóa học - phan 1- năm 2012

Ví dụ nếu dùng phương pháp hiệu chỉnh bằng sai số chuẩn độ thì trước hết chúng ta phải tính được nồng độ gần đúng của NaOH (Cgđ,NaOH) theo qui tắc đương lượng.

Sau đó áp dụng phương trình sai số của phép chuẩn độ bazơ mạnh bằng axit mạnh để tính sai số của phép chuẩn độ theo công thức, rồi hiệu chỉnh tính lại VTĐ

và tinh được chinh xac giá trị nông đô cua NaOH.

Hoặc các em có thể áp dụng phương trình bảo toàn điện tích hay phương trình bảo toàn proton (ĐKP) để tính được nồng độ chính xác của NaOH khi biết pH.

Tuy nhiên đối với học sinh phổ thông chưa được học để thiết lập được phương trình sai số chuẩn độ axit – bazơ, vì vậy GV cần vận dụng kiến thức về chuẩn độ axit – bazơ một cách hợp lí để hướng dẫn các em giải bài toán này mà không cần phải sử dụng đến phương trình sai số (quá nặng đối với các em) và cũng không nên dùng phương trình bảo toàn điện tích (thiên về ý nghĩa vật lí hơn là tư duy hóa học).

Trước hết giáo viên cần hướng dẫn cho các em xác định được giá trị pH tại thời điểm chỉ thị đổi màu (dựa vào khoảng chuyển màu của chỉ thị và dựa vào thứ tự chuẩn độ, HS biêt đươc trong trường hợp nay giá trị pH tại điểm kết thúc chuẩn độ băng 8,00. Vơi pH = 8,00 > 7,00, do đó còn dư NaOH theo phản ứng:

H+ + OH – →¬ H2O

C 148

48.10.5 3−

148

.100 oC

C’ _ 148

48.10.5.100 3−−oC

pH = 8,00→ [OH –] = 10 –6 >> [H+] = 10 –8 → có thể coi COH – dư = [OH –]

148

48.10.5.100 3−−oC = 10 –6 → Co M310.4015,2 −≈

Như vậy với cách giải này các em hiểu được bản chất của các quá trình xảy ra trong dung dịch, nắm được hiện tượng hóa học, giúp phát triển tư duy hóa học.

Cũng cần lưu ý rằng: với [OH –] = 10 –6 710−≈ (ta không thể nói 10 –6 >>10 –7)

khó có thể thuyết phục để các em chấp nhận COH(dư) = [OH –], nhưng nếu so sánh

50

Page 45: 1ygm3hwjpgu090312060334 tài liệu bồi dưỡng giáo viên trường thpt chuyên   mon hóa học - phan 1- năm 2012

[OH –] = 10 –6 >> [H+] = 10 –8, có nghĩa là sự phân li của H2O có thể bỏ qua, do đó sự chấp nhận COH(dư) = [OH –] là hợp lí.

Như vậy từ giá trị pH chuyển màu của chỉ thị cho phép đánh giá được nồng độ của chất tham gia phản ứng. Trên cơ sở này giáo viên có thể nâng mức độ phức tạp của bài toán nếu sử dụng hai chỉ thị sẽ cho phép xác định nồng độ của cả chất phân tích và chất chuẩn.

Ví dụ 5 [4]

Chuẩn độ 50,00 ml dung dịch HCl bằng dung dịch NaOH đến đổi màu phenolphtalein (pT = 9,00) thì phải dùng 25,00 ml NaOH. Nếu chuẩn độ đến xuất hiện màu vàng của metyl da cam (pT = 4,4) thì cần dùng 24,50 ml NaOH. Tính nồng độ HCl và NaOH.

Phân tích:

Với bài toán này, nếu sử dụng phương trình bảo toàn điện tích các em chỉ cần thiết lập hệ 2 phương trình 2 ẩn số là CHCl và CNaOH ở hai giá trị pH khác nhau, từ đó sẽ tính được nồng độ các chất tham gia phản ứng. Nhưng với cách giải như vậy chỉ thiên về công cụ tính toán, nặng về ý nghĩa vật lí mà không chú ý đến tư duy hóa học. Chính vì vậy giáo viên nên hướng dẫn cho các em phân tích bản chất các quá trình xảy ra trong dung dịch. Khi chuẩn độ đến pH = 9,00 > 7,00 → dư NaOH. Vì [OH –] = 10 –5 >> [H+] = 10 –9 do đó:

COH –(dư) =75

.50.25 0CC − = [OH –] = 10 –5 (1)

Nếu chuẩn độ đến pH = 4,4 < 7,00 → dư HCl (M)

Vì pH = 4,4 → CH+

(dư) = 5,74

.5,24.50 0 CC − = [H+] = 10 –4,4

(2)

Từ (1) và (2) → CHCl = Co = 3,70.10 –3(M) và CNaOH = C = 7,43.10 –3(M)

Từ ví dụ 4 và 5 ta thấy các bài toán đều giới hạn trong phạm vi là phép chuẩn độ đều kết thúc ở các giá trị pH mà tại đó có thể bỏ qua quá trình phân li của H2O (pH = 8,0; 9,0; 4,4). Nhưng trên thực tế có thể trung hòa đến giá trị pH bất kì, do đó giáo viên cần khai thác thêm trường hợp nâng cao này.

51

Page 46: 1ygm3hwjpgu090312060334 tài liệu bồi dưỡng giáo viên trường thpt chuyên   mon hóa học - phan 1- năm 2012

Ví dụ 6

Chuẩn độ 100,00 ml dung dịch HCl bằng dung dịch NaOH 1,00.10 –2M đến đổi màu metyl đỏ (pT = 6,2) thì hết 45,00 ml NaOH. Tính CHCl.

Phân tích:

Đây cũng là dạng bài toán cơ bản giống như ví dụ 5, tuy nhiên tại giá trị pH đổi màu của chỉ thị metyl đỏ, pH = 6,2 (dư axit) thì sự phân li của nước là đáng kể, do đó trong trường hợp này để tính chính xác CHCl, HS cần tính đến cân bằng phân li của nước. Điều này giúp cho các em, nhất là HS trường chuyên có tư duy hóa học đúng đắn.

Trên cơ sở các bài toán tính nồng độ các chất phản ứng, giáo viên có thể thay bằng các bài toán tính thể tích của dung dịch thuốc thử, tính hằng số cân bằng v.v.. từ giá trị pH đổi màu của chỉ thị.

Ví dụ 7

Chuẩn độ 25,00 ml dung dịch axit benzoic đến đổi màu chỉ thị metyl đỏ

(pT = 6,2) thì hết 20,70 ml dung dịch NaOH 0,100 M. Tính chính xác nồng độ

dung dịch axit benzoic.

Phân tích

Cũng tương tự như trên GV cần hướng dẫn các em xét các quá trình xảy ra

trong hệ. Khác với phép chuẩn độ axit mạnh, bazơ mạnh, trong bài toán này các

em không cần phải đánh giá gần đúng pHTĐ vẫn có thể xác định được phép chuẩn

độ kết thúc trước ĐTĐ vì khi chuẩn độ axit yếu bằng bazơ mạnh thì pHTĐ > 7,0 do

đó pHcuối = 6,2 < 7,0. Do đó tại điểm cuối chuẩn độ dư axit:

C6H5COOH + OH – → C6H5COO – + H2O

n (số mmol) 25Co 20,7.0,1

n’ 25Co –01.20,7 – 0,1.20,7

TPGH: C6H5COOH: 7,45

7,20.1,0.25 −oC M; C6H5COO –:

20 7

45 7

,

, M

Do pH = 6,2 ≈ 7,0 cho nên phải kể đến cân bằng phân li của nước, tức là phải

tính theo ĐKP với mức không là C6H5COOH để tính được 6 5C H COOH 0C = C .

52

Page 47: 1ygm3hwjpgu090312060334 tài liệu bồi dưỡng giáo viên trường thpt chuyên   mon hóa học - phan 1- năm 2012

Như vậy với bài này, ngoài ĐKP, các em cần phải biết đến khái niệm phân số

nồng độ -6 5

aC H COO

a

Kα =

K + h, cho nên ví dụ này chỉ nên dành cho đối tượng là HS

các trường chuyên.

Ví dụ 8 [2]

Cân 1,250 g axit yếu HA, hòa tan thành 50,00 ml dung dịch. Dùng dung dịch

chuẩn NaOH 0,090 M để chuẩn độ dung dịch HA. Biết rằng khi thêm 8,240 ml

NaOH vào dung dịch phân tích thì pH = 4,30. Nếu thêm 41,20 ml NaOH vào hỗn

hợp chuẩn độ thì đạt được điểm tương đương.

1/ Tính khối lượng mol phân tử HA.

2/ Tính hằng số axit của HA.

53

Page 48: 1ygm3hwjpgu090312060334 tài liệu bồi dưỡng giáo viên trường thpt chuyên   mon hóa học - phan 1- năm 2012

Phân tích:

Để tính được khối lượng mol phân tử của axit HA, cần phải biết nồng độ gốc Co của axit. Giá trị này HS có thể tính được ngay vì từ thể tích NaOH cho vào để đạt đến ĐTĐ → VTĐ = 41,20 ml. Từ đó các em tính được Co và tính được MHA.

Để tính hằng số phân li axit của axit HA, GV cần dẫn dắt cho HS xác định được điểm kết thúc chuẩn độ là trước điểm tương đương do tại thời điểm thêm 8,24 ml NaOH thì pHc = 4,30 < 7,00 < pHTĐ → dư HA, nghĩa là thành phần của hệ tại điểm cuối chuẩn độ gồm hệ đệm HA 0,051 M và A– 0,0127 M. Từ đó sẽ tính được pKa theo phương trình:

pKa = pH –log 904,4051,0

0127,0lg3,4 =−=

a

b

C

C

→ Ka = 1,25.10 –5

Ở đây điều kiện sử dụng phương trình pH = pKa + lga

b

C

C hoàn toàn thỏa

mãn.

Ví dụ 9 [8]

Tính thể tích dung dịch HCl 0,71 M cần tiêu thụ khi chuân đô 100 ml dung dịch chứa KCN 0,120 M và NH3 0,15 M đên pH = 9,24. Cho biết pKa (HCN) = 9,35; pKa(NH4

+) = 9,24.

Phân tích

Đây là phép chuẩn độ hỗn hợp hai đơn bazơ yếu bằng axit mạnh, nhưng hai bazơ này có hằng số phân li axit của axit liên hợp rất gần nhau vì thế phải chuẩn độ tông hai bazơ. Cung tương tư bai trên, nêu có kiến thức về chuẩn độ axit bazơ thì co thê tính VHCl bằng cách hiệu chỉnh thể tích thông qua sai số chuẩn độ. Tuy nhiên với đối tượng là học sinh phổ thông chưa được trang bị kiến thức về phương trình sai số, nên giáo viên cần hướng dẫn các em giải theo cách xác định % mỗi bazơ đã bị chuẩn độ tại pH = 9,24:

5,01010

10

][][

][24,924,9

24,9

43

4 =+

=+

=+ −−

+

+

hK

h

NHNH

NH

a

563,01010

10

][][

][35,924,9

35,9

=+

=+

=+ −−

− hK

h

CNHCN

HCN

a

54

Page 49: 1ygm3hwjpgu090312060334 tài liệu bồi dưỡng giáo viên trường thpt chuyên   mon hóa học - phan 1- năm 2012

tức là tại pH = 9,24 có 50% lượng NH3 và 56,3% CN – đã bị chuẩn độ, từ đó các em dễ dàng tính được VHCl.

Cách giải này đơn giản lại phù hợp với trình độ nhận thức của các em. Để củng cố thêm cho học sinh giáo viên có thể ra thêm bài tập bằng cách thay hỗn hợp đơn bazơ bằng hỗn hợp đơn axit hoặc có thể thêm vào đó một đơn axit (bazơ), thay đổi yêu cầu tính toán v.v...

III. Dạng bài toán xác định bước nhảy chuẩn độ (BNCĐ), chọn chỉ thị cho phép chuẩn độ, tính sai số chuẩn độ

Trong chuẩn độ thể tích nói chung, chuẩn độ axit – bazơ nói riêng 3 nội dung này có quan hệ mật thiết và liên hoàn, vấn đề quan trọng đầu tiên là phải chọn được chỉ thị thích hợp sao cho điểm kết thúc chuẩn độ càng gần với ĐTĐ càng tốt, tức là pH chuyển màu của chỉ thị phải gần với pHTĐ. Tuy vậy, do sự xuất hiện BNCĐ trên đường cong chuẩn độ nên không nhất thiết phải chọn chất chỉ thị có pT ≈ pHTĐ mà có thể chọn bất kì chất chỉ thị nào có chỉ số chuẩn độ nằm trong BNCĐ tương ứng với sai số cho phép. Như vậy để có được kết quả chuẩn độ chính xác, cần thiết phải chọn được chỉ thị thích hợp. Nếu HS đã được trang bị đầy đủ kiến thức về chuẩn độ axit –bazơ như sinh viên đại học thì các em có thể tính sai số đối với từng chỉ thị hoặc tính BNCĐ theo phương trình sai số từ đó cho phép chọn chỉ thị một cách hợp lí. Tuy nhiên đối với HSPT, kiến thức về chuẩn độ được trang bị hết sức sơ lược, vì vậy GV cần vận dụng lí thuyết về chuẩn độ để có thể hướng dẫn các em giải quyết vấn đề một cách linh hoạt tùy thuộc vào từng bài cụ thể.

Ví dụ 10 (Đề thi Quốc gia ITALIA)[2]

Để chuẩn độ CH3COOH bằng NaOH thì trong các chất chỉ thị sau đây, chất nào là tốt nhất?

A. Metyl da cam pKa=3,7.

B. Metyl đỏ pKa = 5,1.

C. Bromthymol xanh pKa = 7,0.

D. Phenolphtalein pKa = 9,4.

Phân tích:

Đây la phep chuân đô môt đơn axit yêu băng bazơ manh, do đó tai ĐTĐ thanh phân cua hê la môt đơn bazơ yêu → pHTĐ >7.

55

Page 50: 1ygm3hwjpgu090312060334 tài liệu bồi dưỡng giáo viên trường thpt chuyên   mon hóa học - phan 1- năm 2012

Đê chon chi thi thich hơp cho môt phep chuân đô co thê căn cư vao pHTĐ, vao VTĐ (thê tich thuôc thư tiêu thu đê đat đên ĐTĐ) hoăc căn cư vao BNCĐ. Muôn tinh đươc VTĐ hoăc BNCĐ cac em phai co sô liêu vê nông đô cac chât phan ưng va thê tich chât phân tich. Nhưng ơ đây bai ra chi hoi chung chung, vi vây HS chi cân chon chi thi nao co pT ma cu thê ơ bai nay la gia tri pKa > 7 la đươc.

Ví dụ 11

Tính bước nhảy pH (BNCĐ) của phép chuẩn độ 100 ml dung dịch HCl 0,100 M bằng dung dịch NaOH 0,100 M nếu chấp nhận sai số chuẩn độ không vượt quá 0,1%.

Phân tích:

Mặc dù chưa được học về phương trình sai số, nhưng GV có thể giải thích để các em nắm được khái niệm rất đơn giản về BNCĐ (hay bước nhảy pH) là khoảng pH thay đổi đột ngột xung quanh ĐTĐ [9].

Ở đây để xác định được BNCĐ ứng với sai số không vượt quá 0,1%, GV chỉ cần hướng dẫn HS tính các giá trị pH ứng với thể tích NaOH cho vào thiếu (Vđ: thể tích đầu bước nhảy) và thừa (Vc: thể tích cuối bước nhảy) 0,1% so với thể tích

NaOH cần thiết để đạt đến ĐTĐ (VTĐ): VTĐ =1,0

100.1,0= 100 ml, suy ra:

Vđ = VTĐ.99,99% = 99,9 ml

Vc = VTĐ.100,1% = 100,1 ml.

Tư các giá trị Vđ và Vc các em dễ dàng tính được pH tương ưng la 4,30 va 9,70. Điêu đo co nghia la với sai số không vượt quá 0,1% thì BNCĐ trong phép chuẩn độ trên là 4,3 – 9,7.

Ví dụ 12

Chuẩn độ 100 ml dung dịch HCl 0,100 M bằng dung dịch chuẩn NaOH 0,100 M. Có thể dùng chất nào trong 3 chất sau: metyl da cam, metyl đỏ, phenolphtalein làm chỉ thị thích hợp cho phép chuẩn độ trên với sai số không vượt quá 0,1%.

Phân tích:

Đây là phép chuẩn độ axit mạnh bằng bazơ mạnh, pHTĐ =7,00. Theo nguyên tắc chúng ta chỉ chọn chi thi nào có chỉ số chuẩn độ pT ≈ pHTĐ (chất có sự đổi màu rõ nhất tại giá trị pH≈ pHTĐ ). Như vậy trong số 3 chỉ thị trên cùng lắm chúng ta chỉ “dám” chọn chỉ thị metyl đỏ có pT = 6,2 ≈ 7,00 = pHTĐ cho phép chuẩn độ này. Nhưng với kiến thức về chuẩn độ axit bazơ, chúng ta đã biết là sự xuất hiện

56

Page 51: 1ygm3hwjpgu090312060334 tài liệu bồi dưỡng giáo viên trường thpt chuyên   mon hóa học - phan 1- năm 2012

BNCĐ trên đường cong chuẩn độ cho phép mở rộng phạm vi chọn chỉ thị: có thể chọn bất kì chỉ thị nào có chỉ số chuẩn độ pT (hay có pH tại điểm chuyển màu ro nhất) nằm trong BNCĐ ứng với sai số cho phép. Chính vì vậy để chọn được chỉ thị thích hợp chúng ta có thể tính BNCĐ ứng với sai số cho trước. Với BNCĐ đã tính được là 4,3 đến 9,7 thì đối với phép chuẩn độ trên hoàn toàn có thể dùng được cả 3 chất chỉ thị.

Ví dụ 13 [5]

Đánh giá sai số khi chuẩn độ 15,00 ml dung dịch NH3 0,020 M bằng dung dịch HCl 0,030 M đến xuất hiện màu đỏ của metyl da cam (pT = 4,0).

Phân tích

Việc đánh giá sai số của phép chuẩn độ rất quan trọng vì từ đó biết được việc chọn chỉ thị đã phù hợp chưa, kết quả thu được từ phép chuẩn độ có đáng tin cậy, tin cậy ở mức độ nào để có cách xử lí số liệu.

Để tính được sai số chuẩn độ, GV cần hướng dẫn để các em hiểu khái niệm về sai số: Sai số chuẩn độ là tỉ số % giữa lượng thuốc thử đã cho dư hoặc còn thiếu so với lượng cần thiết để chuẩn độ đến điểm tương đương. Như vậy:

q = 100⋅−

V

VVc

trong đó Vc: là thể tích thuốc thử tại điểm cuối chuẩn độ

VTĐ: là thể tích thuốc thử tại điểm tương đương

Như vậy để tính được q, các em phải tính được VTĐ từ phương trình phản ứng

chuẩn độ: VTĐ =03,0

02,0.15= 10 ml

Để tính Vc cần phải xác định được thành phần của hệ tại thời điểm chỉ thị đổi màu. Muốn thế cần phải tính được pHTĐ theo quá trình phân ly của NH4

+. Từ giá trị pHTĐ = 5,58 > pHc = 4,0 → dừng chuẩn độ sau điểm tương đương → dư H+

theo phan ưng:

NH3 + H+ →¬ NH4+

C cV+15

02,0.15

c

c

V

V

+15

03,0.

C’ _ c

c

V

V

+−

15

3,003,0

cV+15

3,0

57

Page 52: 1ygm3hwjpgu090312060334 tài liệu bồi dưỡng giáo viên trường thpt chuyên   mon hóa học - phan 1- năm 2012

Từ thành phần của hệ tại ĐCCĐ gồm: H+ c

c

V

V

+−

15

45,003,0 M và NH4

+

cV+15

45,0M

Các em sẽ dễ dàng tính được Vc = 10,083 → q = %83,010010

00,10083,10 =⋅−

Như vây măc du cach giai dai hơn nhưng phu hơp vơi trinh đô cua HSPT, giup cac em phat triên đươc tư duy hoa hoc.

Ví dụ 14

Có thể dùng phenolphtalein làm chỉ thị cho phép chuẩn độ 25,00 ml dung dịch axit benzoic 0,1 M bằng dung dịch NaOH 0,2 M được không, nếu sai số cho phép không vượt quá 0,1%?

Phân tích

Cũng giống như dạng bài chọn chỉ thị thích hợp cho phép chuẩn độ axit mạnh, bazơ mạnh. Để giải quyết bài toán này, hoặc phải tính được pHTĐ để so sánh với giá trị pT của chỉ thị, hoăc phai tinh BNCĐ va xem gia tri pT có nằm trong BNCĐ không? Đối với HS trường chuyên, việc xác định pHTĐ là hoàn toàn có thể thực hiện được và các em dễ dàng tính được pHTĐ = 8,5 ≈ 9,00 = pT của chỉ thị phenolphtalein. Do đó có thể chọn được phenolphtalein làm chỉ thị cho phép chuẩn độ trên.

Tuy nhiên không phải lúc nào cũng chọn được chỉ thị có pT xấp xỉ pHTĐ, vì vậy đê đanh gia chinh xac hơn cần phải tính được BNCĐ từ các giá trị thể tích NaOH cho vào thiếu hoặc thừa so với VTĐ là 0,1%. Từ hai giá trị thể tích này các em sẽ tính được giá trị pHđ = 7,2 và pHc = 9,8. Mặc dù pHđ = 7,2 ≈ 7,00 lẽ ra cần phải đánh giá chính xác theo ĐKP vì phải kể đến cân bằng phân li của H2O. Nhưng nếu chỉ cần so sánh với pTphenolphtalein = 9,00 thì có thể chấp nhận được giá trị gần đúng này.

Vậy BNCĐ = 7,2 – 9,8 → có thể chọn được phenolphtalein làm chỉ thị cho phép chuẩn độ trên vì pTphenolphtalein thuộc BNCĐ.

Vi du 15

58

Page 53: 1ygm3hwjpgu090312060334 tài liệu bồi dưỡng giáo viên trường thpt chuyên   mon hóa học - phan 1- năm 2012

Tính pH của dung dịch NaOH 0,01 M và CH3COONa 0,02 M. Có khả năng chuẩn độ riêng NaOH trong hỗn hợp trên dùng phenolphtalein làm chỉ thị (pT = 8,0) được không? (dùng chất chuẩn là HCl 0,100 M).

Phân tích

Để trả lời câu hỏi có thể chuẩn độ riêng NaOH trong hỗn hợp trên khi dùng phenolphtalein làm chỉ thị được không, đối vơi HS trương chuyên, hoăc vơi đôi tuyên quôc gia, GV co thê hương dân theo hai cach sau:

Cách 1: Xét xem tại điểm dừng chuẩn độ ứng với giá trị pH = 8 thi phần trăm lượng CH3COO – đã bị chuẩn độ là bao nhiêu, bằng cách tính tỉ số:

4874,4

8

33

3 10.75,51010

10

][][

][ −−−

− =+

=+

=+ hK

h

COOHCHCOOCH

COOHCH

a

hay

0,0575%.

Như vậy khi dừng chuẩn độ tại pH = 8 lượng CH3COO – bị chuẩn độ khá nhỏ, nên có thể coi chuẩn độ riêng được NaOH trong hỗn hợp trên.

Cach 2: Hoặc GV có thể hướng dẫn các em tính pH của dung dịch khi vừa chuẩn độ hết NaOH tức là tính pH của dung dịch CH3COO – rồi so sánh với giá trị pH dừng chuẩn độ (pH = 8) để rút ra kết luận. Nhưng đê tinh đươc pH cua dung dich CH3COO – thi bai ra phai cho thêm thê tich cua hôn hơp đê HS tinh lai nông đô cua CH3COO – tai thơi điêm chuân đô hêt NaOH. Gia sư bai ra cho thê tich cua dung dich chuân đô la 50 ml → thê tich dung dich cân đê chuân đô hêt NaOH la

V1 = ml51,0

01,0.50 = . Khi đo MCCOOCH

0182,055

50.02,03

==− va HS dễ dàng

tính được pH của dung dịch CH3COO –.

0,0182 M bằng 8,5 trong khi pH dừng chuẩn độ là 8. Như vậy CH3COO –

cũng bị chuẩn độ môt lượng nhỏ, do đó có thể chuẩn độ riêng được NaOH trong hỗn hợp khi dùng chất chỉ thị là phenolphtalein với chất chuẩn là HCl 0,1M.

IV. Dạng bài toán vẽ đường cong chuẩn độ, xác định thành phần của hệ chuẩn độ

Ví dụ 16

Chuẩn độ 100 ml dung dịch HCl 0,100 M bằng dung dịch NaOH 0,100 M. Hãy vẽ dạng đường cong chuẩn độ của phép chuẩn độ trên.

59

Page 54: 1ygm3hwjpgu090312060334 tài liệu bồi dưỡng giáo viên trường thpt chuyên   mon hóa học - phan 1- năm 2012

Phân tích:

Trước hết GV cần giải thích cho HS hiểu được khái niệm về đường cong chuẩn độ chính là đường biểu diễn sự biến thiên pH của dung dịch theo VNaOH

thêm vao trong quá trình chuẩn độ. Từ đó GV cần hướng dẫn cho các em thiết lập sự phụ thuộc của pH theo thể tích của thuốc thử ở từng thời điểm. Ví dụ:

* Trước ĐTĐ → dư axit:

H+ + OH – →¬ H2O

C o

oo

VV

VC

+ oVV

CV

+

C’ o

oo

VV

CVVC

+−

_

Chấp nhận [H+] = CHCl(dư) = o

oo

VV

CVVC

+−

* Sau ĐTĐ → dư OH – với CNaOH(dư) = o

oo

VV

VCCV

+−

* Tại ĐTĐ: pH = 7,00.

Như vậy biết thể tích thuốc thử (V), chúng ta sẽ tính được pH, từ đó sẽ vẽ được đường chuẩn độ. Vì ở đây chỉ yêu cầu HS vẽ dạng đường cong nên không yêu cầu các em phải kể đến sự phân li của nước và GV nên hướng dẫn để các em có thể chú ý xác định một số điểm đặc biệt như khi chưa chuẩn độ (V = 0 ml), điểm đầu và cuối bước nhảy (pHđ, pHc) tùy theo sai số chọn và ĐTĐ (pHTĐ), v.v….

Ví dụ, đối với phép chuẩn độ đã cho, các em tính được các giá trị pH tương ứng theo VNaOH và ghi số liệu thu được theo bảng sau:

VNaOH 0 10 50 90 99 99,9 100 100,1 101 110

pH 1 1,1 1,48 2,28 3,30 4,30 7,0 9,70 10,7 11,68

Từ các số liệu thu được, HS vẽ đồ thị biểu diễn sự phụ thuộc pH = f(V), từ đó HS có thể nhận xét khoảng bước nhảy pH.

Ví dụ 17 (Đề thi Olympic Quốc tế 2003 – Hy Lạp) [6]

60

Page 55: 1ygm3hwjpgu090312060334 tài liệu bồi dưỡng giáo viên trường thpt chuyên   mon hóa học - phan 1- năm 2012

Dung dịch H3PO4 và/hoặc NaH2PO4 được chuẩn độ với dung dịch chuẩn bazơ mạnh. Kết hợp thành phần các dung dịch này với các đường cong chuẩn độ (pH theo thể tích dung dịch chuẩn độ) nêu trong hình (với H3PO4: pK1 = 2,15; pK2 = 7,2; pK3 = 12,0).

pH

Thể tích dung dịch chuẩn độ (mL)

Cho biêt:

a) Mẫu chỉ chứa H3PO4.

b) Mẫu chứa cả hai theo tỉ lệ mol H3PO4: NaH2PO4 là 2:1.

c) Mẫu chứa cả hai theo tỉ lệ mol H3PO4: NaH2PO4 là 1:1.

Phân tích

Đây là dạng bài phân tích dang đường cong chuẩn độ để xác định thành phần của dung dịch phân tich. Như vây đê lam đươc bai tâp nay, HS phai hiêu đươc cach xây dưng đương cong chuân đô, phai phân tich đươc tưng giai đoan chuân đô trên đương cong chuân đô, phai năm đươc khai niêm BNCĐ va kha năng chuân đô riêng tưng nâc cung như điêu kiên đê môt phep chuân đô co thê thưc hiên đươc. Đây la môt yêu câu rât cao đôi vơi HS, vi dang nay rât it găp đôi vơi cac em. Trươc hêt GV cân hương dân đê cac em xac đinh đươc đôi vơi axit 3 chưc

H3PO4 co ti lê 05,52,7

15,2

2

1 1010

10 == −

a

a

K

K > 104;

3

2

a

a

K

K > 104 nhưng vi Ka3 =10 –12,0

(rât nho), nên co kha năng chuân đô riêng nâc 1 va nâc 2, nhưng không chuân đô đươc nâc 3. Va đây la phep chuân đô môt axit ba chưc (hoăc hôn hơp) nên trên truc hoanh cua ca 4 hinh ve đêu co 3 điêm ưng vơi cac thê tich cua cac thuôc thư la 2 ml, 4 ml, 6 ml ưng vơi 3 điêm tương đương. Va tuy thuôc vao thanh phân cua mâu phân tich se co cac ĐTĐ ưng vơi cac thê tich khac nhau.

61

Page 56: 1ygm3hwjpgu090312060334 tài liệu bồi dưỡng giáo viên trường thpt chuyên   mon hóa học - phan 1- năm 2012

Tiêp theo GV cân phân tich dang đương cong đê cac em hiêu đươc nhưng đoan gần như thăng đưng trên đương cong chuân đô đăc trưng cho BNCĐ va ưng vơi cac ĐTĐ đê suy ra thanh phân cua hê phân tich.

Như vậy căn cứ vao thê tich tiêu thu V1 (ứng với ĐTĐ1), V2 (ứng với ĐTĐ2), HS se tim đươc cac dang đương cong tương ưng vơi tưng mâu phân tich: Đôi vơi mâu phân tich chưa H3PO4 và H2PO4

– có tỉ lệ 1:1 ưng vơi đương cong D; mâu phân tich chưa ca 2 chât theo ti lê mol la H3PO4: H2PO4

– = 2:1 ưng vơi đương cong B và mẫu chỉ gồm H3PO4 ứng với đường cong A.

Ví dụ 18 (Đề thi vòng 3 Olympic Đức 2000)[6]

Cho hai hỗn hợp A và B. Hỗn hợp A chứa Na2CO3 và NaHCO3. Hỗn hợp B chứa Na2CO3 và NaOH. Hòa tan một trong hai hỗn hợp này vào nước, và pha thành 100 ml dung dịch. Chuẩn độ 20,00 ml dung dịch thu được bằng dung dịch HCl 0,200 M với chất chỉ thị phenolphtalein, hết 36,15 ml HCl. Nếu sử dụng chất chỉ thị metyl da cam thì thể tích HCl tiêu thụ là 43,8 ml.

1/ Hãy cho biết phản ứng nào xảy ra hoàn toàn khi dung dịch chuyển màu.

2/ Hãy cho biết (có giải thích) hỗn hợp phân tích là hỗn hợp A hay B.

3/ Hãy xác định thành phần phần trăm khối lượng của hỗn hợp đã phân tích. Cho biết: khoảng chuyển màu của phenolphtalein: pH = 8,3 – 10; của metyl da cam: pH = 3,1 – 4,4.

Phân tích

Đê giai đươc bài này, học sinh cần phải xac đinh đươc giá trị pH ma tai đo môi chỉ thị chuyên mau trong từng phép chuẩn độ và cac em phai viêt đươc phản ứng xảy ra khi chi thi thay đôi tinh hiêu.

Do thành phần của ca hai hôn hơp A va B đêu chưa CO32 –, do đo GV cân

hương dân đê HS xac đinh đươc kha năng chuân đô riêng tưng nâc cua đa bazơ tư

viêc xet ti lê giưa 2

1

a

a

K

K. Sau đo cac em phai liên hê giưa gia tri pH chuyên mau

cua phenolphtalein (pT = 8,3) vơi cac gia tri pKa1, pKa2 đê thây răng khi chất chi thi phenolphtalein mât mau, phep chuân đô dưng ơ nâc 1 tạo thành HCO3

–. Và tai gia tri pH chuyên mau cua metyl da cam (pT = 4,0), phep chuân đô dưng ơ nâc 2.

62

Page 57: 1ygm3hwjpgu090312060334 tài liệu bồi dưỡng giáo viên trường thpt chuyên   mon hóa học - phan 1- năm 2012

Đê xac đinh hôn hơp phân tich la A hay B, cac em phai căn cư vao quan hê thê tich tiêu thu cua thuôc thư (HCl) tai hai điêm dưng chuân đô la V1 (khi dung phenolphtalein) va V2 (khi dung metyl da cam):

Nêu mâu phân tích chi co CO32 – thi V2 ≈ 2V1.

Nêu trong mâu chưa CO32 – va HCO3

– thi V2 > 2V1.

Và nêu mâu phân tích gồm CO32 – va OH – thi V2 < 2V1.

Theo bài ra ta thây V2 < 2 V1. Vây hôn hơp phân tich la hôn hơp B.

Ví dụ 19 (Đề thi chọn đội tuyển quốc gia Việt Nam dự thi Olympic Hóa học Quốc tế, 2011) [7]

Có 4 lọ hóa chất (A, B, C, D) bị mất nhãn, mỗi lọ chứa có thể là dung dịch của một trong các chất: HCl, H3AsO4, NaH2AsO4, cũng có thể là dung dịch hỗn hợp của chúng. Để xác định các lọ hóa chất trên, người ta tiến hành chuẩn độ 10,00 ml mỗi dung dịch bằng dung dịch NaOH 0,120 M, lần lượt với từng chất chỉ thị metyl da cam (pH = 4,40), phenolphtalein (pH = 9,00) riêng rẽ.

Kết quả chuẩn độ thu được như sau:

Dung dịch

chuẩn độ

VNaOH = V1 (ml) VNaOH = V2 (ml)

Dùng chỉ thị metyl da cam Dùng chỉ thị phenolphtalein

A

B

C

D

12,50

11,82

10,75

0,00

18,20

23,60

30,00

13,15

1. Hãy biện luận để xác định thành phần định tính của từng dung dịch A, B, C, D.

2. a) Tính nồng độ ban đầu của chất tan trong dung dịch C.

b) Tính số mol Na3AsO4 cần cho vào 10,00 ml dung dịch C để thu được hỗn hợp có pH = 6,50 (coi thể tích của dung dịch không thay đổi khi thêm Na3AsO4 và bỏ qua sự phân li của nước).

Phân tích:

Tương tự như ví dụ 18, để biện luận hệ, GV cần hướng dẫn để HS xác định khả năng chuẩn độ riêng được nấc 1 và nấc 2, với 2 chỉ thị đã dùng. Và cũng trên

63

Page 58: 1ygm3hwjpgu090312060334 tài liệu bồi dưỡng giáo viên trường thpt chuyên   mon hóa học - phan 1- năm 2012

cơ sở so sánh mối quan hệ giữa V1 và V2 HS sẽ không khó khăn để xác định thành phần của từng dung dịch.

NHẬN XÉT

Qua một số bài tập cũng như một số đề thi trong và ngoài nước, môt lân nưa chung ta thây ro vi tri va vai tro quan trọng cua phương phap chuân đô axit – bazơ trong nội dung chương trinh chuyên và trong bôi dương đôi tuyên dư thi HSG quôc gia va quôc tê. Vi vây cân thiêt phai trang bi cho GV và HS những kiến thức nâng cao ngang tầm quốc tế, nhưng vẫn đảm bảo mức độ hợp lý, phù hợp với trình độ học sinh phổ thông.

TAI LIÊU THAM KHAO

[1] Bài tập chuẩn bị Olympic lần thứ 31 của Thái Lan, 1999.

[2] Hoang Minh Châu, Ngô Thi Thân, Ha Thi Diêp, Đao Đinh Thưc (hiêu đinh tiêng Đưc), Trân Thanh Huê, Nguyên Trong Tho, Pham Đinh Hiên – Olympic Hoa hoc Viêt Nam va Quôc tê – Tâp V. NXB Giao duc, 2003.

[3] Nguyễn Tinh Dung – Hóa học phân tích phần III. Các phương pháp định lượng hóa học. NXB Giáo dục, 2001.

[4] Nguyên Tinh Dung – Bai tâp hoa hoc phân tich. NXB Giao duc, 1982.

[5] Đao Thi Phương Diêp, Đô Văn Huê – Giao Trinh Hoa hoc phân tich, cac phương phap đinh lương hoa hoc. NXB Đai hoc Sư pham, 2007.

[6] Đê thi Olympic Quôc tê tư năm 2002 đên năm 2009.

[7] Đề thi chọn đội tuyển Quốc gia dự thi Olympic Quốc tế năm 2011.

[8] Sơ Giao duc va đao tao Thanh phô Hô Chi Minh. Trương THPT chuyên Lê Hông Phong – Tuyên tâp đê thi Olympic 30 – 4 lân thư XIV. NXB Giao duc, 2008.

[9] Lê Xuân Trong, Nguyên Hưu Đinh, Tư Vong Nghi, Đô Đinh Rang, Cao Thi Thăng – Hoa hoc 12 nâng cao. NXB Giao duc, 2008.

64

Page 59: 1ygm3hwjpgu090312060334 tài liệu bồi dưỡng giáo viên trường thpt chuyên   mon hóa học - phan 1- năm 2012

MéT Sè VÊN §Ò ¸P DôNG C¸C PH¦¥NG PH¸P PH¢N TÝCH C¤NG Cô

PGS.TS Đặng Xuân Thư

Khoa Hóa học – Trường Đại học Sư phạm Hà Nội

A. PHƯƠNG PHÁP QUANG PHỔ HẤP THỤ PHÂN TỬ

I. CƠ SỞ LÝ THUYẾT

1. Phổ điện từ (Electromagnetic spectrum)

– Ánh sáng là hiện tượng điện từ, nghĩa là nó đồng thời truyền qua không

gian cả sóng từ lẫn điện từ. Toán học có thể mô tả hiện tượng sóng theo phương

trình Maxwell.

– Để mô tả ánh sáng như hiện tượng sóng thì các tham số quan trọng nhất là:

tần số ν (frequency) hoặc bước sóng λ (độ dài sóng: wavelength); trong khi coi

ánh sáng có tính chất hạt (photon) thì tính chất đặc thù của nó là năng lượng E:

E = hν (1.1)

ở đây h là hằng số Planck (h = 6,626.10 –34Js = 6,626.10 –27ec.giây).

Bước sóng phụ thuộc vào tần số ν và tốc độ v của ánh sáng:

λ = v/ν ⇔ λν = v = c (1.2)

Trong chân không tốc độ ánh sáng không đổi là v = c = 2,998.108 ms –1. Tốc

độ ánh sáng chỉ giảm nhẹ trong không khí (có thể coi v=c) nhưng giảm nhiều

trong các môi trường đậm đặc hơn như nước hay thuỷ tinh.

Phổ điện từ bao gồm một vùng rộng của bước sóng hay năng lượng:

65

Page 60: 1ygm3hwjpgu090312060334 tài liệu bồi dưỡng giáo viên trường thpt chuyên   mon hóa học - phan 1- năm 2012

Năng lượng E

(J.mol –1)

Bước sóng λ (m)

Số sóng ν(cm –1)

Vùng phổHiện tượng

quan sát được

Tia γ Chuyển dịch hạt nhân

109 10 –10 108

Tia x Chuyển dịch electron trong107 10 –8 106

Tử ngoại và khả kiến

Chuyển dịch electron hoá trị

105 10 –6 104

Hồng ngoại Dao động103 10 –4 102

Sóng ngắn

(sóng viba)

Chuyển động quay

10 10 –2 1Sóng radio, TV...

– 10 – 200 nm: vùng tử ngoại chân không (vì trong vùng này không khí cũng hấp thụ, nên các thiết bị đo phải dùng trong chân không).

– 200 – 380 nm: vùng tử ngoại gần.

– 380 – 780 nm: vùng khả kiến (vùng nhìn thấy).

– 780 nm (0,78 µm) – 2,5 µm: vùng hồng ngoại gần.

– 2,5 µm – 16 µm vùng hồng ngoại giữa hay vùng hồng ngoại NaCl: là vùng

thường dùng nhất trong phương pháp hồng ngoại.

– 16 µm – 300 µm là vùng hồng ngoại xa.

Màu của các bước sóng trong vùng nhìn thấy là vùng rất hẹp của bức xạ điện từ:

Màu của hợp chất quan sát được

Màu ánh sáng bị hấp thụ

Bước sóng gần đúng của ánh sáng bị hấp thụ

Green Red 700 nm

Blue –green Orange –red 600 nm

Violet Yellow 550 nm

Red –violet Yellow –green 530 nm

Red Blue –green 500 nm

Orange Blue 450 nm

Yellow Violet 400 nm

2. Khả năng hấp thụ ánh sáng (hấp thụ bức xạ điện từ)

66

Page 61: 1ygm3hwjpgu090312060334 tài liệu bồi dưỡng giáo viên trường thpt chuyên   mon hóa học - phan 1- năm 2012

E1

E1

E1

Khi bức xạ đi qua một lớp trong suốt các chất rắn, lỏng hay khí thì xảy ra một sự hấp thụ chọn lọc bức xạ với các tần số xác định. Năng lượng của bức xạ được truyền cho phân tử hay nguyên tử chất hấp thụ và chuyển các chất này từ trạng thái cơ bản (có năng lượng EO) sang trạng thái kích thích (có năng lượng EJ) trạng thái này chỉ tồn tại trong một thời gia rất ngắn 10 –8 – 10 –9 giây. Sau đó từ trạng thái kích thích các chất hấp thụ sẽ chuyển trở lại trạng thái cơ bản.

M + hν M* M* M + Q

Năng lượng Q giải toả có thể dưới dạng nhiệt, hay có thể dưới dạng làm biến đổi hoá học (phản ứng quang hoá) hay có thể kèm theo sự phát huỳnh quang hoặc lân quang.

3. Hiện tượng hấp thụ phân tử trong đo quang

Nội năng của phân tử: E = Ee + Ev + Ej

trong đó Ee đặc trưng cho sự chuyển electron từ mức năng lượng này sang mức năng lượng khác. Ev năng lượng dao động các hạt nguyên tử trong phân tử. Ej năng lượng quay các nguyên tử, nhóm nguyên tử trong phân tử.

các mức năng lượng quay

các mức năng lượng dao động

các mức năng lượng điện từ

67

Page 62: 1ygm3hwjpgu090312060334 tài liệu bồi dưỡng giáo viên trường thpt chuyên   mon hóa học - phan 1- năm 2012

Khi chiếu một bức xạ vào phân tử thì phân tử hấp thụ mang tính chọn lọc

∆E = E2 – E1 = hν có tính gián đoạn, tuy nhiên trong phân tử các mức năng lượng

Ee; Ev; Ej xếp chồng chéo lên nhau do đó phổ phân tử là phổ đám (khác với phổ hấp thụ nguyên tử là phổ vạch).

Vì Ee > Ev > Ej nên khi phổ điện từ thu được trong trường hợp hν≥ Ee thì

bao gồm cả phổ electron lẫn với phổ dao động nguyên tử hay nhóm nguyên tử trong phân tử và phổ quay.

Còn nếu Ee ≥ hν ≥ EV thì phổ thu được bao gồm phổ dao động và phổ quay

phân tử. Nếu Ev ≥ hν ≥ Ej thì chỉ thu được phổ quay của phân tử chất hấp thụ (IR

"infrare").

4. Các định luật cơ bản trong quang phổ hấp thụ phân tử

4.1. Định luật Bia (Beer)

Định luật hợp nhất Buge – Lambe – Bia biểu diễn qua biểu thức định lượng:

Il = I0.e –εlC (2.6)

hay: lCI

IlgA

l

ε== 0 (2.7)

ở đây, ε (mol –1.cm –1.lít) là hệ số hấp thụ mol phân tử khi nồng độ được biểu

diễn theo mol/lít và bề dày lớp dung dịch tính theo cm.

Theo tính toán lượng tử thì hệ số hấp thụ mol phân tử lớn nhất chỉ nằm trong khoảng 105 đến 1,2.105, ít có trường hợp lớn hơn.

Tỉ số giữa cường độ dòng sáng đi ra khỏi lớp dung dịch I l với cường độ dòng sáng đơn sắc ban đầu IO gọi là độ truyền quang của dung dịch:

T = Il/IO = 10 –εlC A = –lgT

Các đại lượng A, T phụ thuộc vào: bản chất chất hấp thụ ánh sáng, bước sóng của ánh sáng, nồng độ của chất hấp thụ, nhiệt độ, bề dày cuvét (bề dày lớp dung dịch mà ánh sáng đi qua), dung môi.

4.2. Định luật cộng tính

Định luật cộng tính có ý nghĩa quan trọng trong phép xác định đo quang của các hệ chứa nhiều cấu tử không tương tác hoá học với nhau và đều có khả năng hấp thụ ánh sáng ở bước sóng đang xét.

68

Page 63: 1ygm3hwjpgu090312060334 tài liệu bồi dưỡng giáo viên trường thpt chuyên   mon hóa học - phan 1- năm 2012

– Đối với mật độ quang và độ truyền quang: nn

i ii 1 i 1

A A T T= =

= =∑ ∏

II. ỨNG DỤNG PHÉP ĐO PHỔ HẤP THỤ PHÂN TỬ VÙNG UV –Vis

1. Nghiên cứu thuốc thử hữu cơ (xác định hằng số phân li axit của thuốc thử)

Một trong những ứng dụng quan trọng của phép đo phổ hấp thụ vùng UV –Vis là xác định hằng số phân li axit. Xét cân bằng phân li của chỉ thị axit – bazơ nồng độ C, trong trường hợp đơn giản phân tử axit HIn.

HIn →¬ H+ + In – Ka (1)

Cả hai dạng axit HIn và bazơ liên hợp In – đều hấp thụ trong vùng UV – Vis

với hệ số hấp thụ mol phân tử εHIn và εIn ở các bước sóng hấp thụ cực đại λHIn và

λIn. Ở các pH khác nhau cân bằng (1) sẽ tạo ra nồng độ cân bằng khác nhau của

các dạng axit và bazơ.

Ở một giá trị pH xác định theo định luật cộng tính:

A = εHInl[HIn] + εInl[In] = εHInlαHInC + εInlαInC

Ta có thể xác định bằng hai phương pháp:

Phương pháp 1: Đo ở một bước sóng, về nguyên tắc có thể đo ở các bước sóng bất kì (trừ điểm đẳng quang: là điểm tại bước sóng đó tất cả các dạng tồn tại của chất nghiên cứu đều có hệ số hấp thụ mol bằng nhau), tuy nhiên ta nên chọn bước sóng tại đó cả hai dạng axit và bazơ hấp thụ không quá bé. Ở vùng pH thấp

(xem đường (1)) chỉ thị chỉ tồn tại dưới dạng axit, khi đó αHIn ≈ 1,0 còn αIn ≈ 0,0;

69

X

A

λ (nm)

Phổ hấp thụ của dung dịch HIn ở các pH khác nhau: (1) pH = 1,0

(2) pH = 3,5

(3) pH = 5,0

(4) pH = 6,5

(5) pH = 10,51

2

3

4

5

Page 64: 1ygm3hwjpgu090312060334 tài liệu bồi dưỡng giáo viên trường thpt chuyên   mon hóa học - phan 1- năm 2012

ta đo được A' = εHInlC. Còn khi ở pH cao (xem đường (5)) chỉ thị chỉ tồn tại dưới

dạng bazơ, khi đó αIn ≈ 1,0 còn αHIn ≈ 0,0; ta đo được A'' = εInlC.

AHIn HIn In In HIn In

HIn In a a

Aa

KA ' A '' hlC A lC. lC. A '. A ''. A '. A ''.

K h K h

K A A ' A A 'pK pH lg

h A '' A A '' A

= = ⇒ = ε α + ε α = α + α = +ε ε + +

− −⇒ = ⇒ = −− −

Phương pháp 2: Đo ở hai bước sóng hấp thụ cực đại của hai dạng axit và bazơ.

– Ở bước sóng hấp thụ cực đại dạng axit λHIn tiến hành đo mật độ quang của

dung dịch có pH trung gian, dung dịch có pH thấp và dung dịch có pH cao:

]In[l]HIn[lAHInHIn

HInInHIn

λλε+ε=λ ; lCA;lCA

HInHInHInHIn

InInHInHIn

λλλλε=ε=

– Ở bước sóng hấp thụ cực đại dạng bazơ λIn tiến hành đo mật độ quang của

dung dịch có pH trung gian, dung dịch có pH thấp và dung dịch có pH cao:

]In[l]HIn[lAInIn

InInHIn

λλε+ε=λ ; lCA;lCA

InInInIn

InInHInHIn

λλλλε=ε=

Mặt khác: [ ][ ]

[ ]HIn

InHK a

−+= . Từ các phương trình trên ta thu được:

HInHIn In

In

HInIn HIn

In

HIn HIn

a

In In

AA A

ApK pH lg

AA A

A

λλ λ

λ

λλ λ

λ

= −

Để có thể xác định chính xác hằng số phân li axit, nên chọn vùng pH có sự đóng góp mật độ quang của các dạng liên hợp của chất nghiên cứu là tương đương.

Hệ quả: Có thể sử dụng để xác định pH của dung dịch nếu biết pK của thuốc thử:

HInHIn In

In

HInIn HIn

In

HIn HIn

a

In In

AA A

ApH pK lg

AA A

A

λλ λ

λ

λλ λ

λ

= +

70

Page 65: 1ygm3hwjpgu090312060334 tài liệu bồi dưỡng giáo viên trường thpt chuyên   mon hóa học - phan 1- năm 2012

Để pH xác định chính xác nên chọn chỉ thị có pKa ≈ pH, khi đó sự đóng góp

mật độ quang của các dạng axit và bazơ là tương đương, vì vậy nên chọn chỉ thị có khoảng chuyển màu chứa pH của dung dịch cần xác định.

2. Các phương pháp đo quang định lượng vùng UV – Vis

2.1. Xác định nồng độ bằng phương pháp đường chuẩn

Phương pháp đường chuẩn được áp dụng thuận tiện cho việc phân tích hàng loạt, cho kết quả nhanh chóng. Xây dựng đường chuẩn A = f(C), dựa trên cơ sở

định luật Bia dạng phụ thuộc tuyến tính: A = (a ± εa) + (b ± εb) (dùng phần mềm

Excell hoặc xử lí hồi qui tuyến tính theo nguyên lí bình phương tối thiểu).

Thí dụ: Khi xác định nồng độ Fe3+ trong một mẫu nước bằng axit sulfosalixilic ở pH = 10, người ta sử dụng phương pháp đường chuẩn. Kết quả thực nghiệm thu được trong bảng sau:

Nồng độ M A Nồng độ M A Nồng độ M A

1.10 –5 0,100 5.10 –5 0,498 9.10 –5 0,888

2.10 –5 0,201 6.10 –5 0,595 10.10 –5 0,989

3.10 –5 0,302 7.10 –5 0,703

4.10 –5 0,402 8.10 –5 0,790 Mẫu 0,536

71

Page 66: 1ygm3hwjpgu090312060334 tài liệu bồi dưỡng giáo viên trường thpt chuyên   mon hóa học - phan 1- năm 2012

Phương trình đường chuẩn thu được: A = 9850.C + 0,0051

Thay Anc = 0,536 vào phương trình trên thu được: C = 5,39.10 –5 mol/lít.

Ưu điểm: xác định nhanh, hàng loạt mẫu nghiên cứu.

Nhược điểm: Không loại trừ được các yếu tố ảnh hưởng của các chất lạ trong dung dịch nghiên cứu.

2.2. Xác định nồng độ bằng phương pháp thêm

Phương pháp 1: Lấy cùng lượng dung dịch nghiên cứu VX có nồng độ CX vào hai bình định mức cùng dung tích, thêm vào bình thứ hai một lượng chính xác chất phân tích (Ca). Thực hiện phản ứng màu cho cả hai dung dịch trong cùng điều kiện, định mức đến vạch. Tiến hành đo mật độ quang của hai dung dịch ở cùng bước sóng chọn lọc (bước sóng tối ưu), dùng hai cuvet như nhau, so sánh với cùng một dung dịch so sánh.

Theo định luật Bia:

AX = εlCX (dung dịch không thêm lượng chất phân tích)

Aa = εl(Ca + CX) (dung dịch có thêm lượng chất phân tích)

⇒Xa

aXX AA

CAC

−=

Cũng có thể xác định CX bằng phương pháp đường chuẩn khi dùng phương pháp thêm. Tiến hành xây dựng đường chuẩn theo thực nghiệm sau:

Nồng độ C CX CX + C1 CX + C2 CX + C3 ...

Mật độ quang A AX A1 A2 A3 ...

Xác định nồng độ theo phương pháp thêm bằng ngoại suy đồ thị

72

CX 0 C

1 C

2 C

3

A3

A2

A1

AX

Page 67: 1ygm3hwjpgu090312060334 tài liệu bồi dưỡng giáo viên trường thpt chuyên   mon hóa học - phan 1- năm 2012

Phương pháp 2: Lấy cùng lượng dung dịch mẫu nghiên cứu VX chứa chất hấp thụ có nồng độ CX vào hai cuvet như nhau, thêm vào cuvet thứ hai một lượng chính xác thể tích Va chất hấp thụ có nồng độ chính xác Ca. Tiến hành đo mật độ quang của hai dung dịch ở cùng bước sóng chọn lọc (bước sóng tối ưu), so sánh với cùng một dung dịch so sánh.

AX = εlCX (dung dịch không thêm lượng chất phân tích)

Aa = εl

++

aX

aaXX

VV

VCVC (dung dịch có thêm lượng chất phân tích)

⇒ ( )X a a

Xa X a X X a X a X

X X a a a a

A C V 1C

A V V A V A V V VA A C V C V

= =+ − + − ÷

Ưu điểm của phương pháp thêm là loại bỏ được các yếu tố cản trở có mặt

trong mẫu, xác định được hàm lượng nhỏ của chất nghiên cứu trong mẫu.

2.3. Chuẩn độ đo quang

Tiến hành chuẩn độ mẫu trong đó ít nhất có một cấu tử có khả năng hấp thụ năng lượng ánh sáng thì về nguyên tắc có thể sử dụng phương pháp đo quang để xác định điểm cuối chuẩn độ.

Giả sử dùng chất B để chuẩn độ chất A tạo ra chất C theo phương trình phản ứng sau: A + B C. Tuỳ thuộc khả năng hấp thụ ánh sáng ta có các trường hợp sau:

2.3.1. Trường hợp A hấp thụ ánh sáng (εA > 0), B và C không hấp thụ ánh

sáng (εC = εB = 0). Trong quá trình chuẩn độ ta theo dõi nồng độ của chất A ⇒ trong quá trình chuẩn độ giá trị mật độ quang giảm đến tương đương mật độ quang A = 0 sau đó không đổi (hình 2.1).

2.3.2. Trường hợp (εB > 0), (εC = εA = 0): (hình 2.2)

A

VB

CA

Vtđ

A

VB

CB

Vtđ

73

Page 68: 1ygm3hwjpgu090312060334 tài liệu bồi dưỡng giáo viên trường thpt chuyên   mon hóa học - phan 1- năm 2012

Hình 2.1. Chuẩn độ đo quang dung dịch A hấp thụ, B và C không hấp thụ ánh sáng

Hình 2.2. Chuẩn độ đo quang dung dịch B, hấp thụ, A và C không hấp thụ ánh sáng

2.3.3. Trường hợp (εC > 0), (εB = εA = 0): (hình 2.3)

2.3.4. Trường hợp (εA = 0), (0 < εB < εC): (hình 2.4)

Hình 2.3. Chuẩn độ đo quang dung dịch C hấp thụ, A và B không hấp thụ ánh sáng

Hình 2.4. Chuẩn độ đo quang dung dịch B và C hấp thụ, A không hấp thụ ánh sáng

(εC > εB)

2.3.5. Trường hợp (εA = 0), (0 < εC < εB): (hình 2.5)

2.3.6. Trường hợp (εC = 0), (εA >0; εB >0): (hình 2.6)

Hình 2.5. Chuẩn độ đo quang dung dịch B và C hấp thụ, A không hấp thụ ánh sáng

(εC < εB).

Hình 2.6. Chuẩn độ đo quang dung dịch A và B hấp thụ, C không hấp thụ ánh sáng.

3. Nghiên cứu cân bằng tạo phức

3.1. Xác định thành phần phức chất

3.1.1. Phương pháp hệ đồng phân tử (phương pháp biến đổi liên tục hay phương pháp Oxtromưxlenco)

74

A

VB

CB

Vtđ

CC

A

VB

CC

Vtđ

CC=const

A

VB

CB

Vtđ

CC

A

VB

CC

Vtđ

CC=const

CB

Page 69: 1ygm3hwjpgu090312060334 tài liệu bồi dưỡng giáo viên trường thpt chuyên   mon hóa học - phan 1- năm 2012

Dựa trên việc xác định tỉ số các nồng độ phân tử của các chất tác dụng tương ứng với hiệu suất cực đại của phức đặc trưng bởi một cực đại. Giả sử có sự tạo phức:

M + nR →¬ MRn]MR[

]R][M[K

]R][M[

]MR[K

n

n

cbnn

cb =⇒= −1

Gọi tổng nồng độ ban đầu của M và R là m = const, nồng độ này được duy trì nhờ việc chuẩn bị 2 dung dịch M và R có nồng độ gốc bằng nhau và tiến hành pha các dung dịch với các thể tích khác nhau của M và R nhưng tổng VM + VR không đổi. Gọi x là nồng độ phần mol của R và (1 –x) là nồng độ phần mol của M

M + nR →¬ MRn

C m(1 –x) mx

[ ] (m(1 –x) –C3 (mx –nC3) C3

Thay vào phương trình

]MR[

]R][M[K

]R][M[

]MR[K

n

n

cbnn

cb =⇒= −1

Với C1 = [M] = m(1 –x) – C3; C2 = [R] = mx – nC3; C3 = Kcb.C1.C2n là

nồng độ cân bằng của phức MRn. Ta có: C1, C2, C3 là các hàm phụ thuộc vào x. Ta tìm điều kiện để C3 cực đại là khi dC3/dx = 0.

212111

2

1212

12

212

13

32

31

0 CnC)CnC(m.C.C.K

C.m.C.KC).m(KC.dx

dC.nC.KC.

dx

dCK

dx

dC

;mdx

dCnm

dx

dC

;mdx

dCm

dx

dC

ncb

ncb

ncb

ncb

ncb

=⇔=−=

+−=+=

=−=

−=−−=

−−

nC1 = n(m(1 –x) – C3) = (mx – nC3)

⇒ mn – mnx – nC3 = mx – nC3 ⇒ n(1 –x) = x

⇒ n = x/(1 –x) = mx/m(1 –x) = CR/CM.

Phương pháp thực nghiệm: Chuẩn bị các dãy dung dịch có x biến thiên từ 0 – 1 với các tổng nồng độ m khác nhau. Tiến hành đo mật độ quang các dung dịch, xây

75

Page 70: 1ygm3hwjpgu090312060334 tài liệu bồi dưỡng giáo viên trường thpt chuyên   mon hóa học - phan 1- năm 2012

dựng đồ thị sự phụ thuộc mật độ quang vào x/(1 –x). Tại điểm cực đại chính là tỉ lệ tạo phức giữa M và R 1:n =(1 –x):x.

Giới hạn áp dụng: Cho các trường hợp tạo 1 phức bền, M, R không bị phân li hay bị thuỷ phân, cho các phức 1:1, 1:2 hay tối đa là 1:3 không nên áp dụng cho các phức có tỉ lệ cao. Nên thực nghiệm ở nhiều tổng nồng độ khác nhau và ở các bước sóng khác nhau trong điều kiện cố định pH, lực ion để tăng độ tin cậy của kết quả thực nghiệm. Với các phức đa nhân MpRq tỉ số p/q là thập phân khó xác định chính xác.

3.1.2. Phương pháp tỉ số mol

Dựa trên cân bằng: mM + R →¬ MmR Kcb (1)

hay: M + nR →¬ MRn K'cb (2)

Khi tiến hành thực nghiệm với hai dãy thí nghiệm

– Dãy thứ nhất: nồng độ cation kim loại hằng định và nồng độ thuốc thử R thay đổi.

– Dãy thứ hai: nồng độ thuốc thử R hằng định và nồng độ cation kim loại thay đổi.

76

x/(1 –x)

A

CM, C

R

CM hằng định

CR hằng định

A

Agh

A'gh Đồ thị biểu diễn sự phụ thuộc mật độ

quang vào nồng độ của R (khi nồng độ M hằng định), nồng độ của M (khi nồng độ R hằng định)

Page 71: 1ygm3hwjpgu090312060334 tài liệu bồi dưỡng giáo viên trường thpt chuyên   mon hóa học - phan 1- năm 2012

+ Trong dãy thí nghiệm thứ nhất: khi nồng độ R đủ lớn có thể coi toàn bộ M

đã đi vào phức, khi đó nồng độ của phức MmR = MCm

1(cân bằng (1), và MRn =

CM (2) theo định luật Bia:

Agh = m

1εlCM cho cân bằng (1) hay Agh = εlCM cho cân bằng (2).

+ Trong dãy thí nghiệm thứ hai: khi nồng độ M đủ lớn có thể coi toàn bộ R đã

đi vào phức, khi đó nồng độ của phức MmR = CR (cân bằng (1), và MRn = n

1CR

theo định luật Bia:

A'gh = εlCR cho cân bằng (1) hay A'gh = n

1εlCR cho cân bằng (2).

Kết hợp kết quả của hai dãy thí nghiệm ta có:

+ Với cân bằng (1): m = Rgh

Mgh

C.A

C.'A

+ Với cân bằng (2): n = Mgh

Rgh

C.'A

C.A

Cũng có thể xác định thành phần của phức dựa trên đồ thị sự phụ thuộc mật độ quang vào nồng độ của R (cho cân bằng (1)) và nồng độ của M (cho cân bằng (2)).

Phương pháp này thường chỉ áp dụng cho các phức bền.

3.1.3. Phương pháp chuyển dịch cân bằng

Sử dụng phương pháp này để xác định thành phần các phức đơn nhân.

M + nR →¬ MRn β

[ ][ ][ ] n

n

RM

MR=β ⇒

[ ][ ] [ ]RlgnlgM

MRlg n +β=

77

Page 72: 1ygm3hwjpgu090312060334 tài liệu bồi dưỡng giáo viên trường thpt chuyên   mon hóa học - phan 1- năm 2012

Khi nồng độ của M cố định, sự biến đổi mức độ tạo phức của ion M phụ thuộc vào nồng độ cân bằng của cấu tử R. Giả sử có sự phụ thuộc tuyến tính giữa mức độ tạo phức với mật độ quang của dung dịch. Phương trình trên có thể biểu diễn dưới dạng:

[ ][ ] [ ]

[ ][ ] [ ]Rlgnlg

AA

Alg

M

MRlghay

RlgnlgAA

Alg

M

MRlg

gh

n

gh

n

+β=∆−∆

∆=

+β=−

=

Khi nồng độ R đủ lớn có thể coi nồng độ [R] = CR

Tại điểm cắt trục tung của đồ thị ta có thể xác định hằng số bền của phức lgβ.

Phương pháp này có thể áp dụng xác định cho các phức đơn nhân theo từng nấc.

3.1.4. Phương pháp tỉ số các độ dốc (phương pháp Garvay – Menhing)

Xét phản ứng tạo phức: mM + nR →¬ MmRn nmRMβ

Khi tiến hành thực nghiệm với hai dãy thí nghiệm

– Dãy thứ nhất: nồng độ cation kim loại M hằng định lớn hơn đáng kể so với nồng độ thuốc thử R thay đổi.

– Dãy thứ hai: nồng độ thuốc thử R hằng định lớn hơn đáng kể so với nồng độ cation kim loại M thay đổi.

78

CR

CM hằng định

AA

gh

tgα=n

lgβ

CM, C

R

CM hằng định

CR hằng định

A

Agh

A'gh

α α’

Page 73: 1ygm3hwjpgu090312060334 tài liệu bồi dưỡng giáo viên trường thpt chuyên   mon hóa học - phan 1- năm 2012

+ Với dãy thứ nhất khi nồng độ M rất lớn so với R, có thể coi toàn bộ R đã đi

vào phức MmRn, do đó n[MmRn] = CR ⇒ mật độ quang dung dịch

A = [ ]n

CKRMl R

nm =ε

Khi biểu diễn sự phụ thuộc mật độ quang A = f(CR) thì hệ số góc của đồ thị:

tgα = n

K

+ Với dãy thứ nhất khi nồng độ R rất lớn so với M, có thể coi toàn bộ R đã đi

vào phức MmRn, do đó m[MmRn] = CM ⇒ mật độ quang dung dịch

A = [ ]m

CKRMl R

nm =ε

Khi biểu diễn sự phụ thuộc mật độ quang A = f(CR) thì hệ số góc của đồ thị:

tgα’ = m

K

Vì vậy ta có thành phần của phức có thể xác định: n

m

'tg

tg =αα

. Phương pháp

này càng chính xác khi phức tạo ra càng bền.

III. MỘT SỐ BÀI TẬP VẬN DỤNG

Bài 1. Phổ hấp thụ UV –Vis thường được dùng để xác định nồng độ một chất trong dung dịch bằng cách đo mật độ quang (độ hấp thụ quang) tại bước sóng nhất định của ánh sáng vùng UV –Vis. Định luật Beer –Lambert cho biết mật độ quang

tỉ lệ trực tiếp với nồng độ mol/L tại một bước sóng cho trước: A = ε.l.C (ε là độ

hấp thụ mol hay hệ số tắt tính theo L.mol –1.cm –1, l là chiều dài đường truyền quang học qua lớp hấp thụ tính theo cm; C là nồng độ mol của chất hấp thụ); hay

A = o

10l

IA lg

I= (Io và Il lần lượt là cường độ ánh sáng tới lớp hấp thụ và đi ra khỏi

lớp chất hấp thụ).

79

Page 74: 1ygm3hwjpgu090312060334 tài liệu bồi dưỡng giáo viên trường thpt chuyên   mon hóa học - phan 1- năm 2012

Hai cấu tử A và B không hấp thụ năng lượng ánh sáng trong vùng nhìn thấy, nhưng chúng tạo được hợp chất phức màu AB có khả năng hấp thụ cực đại ở bước sóng 550 nm. Người ta chuẩn bị dung dịch X chứa 1.10 –5 M cấu tử A và 1.10 –2 M cấu tử B rồi đem đo mật độ quang của dung dịch thu được so với H2O ở 550 nm thì mật độ quang A = 0,450 với cuvet có bề dày quang học l = 20 mm. Khi chuẩn bị dung dịch Y bằng cách trộn hai thể tích tương đương của dung dịch chứa 6.10 –5 M cấu tử A và dung dịch chứa 7.10 –5 M cấu tử B rồi tiến hành đo quang ở 550 nm so với H2O (l = 2cm) giá trị mật độ quang thu được là A = 1,242.

a. Tính hằng số không bền của phức AB.

b. Tính độ hấp thụ mol của AB tại bước sóng 550 nm.

c. Tính giá trị mật độ quang của dung dịch Z thu được khi trộn các thể tích bằng nhau của các dung dịch A, B có nồng độ 1.10–4 M với cuvet có bề dầy 0,1 dm ở bước sóng 550 nm.

Hướng dẫn giải:

a. Trong dung dịch X chứa A và B nên có cân bằng:

A + B →¬ AB K

Vì CA << CB nên giả sử có sự tạo phức hoàn toàn giữa A và B: [AB] = CA = 10 –5 mol.L –1.

Theo định luật Beer –Lambert: A = εAB.l.[AB] hay εAB.l.[AB] = εAB.l.10 –5 =

0,450 ⇒ εAB.l = 4,5.104.

Trong dung dịch Y nồng độ đầu của A và B lần lượt là 3.10 –5M và 3,5.10 –5M. Trong dung dịch Y có cân bằng:

A + B →¬ AB K

Mật độ quang của dung dịch Y: A = εAB.l.[AB] = 4,5.104[AB] = 1,242 ⇒ [AB] = 2,76.10 –5 mol.L –1.

80

Page 75: 1ygm3hwjpgu090312060334 tài liệu bồi dưỡng giáo viên trường thpt chuyên   mon hóa học - phan 1- năm 2012

Theo định luật tác dụng khối lượng: [ ]

[ ] [ ]AB

KA . B

=

[ ][ ]( ) [ ]( ) ( ) ( )

-5

5 5 5 -5 5 -5

AB 2,76.10 K

3.10 AB 3,5.10 AB 3.10 2,76.10 3,5.10 2,76.10− − − −= =

− − − −

= 1,554.106

KKB = K –1 = 6,435.10 –7.

Kiểm tra giả thiết trong dung dịch X:

Trạng thái giới hạn: AB 1.10 –5 mol.L –1; B (1.10 –2 –1.10 –5) = 9,99.10 –3

mol.L –1; ta có cân bằng:

AB →¬ A + B KKB = 6,435.10 –7

1.10 –5 9,99.10 –3

1.10 –5 – x x 9,99.10 –3 + x

[ ] [ ][ ]

3

KB 5

A . B x(9,99.10 x)K

AB 1.10 x

+= =− = 6,435.10 –7

Giả sử x << 1.10 –5 ⇒ x = 6.441.10 –10 mol.L –1 thoả mãn điều kiện x << 1.10–5.

Vậy giả thiết [AB] = 1.10 –5 mol.L –1 là phù hợp.

b. Ta có εAB.l = εAB.2 = 4,5.104 ⇒ εAB = 2,25.104 L.mol –1.cm –1.

c. Trong dung dịch Z có nồng độ đầu A và B đều là 5.10 –5 mol.L –1.

Trong dung dịch Z có cân bằng: A + B →¬ AB K

Trạng thái giới hạn: AB 5.10 –5 mol.L –1.

Ta có cân bằng:

AB →¬ A + B KKB = 6,435.10 –7

5.10 –5

5.10 –5 – x x x

[ ] [ ][ ]KB 5

A . B x.xK

AB 5.10 x−= =−

= 6,435.10 –7

⇒ x = 5,36.10 –6 mol.L –1. [AB] = 4,464.10 –5

81

Page 76: 1ygm3hwjpgu090312060334 tài liệu bồi dưỡng giáo viên trường thpt chuyên   mon hóa học - phan 1- năm 2012

Mật độ quang của dung dịch Z: A = εAB.l.[AB] = 2,25.104.1.4,434.10 –6 = 1,004.

Bài 2. Phổ hấp thụ phân tử vùng UV –Vis có độ hấp thụ quang (mật độ quang A) phụ thuộc tuyến tính vào nồng độ. Ngoài ra độ hấp thụ quang còn có tính cộng tính trong dung dịch chứa các cấu tử màu không tương tác hóa học với nhay. Hãy thiết lập công thức tính hằng số phân ly của thuốc thử HR và tính hằng số phân ly của HR biết rằng khi đo mật độ quang ở cùng một bước sóng đối với các dung dịch có tổng nồng độ thuốc thử như nhau (Co mol/L) và có pH khác nhau. Kết quả thực nghiệm cho dưới đây:

– pH < 2 thì được A1 = 0,016.

– pH = 7,33 được A2 = 0,442.

– pH > 11 được A3 = 0,655.

Hướng dẫn giải:

Từ cân bằng: HR →¬ H+ + R – Ka

[ ] [ ]H . R R

K hay pK pH lgHR HR

+ − − = = −

Ta đi tìm [H+] và [R –]

– Khi pH < 2: Thì giả sử toàn bộ thuốc thử ở dạng axit HR và nồng độ của nó

tỷ lệ với mật độ quang

11 HR HR HR HR o HR

o

AA = .[HR].l = .C .l = .C .l (1)

C .lε ε ε ⇒ ε =

– Khi pH > 11: Thì giả sử toàn bộ thuốc thử ở dạng bazơ R – và nồng độ của

nó tỷ lệ với mật độ quang:

- - - - -

- 33 oR R R R R

o

AA = [R ].l = .C .l = .C .l (2)

C .l ε ε ε ⇒ ε =

Đo ở pH=7,33 là đo ở pH trung gian trong dung dịch tồn tại ở cả hai dạng cho nên:

82

Page 77: 1ygm3hwjpgu090312060334 tài liệu bồi dưỡng giáo viên trường thpt chuyên   mon hóa học - phan 1- năm 2012

( )

[ ]

-

- -312 HR R

o o

- -2 o 2 1 3

-2 1

3 2

2 1

3 2

8

AAA = [HR].l + [R ].l = [HR].l + [R ].l (3)

C .l C .l

A C A [HR] + [R ] A .[HR] + A .[R ]

A A[R ]

[HR] A A

R A A 0, 442 0,016pK pH lg pH lg 7,33 lg 7,03

HR A A 0,655 0, 442

K 9,35.10

ε ε

⇒ = =

−⇒ =−

− − = − = − = − =− −

=

Bài 3. Hiện nay, trong các phòng thì nghiệm hiện đại thì pH thường được đo bằng điện cực thủy tinh, các trường hợp còn lại thì được đo bằng phương pháp quang học kết hợp với dùng chất chỉ thị. Ví dụ như khi xác định pH trong mẫu nước biển. Vì hàm lượng muối tan trong nước biển là rất cao dẫn đến sai số hệ thống rất lớn khi đo pH bằng điện cực thủy tinh. Thymol xanh (M = 466,59 g·mol –1) là chỉ thị có màu thuộc dạng axit hai lần axit. Nồng độ của dạng không phân li, H2In có thể bỏ qua ở pH của nước biển. Ở 298 K, hằng số phân li nấc thứ 2 của thymol xanh, đã được hiệu chỉnh, là Ka2 = 10–8.09. Hệ số hấp thụ phân tử (ελ) của

HIn– và In2 – tại hai bước sóng (λ) như trong bảng sau:

Ion ε436 (L.mol –1.cm –1) ε596 (L.mol –1.cm –1)

HIn – 13900 44,2

In2 – 1930 33800

Tiến hành đo mẫu nước biển trong tế bào quang học (cuvet) loại 10,00 cm, kết quả như sau:

Hấp thụ tại bước sóng 436 nm

Hấp thụ tại bước sóng 596 nm

Mẫu nước biển 0,052 0,023

Mẫu có thêm chỉ thị thymol xanh 0,651 0,882

Tính pH của mẫu và nồng độ mol/l của thymol xanh trong mẫu. Vì giá trị Ka2

đã được hiệu chỉnh theo nồng độ muối, do vậy có thể bỏ qua hệ số hoạt độ (coi như hệ số hoạt độ bằng 1,000).

Hướng dẫn giải:

Trong dung dịch có cân bằng:

83

Page 78: 1ygm3hwjpgu090312060334 tài liệu bồi dưỡng giáo viên trường thpt chuyên   mon hóa học - phan 1- năm 2012

HIn – →¬ H+ + In2 – Ka2

2 2 2

a 2 a 2 a2

H . In In InK hay pK pH lg pH pK lg

HIn HIn HIn

+ − − −

− − −

= = − ⇔ = +

Có hai phương trình liên hệ theo độ hấp thụ quang theo định luật cộng tính:

Ở bước sóng 436 nm:

A436 = Amẫu – Anước = 0,651 – 0,052 = 10,00[13900(HIn –) + 1930(In2 –)]

Ở bước sóng 596 nm:

A596 = Amẫu – Anước = 0,882 – 0,023 = 10,00[44,2(HIn –) + 33800(In2 –)]

Giải hệ phương trình trên, ta thu được:

(HIn –) = 3,96.10 –6 mol.L –1 và (In2 –) = 2,54. 10 –6 mol.L –1.

Thay các giá trị trên vào phương trình hằng số phân li của axit:

2 6

a 2 6

In 2,54.10pH pK lg 8,090 lg

3,96.10HIn

− −

−−

= + = +

; pH = 7,897.

Nồng độ của chất chỉ thị nên được biểu diễn theo 3 chữ số có nghĩa và pH thì được lấy tới con số thứ 3 sau dấu phẩy.

Bài 4: Người ta nung chảy 0,2000 gam quặng chứa Al2O3 và 5 % nước với Na2CO3,. Sau 1 giờ sẽ tạo ra aluminat natri (NaAlO2), đem hỗn hợp ra cho dung dịch HCl để chuyển aluminat natri hoàn toàn hành thành dung dịch Al3+ và pha loãng đến thể tích 200,00 ml trong bình định mức. Để xác định Al3+ người ta tiến hành như sau:

– Lấy 2,00 ml dung dịch Al3+ để pha chế phức màu (bằng thuốc thử R không

hấp thụ bức xạ điện từ ở bước sóng đang xét), rồi pha loãng đến thể tích 50,00 ml, đo mật độ quang được ở bước sóng hấp thụ cực đại của phức AlR được AX = 0,250.

– Lấy 2,00 ml dung dịch Al3+, cho thêm 5,00 ml AlCl3 5.10 –4 M để pha chế dung dịch mầu, rồi cũng pha loãng đến thể tích 50,00 ml và đo giá trị mật độ quang được: AX+ a= 0,550 (ở cùng bước sóng như trên).

Hãy viết phương trình phản ứng xảy ra và tính hàm lượng % Al trong mẫu khan. Biết phức tạo thành theo tỷ lệ 1:1 và cả 2 dung dịch phức trên được pha chế ở cùng điều kiện để Al3+ tham gia tạo phức hoàn toàn.

84

Page 79: 1ygm3hwjpgu090312060334 tài liệu bồi dưỡng giáo viên trường thpt chuyên   mon hóa học - phan 1- năm 2012

Hướng dẫn giải:

Al2O3 + Na2CO3 →¬ 2NaAlO2 + CO2

NaAlO2 + 4HCl →¬ AlCl3 + NaCl + 2H2O

(Al3+ + R →¬ Al –R)

Phức tạo thành theo tỷ lệ 1:1 CPhức = CAl3+

– Theo định luật Beer:

oX X X X

44o

X+a X X X

2,00.CA = .l.C .l.

50,00

2,00.C 5,00.5.105,00.5.10A = .l. C .l.

50 50

−−

ε = ε

+ε + = ε ÷ ÷

43X+a o

oX o

A 2,00.C 5,00.5.10 0,550C 4,167.10 M

A 2,00.C 0,250

−−+= = ⇒ =

– Khối lượng Al: mAl = MAl.V.Co = 27.0,200.4,167.10 –3 = 0,0225 gam.

– Thành phần % theo khối lượng của nhôm: 11,25%

Bài 5. Một nhà hóa học tiến hành phân tích một hợp kim màu trắng bạc gồm kim loại đồng và niken. Ông cân 3,1422 gam mẫu và hòa tan hoàn toàn vào dung dịch axit nitric đặc trong 4 giờ trong tủ hút. Pha loãng dung dịch thu được thành 100,00 ml trong một bình định mức. Trong quá trình hòa tan có một chất khí màu nâu tạo thành và không có sản phẩm khí khác.

a) Viết phương trình hóa học cho phản ứng hòa tan.

Để xác định thành phần của hợp kim: đầu tiên chuẩn bị dung dịch Na2S2O3

bằng cách hòa tan 6 gam Na2S2O3.5H2O vào 1,0 lít nước. Sau đó cân 0,08590 gam KIO3 hòa tan vào nước pha thành 100,00 ml dung dịch gốc trong bình định mức. Lấy 10,00 ml dung dịch gốc này thêm vào đó 5 ml dung dịch axit clohydric 20% và 2 gam KI rắn. Chuẩn độ mẫu này bằng dung dịch Na2S2O3. Tiến hành song song các thí nghiệm tương tự xác định được thể tích dung dịch Na2S2O3 cần để đạt đến điểm tương đương của phản ứng là 10,46 ml.

b) Viết tất cả các phản ứng xảy ra và xác định nồng độ của dung dịch Na2S2O3; nên sử dụng chỉ thị nào trong phản ứng chuẩn độ?

85

Page 80: 1ygm3hwjpgu090312060334 tài liệu bồi dưỡng giáo viên trường thpt chuyên   mon hóa học - phan 1- năm 2012

Khi rửa dụng cụ thì thấy có một lượng kết tủa trắng xuất hiện ở mẫu thứ nhất. Anh ta nhớ rất rõ rằng anh ta thêm nhiều hơn lượng Na2S2O3 cần thiết so với lượng cần để đạt đến điểm tương đương.

c) Viết phương trình hóa học của phản ứng sinh ra kết tủa.

Thí nghiệm tiếp theo lại dùng dung dịch gốc màu xanh rêu đã chuẩn bị đầu tiên. Lấy 1,000 ml dung dịch này vào trong bình nón, thêm vào đó 20 ml dung dịch axit acetic 5% và 2 gam KI rắn. Đợi khoảng 5 phút. Dung dịch chuyển sang màu nâu và có kết tủa nhạt màu xuất hiện.

d) Viết phương trình hóa học giải thích sự biến đổi màu của dung dịch và sự xuất hiện kết tủa. Tại sao anh ta phải đợi? Tại sao lại là một sai lầm nếu như anh ta đợi lâu hơn?

Sau đó, anh ta chuẩn độ mẫu bằng dung dịch Na2S2O3 thu được. Thể tích cần để đạt đến điểm tương đương là 16,11 ml. Bây giờ anh ta có thể tính được thành phần của mẫu hợp kim.

e) Thành phần phần trăm về khối lượng của mỗi kim loại trong hợp kim là bao nhiêu?

Do là một nhà hóa phân tích giỏi nên ông ta không thoả mãn với kết qủa thu được và cố gắng xác định thành phần của hợp kim bằng phương pháp complexon. Trong phương pháp này ông ta không quan tâm đến kết qủa đã thu được bằng phương pháp iot. Đầu tiên, ông ta hòa tan 3,6811 gam Na2EDTA.2H2O (M = 372,25 g/mol) thành 1,0000 lít dung dịch. Rồi lấy 0,2000 ml dung dịch gốc màu xanh rêu, thêm vào đó 20 ml nước và 2 ml dung dịch amoniăc 25%. Dung dịch chuyển sang màu xanh tím đậm.

f) Tiểu phân nào gây ra màu đó? Mục đích của việc thêm amoniăc là gì?

Trong một vài thí nghiệm song song, tính trung bình, thể tích để đạt đến điểm tương đương là 10,21 ml.

g) Kết quả này liệu có sai lệch so với kết luận trước đó về thành phần của mẫu hợp kim không?

Nhà phân tích vẫn chưa thỏa mãn và cũng nghi ngờ rằng ông cân mẫu sai, vì thế ông lại dùng máy phân tích trắc quang của phòng thí nghiệm. Phòng thí nghiệm mà ông đang làm việc là một phòng thí nghiệm tốt nên ông dễ dàng tìm thấy, điều chế và chuẩn hóa được dung dịch CuCl2 0.1024 mol/lít và dung dịch NiCl2 0,1192 mol/lít. Đầu tiên ông đo phổ hấp thụ của dung dịch CuCl2 bằng cách

86

Page 81: 1ygm3hwjpgu090312060334 tài liệu bồi dưỡng giáo viên trường thpt chuyên   mon hóa học - phan 1- năm 2012

dùng cuvet thạch anh dày 1,000 –cm và lưu ý đến giá trị độ hấp thụ ở một vài bước sóng mà theo ông là phù hợp:

λ (nm) 260 395 720 815

A 0,6847 0,0110 0,9294 1,428

Sau đó ông đo độ hấp thụ của dung dịch NiCl2 ở cùng một bước sóng trong cùng một cuvet:

λ (nm) 260 395 720 815

A 0,0597 0,6695 0,3000 0,1182

Ông pha loãng 5,000 ml dung dịch xanh rêu ban đầu đến 25,00 ml trong một bình định mức và đo độ hấp thụ. Ông đọc được giá trị là 1,061 ở 815 nm và 0,1583 ở 395 nm.

h) Tại sao ông phải pha loãng dung dịch. Nếu chỉ dựa vào những kết qủa của phương pháp phân tích trắc quang thì thành phần của hợp kim như thế nào?

Tiếp theo ông đo độ hấp thụ quang ở 720 nm và nhận được giá trị 0.7405.

i) Giá trị này có phù hợp với những kết luận trước đó không?

Cuối cùng ông chỉnh thiết bị đến 260 nm. Ông rất ngạc nhiên khi đọc được giá trị là 6.000.

j) Ông muốn thấy giá trị bao nhiêu?

Ông quyết định đo độ hấp thụ quang ở bước sóng này trong cuvet thạch anh nhỏ hơn có độ dày 1,00 –mm nhưng ông vẫn nhận được giá trị là 6.000.

k) Giải thích sự xuất hiện giá trị này và hãy tìm một phương pháp để xác nhận điều đó bằng cách sử dụng các hóa chất và dụng cụ đã được nhà hóa học của chúng ta sử dụng.

Hướng dẫn giải:

a) Cu + 4HNO3 = Cu(NO3)2 + 2NO2 + 2H2O

Ni + 4HNO3 = Ni(NO3)2 + 2NO2 + 2H2O

b) IO3 – + 5I – + 6H+ = 3I2 + 3H2O

I2 + I – ⇌ I3 –

I2 + 2S2O32 – = 2I – + S4O6

2 –

I3 – + 2S2O3

2 – = 3I – + S4O62 –

87

Page 82: 1ygm3hwjpgu090312060334 tài liệu bồi dưỡng giáo viên trường thpt chuyên   mon hóa học - phan 1- năm 2012

- -53 -1

0,0895g 10,00mln(IO )= . =4,014.10 mol

214,00g.mol 100,00ml

n(I2) = 3n(IO3 –) = 1,2042.10 –4 mol

n(S2O32 –) = 2n(I2) = 2,4084.10 –4 mol

c = n(S2O32 –) / V(S2O3

2 –) = 2,4084.10 –4 mol / 0,01046 lít = 0,02302 mol/lít. Sử dụng chỉ thị là dung dịch hồ tinh bột.

c) S2O32 – + H+ = HSO3

– + S

d) 2Cu2+ + 4I – = 2CuI + I2

I2 + I – ⇌ I3 –

Phản ứng khử giữa Cu2+ và I – không phải xảy ra ngay lập tức. Ở những điều kiện đã nêu thì phải cần đến 5 phút phản ứng mới hoàn tất. Đợi trong nhiều giờ là cả một sai lầm do I – bị oxy chậm bởi oxy không khí.

e) n(S2O32 –) = c.V = 0,02302 mol/lít.0,01611 lít = 3,7085.10 –4 mol

n(Cu2+) (trong 1,000 ml dung dịch mẫu) = 2n(I2) = n(S2O32 –) = 3,709.10 –4 mol

m(Cu) = 3,709.10 –2 mol = 63,55 g/mol = 2,357 g

Lượng đồng trong mẫu hợp kim là 2,357 / 3,1422 = 75,01% khối lượng.

f) Màu sắc phức amin của cả Cu2+ và Ni2+ đều là màu xanh tím (Thực ra màu sắc của phức Cu2+ đậm hơn). Amoniac cần để chỉnh pH về một giá trị thích hợp để sự tạo phức với EDTA xảy ra hoàn toàn và không tạo kết tủa hidroxit của Cu2+ và Ni2+.

g) -4

-1

3,6811g 10,21mln(EDTA)= . =1,010.10 mol

372,25g.mol 1000,00mcl

n(Cu) + n(Ni) = 1,010.10 –4 mol. 100,0 ml/0,2000 ml = 0,05048 mol

Từ khối lượng của mẫu hợp kim:

M(Cu).n(Cu) + M(Ni).n(Ni) = 3,1422g

Giải phương trình đối với n(Cu) và n(Ni) ta được

n(Ni) = 0,0136 mol ⇒ m(Ni) = 0,796g

n(Cu) = 0,0369 mol ⇒ m(Cu) = 2,35g

Kết quả này khớp với kết quả mà trước đó đã được tính theo phương pháp chuẩn độ iot.

88

Page 83: 1ygm3hwjpgu090312060334 tài liệu bồi dưỡng giáo viên trường thpt chuyên   mon hóa học - phan 1- năm 2012

h) Dung dịch mẫu chưa pha loãng sẽ cho giá trị độ hấp thụ quang lớn hơn 2,0; vốn không thể dễ dàng xác định.

Hệ số hấp thụ mol đối với Cu2+ và Ni2+:

Bước sóng (nm) 260 395 720 815

2Cu +λε (lít.mol –1.cm –1) 0,6847

= 6,6870,1024 .1,000

0,107 9,076 13,95

2Ni +λε (lít.mol –1.cm –1) 0,501 5,617 2,517 0,9916

Nồng độ của dung dịch mẫu đã pha loãng có thể được xác định bằng cách giải hệ phương trình tương ứng với các giá trị đo ở các bước sóng khác nhau theo định luật cộng tính:

A (815 nm) = 13,95.1,000.CCu + 0,9916.1,000.CNi = 1,061

A (395 nm) = 0,107.1,000.CCu + 0,9916.1,000.CNi = 0,1583

CCu = 0,07418 mol/lít; CNi = 0,02677 mol/lít đối với dung dịch loãng.

Nồng độ của dung dịch mẫu ban đầu lớn gấp 5 lần: CCu = 0,3709 mol/lít;

CNi = 0,1338 mol/lít

Đối với toàn bộ thể tích của dung dịch mẫu (100,0 ml): nCu = 0,03709 mol;

nNi = 0,01338 mol.

Kết quả này hoàn toàn khớp với kết quả chuẩn độ.

i) Giá trị của độ hấp thụ ở 720 nm sẽ là:

A (720 nm) = 9,076.1,000.CCu + 2,217.1,000.CNi = 0,7404

Kết quả này hoàn toàn khớp với giá trị đã tính ở trên.

j) Giá trị của độ hấp thụ ở 260 nm

A (260 nm) = (ε(260 nm, Cu).c(Cu) + ε(260 nm, Ni).c(Ni)).1,000 cm =

0,5093

Kết quả này không khớp với kết quả thực nghiệm.

k) Máy phổ kế trắc quang đọc giá trị là 6,000 có nghĩa là thực tế không hề có

ánh sáng đi xuyên qua mẫu. Điều này vẫn không thay đổi khi ta dùng cuvet có độ

dài nhỏ hơn.

Độ hấp thụ mol của Cu2+ và Ni2+ được đo bằng cách sử dụng hai dung dịch

CuCl2 và NiCl2. Axit nitric được sử dụng để hoà tan đồng xu nên nồng độ của

89

Page 84: 1ygm3hwjpgu090312060334 tài liệu bồi dưỡng giáo viên trường thpt chuyên   mon hóa học - phan 1- năm 2012

anion nitrat sẽ rất cao trong dung dịch mẫu. Quan sát thực nghiệm có thể được

giải thích nếu như ion nitrat hấp thụ ở 260 nm. Điều này có thể được xác nhận

bằng cách ghi phổ UV –Vis của một mẫu axit nitric

B. PHƯƠNG PHÁP PHÂN TÍCH ĐIỆN HOÁ

I. CƠ SỞ LÝ THUYẾT

1. Tế bào điện hoá (ô điện hoá)

Là một hệ gồm 2 điện cực là hai vật dẫn điện loại một (vật dẫn điện electron:

dây dẫn) nhúng vào một hay hai dung dịch điện li hoặc chất điện li nóng chảy (vật

dẫn loại 2: dẫn điện nhờ ion).

90

Page 85: 1ygm3hwjpgu090312060334 tài liệu bồi dưỡng giáo viên trường thpt chuyên   mon hóa học - phan 1- năm 2012

∗ Phân loại tế bào điện hoá: 2 loại

– Tế bào Galvani (hay ô Galvani):

Những tế bào điện hoá sinh ra dòng điện nhờ phản ứng oxi hoá – khử tự phát xảy ra trong đó (hoá năng biến thành điện năng). Khi đó phản ứng hoá học trong tế bào có ∆G < 0 và E > 0.

+ Tại catot xảy ra quá trình khử: Ox1 + n1e → Kh1 đóng vai trò cực dương có thế điện cực là E1.

+ Tại anot xảy ra quá trình oxi hóa: Kh2 → Ox2 + n2e đóng vai trò cực âm có thế điện cực là E2.

Sức điện động của tế bào Galvani: E = E1 – E2

– Tế bào điện phân:

Là những tế bào điện hoá trong đó xảy ra quá trình oxi hoá – khử cưỡng bức dưới tác động của nguồn điện ngoài. Khi đó phản ứng trong tế bào điện hoá xảy ra kể cả các phản ứng có ∆G > 0 và E < 0.

+ Tại catot xảy ra quá trình khử: Ox1 + n1e → Kh1 đóng vai trò cực âm có thế điện cực là E1. Điều kiện để phản ứng xảy ra trên catot: EC < E1

+ Tại anot xảy ra quá trình oxi hóa: Kh2 → Ox2 + n2e đóng vai trò cực dương có thế điện cực là E2. Điều kiện để phản ứng xảy ra trên anot: EA > E2.

Vì vậy thế cần đặt lên hệ điện phân: U = EA – EC ≥ E2 – E1

2. Điện cực

2.1. Điện cực so sánh

Tiêu chuẩn của một điện cực so sánh:

– Phản ứng quyết định thế điện cực phải hoàn toàn thuận nghịch.

– Điện cực phải rất ít bị phân cực, nghĩa là rất ít bị thay đổi khi có dòng điện đi qua.

– Phải có độ lặp lại cao và có thế ổn định khi bảo quản lâu dài cũng như khi làm việc trong các điều kiện khác nhau.

2.1.1. Điện cực calomen

Cấu tạo: Hg | Hg2Cl2 | KCl

Phản ứng điện cực: Hg2Cl2 + 2e →¬ 2Hg + 2Cl –

91

Page 86: 1ygm3hwjpgu090312060334 tài liệu bồi dưỡng giáo viên trường thpt chuyên   mon hóa học - phan 1- năm 2012

Thế điện cực phụ thuộc vào nồng độ Cl –: ( )2 2

oCal Hg Cl 2

2Hg

RT 1E E ln

2F Cl−= +

Điện cực calomen bão hoà (SCE: Saturated Calomen Electrode) ở 250C có ESCE = 0,242 V.

Sơ đồ xác định thế điện cực của điện cực calomen bão hoà:

Pt (H2 p =1atm)| H+ 1 M || KCl bão hoà | Hg2Cl2 | Hg

2.1.2. Điện cực Bạc – Bạc clorua

Cấu tạo: Ag | AgCl | KCl

Phản ứng điện cực: AgCl + e →¬ Ag + Cl –

Thế điện cực phụ thuộc vào nồng độ Cl –:

oAgCl

Ag

spo oAgCl AgCl sp

Ag Ag

o oAgCl spAg

AgAg

RT 1E E ln

F (Cl )

KRT RT RT 1E ln E ln K ln

F F F(Ag ) (Ag )

RTE E ln K

F+

+ +

= +

= − = − −

⇒ = −

2.2. Điện cực chỉ thị

Điện cực chỉ thị là điện cực mà thế của nó trực tiếp hay gián tiếp phụ thuộc vào nồng độ chất nghiên cứu.

Các yêu cầu của điện cực chỉ thị:

+ Thế điện cực phải lặp lại và thiết lập nhanh, điện cực phải làm việc thuận nghịch với chất nghiên cứu.

+ Điện cực phải có độ bền hoá học để điện cực không tác dụng với các cấu tử khác trong dung dịch nghiên cứu.

92

Page 87: 1ygm3hwjpgu090312060334 tài liệu bồi dưỡng giáo viên trường thpt chuyên   mon hóa học - phan 1- năm 2012

Các loại điện cực chỉ thị:

+ Điện cực trơ.

+ Điện cực kim loại.

+ Điện cực màng chọn lọc ion.

2.2.1. Điện cực hiđro (Thuộc loại điện cực trơ)

Phản ứng điện cực: H2 → 2H+ + 2e

Cấu tạo: Pt, H2 (p atm) | H+ a M ||

– Bản Pt mạ bột Pt (đen Pt) có diện tích bề mặt 1cm2 hấp thụ H2 bão hòa nhúng trong dung dịch axit (thường là HCl nồng độ 1 mol/l).

– Luồng khí H2 có áp suất không đổi (thường là 1 atm).

22 2 2

2

22o o

H2H 2HH H H

2o2H

H

HRT RTE E ln ;Khi p 1 E E ln H ;

2F p 2F

RT 2,303RTKhi H 1 E E 0; Khi H 1 E ln H pH

2F F

+ +

+

++

+ + +

= + = ⇒ = +

= ⇒ = = ≠ ⇒ = = −

Phương trình Nernst:

2.2.2. Điện cực quinhiđron

Quinhiđron là hỗn hợp đồng phân tử của Quinon và Hiđroquinon ít tan trong nước phủ trên bản cực Pt.

Phản ứng điện cực: Q + 2H+ + 2e →¬ H2Q

93

Page 88: 1ygm3hwjpgu090312060334 tài liệu bồi dưỡng giáo viên trường thpt chuyên   mon hóa học - phan 1- năm 2012

[ ][ ]

[ ][ ]2 2

2

2

o ' o'Q Q Q Q

H Q H Q2 2

o 'Q Q

H Q

Q H QRT RTE E ln ;Khi 1 E E ln H ;

2F H Q H Q F

2,303RTE E pH

F

++

= + = ⇒ = +

= −

Điện cực quinhiđron làm việc thuận nghịch với ion H+ với Eo = 0,699 V ở 25oC.

Nhược điểm:

– Chỉ làm việc trong môi trường axit hay trung tính, khi pH > 8 thì H2Q phân ly nên không làm việc thuận nghịch.

– Độ chính xác giảm khi làm việc trong môi trường lực ion cao.

2.2.3. Điện cực kim loại

a. Điện cực kim loại loại một:

– Dùng dây hay bản kim loại nhúng trong dung dịch muối tan của kim loại đó. Ag, Hg, Cd... là các điện cực thuận nghịch và lặp lại.

Mn+ + ne → M

M | Mn+ a M ||

Phương trình Nernst: n no n

M MM M

RTE E ln M

nF+ +

+ = +

– Với nhiều điện cực, độ lặp lại tốt khi dùng điện cực hỗn hống kim loại thay cho kim loại tinh khiết. Trong các loại điện cực chỉ thị thì loại điện cực đo thế oxi hoá – khử có vị trí đặc biệt (Pt, Au, Ir hay graphit). Thế điện cực của loại điện cực này phụ thuộc vào tỉ lệ nồng độ dạng oxi hoá và dạng khử của cặp oxi hoá – khử.

Cd(Hg) | Cd2+ a M ||

Phương trình Nernst: [ ]2 2

2o

Cd CdCd(Hg) Cd(Hg)

CdRTE E ln

2F Cd(Hg)+ +

+ = +

b. Điện cực kim loại loại hai: Các bản hay dây kim loại có phủ bên ngoài một lớp muối ít tan của kim loại đó và được nhúng vào dung dịch muối chứa anion có trong muối ít tan phủ trên kim loại (điện cực calomen, điện cực Ag/AgCl...).

94

Page 89: 1ygm3hwjpgu090312060334 tài liệu bồi dưỡng giáo viên trường thpt chuyên   mon hóa học - phan 1- năm 2012

2.2.3. Điện cực màng chọn lọc ion

Vật liệu chế tạo điện cực là màng chất rắn hay màng chất lỏng có chứa ion cần xác định. Điện cực này làm việc không phải do phản ứng điện hoá có sự vận chuyển ion mà do hiệu số điện thế xuất hiện trên bề mặt ngăn cách pha và sự trao đổi cân bằng giữa màng và dung dịch (điện cực thuỷ tinh đo pH là điện cực điển hình của loại này).

Trong đó quan trọng nhất là điện cực thủy tinh.

a) Cấu tạo điện cực thủy tinh:

Ag | AgCl | HCl 0,1 M || màng thủy tinh || KCl bão hòa | Hg2Cl2 | Hg

Đặc trưng của điện cực thủy tinh là chức phận hiđro:

– Bầu thủy tinh (màng thủy tinh) bị hiđrat hóa.

– Trao đổi ion:

H+DD + Me+

TT →¬ H+TT + Me+

DD

H+TT + Me+

DD →¬ H+DD + Me+

TT

– Tùy theo chất liệu thủy tinh và nhiệt độ mà hằng số trao đổi:

( ) ( )( ) ( )

DD TT

TT DD

H MeK

H Me

+ +

+ +=

– Thế điện cực: ( ) ( )o'TT TT DD DD

RTE E ln H K Me

F+ + = + +

95

Page 90: 1ygm3hwjpgu090312060334 tài liệu bồi dưỡng giáo viên trường thpt chuyên   mon hóa học - phan 1- năm 2012

Trong điều kiện nhiệt độ, chất liệu thủy tinh đồng nhất, nồng độ cation Me+ trong

dung dịch nhỏ, khi đó có thể chấp nhận gần đúng: ( )o'TT TT DD

RTE E ln H

F+= +

* Trong những năm gần đây xuất hiện nhiều loại điện cực màng chọn lọc để xác định hoạt độ hoặc nồng độ các cation (Na+, K+, Ca2+, Mg2+, Zn2+...) và các anion (Cl –, Br –, I –, F –, NO3

–, S2 –...).

Ta có: sample sample

internalsolution internalsolution

a aRTΔG = -nFE = -RTln E = ln

a nF a⇒

II. BÀI TẬP ỨNG DỤNG

Câu 1: Tính toán sai số tương đối sẽ mắc phải khi xác định nồng độ ion H+

trong dung dịch NaOH 10 –2 M bằng phương pháp sử dụng điện cực thủy tinh pH. Biết hằng số trao đổi KH,Na = 10 –10; giả sử hệ số hoạt độ của tất cả các ion đều bằng 1.

Hướng dẫn giải:

– Trong dung dịch có các quá trình:

NaOH → Na+ + OH –

H2O →¬ H+ + OH –

– Nồng độ của ion Na+ và OH – trong dung dịch: [Na+] = 10 –2 M;

[OH –] = 10 –2M, vì vậy nồng độ H+ trong dung dịch: 12WK

H 10 MOH

+ −−

= =

– Khi xác định bằng điện cực thủy tinh thường sử dụng phương trình liên hệ:

96

Page 91: 1ygm3hwjpgu090312060334 tài liệu bồi dưỡng giáo viên trường thpt chuyên   mon hóa học - phan 1- năm 2012

( )o'TT TT DD

RTE E ln H

F+= +

Trong khi thực tế sự phụ thuộc của thế theo phương trình:

( ) ( )o'TT TT DD DD

RTE E ln H K Me

F+ + = + +

Vậy sai số tương đối khi xác định bằng điện cực thủy tinh:

( ) ( ) ( )( )

12 10 2 12

DD DD DD12

DD

H K Me H 10 10 .10 10q .100% .100% 100%

10H

+ + + − − − −

−+

+ − + − = = =

Câu 2. Nồng độ của ion Ca2+ trong mẫu nước biển được xác định bằng điện cực màng chọn lọc ion bằng phương pháp thêm chuẩn. 10 ml dung dịch mẫu được chuyển vào bình định mức 100,0ml và pha loãng tới vạch. Lấy 50,0 ml dung dịch trên cho vào cốc được nhúng điện cực chọn lọc ion Ca2+ và điện cực so sánh, thế đo được là –0,05290V. Thêm 1,00 ml dung dịch Ca2+ nồng độ 5,00.10 –2 M vào cốc. Thế đo được lúc này là –0,04417 V. Giải thích quá trình thực nghiệm, thiết lập phương trình tính nồng độ Ca2+ và tính nồng độ Ca2+ trong mẫu nước biển. Tính sai số có thể mắc phải nếu trong dung dịch có chứa Mg2+ nồng độ 10 –4M.

Biết 2 2Ca / Mgk + + = 0,032.

Hướng dẫn giải

Thế của hai phép đo được biểu diễn theo phương trình Nernst:

( )Cell XX

0,05916E K lg C 0,05290V

2= + = −

và ( ) X X S SCell S

T

V C V .C0,05916E K lg 0,04417V

2 V

+= + = − ÷

;

ở đây VT = VX + VS = 50,00 + 1,00 = 51,00 ml

Ta có:

( ) ( ) X X S SCell Cell XS X

T

V C V .C0,05916E E E lg lg C 0,00873V

2 V

+∆ = − = − = ÷

X X S S S SX

T X T T X

V C V .C V .CV2 Elg lg

0,05916 V .C V V .C

+∆⇒ = = + ÷ ÷

97

Page 92: 1ygm3hwjpgu090312060334 tài liệu bồi dưỡng giáo viên trường thpt chuyên   mon hóa học - phan 1- năm 2012

Thay các giá trị: VT = 51,00 ml; VX = 50,00 ml; VS = 1,00 ml thu được: CX = 9,88.10 –4 mol/L.

Nồng độ ban đầu của Ca2+ trong mẫu: C = 10CX = 9,88.10 –3 mol/L.

Nếu trong dung dịch có mặt Mg2+ khi đó phương trình Nernst có dạng:

( ) ( )Cell X Ca,Mg MgX

0,05916E K lg C k .C 0,05290V

2= + + = − ;

( ) X MgX X S SCell Ca,MgS

T T

V CV C V .C0,05916E K lg k . 0,04417V

2 V V

+= + + = − ÷

( ) ( )Cell CellS XE E E∆ = − =

( )X MgX X S SCa,Mg X Ca,Mg Mg

T T

V CV C V .C0,05916lg k . lg C k .C 0,00873V

2 V V

+= + − + = ÷

X X S S Ca,Mg X Mg

T X Ca,Mg T Mg

V C V .C k V C2 Elg

0,05916 V .C k V C

+ +∆⇒ = ÷ ÷+

Thay các giá trị tính được CX = 9,8447.10 –4 mol/L hay nồng độ Ca2+ trong mẫu là 9,8447.10 –3 mol/L.

Sai số tương đối mắc phải là: ( )4 4

4

9,88.10 9,8447.10q .100% 0,36%

9,8447.10

− −

−= =

Câu 3. Trong quá trình điện phân do một số nguyên nhân nên xảy ra hiện tượng quá thế ở các điện cực. Khái niệm quá thế được định nghĩa là lượng thế cần đặt thêm vào điện cực so với lí thuyết để phản ứng điện cực có thể xảy ra; quá thế phụ thuộc vào bản chất chất tham gia phản ứng điện cực, bản chất điện cực, mật độ dòng điện...

Cho dòng điện đi qua dung dịch điện phân gồm: Zn(ClO4)2 0,10 M và Pb(ClO4)2 0,01 M trong dung dịch đệm có pH= 4. Dùng hai điện cực platin phẳng.

– Viết các phương trình phản ứng xảy ra ở hai điện cực, dự đoán hiện tượng xảy ra.

– Nếu kết thúc điện phân khi [ Pb2+] = 10 – 4 ion –g/L thì điện áp tác dụng lên hai điện cực phải bằng bao nhiêu? (coi điện trở của bình điện phân và quá thế không thay đổi trong quá trình điện phân).

98

Page 93: 1ygm3hwjpgu090312060334 tài liệu bồi dưỡng giáo viên trường thpt chuyên   mon hóa học - phan 1- năm 2012

– Tính xem khi khí H2 thoát ra thì chì đã tách ra hoàn toàn chưa? Tại thời điểm này, thì chì đã tách ra được bao nhiêu %?

– Tính thời gian điện phân, khi ngừng điện phân ở catot, lượng chì tách ra được 0,414 g.

Cho biết: Pb =207; độ giảm thế của bình điện phân là 0,35v; I = 0,2 A và ở 25 0C:

2

o

PbPb

E + = –0,130V; 2

o

ZnZn

E + = –0,760V; 2

2

o

O ,HH O

E + = 1,230 V; 2

o

2HH

E + = 0V;

Các giá trị quá thế:

Pb(Pt )η = 0,0005V; Zn(Pt )η = 0,00085V; 2H (Pt )η = 0,197V;

2O (Pt )η = 0,470V.

Hướng dẫn giải

a. Viết các phản ứng xảy ra ở điện cực:

* Ở catot (–):

Zn2+ + 2e = Zn (1) Pb2+ + 2e = Pb (2) 2H+ + 2e = H2 (3)

Để quá trình (1) xảy ra thế đặt trên catot phải thỏa mãn:

2 2 2 2

o 2C Zn Zn Zn Zn

Zn Zn

0,0592E E E lg Zn

2+ + + +

+ ≤ + η = + + η =

0,7896 0,00085 0,79045V= − − = −

Để quá trình (2) xảy ra thế đặt trên catot phải thỏa mãn:

2 2 2 2

o 2C Pb Pb Pb Pb

Pb Pb

0,0592E E E lg Pb

2+ + + +

+ ≤ + η = + + η =

0,1892 0,0005 0,1897V= − − = −

Để quá trình (3) xảy ra thế đặt trên catot phải thỏa mãn:

2 22 2

o 2C H H2H 2H

H H

E E E 0,0592lg H 0,2368 0,197 0,4338V+ ++ ≤ + η = + + η = − − = −

Thứ tự bị khử ở catot: Pb2+; H+ và Zn2+

Vậy ở catot xảy ra phản ứng 1 khử Pb2+ thành Pb kim loại.

Vậy ở catot (–) xảy ra sự điện phân: Pb2+ + 2e = Pb

* Ở anot (+) xảy ra phản ứng điện phân nước.:

99

Page 94: 1ygm3hwjpgu090312060334 tài liệu bồi dưỡng giáo viên trường thpt chuyên   mon hóa học - phan 1- năm 2012

2H2O – 4e = O2 + 4H+

2 22 2

2 2

4oA O OO ,H O ,H

H O H O

0,0592E E lg p H 0,9932 0,47 1,4632V

4E + +

+ ≥ + η = + = + =

b. Điện áp tối thiểu phải đặt vào hai điện cực (Coi điện trở của bình điện phân và quá thế không đổi) khi điện phân còn [Pb2+]= 10 –4 ion –g/l:

2 2 2 2

o 2C Pb Pb Pb Pb

Pb Pb

0,0592E E E lg Pb 0,2484 0,0005 0, 2489V

2+ + + +

+ ≤ + η = + + η = − − = −

Trong dung dịch đệm nên pH không đổi, thế đặt vào anot không đổi: 1,464V

⇒ Điện áp tối thiểu cần đặt: E = EA – EC +IR = 1,4632 – (–0,2489) + 0,35 =

2,0621 V

c. Khi có khí hydro thoát ra thì chì đã kết tủa hoàn toàn hay chưa:

Khi đó:

2 2 2 22

2

o 2H2H Pb Pb Pb Pb

H Pb Pb

0,0592E 0,4338 E E lg Pb

2+ + + + +

+ + η = − = + η = + + η

[Pb2+] =10 –6,59. Vậy khí có khí hydro thoát ra thì chì coi như bị điện phân hết.

Lượng Pb đã tách ra: (10 –2 –10 –6,59)/10 –2.100% = 99,9975%.

d. Tính thời gian điện phân:m .n.FM I.t 0,414.2.96500Pbm = x ® t= = =1930 giayPb n F M.I 207.0,2

Câu 4. Axit cianhidric là một axit yếu có hằng số phân li Ka = 4.93×10–10.

Biết CN – có khả năng tạo phức bền với nhiều cation kim loại, trong đó có Ni2+. Tính cân bằng trong dung dịch mạ điện chứa Ni(ClO4)2 0,010M và KCN 1,00 M và tính điện áp cần áp lên catot để quá trình mạ điện được thực hiện. Biết bình

điện phân có catot là vật cần mạ và anot là Ni kim loại; 24Ni(CN)

lg −β = 30,22; 2

o

NiNi

E +

= –0,257 V.

Người ta mạ niken lên mẫu vật kim loại bằng phương pháp mạ điện trong bể mạ chứa dung dịch cho ở 7. Điện áp được đặt lên các điện cực của bể mạ là 2,5 V. Cần mạ 10 mẫu vật kim loại hình trụ; mỗi mẫu có bán kính 2,5cm, cao 20cm. Người ta phủ lên mỗi mẫu một lớp niken dày 0,4mm. Hãy tính điện năng (theo KWh) phải tiêu thụ.

100

Page 95: 1ygm3hwjpgu090312060334 tài liệu bồi dưỡng giáo viên trường thpt chuyên   mon hóa học - phan 1- năm 2012

Cho biết: Niken có khối lượng riêng D = 8,9 g/cm3; khối lượng mol nguyên tử là 58,7(g/mol): hiệu suất dòng bằng 90%; 1KWh = 3,6.106J.

Hướng dẫn giải:

– Trong bình điện phân (bể mạ) có các quá trình:

Ni2+ + 4CN – →¬ Ni(CN)42 –

4β = 1030,22

0,01 1,00

– 0,96 0,01

4β lớn nên coi phản ứng tạo phức hoàn toàn và nồng độ CN – lớn nên quyết

định môi trường:

CN – + H2O →¬ HCN + OH – KB = 2,03.10 –5

Tính theo cân bằng được: [OH –] = [HCN] = 4,40.10 –3 M; [CN –] = 0,9556 M

[Ni2+] = 0,01. 2Ni +α = 4

4

10,01.

1 . CN− + β = 7,23.10 –33 << 0,01.

Trong dung dịch chứa: Ni(CN)42 – 0,01 M.

Phản ứng điện cực để xảy ra quá trình mạ:

Tại catot: Ni(CN)42 – + 2e →¬ Ni + 4CN –

Tại anot: Ni + 4CN – →¬ Ni(CN)42 – + 2e

Nên thành phần dung dịch không đổi.

Vậy

2

o 2 33C Ni

Ni

0,0592 0,0592E E lg Ni 0,227 lg 7,23.10 1,117V

2 2+

+ − ≤ + = − + = −

b) Thể tích của 1 mẫu vật kim loại hình trụ là:

V = πr2h = 3,14 × (2,5)2 × 20 = 392,5 (cm3).

Lớp phủ niken ở mỗi mẫu vật có bề dày 0,4 mm nên ở mỗi mẫu vật này bán kính

tăng tới 2,5 + 0,04 = 2,54 (cm); chiều cao tăng tới 20,0 + (0,04×2) = 20,08 (cm).

Vậy thể tích của mỗi mẫu vật này tăng thêm một lượng là:

∆V = V '− V = [ 3,14. (2,54)2. 20,08] – 392,5 ∆V = 14,281(cm3)

101

Page 96: 1ygm3hwjpgu090312060334 tài liệu bồi dưỡng giáo viên trường thpt chuyên   mon hóa học - phan 1- năm 2012

Tổng số thể tích tăng thêm cuả cả 10 mẫu vật là:

V = 10 ∆V = 10 × 14,281cm3 = 142,81 cm3. Đây cũng chính là thể tích

niken phải phủ lên 10 mẫu vật cần mạ; khối lượng tương ứng là:

m = V.D =142,81.8,9 = 1271,01 (gam) hay 1271,01

58,7= 21,6526 (mol)

Từ biểu thức của định luật Farađay:

m = 96500

AIt

n⇒ It = (m/A).96500n (1)

Số điện năng tương ứng là: w = ItU = m

A× 96500n.U (2)

Với Ni ta có n = 2; theo trên đã có m

A = 21,6526 (mol); theo đề bài U = 2,5 V.

Thế các trị số này vào (2), ta có w = 21,6526.96500.2.2,5 = 10447379,5 (J)

Vì hiệu suất dòng điện là 90% và 1 kWh = 3,6.106J nên số điện năng thực tế

cần dùng là: W = 90W × 100 × 6

1

3,6 10× =

90

W × 100 × 6

1

3,6 10×⇒ W = 3,2245kWh.

Câu 5. Điện phân hỗn hợp dung dịch X chứa Cu(NO3)2 0,10 mol/L, Cd(NO3)2

0,10 mol/L.

1. Axit hóa dung dịch bằng HCl đến nồng độ 1,00 mol/L được dung dịch Z, giả sử thể tích dung dịch không đổi khi axit hóa. Có thể tách hoàn toàn Cu2+ khi có Cd2+ trong dung dịch Z bằng cách điện phân dung dịch Z trong bình điện phân với hai điện cực platin phẳng, được không?

2. Hòa tan 6,5 gam KCN vào 100ml dung dịch X (coi thể tích không đổi khi hòa tan KCN vào dung dịch) thu được dung dịch Y. Tiến hành điện phân dung dịch Y trong bình điện phân với hai điện cực platin phẳng. Tính nồng độ cân bằng trong dung dịch khi thế đặt trên catot lúc này là –1,00 V.

Các quá trình đều thực hiện ở 25oC.

Cho:

2

o

CuCu

E + = 0,337V; 2

o

CuCu

E ++

= 0,521V; 2

o

CdCd

E + = –0,402V; 2

o

(CN) ,2H2HCN

E + = 0,370V;

102

Page 97: 1ygm3hwjpgu090312060334 tài liệu bồi dưỡng giáo viên trường thpt chuyên   mon hóa học - phan 1- năm 2012

2-

oCl

2Cl

E 1,360V= ; 2

2

o

O ,HH O

E + = 1,230V; 2

o

2HH

E + = 0,00V; 2O (Pt )η = 0,470V;

2H (Pt)η = 0,197 V;

Ksp(CuCN) = 3,5·10 –19; KHCN = 10 –9,31

2Cu(CN)lg −β =24,0; 2

3Cu(CN)lg −β = 28,59; 3

4Cu(CN)lg −β = 30,29;

2Cu(CN)lgβ = 25,0.

Cd(CN)lg +β = 6,01;

2Cd(CN)lgβ = 11,12;

3Cd(CN)lg −β = 15,65; 2

4Cd(CN)lg −β = 17,92.

Hướng dẫn giải:

1. Dung dịch Z: Cu(NO3)2 0,10 mol/L, Cd(NO3)2 0,10 mol/L và HCl 1,0 M.

Ta có nồng độ các ion trong dung dịch: Cu2+ 0,10 M; Cd2+ 0,10 M; H+ 1,0 M; Cl – 1,0 M.

Các phản ứng điện phân:

+ Tại catot điều kiện xảy ra các quá trình điện phân:

2+ 2+

2+ 2+

+ +2 2

2 2

2+ o 2C Cu Cu

Cu Cu

2+ o 2C Cd Cd

Cd Cd

o2 C H H2H 2H

H H

0,0592Cu +2e Cu E E E lg Cu 0,3074V

20,0592

Cd +2e Cd E E E lg Cd 0,4316V2

0,05922H +2e H E E E lg H 0,197V

1

+

+

+ +

→ ≤ = + =

→ ≤ = + = −

→ ≤ + η = + + η = −

Thứ tự điện phân là Cu2+; H+ và cuối cùng là Cd2+.

+ Tại anot:

+ +2 22 2

2 2

2 2

o2 2 A O O4H ;O 4H ;O

2H O 2H O

o2 A Cl Cl

2Cl 2Cl

0,0592O 4e 4H 2H O E E E lg H 1,700V

1

0,0592 1Cl 2e 2Cl E E E lg 1,360V

2 Cl− −

+ +

−−−

+ + → ≥ + η = + + η =

+ → ≥ = + =

xảy ra quá trình điện phân Cl –.

– Khi điện phân Cu2+:

Cu2+ + 2Cl – → Cu + Cl2

Do đó nồng độ H+ không thay đổi.

– Khi bắt đầu điện phân H+ thì nồng độ của Cu2+ còn lại là:

103

Page 98: 1ygm3hwjpgu090312060334 tài liệu bồi dưỡng giáo viên trường thpt chuyên   mon hóa học - phan 1- năm 2012

2+ 2+

o 2

Cu CuCu Cu

0,0592E E lg Cu 0,197V

2+ = + = − ⇒ [Cu2+] = 10 –18,04 M. Nên

khi chưa điện phân đến Cd2+ thì Cu2+ đã bị điện phân hoàn toàn.

2. Nồng độ CN – trong dung dịch khi hòa tan KCN là 1,00 M rất lớn so với nồng độ của Cu2+ và Cd2+. Mặt khác Cd2+ có khả năng tạo phức bền với CN – và có

4 3 2 1; ;β >> β β β nên giả sử Cd2+ tạo phức Cd(CN)42 – là chủ yếu. Đối với Cu2+ là

chất oxi hóa, trong khi CN – có tính khử và Cu+ tạo phức bền hơn Cu2+ nên có thể

oxi hóa CN – tạo Cu+; mặt khác 4 3 2;β >> β β nên chủ yếu tạo phức Cu(CN)43 –.

Các quá trình trong dung dịch:

Cd2+ + iCN – →¬ Cd(CN)i2 –i βi

(1)

Cu2+ + e → Cu+ (2)

Cu2+ + 2CN – →¬ Cu(CN)2 (3)

Cu+ + 4CN – →¬ Cu(CN)43 – (4)

HCN →¬ H+ + CN – Ka = 10 –9,31 (5)

(CN)2 + 2e + 2H+ →¬ 2HCN (6)

Tổ hợp các phương trình từ (2) đến (5) ta có:

2Cu(CN)2 + 6CN – →¬ 2Cu(CN)43 – + (CN)2

K = K22.K3

–2.K42.Ka

–2.K6 –2 = 2.1034 rất lớn nên phản ứng coi như hoàn toàn.

Trạng thái giới hạn của hệ là: Cd(CN)42 – 0,1 M và Cu(CN)4

3 – 0,1 M và CN –

0,1 M.

– Phản ứng thủy phân của CN – không làm thay đổi nồng độ CN –:

CN – + H2O →¬ HCN + OH – K = 10 –4,69

⇒ [OH –] = [HCN] = 1,419.10 –3

– Khi thế đặt trên catot là –1,0 V, khi đó nồng độ cân bằng của [Cd2+] và [Cu+] có thể được tính theo phương trình Nernst:

2 2

o 2

Cd CdCd Cd

0,0592E E lg Cd

2+ +

+ = + = – 1,00 V ⇒ [Cd2+] = 10 –20,20

– Đối với đồng: ta có Cu2+ + 2e → Cu Eo = 0,337 V (7)

104

Page 99: 1ygm3hwjpgu090312060334 tài liệu bồi dưỡng giáo viên trường thpt chuyên   mon hóa học - phan 1- năm 2012

và Cu2+ + e → Cu+ Eo = 0,521 V (8)

⇒ Cu+ + e → Cu o

CuCu

E + = 0,153 V

Theo phương trình Nernst: o

Cu CuCu Cu

E E 0,0592lg[Cu ]=-1,00V+ ++= + ⇒ [Cu+]

= 10 –19,48

– Giả sử nồng độ CN – không đổi 0,1 M ta có [Cd(CN)42 –] = β4.[Cd2+].[CN –]4

= 5,25.10 –7 M. Vậy có thể coi Cd2+ đã bị điện phân hoàn toàn:

– Nếu Cu+ đã bị điện phân thi nồng độ Cu(CN)43 – phải nhỏ hơn 0,1 M.

Ta có: [Cu(CN)43 –] = β4.[Cu+].[CN –]4 = 1030,29. 10 –19,48.0,14 >> 0,1 nên Cu+

chưa bị điện phân.

+ Tại catot: Cd(CN)42 – + 2e →¬ Cd + 4CN –

+ Tại anot: 2

o

(CN) ,2H2HCN

E + =0,370V < 2

2

o

O ,HH O

E + + 2O (Pt )η = 1,700V nên CN – bị

oxi hóa:

2CN – → (CN)2 + 2e

+ Phản ứng điện phân:

Cd(CN)42 – → Cd + 2CN – + (CN)2

Nồng độ CN – lúc này là 0,1 + 2.0,1 = 0,3 M.

Vậy trong dung dịch chứa các cấu tử:

[Cd2+] = 10 –20,2 M; [Cd(CN)+] = 106,01.10 –20,2.0,3 = 1,94. 10 –15 M;

[Cd(CN)2] = 1011,12.10–20,2.0,32 = 7,49.10 –11 M; [Cd(CN)3–] = 1015,65.10 –20,2.0,33 =

7,61.10 –7 M;

[Cd(CN)42 –] = 1017,92.10 –20,2.0,34 = 4,25.10 –5 M.

Tổng nồng độ Cd2+ còn lại: 4,32.10 –5 M chiếm 0,043% lượng Cd2+ ban đầu nên giả thiết bị điện phân hết có thể chấp nhận được.

+ 302 3 4

2 3 4

1[Cu ]=0,1. 5,94.10 M

1 . CN . CN . CN

− − −=

+ β + β + β

[Cu(CN)2 –] = 1024. 5,94.10 –30.0,32 = 5,436.10 –7 M;

[Cu(CN)32 –] = 1028,59. 5,94.10 –30.0,33 = 6,24.10 –3 M

105

Page 100: 1ygm3hwjpgu090312060334 tài liệu bồi dưỡng giáo viên trường thpt chuyên   mon hóa học - phan 1- năm 2012

[Cu(CN)43 –] = 1030,29. 5,94.10 –30.0,34 = 0,0938 M.

Kiểm tra tổng nồng độ CN –:

[CN –] = 1,0 – 0,1 – 0,2 – 2.5,436.10 –7 – 3.6,24.10 –3 – 4.0,0938 – 7,49.10 –11 – 2.7,49.10 –11 – 3. 7,61.10 –7 – 4.4,25.10 –5 = 0,306 M là chấp nhận được so với 0,3 M.

Câu 6. Dung dịch A gồm AgNO3 0,050 M và Pb(NO3)2 0,100 M.

1. Tính pH của dung dịch A.

2. Thêm 10,00 ml KI 0,250 M và HNO3 0,200 M vào 10,00 ml dung dịch A. Sau phản ứng người ta nhúng một điện cực Ag vào dung dịch B vừa thu được và ghép thành pin (có cầu muối tiếp xúc hai dung dịch) với một điện cực có Ag nhúng vào dung dịch X gồm AgNO3 0,010 M và KSCN 0,040 M.

a) Viết sơ đồ pin.

b) Tính sức điện động Epin tại 250C.

c) Viết phương trình phản ứng xảy ra khi pin hoạt động.

d) Tính hằng số cân bằng của phản ứng.

Cho biết: Ag+ + H2O AgOH + H+ (1) ; K1= 10 –11,70

Pb2+ + H2O PbOH+ + H+ (2) ; K2= 10 –7,80

Chỉ số tích số tan pKs: AgI là 16,0 ; PbI2 là 7,86 ; AgSCN là 12,0.

;

RT ln = 0,0592 lgF= 0 ,799 VE0

Ag+/Ag

3. Epin sẽ thay đổi ra sao nếu: a) thêm một lượng nhỏ NaOH vào dung dịch B; b) thêm một lượng nhỏ Fe(NO3)3 vào dung dịch X?

Hướng dẫn giải:

1. Ag+ + H2O ⇌ AgOH + H+ ; K1 = 10 –11,7 (1)

Pb2+ + H2O ⇌ PbOH+ + H+ ; K2 = 10 –7,8 (2)

Do K2 >> K1 nên cân bằng 2 quyết định pH của dung dịch

Pb2+ + H2O ⇌ PbOH + H+ ; K2 = 10 –7,8 (2)

C 0,10

[ ] 0,10 − x x x

8,72

10x1,0

x −=−

⇒ x = 10 –4,4 = [H+] ; pH = 4,40

2. a) Dung dịch B: Thêm KI: CAg+ = 0,025 M; CPb2+ = 0,050

106

Page 101: 1ygm3hwjpgu090312060334 tài liệu bồi dưỡng giáo viên trường thpt chuyên   mon hóa học - phan 1- năm 2012

CI – = 0,125M; CH+ = 0,10M

Ag+ + I− → AgI ↓

0,025 0,125

– 0,10

Pb2+ + 2 I− → PbI2 ↓

0,05 0,10

– –

Trong dung dịch có đồng thời hai kết tủa AgI ↓ và PbI2 ↓

AgI ↓ ⇌ Ag+ + I− ; Ks1 = 1.10 –16 (3)

PbI2 ↓ ⇌ Pb2+ + 2 I− ; Ks2 = 1.10 –7,86 (4)

Ks1 << Ks2, vậy trong dung dịch cân bằng (4) là chủ yếu. Sự tạo phức hiđroxo của Pb2+ là không đáng kể vì có H+ dư:

Pb2+ + H2O ⇌ PbOH + H+; K2 = 10 –7,8

[ ][ ] [ ] [ ]++−

+

+

<<→== 28,61

8,7

2PbPbOH10

10

10

Pb

PbOH

Trong dung dịch PbI2↓ ⇌ Pb2+ + 2 I− Ks2 = 1.10 –7,86

x 2 x

(2x)2x = 10 –7,86 ⇒ x = 1,51.10 –3M ⇒ 2x = [I−] = 2,302. 10 –3M

[ ] [ ] M10.31,310.02,3

10.1

I

KAg 14

3

161s −

−+ ===

E của cực Ag trong dung dịch A: Ag+ + e ⇌ Ag

[ ]V001,0E

10.31,3lg0592,0799,0Aglg0592,0EE

1

140

AgAg1

=

+=+= −++

Dung dịch X: Ag+ + SCN− ⇌ AgSCN↓ ; 1012,0

0,010 0,040

– 0,030 0,010

AgSCN↓ ⇌ Ag+ + SCN− ; 10 –12,0

107

Page 102: 1ygm3hwjpgu090312060334 tài liệu bồi dưỡng giáo viên trường thpt chuyên   mon hóa học - phan 1- năm 2012

0,030

x (0,030 + x)

x(0,030 + x) = 10 –12

[ ] 112

12

10.33,310x3

10xAg −

−+ ===

[ ]V179,0E

10.33,3lg0592,0799,0Ag lg 0,0592 0,799 E

2

112

=+=+= −+

Vì E2 > E1, ta có pin gồm cực Ag trong X là cực +, cực Ag trong B là cực –

Sơ đồ pin:

AgI↓ AgSCN↓

PbI2↓ SCN− 0,03 M

b) Epin = 0,179 – 0,001 = 0,178V

c) Phương trình phản ứng: Ag + I– ⇌ AgI↓ + e

AgSCN + e ⇌ Ag↓ + SCN–

AgSCN + I– ⇌ Ag↓ + SCN–

d) AgSCN

AgI

12S 4

16S

K 10K 10

K 10

−= = =

3. a) Khi thêm lượng nhỏ NaOH vào dung dịch B, có thể xảy ra 3 trường hợp:

– Lượng NaOH quá ít không đủ để trung hoà HNO3: Sự tạo phức hiđroxo của Pb2+ vẫn không đáng kể, do đó Epin không thay đổi.

– Lượng NaOH đủ để trung hoà HNO3: Có sự tạo phức hiđroxo của Pb2+ do

đó [Pb2+] giảm, nồng độ I – sẽ tăng lên, do đó nồng độ Ag+ giảm xuống, E1 giảm;

vậy Epin tăng.

– Lượng NaOH đủ dư để trung hoà hết HNO3 và hoà tan PbI2 tạo thành

PbO2–, do đó [Pb2+] giảm

và Epin tăng. PbI2 + 4 OH– → PbO2– + 2 H2O + 2 I–

b) Thêm ít Fe3+ vào dung dịch X: Fe3+ + SCN– → FeSCN2+

108

Ag Ag

Page 103: 1ygm3hwjpgu090312060334 tài liệu bồi dưỡng giáo viên trường thpt chuyên   mon hóa học - phan 1- năm 2012

Nồng độ ion SCN– giảm, do đó nồng độ ion Ag+ tăng, E2 tăng dẫn đến Epin

tăng.

109

Page 104: 1ygm3hwjpgu090312060334 tài liệu bồi dưỡng giáo viên trường thpt chuyên   mon hóa học - phan 1- năm 2012

Ph n 3ÇHãA HäC V¤ C¥

MỘT SỐ TÍNH CHẤT VÀ ỨNG DỤNG CỦA CROM

TS Vi Anh Tuấn – Khoa Hóa học

Trường Đại học KHTN – Đại học Quốc gia Hà Nội

Crom là nguyên tố kim loại chuyển tiếp thuộc nhóm VIB của bảng hệ thống tuần hoàn. Crom có cấu hình electron ở lớp vỏ hóa trị là 3d54s1. Số oxi hóa đặc trưng của crom là +3 và +6. Tên gọi của crom xuất phát từ tiếng Hy Lạp "chroma" có nghĩa là mầu sắc vì các hợp chất của crom đều có mầu.

Crom là kim loại mầu trắng bạc. Do có số electron độc thân tối đa nên độ bền của liên kết trong tinh thể crom là lớn nhất, vì vậy crom có nhiệt độ sôi, nhiệt độ nóng chảy và độ cứng rất cao (to

nc = 1907oC; tos = 2671oC, độ cứng theo thang

Mohs: 8,5).

1. Điều chế crom

Khoáng vật chính của crom là sắt cromit (FeCr2O4). Những nước giầu mỏ quặng crom là Nam Phi, Kazakhstan, Ấn Độ, Nga, Thổ Nhĩ Kỳ và Zimbabwe. Nước ta có một mỏ sa khoáng cromit khá lớn ở Cổ Định, Thanh Hóa.

Trong công nghiệp lượng lớn crom được sản xuất từ quặng dưới dạng hợp kim ferocrom chứa 50 –70% crom, được sản xuất bằng cách dùng than cốc khử quặng cromit trong lò điện:

Fe(CrO2)2 + 4 C to Fe + 2 Cr + 4 CO

Để điều chế crom tinh khiết người ta phải tách sắt ra khỏi quặng cromit theo quy trình sau: Quặng cromit được đun nóng chảy với hỗn hợp canxi cacbonat và natri cacbonat và có mặt không khí. Crom bị oxi hóa thành cromat và sắt chuyển

110

Page 105: 1ygm3hwjpgu090312060334 tài liệu bồi dưỡng giáo viên trường thpt chuyên   mon hóa học - phan 1- năm 2012

thành Fe2O3. Hòa tan hỗn hợp vào nước, tách phần oxit sắt không tan thu được dung dịch cromat. Thêm axit sunfuric để chuyển cromat thành dicromat:

4 FeCr2O4 + 8 Na2CO3 + 7 O2 to 8 Na2CrO4 + 2 Fe2O3 + 8 CO2

2 Na2CrO4 + H2SO4 → Na2Cr2O7 + Na2SO4 + H2O

Khử dicromat thành crom (III) oxit bằng cacbon. Tiếp theo khử crom (III) oxit bằng nhôm sẽ thu được crom kim loại:

Na2Cr2O7 + 2 C to Cr2O3 + Na2CO3 + CO

Cr2O3 + 2 Al to 2 Cr + Al2O3

Câu 1.1. (a) Viết các phương trình hoá học từ Na2Cr2O7, C (than đá), Al (bột nhôm) và các điều kiện cần thiết để thu được Cr.

(b) CrO2Cl2 (cromyl clorua) là một hoá chất quan trọng. Hãy viết các phương trình hoá học tạo ra CrO2Cl2 từ:

i. CrO3 tác dụng với axit HCl.

ii. Cho K2Cr2O7 tác dụng với KCl trong H2SO4 đặc, nóng.

(c) Thêm chất thích hợp và hoàn thành các phương trình hoá học sau:

i. KNO2 + KNO3 + ? → K2CrO4 + NO

ii. NaNO2 + ? + NaI → I2 + NaHSO4 + NO + H2O

iii. HNO3 + P2O5 → ? + N2O5

(a. Na2Cr2O7 + 2 C to Na2CO3 + Cr2O3 + CO

Cr2O3 + 2 Al to 2 Cr + Al2O3

b. i. CrO3 + 2 HCl → CrO2Cl2 + H2O

ii. K2Cr2O7 + 4 KCl + 3 H2SO4 → 3 K2SO4 + 2 CrO2Cl2 + 3 H2O

c. i. 3 KNO2 + KNO3 + Cr2O3 → 2 K2CrO4 + 4 NO

ii. 2 NaNO2 + 4 H2SO4 + 2 NaI → I2 + 4 NaHSO4 + 2 NO + 2 H2O

iii. 2 HNO3 + P2O5 → 2 HPO3 + N2O5)

Câu 1.2. (a) Tại sao crom có khả năng thể hiện nhiều trạng thái oxi hoá? Cho biết những số oxi hoá phổ biến của crom?

(b) Nêu và giải thích sự biến đổi tính chất axit – bazơ trong dãy oxit: CrO, Cr2O3, CrO3. Viết phương trình hoá học của các phản ứng để minh hoạ.

111

Page 106: 1ygm3hwjpgu090312060334 tài liệu bồi dưỡng giáo viên trường thpt chuyên   mon hóa học - phan 1- năm 2012

(c) Viết phương trình ion của các phản ứng điều chế Al2O3 và Cr2O3 từ dung dịch gồm kali cromit và kali aluminat.

(a. Cấu hình electron của crom là [Cr]3d54s1, do nguyên tử crom có 6 electron hóa trị nên crom thể hiện nhiều trạng thái oxi hóa; Số oxi hóa phổ biến của crom là +2, +3, +6.

b. Đi từ CrO – Cr2O3 – CrO3 thì tính axit tăng dần, tính bazơ giảm dần.

Giải thích: khi đi từ CrO – Cr2O3 – CrO3 thì số oxi hóa của nguyên tử crom tăng do đó tính cộng hóa trị của liên kết Cr –O tăng, làm cho tính bazơ giảm, tính axit tăng.

Trong thực tế CrO là oxit axit, Cr2O3 là oxit lưỡng tính và CrO3 là oxit axit.

CrO + 2 H+ → Cr2+ + H2O

Cr2O3 + 6 H+ → 2 Cr3+ + 3 H2O

Cr2O3 + 2 OH – (nóng chảy) → 2 CrO2 – + H2O

CrO3 + 2 OH – → CrO42 – + H2O

c. Cách 1: Thêm H2O2 và NaOH dư vào dung dịch:

2 Cr(OH)4 – + 3 H2O2 + 2 OH – → 2 CrO4

2 – + 8 H2O

Thêm tiếp CO2 đến dư vào dung dịch thu được:

Al(OH)4 – + CO2 → Al(OH)3↓ + HCO3

Lọc lấy kết tủa đem nung thu được Al2O3 tinh khiết:

2 Al(OH)3 → Al2O3 + 3 H2O

Thêm H2SO4 đến dư vào phần dung dịch lọc, cô cạn dung dịch để kết tinh K2Cr2O7.

2 CrO4 – + 2 H+ → Cr2O7

2 – + H2O

Nung K2Cr2O7 với C thu được Cr2O3

K2Cr2O7 + 2 C → K2CO3 + Cr2O3 + CO

Cách 2: Thêm H2O2 và KOH dư vào dung dịch:

2 Cr(OH)4 – + 3 H2O2 + 2 OH – → 2 CrO4

2 – + 8 H2O

Thêm tiếp CO2 đến dư vào dung dịch thu được:

Al(OH)4 – + CO2 → Al(OH)3↓ + HCO3

112

Page 107: 1ygm3hwjpgu090312060334 tài liệu bồi dưỡng giáo viên trường thpt chuyên   mon hóa học - phan 1- năm 2012

Lọc lấy kết tủa đem nung thu được Al2O3 tinh khiết:

2 Al(OH)3 → Al2O3 + 3 H2O

Thêm H2O2 và H2SO4 đến dư vào phần dung dịch lọc:

Cr2O72 – + 8 H+ + 3 H2O2 → 2 Cr3+ + 3 O2 + 7 H2O

Thêm NH3 loãng để kết tủa Cr3+:

Cr3+ + 3 NH3 + 3 H2O → Cr(OH)3↓ + 3 NH4+

Lọc lấy kết tủa đem nung thu được Cr2O3 tinh khiết:

2 Cr(OH)3 → Cr2O3 + 3 H2O)

Câu 1.3. Hòa tan hoàn toàn 0,1331 gam một mảnh hợp kim chứa sắt và crom bằng dung dịch axit sunfuric loãng, dư để tạo thành các ion Fe2+ và Cr3+. Chuẩn độ dung dịch thu được bằng dung dịch pemanganat ở nhiệt độ phòng thấy vừa hết 20,08 ml dung dịch. Biết rằng ở điều kiện thí nghiệm Cr3+ không tham gia phản ứng.

Trong một thí nghiệm khác, người ta chuẩn độ 10 ml dung dịch axit oxalic 0,0500 M trong môi trường axit sunfuric bằng dung dịch pemanganat ở trên thấy vừa hết 9,75 ml dung dịch.

(a) Hãy viết các phản ứng xảy ra ở dạng ion rút gọn.

(b) Hãy tính thành phần % theo khối lượng của sắt và crom trong hợp kim ban đầu. (Cr = 52,00; Fe = 55,85).

Một bình điện phân chứa 100 lit dung dịch CrO3 nồng độ 230 g/lit. Tiến hành điện phân dung dịch trong khoảng thời gian 10 giờ với cường độ dòng điện 1500 A. Sau khi kết thúc thí nghiệm thấy khối lượng của catot tăng lên 670 gam, biết rằng có khí thoát ra ở cả catot và anot.

(c) Hãy cho biết khí thoát ra ở anot và ở catot?

(d) Hãy tính hiệu suất của quá trình điện phân điều chế crom.

(e) Hãy tính thể tích khí thoát ra ở catot và ở anot ở 25°C và 1,00 atm.

(a. Fe + 2 H+ → Fe2+ + H2

Cr + 3 H+ → Cr3+ + 3/2H2

5 Fe2+ + MnO4 – + 8 H+ → 5 Fe3+ + Mn2+ + 4 H2O

5 H2C2O4 + 2 MnO4 – + 6 H+ → 10 CO2 + 2 Mn2+ + 8 H2O

113

Page 108: 1ygm3hwjpgu090312060334 tài liệu bồi dưỡng giáo viên trường thpt chuyên   mon hóa học - phan 1- năm 2012

b. MnO4 –: 0,02051 M;

Fe: 86,4%; Cr: 13,6%;

c. CrO3 + H2O → 2 H+ + CrO42 –

CrO42 – + 8 H+ + 6 e → Cr + 4 H2O

Anot (+): 2 H2O → O2↑ + 4 H+ + 4e

Catot (–): 2 H+ + 2e → H2↑

d. Có: molF

Itne 7,559

96485

3600101500 =××==

Số mol electron do cromat nhận là:

molne 3,77600,52

670' =×=

Hiệu suất của quá trình điện phân điều chế crom:

%8,131007,559

3,77'

=×==e

e

n

nH

e. Có: molnn

n eeH 2,241

2

3,777,559

2

'

2=−=−=

moln

n eO 9,139

4

7,559

42===

=> litVH310.89,5

1

298082,02,2412

=××=

litVO310.42,3

1

298082,09,1392

=××= )

Câu 1.4. Nguồn nguyên liệu chính để điều chế crom là quặng cromit (FeCr2O4), trong cromit sắt có hóa trị II.

(a) Hãy xác định số oxi hóa của crom trong các cấu tử sau: Cr2O3, CrO42–,

Cr2O72–, FeCr2O4.

(b) Hãy viết phương trình phản ứng khử quặng cromit bằng cacbon, biết sản phẩm thu được là sắt, crom và cacbon dioxit.

Quy trình điều chế muối crom (III) từ quặng cromit như sau: Đầu tiên oxi hóa cromit bằng không khí trong natri hidroxit nóng chảy tạo thành natri cromat (Na2CrO4); Hòa tan sản phẩm vào nước, axit hóa dung dịch bằng dung dịch H2SO4

114

Page 109: 1ygm3hwjpgu090312060334 tài liệu bồi dưỡng giáo viên trường thpt chuyên   mon hóa học - phan 1- năm 2012

thu được dung dịch có mầu da cam; Cô cạn dung dịch, lấy chất rắn đem khử bằng cacbon thu được Cr2O3; Hòa tan oxit này bằng dung dịch axit sunfuric thu được dung dịch muối crom (III).

(c) Hãy viết các phương trình phản ứng xảy ra trong quy trình trên.

Để xác định hàm lượng crom trong mẫu quặng cromit, người ta sử dụng phương pháp chuẩn độ iot – thiosunfat. Oxi hóa 5,00 g mẫu quặng cromit bằng oxi trong dung dịch natri hidroxit, chuyển toàn bộ dung dịch thu được (chứa natri cromat) vào bình định mức 1,00 lit và định mức đến vạch bằng nước cất (dung dịch A). Lấy 25,00 ml dung dịch A cho vào bình nón có chứa dung dịch đệm. Thêm khoảng 3 gam KI. Đậy kín miệng bình và để trong tối cho phản ứng xảy ra hoàn toàn, sản phẩm tạo thành là I3

– và Cr3+. Chuẩn độ lượng I3 – sinh ra bằng

dung dịch Na2S2O3 0,1000 M, sản phẩm tạo thành là I – và ion tetrathionat (S4O6

2–), thấy vừa hết 23,10 ml dung dịch.

(d) Hãy viết các phương trình phản ứng xảy ra trong quy trình phân tích trên.

(e) Hãy tính nồng độ của natri cromat trong dung dịch A.

(g) Hãy tính thành phần % theo khối lượng của cromit trong mẫu quặng.

(h) Hãy tính thành phần % theo khối lượng của crom trong mẫu quặng.

(a. +3, +6, +6, +3;

b. FeCr2O4 + 2 C → Fe + 2 Cr + 2 CO2

c. 4 FeCr2O4 + 7 O2 + 16 NaOH → 2 Fe2O3 + 8 Na2CrO4 + 8 H2O

2 Na2CrO4 + 2 H2SO4 → 2 NaHSO4 + Na2Cr2O7 + H2O

Na2Cr2O7 + 2 C to Na2CO3 + Cr2O3 + CO

Cr2O3 + 3 H2SO4 → Cr2(SO4)3 + 3 H2O

d. 4 FeCr2O4 + 7 O2 + 16 NaOH → 2 Fe2O3 + 8 Na2CrO4 + 8 H2O

2 CrO42 – + 9 I – + 16 H+ → 2 Cr3+ + 3 I3

– + 8 H2O

I3 – + 2 S2O3

2 – → 3 I – + S4O62 –

e. MCCrO

0308,000,25

3

2

2

11000,010,23

24

=×××

=−

115

Page 110: 1ygm3hwjpgu090312060334 tài liệu bồi dưỡng giáo viên trường thpt chuyên   mon hóa học - phan 1- năm 2012

g. molnOFeCr

0154,08

40,10308,0

42=××=

=> %94,6810000,5

85,2230154,0% 42 =××=OFeCr

h. molnnOFeCrCr 0308,02

42==

=> %04,3210000,5

00,520308,0% =××=Cr )

2. Phức chất của crom

2.1. Phức chất crom (0)

a) Phức hexacacbonyl crom(0), Cr(CO)6

2s 2sσs

σs*

C OCO

2p2pσz

σz*

πx πy

πx* πy*

Giản đồ MO của phân tử CO.

Trong phân tử CO, cặp electron trên MO σz có năng lượng cao hơn những

cặp electron trên MO πx và πy nên có khả năng tạo liên kết σ cho nhận với các

obitan lai hóa d2sp3 trống của nguyên tử crom.

116

Page 111: 1ygm3hwjpgu090312060334 tài liệu bồi dưỡng giáo viên trường thpt chuyên   mon hóa học - phan 1- năm 2012

3d 4s 4p

d2sp3

liªn kÕt

liªn kÕt

Cr(0):

Cr CO

COCr

CO CO CO CO CO CO

Ngoài liên kết σ–cho nhận (Cr←CO), trong hexacacbonyl còn có liên kết

π –cho Cr→CO tạo nên bởi những cặp electron d của nguyên tử crom với những

MO –π* trống của phân tử CO và nhờ liên kết π này, các phân tử cacbonyl kim

loại được làm bền thêm.

π-cho

Cr C Oσ

Vậy phức Cr(CO)6 có cấu hình bát diện đều và nghịch từ.

CrOC

OC CO

CO

CO

CO

Như vậy phức Cr(CO)6 tuân theo quy tắc 18 electron: "Nguyên tử kim loại trong cacbonyl kim loại có khuynh hướng nhận thêm một số electron của phân tử CO để đạt được 18 electron ở vỏ hóa trị". Dưới đây là một số phức cacbonyl kim loại tuân theo quy tắc 18 electron.

Cacbonyl Số electron hóa trị của M

Số electron CO đóng góp

Tổng Hình học phân tử

Ti(CO)7 4 14 18 –

V2(CO)12 5 12+1 18 –

Cr(CO)6 6 12 18 Bát diện đều

Mn2(CO)10 7 10+1 18 2 bát diện chung 1 đỉnh

117

Page 112: 1ygm3hwjpgu090312060334 tài liệu bồi dưỡng giáo viên trường thpt chuyên   mon hóa học - phan 1- năm 2012

Fe(CO)5 8 10 18 Lưỡng tháp tam giác

Co2(CO)8 9 8+1 18 –

Ni(CO)4 10 8 18 Tứ diện

OC Mn

OC

COCO

CO

Mn

OC

COCO

CO

CO

3d 4s 4p

d2sp3

Mn (0):

Mn CO

COMn

CO CO CO CO COπ

σMn-Mnσ

b) Phức crom dibenzen Cr(C6H6)2

Phân tử crom dibenzen có hình bánh kẹp (thuộc loại hợp chất π – cơ kim).

3d 4s 4p

d2sp3

Cr (0):

Cr MO π*

MO πCr

π

σ

π

π*

πE

Gi¶n ®å n¨ng l- î ng c¸c MO π cña benzen

Cr

118

Page 113: 1ygm3hwjpgu090312060334 tài liệu bồi dưỡng giáo viên trường thpt chuyên   mon hóa học - phan 1- năm 2012

Trong hợp chất này, quy tắc 18 electron cũng được tuân theo.

Câu 2.1. Cuối thế kỉ 18 Ludwig Mond phát hiện ra rằng niken ở dạng bột mịn phản ứng với cacbon monoxit tạo thành tetracacbonyl niken, Ni(CO)4, một chất lỏng không mầu, dễ bay hơi. Phức Ni(CO)4 tuân theo quy tắc 18 electron.

(a) Sử dụng quy tắc 18 electron, hãy dự đoán công thức phức cacbonyl của Fe(0) và Cr(0).

(b) Sử dụng quy tắc 18 electron hãy dự đoán công thức phức nitrozyl của Cr(0)?

(c) Hãy giải thích tại sao Mn(0) và Co(0) không thể tạo ra các phức cacbonyl đơn nhân kiểu M(CO)x (M là kim loại), mà chỉ có thể tạo các phức cacbonyl có liên kết kim loại – kim loại?

(d) Hãy cho biết cấu trúc không gian của Ni(CO)4, Mn2(CO)10 và Co2(CO)8.

(e) Hãy cho biết phức V(CO)6 và các phức cho ở phần (a) và (d) là thuận từ hay nghịch từ?

(g) Hãy giải thích tại sao cacbon monoxit liên kết với kim loại bền hơn rất

nhiều so với khi liên kết với bo trong hợp chất kiểu R3BCO; R là gốc ankyl?

(h) Quy tắc 18 electron cũng nghiệm đúng đối với phức của crom và benzen.

i/ Hãy viết công thức của phức.

ii/ Hãy cho biết công thức của phức có cấu trúc tương tự được điều chế bằng phản ứng giữa bột sắt với xiclopentadien? Hãy viết phương trình phản ứng điều chế phức.

(a. Fe(CO)5 và Cr(CO)6;

b. Cr(NO)4

c. Do nguyên tử Mn và Co có số lẻ electron.

d.

CO

NiOC CO

CO OC MnOC

COCO

CO

Mn

OC

COCO

CO

CO OC CoCOOC

OC

Co

OC

CO

CO

CO

e. Fe(CO)5, Cr(CO)6, Ni(CO)4, Mn2(CO)10, Co2(CO)10: nghịch từ;

119

Page 114: 1ygm3hwjpgu090312060334 tài liệu bồi dưỡng giáo viên trường thpt chuyên   mon hóa học - phan 1- năm 2012

V(CO)6 thuận từ;

g. Do trong R3B –CO không có liên kết π – cho.

π-cho

Cr C Oσ

h. i/

Cr

ii/ Fe + 2 C5H6 → Fe(C5H5)2 + H2

Fe

)

2.2. Phức chất crom (II)

Cr(II) axetat là kết tủa mầu đỏ và là hợp chất bền nhất của Cr(II), có cấu tạo đime [Cr(CH3COO)2.H2O]2. Trong phân tử dime liên kết Cr –Cr là liên kết bốn

(1σ, 2π, 1δ).

3d 4s 4p

d2sp3

Cr (II):

ΟCrσ

δ Cr-Cr π Cr-Cr σ Cr-Cr

120

Page 115: 1ygm3hwjpgu090312060334 tài liệu bồi dưỡng giáo viên trường thpt chuyên   mon hóa học - phan 1- năm 2012

Cr Cr

O

O

O

O

H2O OH2

O

O

O

O

Cr(II) axetat được điều chế theo phản ứng:

2 CrCl2 + 4 NaCH3COO + 2 H2O → [Cr(CH3COO)2.H2O]2 + 4 NaCl

Câu 2.2. X là một hợp chất của crom. Phân tích thành phần nguyên tố cho thấy: Cr 27,7%; C: 25,5%; H 4,25% theo khối lượng, phần còn lại là oxi.

(a) Hãy cho biết công thức đơn giản nhất của X.

(b) Biết rằng trong công thức đơn giản nhất có chứa một phân tử nước, hãy cho biết phối tử còn lại là gì? Hãy cho biết số oxi hóa của Cr trong X.

(c) Nghiên cứu từ tính cho thấy X có tính nghịch từ, hãy giải thích tính chất này của hợp chất X và vẽ cấu trúc không gian của X.

(a.

5:8:4:166,2:25,4:125,2:533,016

55,42:

1

25,4:

12

5,25:

52

7,27::: ===OHCCr

=> Công thức kinh nghiệm của X là: CrC4H8O5.

b. Cr(H2O)(CH3COO)2, phối tử còn lại là axetat, số oxi hóa của crom: +2.

c. X là đime: [Cr(H2O)(CH3COO)2]2.

Cr Cr

O

O

O

O

H2O OH2

O

O

O

O

3d 4s 4p

d2sp3

Cr (II):

ΟCrσ

δ Cr-Cr π Cr-Cr σ Cr-Cr

)

121

Page 116: 1ygm3hwjpgu090312060334 tài liệu bồi dưỡng giáo viên trường thpt chuyên   mon hóa học - phan 1- năm 2012

Câu 2.3. Kim loại A có mầu trắng bạc và có ánh kim. Trong công nghiệp, người ta dùng kim loại A để mạ lên các đồ vật bằng kim loại để bảo vệ cho kim loại không bị gỉ, lớp mạ thường chỉ dày khoảng 0,005 mm. Hidroxit của A là chất B có dạng A(OH)2, B là chất kết tủa mầu vàng nhưng thường lẫn tạp chất nên có mầu hung. Khi tiếp xúc với không khí, B chuyển thành C là chất nhầy có mầu lục nhạt, không tan trong nước nhưng tan được trong cả dung dịch kiềm cũng như dung dịch axit. Chất B tan trong dung dịch HCl dư cho dung dịch D có mầu xanh lam nhưng khi cô cạn dung dịch thì lại được muối rắn khan mầu trắng, hút ẩm mạnh. Cho dung dịch D tác dụng với dung dịch NaCH3COO đặc thu được kết tủa ít tan mầu đỏ E. E dạng dime là hợp chất có cấu tạo đặc biệt và là một trong những chất dễ điều chế và bền nhất của A(II).

(a) Tìm các chất từ A đến E và viết các phương trình phản ứng xảy ra.

(b) Nêu bản chất liên kết của A trong dime E.

(a. A: Cr;

4 Cr(OH)2 + O2 + 2 H2O → 4 Cr(OH)3

B C

Cr(OH)3 + 3 H+ → Cr3+ + 3 H2O

Cr(OH)3 + OH – → [Cr(OH)4] –

Cr(OH)2 + 2 HCl → CrCl2 + 2 H2O

B D

2 CrCl2 + 4 NaCH3COO + 2 H2O → [Cr(CH3COO)2(H2O)]2 + 4 NaCl

D E

b. Xem phần lý thuyết)

2.3. Phức chất crom (III)

Do có kích thước bé và điện tích lớn nên ion Cr3+ có khả năng tạo phức rất mạnh, các phức bền thường gặp là [CrX6]3 – (X: F, Cl, SCN, CN), [Cr(C2O4)2] –, NH4[Cr(NH3)2(SCN)4] (muối Reinecke có mầu đỏ đậm).

Trong dung dịch nước, muối CrCl3 tồn tại ở 3 dạng khác nhau:

[Cr(H2O)6]Cl3 [Cr(H2O)5Cl]Cl2 [Cr(H2O)4Cl2]Cl

tím xanh nhạt xanh thẫm

3 AgCl↓ 2 AgCl↓ 1 AgCl↓

122

Page 117: 1ygm3hwjpgu090312060334 tài liệu bồi dưỡng giáo viên trường thpt chuyên   mon hóa học - phan 1- năm 2012

Câu 2.4. Các hợp chất A, B, C có cùng công thức phân tử CrCl3.6H2O. Trong dung dịch nước tồn tại cân bằng giữa chúng như sau:

[Cr(H2O)6]Cl3 [Cr(H2O)5Cl]Cl2.H2O [Cr(H2O)4Cl2]Cl.2H2O.

(A) (B) (C)

Trong một thí nghiệm người ta cho dung dịch chứa 0,32 gam CrCl3.6H2O đi qua một lớp nhựa trao đổi cation dưới dạng H+. Cần 28,8 ml dung dịch NaOH 0,125M để chuẩn độ hết lượng H+ đã chuyển vào dung dịch.

(a) Gọi tên các phức chất A, B và C. Phức chất nào có đồng phân hình học?

(b) Xác định công thức của phức trong dung dịch.

(c) Viết cấu hình electron của Cr trong ion phức xác định được ở (b) và xác định từ tính của phức chất đó.

(a. A: Hexaaquacrom (III) clorua

B: Cloro pentaaqua crom (III) clorua hiđrat

C: Đicloro tetraaqua crom (III) clorua hiđrat (Có đồng phân hình học)

b. [Cr(H2O)6 –nCln]Cl3 –n.nH2O → [Cr(H2O)6 –nCln]3 –n + (3 –n) Cl – + n H2O

1,20.10 –3 1,20.10 –3 mol

[Cr(H2O)6 –nCln]3 –n + (3 –n) R –COOH → ... + (3 – n) H+

1,20.10 –3 1,20.10 –3 (3 – n) mol

H+ + OH – → H2O

3,60.10 –3 mol

Có: nH+ = 1,20.10 –3 (3 – n) = 3,60.10 –3

=> n = 0;

Vậy phức trong dung dịch là [Cr(H2O)6]Cl3 (phức A)

c. Phức thuận từ; BM .87,3)23(3 =+=µ

4p3d 4s

[Cr(H2O)6]3+

sp3d24d

6 H2O )

123

Page 118: 1ygm3hwjpgu090312060334 tài liệu bồi dưỡng giáo viên trường thpt chuyên   mon hóa học - phan 1- năm 2012

Câu 2.5. Thêm từ amoni dicromat vào muối amoni thioxianat nóng chảy sẽ thu được muối Reinecke có công thức NH4[Cr(SCN)x(NH3)y], muối này có thành phần % theo khối lượng của các nguyên tố như sau: Cr 15,5 %; S 38,15 % và N 29,2 %.

(a) Hãy xác định các giá trị x và y trong công thức trên.

(b) Hãy xác định số oxi hóa của crom trong phức.

(c) Hãy cho biết hình học của anion phức.

(d) Hãy vẽ các đồng phân lập thể của anion phức trên.

(a. NH4[Cr(SCN)4(NH3)2];

b. +3;

c. bát diện;

d. đồng phân hình học

CrNCS

NCS SCN

SCN

NH3

NH3

-

CrNCS

NCS SCN

NH3

NH3

SCN

-

)

Câu 2.6. Ion glyxinat, NH2–CH2–COO–, là một phối tử hai càng, tạo phức trisglyxinatocrom(III). Lưu ý rằng các nguyên tử oxi và nitơ của cùng một phối tử chỉ có thể liên kết ở các vị trí cạnh nhau trong cấu hình bát diện.

(a) Hãy cho biết phức trên có bao nhiêu đồng phân hình học?

(b) Hãy cho biết đồng phân hình học nào là bất đối?

Một phức chất khác của crom có thành phần % theo khối lượng của các nguyên tố như sau: 19,5 % Cr; 40,0 % Cl, 4,5 % H và 36,0 % O. Hòa tan 0,533 g phức vào 100 ml nước và thêm tiếp 10 ml dung dịch HNO3 2 M. Thêm lượng dư dung dịch bạc nitrat. Lọc, rửa kết tủa và đem sấy khô thu được 0,287 g chất rắn. Mặt khác đun nóng nhẹ 1,06 g phức ở 100 °C thấy thoát ra 0,144 g nước.

Nhiệt độ đông đặc của dung dịch thu được khi hòa tan 1,33 g phức vào 100ml nước là –0,18 °C. (Hằng số nghiệm lạnh của nước là 1,82 K.kg.mol –1).

(c) Hãy cho biết công thức của phức.

(d) Hãy vẽ tất cả các đồng phân lập thể của phức trên.

124

Page 119: 1ygm3hwjpgu090312060334 tài liệu bồi dưỡng giáo viên trường thpt chuyên   mon hóa học - phan 1- năm 2012

(a. Hai đồng phân hình học (fac và mer).

CrO

N O

O

N

N

bÊt ®èi

CrO

N N

O

O

N

bÊt ®èi

b. Cả hai đồng phân hình học đều bất đối.

c. Gọi CTPT của phức là CrxClyHzOt. Có:

6:12:3:125,2:46,4:128,1:375,00,16

0,36:

01,1

5,4:

45,35

0,40:

0,52

5,19::: ===tzyx

=> CTPT của phức là CrCl3(H2O)6. (M = 266,47)

Thí nghiệm 1:

[Cr(H2O)6 –nCln]Cl3 –n.nH2O → [Cr(H2O)6 –nCln]3 –n + (3 –n) Cl – + n H2O

2,00.10 –3 2,00.10 –3(3 –n) mol

Ag+ + Cl – → AgCl

2,00.10 –3 mol

Có: nAgCl = 2,00.10 –3(3 –n) = 2,00.10 –3

=> n = 2

Thí nghiệm 2:

[Cr(H2O)6 –nCln]Cl3 –n.nH2O → [Cr(H2O)6 –nCln]Cl3 –n + n H2O

3,98.10 –3 7,99.10 –3 mol

Có: nH2O = 3,98.10 –3n = 7,99.10 –3

=> n = 2

Thí nghiệm 3:

[Cr(H2O)6 –nCln]Cl3 –n.nH2O → [Cr(H2O)6 –nCln]3 –n + (3 –n) Cl – + n H2O

4,99.10 –3 4,99.10 –3 4,99.10 –3 (3 –n) mol

Có: dm

ion

m

nkkmt ==∆

125

Page 120: 1ygm3hwjpgu090312060334 tài liệu bồi dưỡng giáo viên trường thpt chuyên   mon hóa học - phan 1- năm 2012

(Coi lượng nước do phức phân li ra không đáng kể)

=> molmolkgK

kgK

k

tmn dm

ion3

110.89,9

..82,1

1,018,0 −− =×=∆=

Có: nion = 4,99.10 –3 + 4,99.10 –3 (3 –n) = 9,89.10 –3

=> n = 2

Kết quả của 3 thí nghiệm phù hợp với nhau, vậy công thức của phức là [Cr(H2O)4Cl2]Cl.2H2O;

d. 2 đồng phân hình học)

Câu 2.7. (a) Hãy vẽ và gọi tên các đồng phân liên kết và đồng phân lập thể của mỗi hợp chất phối trí sau:

i. [Pt(NH3)2(SCN)2].

ii. [Co(en)(H2NCH2COO)2]Cl.

(b) Hãy vẽ giản đồ phân tách mức năng lượng của các obitan d trong trường tinh thể của mỗi phức sau.

i. [Cr(CN)6]3 –.

ii. [Cr(NH3)6]2+.

iii. [Cr(CN)6]4 –.

(a. i.

Pt

NH3

NH3

NCS SCN Pt

SCN

NH3

NCS NH3

Pt

NCS

NH3

SCN NH3Pt

NH3

NH3

SCN NCS

Pt

NH3

NH3

NCS NCS Pt

NCS

NH3

NCS NH3

ii.

126

Page 121: 1ygm3hwjpgu090312060334 tài liệu bồi dưỡng giáo viên trường thpt chuyên   mon hóa học - phan 1- năm 2012

N

Co

N

O N

O N

N

Co

N

ON

ON

O

Co

N

N N

O N

O

Co

N

NN

ON

O

Co

O

N N

N N

O

Co

O

NN

NN

b.

[Cr(CN)6]3- [Cr(NH3)6]2+[Cr(CN)6]4-)

Câu 2.8. Phức kali xiano K1 của nguyên tố A có momen từ µ = 3,8 B.M. Hợp chất có cực đại hấp thụ tương ứng với bước chuyển d –d tại 26700 cm –1. Phản ứng khử ion A2O7

2 – (có mầu da cam) bằng etanol trong môi trường axit tạo ra ion trung tâm của phức xiano. Phức K1 bị khử bởi kali trong amoniac lỏng cho phức xiano K2 (có số phối trí không thay đổi). Bằng cách thay đổi tất cả phối tử trong K2 bằng amoniac, momen từ của phức K3 thu được tăng lên thành µ = 4,9 B.M..

(a) Hãy cho biết A là nguyên tố nào?

(b) Hãy cho biết công thức và gọi tên phức K1.

(c) Hãy biểu diễn sự phân bố của electron trên các obitan d của nguyên tử trung tâm trong phức K1.

Cho NA = 6,022·1023 mol –1; c = 3,0·108 m/s và h = 6,62·10 –34 J.s.

(d) Hãy tính năng lượng tách của trường phối tử ∆ (theo kJ/mol) của phức K1.

(e) Cho các phối tử H2O, Cl – và F – các giá trị năng lượng tách: 158 kJ/mol, 182 kJ/mol và 208 kJ/mol. Hãy cho biết năng lượng tách tương ứng với mỗi phối tử.

(f) Hãy cho biết công thức và gọi tên của phức K2.

(g) Hãy biểu diễn sự phân bố của electron trên các obitan d của nguyên tử trung tâm trong phức K2 và K3, và cho biết phức là spin cao hay spin thấp?

(h) Phức K2 có cấu trúc bát diện lệch. Hãy cho biết tên gọi của hiệu ứng gây ra điều này.

127

Page 122: 1ygm3hwjpgu090312060334 tài liệu bồi dưỡng giáo viên trường thpt chuyên   mon hóa học - phan 1- năm 2012

(i) Hãy cho biết hình học của phức A(acac)2? Chú ý đến hiệu ứng được đề cập ở phần (h).

(a. Cr;

b. K3[Cr(CN)6], kali hexaxiano cromat (III);

c.

; Bµµ 87,3)23(3 =+=

d. 34 8 1 1 231006,62.10 . 3.10 . 26700 6,022.10A

cmE hc N J s m s cm

mυ − − −∆ = = × × × =

3319.10 / 319 /J mol kJ mol= =

e. H2O: 208 kj/mol; Cl –: 158 kj/mol; F –: 182 kj/mol.

f. K4[Cr(CN)6], kali hexaxiano cromat (II);

g.

spin thấp spin cao

h. Hiệu ứng Jahn – Teller;

i. Vuông phẳng)

Câu 2.9. Trong câu hỏi này các phức của Cr3+ và Cr2+ đều có cấu hình bát diện.

Thêm amoniac vào dung dịch Cr(III) tạo thành Cr(OH)3, một kết tủa mầu lục xám. Cr(OH)3 là hidroxit lưỡng tính, phản ứng với axit tạo ion phức mầu tím A, phản ứng với dung dịch NaOH tạo ion phức mầu xanh thẫm B.

(a) Hãy viết công thức của các ion phức A và B.

(b) Hãy viết phương trình phản ứng của Cr(OH)3 với axit và bazơ.

(c) Hãy giải thích tại sao các ion phức A và B lại có mầu khác nhau?

Ion phức A có tính axit (pKa = 3,95). Bazơ liên hợp của A dễ dàng bị dime hóa

với cầu nối là ion hydroxo tạo thành ion phức X có công thức [Cr2(OH)2(H2O)8]a±.

(d) Hãy viết phản ứng phân li axit của A.

128

Page 123: 1ygm3hwjpgu090312060334 tài liệu bồi dưỡng giáo viên trường thpt chuyên   mon hóa học - phan 1- năm 2012

(e) Hãy vẽ cấu trúc không gian và xác định điện tích a± của ion phức X.

C là phức triammintriclorocrom(III). Khi thêm ion oxalat (ox) vào C thì một phối tử Cl – và một phối tử NH3 sẽ được thay thế bằng một phối tử ox để tạo thành ion phức D.

(f) Hãy viết công thức của các ion phức C và D.

(g) Hãy vẽ các đồng phân lập thể của C và D (sử dụng kí hiệu O O cho

phối tử oxalat).

Ion phức A bị khử bằng hỗn hống kẽm – thủy ngân tạo thành phức E mầu xanh da trời rất kém bền của Cr2+.

(h) Hãy vẽ hai giản đồ phân tách năng lượng của các obitan d trong trường tinh thể của phức E.

(i) Hãy tính momen từ của phức E ứng với hai giản đồ trên.

(k) Hãy thiết lập biểu thức tính năng lượng làm bền trong trường tinh thể của E ứng với hai giản đồ trên theo năng lượng tách (Δo) và năng lượng ghép đôi electron (P).

(a. [Cr(H2O)6]3+; [Cr(OH)6]3 –;

b. Cr(OH)3 + 3 H3O+ → [Cr(H2O)6]3+

Cr(OH)3 + 3 OH – → [Cr(OH)6]3 –

c. Do năng lượng tách trong trường phối tử của hai phức khác nhau.

d. [Cr(H2O)6]3+ + H2O [Cr(OH)(H2O)5]2+ + H3O+;

e.

f. C: [CrCl3(NH3)3]; D: [CrCl2(NH3)2ox] –;

g.

129

Page 124: 1ygm3hwjpgu090312060334 tài liệu bồi dưỡng giáo viên trường thpt chuyên   mon hóa học - phan 1- năm 2012

h.

i. BM9,4)24(41 =+=µ

BM8,2)22(22 =+=µ

k. DqE ooolb 65

3

5

3

5

231 −=∆−=∆+∆−= ;

ooolb PDqPE +−=+∆−= 165

242 )

Câu 2.10. Tinh thể đá Ruby có mầu đỏ đậm và được sử dụng làm đồ trang sức. Ít người biết được rằng trái tim của máy phát tia laze đầu tiên, được chế tạo bởi Maiman vào năm 1960, là một tinh thể ruby lớn. Mầu đỏ của đá ruby là do sự hấp thụ ánh sáng của ion Cr3+ có mặt trong tinh thể oxit nhôm (Al2O3) không mầu. Ion Cr3+ có 3 electron ở phân lớp 3d và có khả năng hấp thụ ánh sáng do sự chuyển dịch của electron giữa các obitan của phân lớp 3d có mức năng lượng khác nhau.

(a) Hãy cho biết phổ hấp thụ nào dưới đây là phổ hấp thụ của ruby?

130

Page 125: 1ygm3hwjpgu090312060334 tài liệu bồi dưỡng giáo viên trường thpt chuyên   mon hóa học - phan 1- năm 2012

Ống ruby dùng trong máy phát laze là một ống hình trụ có chiều dài 15,2 cm và có đường kính 1,15 cm. Hàm lượng ion Cr3+ trong ống là 0,050 % theo khối lượng. Khối lượng riêng của Al2O3 là 4,05 g cm –3. Nguyên tử khối của Cr = 52u.

(b) Hãy tính số ion Cr3+ có trong ống ruby.

Trong ruby ion Cr3+ được phối trí bởi 6 ion oxit có cấu hình bát diện. Hình dạng của năm obitan 3d được cho dưới đây. Hình tiếp theo là giản đồ phân tách năng lượng của các obian d thành hai nhóm, một nhóm gồm ba obitan có mức năng lượng thấp hơn (t2g) và một nhóm gồm hai obitan có mức năng lượng cao hơn (eg).

(c) Hãy cho biết những obitan 3d nào (dz2, dxy, dyz, dx2 –y2, dxz) thuộc nhóm t2g

và những obian 3d nào thuộc nhóm eg?

131

Page 126: 1ygm3hwjpgu090312060334 tài liệu bồi dưỡng giáo viên trường thpt chuyên   mon hóa học - phan 1- năm 2012

eg

t2g

(d) Hãy vẽ giản đồ phân tách mức năng lượng của các obitan d trong trường tinh thể của phức Cr3+ trong ruby.

Một viên đá ruby được đặt trên một chiếc cân đòn. Khi cân thăng bằng, người ta đưa một cục nam châm vào ngay phía dưới đĩa cân đựng viên đá ruby.

(e) Hãy cho biết điều gì sẽ xảy ra?

(A) Cục nam châm kéo viên đá ruby xuống.

(B) Cục nam châm không làm viên đá ruby dịch chuyển.

(C) Cục nam châm đẩy viên đá ruby đi lên.

(D) Cục nam châm làm cho viên đá ruby di chuyển lên xuống.

(a. Phổ số 4

b.

ionmolionmolgam

cmgamcm

cmCrN 20123

1

32

3 10.7,3.10.022,6.52

100

05,0.05,4

2

15,114,32,15

)( =×××

××

= −−

+

c. d.

eg

t2gdxy dxz dyz

dz dx - y2 2 2

e. A)

132

Page 127: 1ygm3hwjpgu090312060334 tài liệu bồi dưỡng giáo viên trường thpt chuyên   mon hóa học - phan 1- năm 2012

Câu 2.11. X là ion phức xianua của crom có cấu hình bát diện, tan trong nước

và có tính khử. Biết X có momen từ µ = 2,83 B.M.

(a) Hãy xác định số oxi hóa của crom trong X.

Thay thế toàn bộ phối tử xianua trong X bằng amoniac thu được phức Y. Biết

µX ≠ µY.

(b) Hãy xác định momen từ (theo BM) của Y.

(a. Có: 83,2)2( =+= nnµ

=> n = 2,

Vậy crom có số oxi hóa +2 hoặc +4 trong phức X.

Do X có tính khử nên crom có số oxi hóa +2. X: [Cr(CN)6]4 –

[Cr(CN)6]4-

d2sp3

6CN-

b. BMnnY 90,4)24(4)2( =+=+=µ

[Cr(NH3)6]2+

sp3d2

6 NH3

)

Câu 2.12. (a) Hãy vẽ các đồng phân của các phức sau:

i/ [Cr(NH3)4Cl2]Cl

ii/ [CoPy3Cl3], py là pyridin.

iii/ [Co(SCN)(H2O)5]Cl

iv/ [PtCl(PMe3)3]Br, Me là CH3.

v/ [Co(en)2Cl2]Cl.

133

Page 128: 1ygm3hwjpgu090312060334 tài liệu bồi dưỡng giáo viên trường thpt chuyên   mon hóa học - phan 1- năm 2012

(b) CrCl3 phản ứng với NH3 tạo thành các phức có công thức chung là CrCl3(NH3)x (x = 3 – 6). Hãy viết công thức của các phức trên và so sánh độ dẫn điện của chúng trong các dung dịch có cùng nồng độ.

(a. i/ [Cr(NH3)4Cl2]Cl: 2 đồng phân (cis, trans).

Tetraammin dicloro crom (III) clorua

ii/ [CoPy3Cl3]: 2 đồng phân (mer, fac).

Tricloro tris(pyridin) coban(III)

iii/ [Co(SCN)(H2O)5]Cl: 2 đồng phân (liên kết).

Pentaaqua thioxianato coban(II) clorua

Pentaaqua isothioxianato coban(II) clorua

iv/ [PtCl(PMe3)3]Br: cấu hình vuông phẳng, 2 đồng phân (đồng phân ion hóa)

[PtCl(PMe3)3]Br: Cloro tris (trimetylphotphin)platin (II) bromua.

[PtBr(PMe3)3]Cl: Bromo tris (trimetylphotphin)platin (II) clorua.

v/ [Co(en)2Cl2]Cl: 3 đồng phân (±cis, trans).

Dicloro bis (etylendiamin)coban(III) clorua;

b. Độ dẫn điện: [Cr(NH3)6]Cl3 > [Cr(NH3)5Cl]Cl2 > [Cr(NH3)4Cl2]Cl > [Cr(NH3)3Cl3])

134

Page 129: 1ygm3hwjpgu090312060334 tài liệu bồi dưỡng giáo viên trường thpt chuyên   mon hóa học - phan 1- năm 2012

3. Phương pháp chuẩn độ cromat

Phương pháp chuẩn độ dicromat dựa trên khả năng oxi hoá của ion dicromat trong môi trường axit:

Cr2O72 – + 14 H+ + 6e → 2 Cr3+ + 7 H2O ; E0 = 1,33V

Da cam lục

Ưu điểm:

• K2Cr2O7 là chất gốc.

• không phản ứng với Cl –.

• dung dịch K2Cr2O7 bền theo thời gian, ít bị ảnh hưởng bởi chất hữu cơ.

Nhược điểm:

• không tự chỉ thị (do cường độ mầu của Cr2O72 – không đủ lớn).

• phản ứng tương đối chậm.

Ứng dụng:

• Xác định chất khử: H2O2, S2O32 –, Fe2+, SO3

2 –, I –, rượu etylic, RSH,

vitamin C, COD.

• Xác định chất oxi hoá (chuẩn độ ngược): Fe3+, NO3 –, Cu2+, MnO4

–, IO3 –.

• Xác định chất không có tính oxi hóa – khử: Pb2+, Ba2+ (kết tủa dưới

dạng MCrO4).

Câu 3.1. Lấy 10 ml mẫu dung dịch chứa Cr(III) cho vào bình nón, thêm 5 ml dung dịch H2SO4 3M, 5 ml H3PO4 2M, 5 giọt dung dịch AgNO3 1% và 1 giọt dung dịch MnSO4 1%. Pha loãng dung dịch bằng nước cất tới khoảng 100 ml. Đun trên bếp điện, cho từng lượng nhỏ muối (NH4)2S2O8 vào, lắc dung dịch liên tục, khi dung dịch có mầu đỏ hồng (mầu của Cr2O7

2 – và MnO4 –) thì dừng cho pesunfat.

Tiếp tục đun sôi kỹ cho pesunphat phân hủy hết. Thêm từng giọt dung dịch HCl 6M vào cho tới khi dung dịch mất mầu hồng và chyển sang mầu vàng da cam thì ngừng lại. Đun tiếp 5 phút nữa để đuổi hết Cl2 ra khỏi dung dịch. Để nguội, thêm 5 ml H3PO4 2M, vài giọt chất chỉ thị diphenyl amin, chuẩn độ dung dịch thu được bằng dung dịch Fe2+ 0,032M tới khi dung dịch chuyển từ màu tím sang màu xanh nhạt thấy hết 6,75 ml.

135

Page 130: 1ygm3hwjpgu090312060334 tài liệu bồi dưỡng giáo viên trường thpt chuyên   mon hóa học - phan 1- năm 2012

(a) Viết các phương trình phản ứng xảy ra trong thí nghiệm.

(b) Cho biết vai trò của AgNO3 và MnSO4 thêm vào.

(c) Tính nồng độ của Cr (III) trong dung dịch mẫu.

(d) Nếu S2O82 – không bị phân huỷ hết thì phương pháp mắc sai số dương hay

sai số âm.

(a. 2Cr3+ + 3S2O82 – + 7H2O → Cr2O7

2 – + 6SO42 – + 14H+

2Mn2+ + 5S2O82 – + 8H2O → 2MnO4

– + 10SO42 – + 16H+

2S2O82 – + 2H2O → 4SO4

2 – + O2 + 4H+

2MnO4 – + 10Cl – + 16H+ → 2Mn2+ + 5Cl2 + 8H2O

Cr2O72 – + 6Fe2+ + 14H+ → 2Cr3+ + 6Fe3+ + 7H2O

b. AgNO3: xúc tác; MnSO4: chỉ thị

c. 0,0072M)

Câu 3.2. Dung dịch Cr2O72 – trong môi trường axit có thể oxi hoá cả etanol và

etanal thành axit etanoic (axit axetic) và ion dicromat bị khử thành Cr(III). Dung dịch AgNO3/ NH3 chỉ oxi hoá được etanal, không oxi hoá được etanol. Trong quá trình này, ion Ag+ bị khử thành bạc kim loại.

Có 500,0 ml dung dịch chứa etanol và etanal. Để xác định nồng độ của mỗi chất, đầu tiên người ta tiến hành chuẩn hoá dung dịch K2Cr2O7 trong môi trường axit bằng cách chuẩn độ với dung dịch FeSO4 (ion đicromat oxi hoá Fe2+ thành Fe3+). Chuẩn bị dung dịch chuẩn Fe2+ bằng cách hoà tan 7,43 gam FeSO4.7H2O (MW= 287,0) vào bình định mức 100,0 ml và định mức đến vạch bằng nước cất deion. Chuẩn độ 25,0 ml dung dịch này thấy hết 23,12 ml dung dịch dicromat. Tiếp đó, chuẩn độ 50,0 ml dung dịch mẫu chứa etanol và etanal thấy hết 22,45 ml dung dịch dicromat. Trong một thí nghiệm khác, thêm lượng dư dung dịch AgNO3/ NH3 vào 50,0 ml dung dịch mẫu thấy có 0,234 gam Ag tạo thành.

(a) Viết các phương trình phản ứng xảy ra trong quy trình.

(b) Tại sao phải axit hoá dung dịch dicromat?

(c) Tính nồng độ mol của dung dịch dicromat?

(d) Tính nồng độ của etanol và etanal trong dung dịch mẫu.

(a. 2Cr2O72 – + 3C2H5OH + 16H+ → 4Cr3+ + 3CH3COOH + 11H2O.

136

Page 131: 1ygm3hwjpgu090312060334 tài liệu bồi dưỡng giáo viên trường thpt chuyên   mon hóa học - phan 1- năm 2012

Cr2O72 – + 3CH3CHO + 8H+ → 2Cr3+ + 3CH3COOH + 4H2O

Cr2O72 – + 6Fe2+ + 14H+ → 2Cr3+ + 6Fe3+ + 7H2O

2Ag+ + CH3CHO + H2O → 2Ag + CH3COOH + 2H+

b. Trong môi trường trung tính hoặc kiềm, tính oxi hoá của Cr2O72 – yếu hơn

trong môi trường axit. Mặt khác, trong môi trường trung tính, sản phẩm khử là Cr(OH)3 không tan

c. Fe2+: 0,259M; Cr2O72 –: 0,0467M

d. C2H5OH: 0,716M; CH3CHO: 0,0217M)

Câu 3.3. Hàm lượng Fe2O3 trong quặng hematit được xác định bằng phương pháp chuẩn độ oxi hóa – khử. Trong một thí nghiệm, 4,00 g bột hematit được xử lý bằng dung dịch axit clohidric đặc. Phần không tan được lọc bỏ, khử toàn bộ ion sắt trong phần dung dịch thành Fe2+. Chuẩn độ dung dịch thu được (ở pH = 0) bằng dung dịch dicromat 0,100 mol/lit). Thể tích dung dịch cần đạt tới điểm tương đương là 33,74 ml.

(a) Hãy viết phương trình phản ứng của sắt (III) oxit với axit clohidric.

(b) Hãy chọn những chất khử thích hợp trong số các chất khử cho dưới đây để khử Fe(III) thành Fe(II): bromua, nitrit, sunphit, hỗn hỗng kẽm – thủy ngân và Sn2+.

Cho thế khử chuẩn của một số cặp oxi hóa – khử liên hợp như sau:

Cặp oxi hóa/ khử E° (V) Cặp oxi hóa/ khử E° (V) Cặp oxi hóa/ khử E° (V)

Zn2+/Zn –0,76 Fe3+/Fe2+ 0,77 BrO3 –/Br – 1,44

Sn4+/Sn2+ 0,15 Cr2O72 –/Cr3+ 1,33 Mn3+/Mn2+ 1,51

SO42 –/SO3

2 – 0,17 Cl2/Cl – 1,36 MnO4 –/Mn2+ 1,51

(c) Hãy viết phương trình phản ứng chuẩn độ và tính hằng số cân bằng của phản ứng.

(d) Hãy tính thành phần % theo khối lượng của Fe2O3 có trong mẫu hematit.

(e) Hãy tính thế của dung dịch tại điểm tương đương trong phép chuẩn độ trên.

(g) Hãy chọn những chất chỉ thị oxi hóa – khử thích hợp cho phép chuẩn độ trên từ các chất chỉ thị cho dưới đây.

• sắt phenantrolin (xanh nhạt/ đỏ – E° = 1,14V).

• ferroin (xanh/ đỏ da cam – E° = 1,06V).

• indigocamin (xanh/ vàng – E° = 0,29V).

137

Page 132: 1ygm3hwjpgu090312060334 tài liệu bồi dưỡng giáo viên trường thpt chuyên   mon hóa học - phan 1- năm 2012

• metylen xanh (xanh/ không mầu – E° = 0,53V).

• nilblue sunphat (tím/ không mầu – E° = 0,41V).

(h) Hãy cho biết ion clorua có bị oxi hóa trong quá trình chuẩn độ không?

Một phương pháp khác để định lượng sắt (II) là phương pháp chuẩn độ kali pemanganat. Để tránh tạo thành clo người ta thêm vào dung dịch sắt (II) 10,0 ml dung dịch bảo vệ. Dung dịch này chứa MnSO4, axit sunfuric và axit photphoric đặc.

(i) Hãy tính giá trị pH tối thiểu tại đó ion clorua bị oxi hóa thành clo bởi pemanganat. Biết nồng độ của tất cả các ion khác đều là 1 M.

(j) Hãy cho biết thế khử của cặp MnO4 –, H+/ Mn2+ thay đổi như thế nào khi

thêm MnSO4 vào dung dịch?

(k) Hãy cho biết vai trò của axit photphoric trong dung dịch bảo vệ?

(a. Fe2O3 + 6 HCl → 2 FeCl3 + 3 H2O

b. Chất khử thích hợp: Zn, Sn2+, SO32 –; do chúng có E° < 0,77 V;

c. 6 Fe2+ + Cr2O72 – + 14 H+ → 6 Fe3+ + 2 Cr3+ + 7 H2O

560592,0

)77,033,1(6

10.7,510 ==−

K

d. 40,5%

e. Tại điểm tương đương

=

=−+

++

][6][

][3][2

722

33

OCrFe

CrFe

Có: ][

][log0592,0

2

30

/ 23 +

+

+= ++Fe

FeEE

FeFe

23

142720

2/ ][

]][[log

6

0592,032

72 +

+−

+= +−Cr

HOCrEE

CrOCr

=>

23

142720

2/2

30

/ ][

]][[log0592,06

][

][log0592,07 32

7223 +

+−

+

+

+++= +−++Cr

HOCrE

Fe

FeEE

CrOCrFeFe

138

Page 133: 1ygm3hwjpgu090312060334 tài liệu bồi dưỡng giáo viên trường thpt chuyên   mon hóa học - phan 1- năm 2012

][][

][]][[log0592,06

223

3142720

2/

0

/ 3272

23 ++

++−

++= +−++FeCr

FeHOCrEE

CrOCrFeFe

][6][

][3]][[log0592,06

272

23

3142720

2/

0

/ 3272

23 −+

++−

++= +−++OCrCr

CrHOCrEE

CrOCrFeFe

][2

][log0592,06

3

140

2/

0

/ 3272

23 +

+

++= +−++Cr

HEE

CrOCrFeFe

=>

VEECr

HEE

E CrOCrFeFeCrOCrFeFe

25,17

6

7][2

][log0592,06 0

2/

0

/3

140

2/

0

/ 3272

2332

7223

=+

≈++

=+−++

+−++ +

+

g. Chất chỉ thị thích hợp: sắt phenantrolin, ferroin; do E° của chất chỉ thị nằm trong khoảng bước nhảy thế của đường cong chuẩn độ.

h. Không vì E°(Cl2/Cl –) > E°(Cr2O72 –/Cr3+);

i. 1

][1log

5

0592,051,136,1

8+×+= H;

pH = 1,58;

j. giảm

k. tạo phức với Fe3+; Fe(PO4)23 –)

Câu 3.4. Quy trình phân tích crom trong mẫu thép không gỉ chứa Fe, Cr và Mn.

Pha dung dịch chuẩn FeSO4: Hoà tan 9,9823 gam muối Mohr (FeSO4.(NH4)2SO4.6H2O) vào bình định mức 250 ml có H2SO4 và định mức đến vạch bằng nước cất.

(a) Tính nồng độ mol của dung dịch FeSO4 thu được.

Chuẩn hoá dung dịch KMnO4: Lấy 25,0 ml dung dịch FeSO4 vừa pha chế ở trên cho vào bình nón, thêm 1 ml dung dịch H3PO4 đặc (để tạo phức không mầu với Fe3+), chuẩn độ dung dịch thu được bằng dung dịch KMnO4 thấy vừa hết 24,64 ml.

(b) Tính nồng độ mol của dung dịch KMnO4.

139

Page 134: 1ygm3hwjpgu090312060334 tài liệu bồi dưỡng giáo viên trường thpt chuyên   mon hóa học - phan 1- năm 2012

Chuẩn bị mẫu: Hoà tan 0,2800 gam mẫu thép trong dung dịch hỗn hợp H3PO4

và H2SO4 đặc, đun nóng cho đến khi thu được dung dịch trong suốt mầu xanh.

(Fe → Fe3+, Cr → Cr3+, Mn → Mn2+). Làm lạnh dung dịch đến nhiệt độ phòng.

Thêm 5 ml dung dịch AgNO3 1%, 20 ml dung dịch (NH4)2S2O8 20%. Sau vài phút dung dịch có mầu hồng (pesunfat oxi hoá Mn2+ thành MnO4

–, Cr3+ thành Cr2O72 –,

Ag+ đóng vai trò làm chất xúc tác). Đun sôi dung dịch để phân huỷ hết ion pesunfat còn dư (sinh ra SO4

2 – và O2). Thêm từ từ từng giọt HCl đặc đến khi dung dịch chuyển từ mầu hồng sang mầu vàng (HCl phản ứng chọn lọc với MnO4

–). Sau khi kết thúc quá trình thấy có kết tủa trắng xuất hiện ở đáy bình.

(c) Viết phương trình phản ứng của Cr3+, Mn2+ với S2O82 – trong môi trường

axit.

(d) Viết phương trình phản ứng loại MnO4 – bằng dung dịch HCl đặc.

(e) Hãy cho biết kết tủa trắng là chất gì? Được tạo thành như thế nào?

Tiến hành chuẩn độ: Chuyển dung dịch thu được ở trên vào bình định mức 250 ml rồi định mức đến vạch bằng nước cất (dung dịch A). Lấy 50 ml dung dịch A cho vào bình nón, thêm tiếp 25,0 ml dung dịch FeSO4. Lượng FeSO4 dư được chuẩn độ bằng dung dịch KMnO4 thấy vừa hết 19,89 ml.

(g) Viết phương trình phản ứng của Fe2+ với Cr2O72 –.

(h) Tính % khối lượng của Cr trong mẫu thép (Cr = 52,0).

(a. 0,1019 M;

b. 0,02068 M;

c. 2 Cr3+ + 3 S2O82 – + 7 H2O → Cr2O7

2 – + 6 SO42 – + 14 H+

2 Mn2+ + 5 S2O82 – + 8 H2O → 2 MnO4

– + 10 SO42 – + 16 H+

d. 2 MnO4 – + 10 Cl – + 16 H+ → 2 Mn2+ + 5 Cl2 + 8 H2O

e. AgCl;

h. 15,19%)

Câu 3.5. Crom được điều chế từ quặng FeCr2O4, sắt được tách ra khỏi quặng theo hai bước:

4 FeCr2O4 + 8 Na2CO3 + 7 O2 → 8 Na2CrO4 + 2 Fe2O3 + 8 CO2

2 Na2CrO4 + H2SO4 → Na2Cr2O7 + Na2SO4 + H2O

140

Page 135: 1ygm3hwjpgu090312060334 tài liệu bồi dưỡng giáo viên trường thpt chuyên   mon hóa học - phan 1- năm 2012

Khử đicromat bằng cacbon sẽ thu được crom(III) oxit, khử oxit tạo thành bằng phương pháp nhiệt nhôm sẽ thu được crom.

Na2Cr2O7 + 2 C → Cr2O3 + Na2CO3 + CO

Cr2O3 + 2 Al → Al2O3 + 2 Cr

(a) Hãy tính khối lượng crom được tạo thành từ 2,1 tấn quặng có chứa 72,0% FeCr2O4 (Cr = 52,00; Fe = 55,85; O = 16,00). Giả thiết hiệu suất của cả quá trình là 100%.

Một mẫu thép có chứa Mn và Cr. Mn và Cr trong 5,00 gam mẫu thép được oxi hóa thành MnO4

– và Cr2O72 – và pha loãng thành 100,0 ml dung dịch. Lấy 50,0

ml dung dịch, thêm BaCl2 đến dư và điều chỉnh pH đến giá trị thích hợp, cromat kết tủa hoàn toàn và thu được 5,82 g BaCrO4. 50,0 ml dung dịch còn lại phản ứng vừa đủ với 43,5 ml dung dịch Fe2+ 1,60 M trong môi trường axit.

MnO4 – + Fe2+ + H+ → Mn2+ + Fe3+

Cr2O72 – + Fe2+ + H+ → Cr3+ + Fe3+

(b) Hãy tính thành phần % theo khối lượng của Mn và Cr trong mẫu thép (Mn = 54,94; Ba = 137,33).

(a. Sơ đồ hợp thức: FeCr2O4 → 2 Cr; mCr = 702 kg;

b. MnO4 – + 5 Fe2+ + 8 H+ → Mn2+ + 5 Fe3+ + 4 H2O

Cr2O72 – + 6 Fe2+ + 14 H+ → 2 Cr3+ + 6 Fe3+ + 7 H2O

Cr: 47,8%; Mn: 0,3%)

Câu 3.6. Hàm lượng crom trong một mẫu thép không gỉ (chứa Fe –Cr) được xác định theo quy trình sau:

1. Hòa tan 0,1000 g mẫu thép bằng 20 ml dung dịch axit sunfuric 1,8 M, đun nóng.

2. Thêm tiếp 4 ml dung dịch axit nitric đặc và đun khoảng 10 phút cho đến khi dung dịch chuyển từ mầu xanh sang mầu lục.

3. Thêm 10 ml dung dịch bạc nitrat 0,5% và 6 g amoni pesunfat ((NH4)2S2O8). Đun sôi dung dịch khoảng 20 phút để amoni sunfat bị phân hủy hoàn toàn, dung dịch thu được có mầu da cam.

4. Thêm 10 ml dung dịch NaCl 5%.

141

Page 136: 1ygm3hwjpgu090312060334 tài liệu bồi dưỡng giáo viên trường thpt chuyên   mon hóa học - phan 1- năm 2012

5. Thêm chính xác 20,00 ml dung dịch FeSO4·(NH4)2SO4 1,00×10 –1 mol lit –1.

6. Chuẩn độ dung dịch thu được bằng dung dịch KMnO4 2,00 ×10 –2 mol lit –1.

(a) Hãy viết các phương trình phản ứng xảy ra ở bước 2.

(b) Hãy viết phương trình phản ứng xảy ra ở bước 3.

(c) Hãy viết phương trình phản ứng xảy ra ở bước 4 và cho biết mục đích của quá trình.

(d) Hãy viết phương trình phản ứng xảy ra ở bước 5. Hãy cho biết mầu của dung dịch thu được.

(e) Hãy viết phương trình phản ứng xảy ra ở bước 6.

(f) Phép chuẩn độ ở bước 6 cần 12,00 ml dung dịch pemanganat để đạt tới điểm tương đương. Hãy tính thành phần % theo khối lượng của crom có trong mẫu thép.

(a. Fe2+ + 2 H+ + NO3 – → Fe3+ + NO2 + H2O

Cr2+ + 2 H+ + NO3 – → Cr3+ + NO2 + H2O

xanh lục

b. 2 Cr3+ + 3 S2O82 – + 7 H2O → Cr2O7

2 – + 6 SO42 – + 14 H+

c. Ag+ + Cl – → AgCl↓

Mục đích: loại Ag+ ra khỏi dung dịch, ngăn cản quá trình Cr3+ bị oxi hóa trở lại thành Cr2O7

2 – ở bước sau trong trường hợp amoni pesunfat không bị phân hủy hết;

d. 6 Fe2+ + Cr2O72 – + 14 H+ → 6 Fe3+ + 2 Cr3+ + 7 H2O

0,8 0,8/6 mmol

dung dịch chuyển từ mầu da cam sang mầu lục của Cr3+.

e. 5 Fe2+ + MnO4 – + 8 H+ → 5 Fe3+ + Mn2+ + 4 H2O

1,2 0,24 mmol

f. Tổng số mmol Fe2+ đưa vào dung dịch là: 20× 0,1 = 2 mmol;

=> nCr = 1,6/6 mmol;

142

Page 137: 1ygm3hwjpgu090312060334 tài liệu bồi dưỡng giáo viên trường thpt chuyên   mon hóa học - phan 1- năm 2012

=> %87,13100100

526

6,1

% ==Cr )

Câu 3.7. Để xác định hàm lượng của crom và sắt trong một mẫu gồm Cr2O3

và Fe2O3, người ta đun nóng chảy 1,98 gam mẫu với Na2O2 để oxi hóa Cr2O3 thành CrO4

2 –. Cho khối đã nung chảy vào nước, đun sôi để phân huỷ hết Na2O2. Thêm H2SO4 loãng đến dư vào hỗn hợp thu được và pha thành 100,00 ml, được dung dịch A có màu vàng da cam. Cho dung dịch KI (dư) vào 10,00 ml dung dịch A, lượng I3

– (sản phẩm của phản ứng giữa I– và I2) giải phóng ra phản ứng hết với 10,50 ml dung dịch Na2S2O3 0,40 M. Nếu cho dung dịch NaF (dư) vào 10,00 ml dung dịch A rồi nhỏ tiếp dung dịch KI đến dư thì lượng I3

– giải phóng ra chỉ phản ứng hết với 7,50 ml dung dịch Na2S2O3 0,40 M.

(a) Viết các phương trình phản ứng xảy ra.

(b) Giải thích vai trò của dung dịch NaF.

(c) Tính thành phần % khối lượng của crom và sắt trong mẫu ban đầu. Cho: Fe = 56; Cr = 52.

(a. Cr2O3 + 3 Na2O2 → 2 Na2CrO4 + Na2O (1)

2 Na2O2 + 2 H2O → O2↑ + 4 OH – + 4 Na+ (2)

OH – + H+ → H2O (3)

2 CrO42 – + 2 H+ → Cr2O7

2 – + H2O (4)

Fe2O3 + 6 H+ → 2 Fe3+ + 3 H2O (5)

Cr2O72 – + 9 I – + 14 H+ → 2 Cr3+ + 3 I3

– + 7 H2O (6)

2 Fe3+ + 3 I – → 2 Fe2+ + I3 – (7)

2 S2O32 – + I3

– → S4O62 – + 3 I – (8)

Fe3+ + 3 F – → FeF3 (9)

b. Vai trò của dung dịch NaF: F – có mặt trong dung dịch tạo phức bền, không màu với Fe3+, dùng để che Fe3+.

c. Trong 10 ml dung dịch A: Cr2O72 –: x mol; Fe3+: y mol.

3x + y/2 = 4,2.10 –3/2

3x = 3.10 –3/2

x = 5.10 –4; y = 1,2.10 –3

143

Page 138: 1ygm3hwjpgu090312060334 tài liệu bồi dưỡng giáo viên trường thpt chuyên   mon hóa học - phan 1- năm 2012

%Fe = 33,94%; %Cr = 26,27%)

Câu 3.8. Hòa tan hoàn toàn 1500 mg một mẫu hợp kim chứa bạc, đồng và crom thành 500,00 ml dung dịch chứa Ag+, Cu2+ và Cr3+ (dung dịch A). Lấy 50,00 ml dung dịch A tiến thành theo quy trình sau:

Sau khi tách bạc và đồng, ion crom (III) được oxi hóa theo phản ứng sau:

OH – + Cr3+ + H2O2 → CrO42 – + H2O

Thêm 25,00 ml dung dịch Fe2+ 0,100 M. Phản ứng xảy ra như sau:

H+ + Fe2+ + CrO42 – → Fe3+ + Cr3+ + H2O

để phản ứng với lượng Fe2+ dư cần dùng vừa đủ 17,20 ml dung dịch KMnO4

0,020 M. Phản ứng xảy ra như sau:

H+ + Fe2+ + MnO4 – → Fe3+ + Mn2+ + H2O

Trong một thí nghiệm khác, lấy 200 ml dung dịch A đem điện phân. Cả ba kim loại bị kết tủa hoàn toàn sau khi điện phân 14,50 phút với cường độ dòng 2A. Biết hiệu suất của quá trình điện phân là 90%.

Hãy viết các phương trình phản ứng xảy ra và tính thành phần % theo khối lượng của mỗi kim loại trong hợp kim đầu. Cu = 63,55; Ag = 107,87; Cr = 52,00.

(Đặt số mol Ag, Cu và Cr trong 1500 mg mẫu hợp kim ban đầu lần lượt là x, y và z. Có:

107,87x + 63,55y + 52,00z = 1,5 (1)

10 OH – + 2 Cr3+ + 3 H2O2 → 2 CrO42 – + 8 H2O

z/10 z/10

8 H+ + 3 Fe2+ + CrO42 – → 3 Fe3+ + Cr3+ + 4 H2O

3z/10 z/10

8 H+ + 5 Fe2+ + MnO4 – → 5 Fe3+ + Mn2+ + 4 H2O

Có: 1,0025,002,00172,05103

5103

42 ×=××+=+= −+

zn

zn

MnOFe (2)

Quá trình điện phân:

Ag+ + e → Ag

2x/5 2x/5

144

Page 139: 1ygm3hwjpgu090312060334 tài liệu bồi dưỡng giáo viên trường thpt chuyên   mon hóa học - phan 1- năm 2012

Cu2+ + 2e → Cu

2y/5 4y/5

Cr3+ + 3e → Cr

2z/5 6z/5

Có: 01623,096485

605,1429,0

5

642 =×××=++= zyxne (3)

Từ (1), (2) và (3) suy ra:

x = 4,25.10 –3 mol; y = 1,43.10 –2 mol; z = 2,6.10 –3 mol

=> %Ag = 30,56%; % Cu = 60,58%; % Cr = 8,86%)

Câu 3.9. Trong một nhà máy hóa chất điều chế fomandehit bằng phản ứng oxi hóa metanol, dung dịch chứa metanol và fomandehit được đem phân tích. Để kiểm tra phương pháp người ta tiến hành thí nghiệm với dung dịch chuẩn chứa metanol và fomandehit. Các dung dịch được sử dụng như sau:

• Metanol, 5,00 g/lit

• Fomandehit, 5,00 g/lit

• Kali dicromat, 3,000.10 –2 mol/lit

• Amoni sắt(II) sunphat, 0,2000 mol/lit

• Iot, 0,1000 mol/ lit

• Natri thiosunfat, 0,2000 mol/lit

I. Trộn 10,00 ml dung dịch metanol với 100,00 ml dung dịch kali dicromat, thêm khoảng 100 ml dung dịch axit sunfuric đặc và để yên khoảng 30 phút. Lượng dicromat dư được chuẩn bằng dung dịch Fe2+ với chất chỉ thị axit diphenylamin sunphonic (mầu đổi từ đỏ – tím sang xanh nhạt). Thể tích dung dịch Fe2+ cần dùng để đạt tới điểm tương đương là 43,5 ml.

II. Trộn 10,00 ml dung dịch fomandehit với 50,00 ml dung dịch iot. Thêm dung dịch NaOH cho tới khi dung dịch có môi trường kiềm và để yên khoảng 10 phút. Thêm axit clohidric để trung hòa dung dịch, lượng iot dư được chuẩn bằng dung dịch thiosunphat, với chất chỉ thị hồ tinh bột. Thể tích dung dịch thiosunfat cần dùng để đạt tới điểm tương đương là 33,3 ml.

145

Page 140: 1ygm3hwjpgu090312060334 tài liệu bồi dưỡng giáo viên trường thpt chuyên   mon hóa học - phan 1- năm 2012

(a) Hãy tính tỉ lệ số mol metanol/đicromat và fomandehit /iot tham gia phản ứng với nhau.

(b) Hãy viết phương trình phản ứng của metanol với đicromat trong môi trường axit và của fomandehit với iot trong môi trường kiềm.

III. Biết rằng iot không phản ứng với metanol. Lấy hai mẫu dung dịch chứa metanol và fomandehit chưa biết nồng độ, mỗi mẫu có thể tích 10,00 ml.

Một mẫu được trộn với 100,00 ml dung dịch kali dicromat và axit sunfuric đặc như trong thí nghiệm I. Lượng dicromat dư phản ứng vừa đủ với 4,8 ml dung dịch Fe2+.

Mẫu còn lại được trộn với 50,00 ml dung dịch iot và được tiến hành như thí nghiệm II. Lượng iot dư phản ứng vừa đủ với 16,50 ml dung dịch thiosunfat.

(c) Hãy viết các phương trình phản ứng xảy ra và tính nồng độ (theo g/lit) của metanol và fomandehit trong dung dịch.

(a. Thí nghiệm I:

CH3OH + K2Cr2O7 →...

6 Fe2+ + Cr2O72 – + 14 H+ → 6 Fe3+ + 2 Cr3+ + 7 H2O

Số mol metanol có trong 10,00 ml dung dịch là:

moln OHCH310.56,1

32

501,03

−=×=

Số mol K2Cr2O7 phản ứng với metanol là:

molnOCrK

310.55,16

2,00435,003,01,0

722

−=×−×=

=>1

1

10.55,1

10.56,13

3

722

3 == −

OCrK

OHCH

n

n

Thí nghiệm II:

HCHO + I2 → ...

I2 + 2 S2O32 – → 2 I – + S4O6

2 –

Số mol fomandehit có trong 10,00 ml dung dịch là:

molnHCHO310.67,1

30501,0 −=×=

146

Page 141: 1ygm3hwjpgu090312060334 tài liệu bồi dưỡng giáo viên trường thpt chuyên   mon hóa học - phan 1- năm 2012

Số mol I2 phản ứng với fomandehit là:

molnI310.67,1

2

2,00333,01,005,0

2

−=×−×=

=>1

1

10.67,1

10.67,13

3

2

== −

I

HCHO

n

n

b. CH3OH + Cr2O72 – + 8 H+ → CO2 + 2 Cr3+ + 6 H2O

HCHO + I2 + 3 OH – → HCOO – + 2 I – + 2 H2O

c. Đặt số mol của CH3OH và HCHO có trong 10,00 ml dung dịch mẫu lần lượt là x và y.

CH3OH + Cr2O72 – + 8 H+ → CO2 + 2 Cr3+ + 6 H2O

x x

3 HCHO + 2 Cr2O72 – + 16 H+ → 3 CO2 + 4 Cr3+ + 11 H2O

y 2y/3

6 Fe2+ + Cr2O72 – + 14 H+ → 6 Fe3+ + 2 Cr3+ + 7 H2O

Có: 03,01,010.6,13

2

63

2 42

722×=++=++= −+ y

xny

xn FeOCrK

(1)

HCHO + I2 + 3 OH – → HCOO – + 2 I – + 2 H2O

y y

I2 + 2 S2O32 – → 2 I – + S4O6

2 –

Có: 05,01,010.65,12

3232

2×=+=+= −−

yn

yn OSI

(2)

Từ (1) và (2) suy ra:

x = 6,07.10 –4 mol; y = 3,35.10 –3 mol

=> litgamlit

molgamlmolC OHCH /94,1

01,0

.32.10.07,6 114

3=×=

−−−

litgamlit

molgamlmolCHCHO /05,10

01,0

.30.10.35,3 113

=×=−−−

)

147

Page 142: 1ygm3hwjpgu090312060334 tài liệu bồi dưỡng giáo viên trường thpt chuyên   mon hóa học - phan 1- năm 2012

Câu 3.10. Dung dịch dicromat trong môi trường axit có thể oxi hóa etanol và etanal thành axit etanoic, và ion dicromat bị khử thành ion Cr(III). Dung dịch bạc (I) trong amoniac có thể oxi hóa etanal, nhưng không phản ứng với etanol.

Dung dịch X chứa etanol và etanal chưa biết nồng độ. Để xác định nồng độ của hai chất trong X, đầu tiên người ta tiến hành xác định nồng độ của dung dịch K2Cr2O7 đã được axit hóa. Dung dịch sắt (II) được điều chế bằng cách hòa tan 7,43 g FeSO4.7H2O vào nước để được 100,0 ml dung dịch. 25,0 ml dung dịch này phản ứng vừa đủ với 23,12 ml dung dịch dicromat để đạt tới điểm tương đương.

Chuẩn độ 50,0 ml dung dịch X bằng dung dịch đicromat ở trên thấy vừa hết 22,45 ml dung dịch để đạt tới điểm tương đương.

Thêm lượng dư dung dịch bạc (I) nitrat trong amoniac vào 50,0 ml dung dịch X thấy tạo 0,234 g bạc.

(a) Hãy viết các phương trình ion rút gọn của các phản ứng xảy ra.

(b) Hãy giải thích tại sao lại phải axit hóa dung dịch đicromat?

(c) Hãy tính nồng độ mol của dung dịch K2Cr2O7.

(d) Hãy tính nồng độ mol của etanol và etanal trong X.

(a. 3 C2H5OH + 2 Cr2O72 – + 16 H+ → 3 CH3COOH + 4 Cr3+ + 11 H2O

x 2x/3

3 CH3CHO + Cr2O72 – + 8 H+ → 3 CH3COOH + 2 Cr3+ + 4 H2O

y y/3

CH3CHO + 2 Ag(NH3)2+ + H2O → CH3COO – + 3 NH4

+ + NH3 + 2 Ag

y 2y

6 Fe2+ + Cr2O72 – + 14 H+ → 6 Fe3+ + 2 Cr3+ + 7 H2O

b. Axit hóa để phản ứng xảy ra hoàn toàn, sản phẩm của phản ứng là Cr3+

chứ không phải kết tủa Cr(OH)3.

c. Có: MCFe

2672,01,005,278

43,72 =

×=+

=> MCOCrK

04815,012,23

6

1252672,0

722=

××=

148

Page 143: 1ygm3hwjpgu090312060334 tài liệu bồi dưỡng giáo viên trường thpt chuyên   mon hóa học - phan 1- năm 2012

d. Đặt số mol của etanol và etanal trong 50,0 ml dung dịch X lần lượt là x và y. Có:

molyx

nOCrK

310.081,102245,004815,033

2722

−=×=+= (1)

molynAg310.169,2

87,107

234,02 −===

(2)

Từ (1) và (2) suy ra:

x = 1,079.10 –3 mol; y = 1,085.10 –3 mol;

=> lmolC OHHC /0216,005,0

10.079,1 3

52==

lmolC CHOCH /0217,005,0

10.085,1 3

3==

)

Câu 3.11. Oxi trong không khí oxi hóa chậm Fe(II) trong dung dịch axit thành sắt (III).

(a) Hãy viết phương trình phản ứng oxi hóa Fe2+ bằng oxi.

(b) Hãy cho biết pH của dung dịch ảnh hưởng thế nào đến phản ứng ở (a).

Kesan, một nhà hóa học trẻ, quyết định nghiên cứu ảnh hưởng của oxi không khí đến dung dịch sắt (II) peclorat. Kesan cân 13,4 gam mẫu Fe(ClO4)2.xH2O và hòa tan vào nước để tạo thành 200,0 ml dung dịch A.

Thuốc thử H3L, một phối tử hữu cơ, phản ứng hoàn toàn với ion Fe(III) theo

phương trình: Fe3+ + 2 H3L (aq) → [FeL2]3– + 6 H+

Biết H3L có mầu lục thẫm, phức [FeL2]3– có màu vàng nhạt và H3L không tạo phức với ion sắt (II). Do đó có thể sử dụng H3L để chuẩn độ định lượng ion Fe(III) khi có mặt ion Fe(II). Tại điểm tương đương dung dịch chuyển từ mầu vàng nhạt sang mầu lục nhạt.

Lấy 20,0 ml dung dịch A cho vào bình nón. Tiến hành oxi hóa hoàn toàn sắt (II) thành sắt (III) bằng lượng dư dung dịch kali dicromat trong môi trường axit, ion Cr(III) sinh ra cũng phản ứng với H3L giống như ion Fe(III). Chuẩn độ dung dịch thu được bằng dung dịch H3L 0,3182 M thấy cần dùng 30,20 ml dung dịch để đạt tới điểm tương đương.

(c) Hãy cho biết số oxi hóa của crom trong kali đicromat.

149

Page 144: 1ygm3hwjpgu090312060334 tài liệu bồi dưỡng giáo viên trường thpt chuyên   mon hóa học - phan 1- năm 2012

(d) Hãy viết phương trình ion rút gọn của phản ứng oxi hóa Fe(II) thành sắt (III) bằng đicromat.

(e) Hãy tính nồng độ mol của Fe(II) trong dung dịch A. Giả thiết rằng trong dung dịch A không có mặt Fe(III).

(g) Hãy tính giá trị của x.

Để xác định ảnh hưởng của oxi không khí đến dung dịch sắt (II) peclorat, Kesan lấy hai mẫu dung dịch A đều có thể tích 20,0 ml cho vào hai bình nón sạch, kí hiệu là 1 và 2. Tiến hành chuẩn độ ngay bình nón 1 bằng dung dịch H3L 0,3182 M, kết quả cho thấy mầu lục bền xuất hiện ngay sau khi thêm một giọt dung dịch thuốc thử. Bình nón 2 được để ngoài không khí trong vòng 1 tuần, sau đó tiến hành chuẩn độ thấy hết 11,35 ml dung dịch H3L 0,3182 M để đạt tới điểm tương đương.

(h) Hãy giải thích kết quả khi tiến hành thí nghiệm với bình nón 1.

(i) Hãy tính nồng độ mol của Fe(III) trong bình nón 2.

(k) Hãy tính % lượng ion Fe(II) còn lại trong bình nón 2 so với lượng Fe(II) ban đầu.

(l) Nếu bình nón 2 được để ngoài không khí hai tuần thay vì một tuần, hãy cho biết thể tích dung dịch H3L cần dùng để đạt tới điểm tương đương sẽ lớn hơn, nhỏ hơn hay bằng 22,70 ml?

(a. O2 + 4 Fe2+ + 4 H+ → 4 Fe3+ + 2 H2O

b. Với điều kiện Fe3+ và Fe2+ chưa bị thủy phân thì khi pH tăng thì hiệu suất phản ứng giảm.

c. +6

d. 6 Fe2+ + Cr2O72 – + 14 H+ → 6 Fe3+ + 2 Cr3+ + 7 H2O

x x x/3

e. Đặt số mol Fe2+ có trong 20,0 ml dung dịch A là x.

Fe2+ + 2 H3L (aq) → [FeL2]3– + 6 H+

x 2x

Cr3+ + 2 H3L (aq) → [CrL2]3– + 6 H+

x/3 2x/3

150

Page 145: 1ygm3hwjpgu090312060334 tài liệu bồi dưỡng giáo viên trường thpt chuyên   mon hóa học - phan 1- năm 2012

Có: molx

xn LH ×=×=+= −310.610,93182,003020,03

22

3

=> x = 3,604.10 –3 mol

=> MCFe

1802,00200,0

10.604,3 3

2 ==−

+

g. Có: MxCFe

2180,02,0

02,1875,254

4,13

2 =+=+

=> x = 6,52

h. Kết quả thực nghiệm chứng tỏ trong bình nón 1 không có mặt Fe(III).

i. MCFe

0903,00,20

2

13182,035,11

3 =××

=+

k. %Fe(II) còn lại0 1802 0 0903

100 49 890 1802

, ,, %

,

−= × =

l. Khi toàn bộ lượng Fe2+ bị chuyển thành Fe3+ thì thể tích dung dịch H3L cần dùng để phản ứng vừa đủ với lượng Fe3+ sinh ra là:

mlV LH 65,223182,0

20,201802,0max,3

=××=

Vậy sau hai tuần thì thể tích dung dịch H3L cần dùng sẽ nhỏ hơn 22,70 ml)

Câu 3.12. Cho giản đồ Latimer của crom trong môi trường axit (pH = 0) như sau:

Cr2O72- Cr(V) Cr(IV) [Cr(H2O)6]

3+ [Cr(H2O)6]2+ Cr

+0,293

+0,55 +2,1 -0,912

da cam ®á lôctÝmxanh-0,744

X Y

(a) Hãy tính các giá trị X và Y trong giản đồ trên.

(b) Hãy cho biết Cr (IV) có bị phân hủy dị ly thành Cr (III) và Cr (VI) không?

(c) Cặp Cr2O72 –/Cr3+ được sử dụng làm chất oxi hóa. Hãy viết bán phản ứng

của cặp trên.

151

Page 146: 1ygm3hwjpgu090312060334 tài liệu bồi dưỡng giáo viên trường thpt chuyên   mon hóa học - phan 1- năm 2012

(d) Hãy tính sự thay đổi thế của cặp trên nếu pH của dung dịch tăng từ 1 đến 3 (T = 298 K, E°(Cr2O7

2 –/Cr3+) = 1,33 V). Biết trong khoảng pH này nồng độ của Cr2O7

2 – và Cr3+ coi như không thay đổi.

(e) Chuẩn độ 100 ml dung dịch kali dicromat 0,01667 M bằng dung dịch Fe(II) 0,1 M. Hãy tính thế khử của dung dịch thu được sau khi thêm 100 ml dung dịch Fe(II) 0,1 M. Biết trạng thái cân bằng có pH=1; E° (Fe3+ /Fe2+) = 0,77V.

Trong một thí nghiệm, một bình điện phân chứa 150 lit dung dịch axit cromit H2CrO4. Quá trình điện phân được thực hiện trong khoảng thời gian 8 giờ với cường độ dòng điện 2000A. Vật cần mạ đóng vai trò là catot được phủ 350g crom sau khi thí nghiệm kết thúc. Tại catot bên cạnh crom kết tủa còn xảy ra phản ứng giải phóng khí. Do đó hiệu suất của quá trình điện phân tạo crom nhỏ hơn 100%.

(g) Hãy viết bán phản ứng tạo crom ở catot và tính hiệu suất của quá trình điện phân tạo crom.

(h) Hãy viết các bán phản ứng tạo khí ở catot và anot và tính thể tích khí thoát ra ở anot và catot ở 298 K và 1,013.105 N/m2.

(i) Một loại thuốc trừ sâu chứa đồng (II) oleat Cu(C18H33O2)2 (M = 626). Cho 9,9 g mẫu thuốc trừ sâu này vào hỗn hợp axit sunfuric và axit nitric đặc đun nóng để phá hủy hết phần hữu cơ. Thêm lượng dư kali cromat vào dung dịch thu được để kết tủa đồng dưới dạng muối cromat bazơ CuCrO4.2CuO.2H2O. Chất kết tủa được lọc, rửa và hòa tan trong dung dịch axit:

2 (CuCrO4.2CuO.2H2O) (r) + 10 H+ (aq.) → 6 Cu2+(aq.) + Cr2O72 – (aq.) + 9 H2O

Chuẩn độ lượng đicromat sinh ra cần 15,7 ml dung dịch Fe2+ 0,232 M. Hãy tính thành phần % theo khối lượng của đồng (II) oleat có trong mẫu.

(a. Có: 6 × 0,293 = 0,55 + x + 2,1 + 3 × (–0,744)

=> x = 1,34 V

Có: 3 × (–0,744) = y + 2 × (–0,912)

=> y = –0,408 V;

b. 3 Cr(IV) → 2 Cr(III) + Cr(VI)

Có: VEIVCrOCr

945,02

34,155,00

)(/272

=+=−

Cách 1: 390592,0

)945,01,2(2

10.05,110 ==−×

K

152

Page 147: 1ygm3hwjpgu090312060334 tài liệu bồi dưỡng giáo viên trường thpt chuyên   mon hóa học - phan 1- năm 2012

Cách 2: ∆G0 = –nFE0pin = – 2 × 96485 × (2,1 – 0,945) = –222,88 kJ/mol < 0.

Vậy Cr(IV) tự phân hủy dị ly tạo Cr(III) và Cr(VI).

c. Cr2O72 – + 14 H+ + 6 e → 2 Cr3+ + 7 H2O

d. pH = 1:

VCr

HOCrEE

CrOCrCrOCr19,1)10lg(

6

0592,033,1

][

]][[lg

6

0592,0 14123

142720

// 3272

3272

=+=+= −+

+−

+−+−

pH = 3:

VCr

HOCrEE

CrOCrCrOCr92,0)10lg(

6

0592,033,1

][

]][[lg

6

0592,0 14323

142720

// 3272

3272

=+=+= −+

+−

+−+−

Vậy khi pH tăng từ 1 đến 3 thì thế khử của hệ giảm 0,27V;

e. Cr2O72 – + 6 Fe2+ + 14 H+ → 2 Cr3+ + 6 Fe3+ + 7 H2O

560592,0

)77,033,1(6

10.71,510 ==−×

K

bđ: 8,335 50 mM

pư 8,335 50 16,67 50 mM

cb – – 0,1 16,67 50 mM

Có: ( ) ( ) 56

146272

272

6323

10.71,51,0])[6(][

10.00,5010.67,16 =××

×= −−

−−

OCrOCrK

=> [Cr2O72 –] = 3,997.10 –9 M

=>

VCr

HOCrEEE

CrOCrCrOCrdd 14,1)10.67,16(

)10(10.997,3lg

6

0592,033,1

][

]][[lg

6

0592,023

1419

23

142720

// 3272

3272

=+=+== −

−−

+

+−

+−+−

Hoặc: [Fe2+] = 6[Cr2O72 –] = 2,398.10 –8 M

VFe

FeEEE

FeFeFeFedd 14,110.398,2

10.50lg0592,077,0

][

][lg0592,0

8

3

2

30

// 2323 =+=+== −

+

+

++++

g. Catot: CrO42 – + 8 H+ + 6 e → Cr + 4 H2O

molne 59796485

360082000 =××=

153

Page 148: 1ygm3hwjpgu090312060334 tài liệu bồi dưỡng giáo viên trường thpt chuyên   mon hóa học - phan 1- năm 2012

=>%76,6100

00,526

1597

350 =×××

h. Catot: 2 H+ + 2 e → H2

Hoặc: 2 H2O + 2e → H2 + 2 OH –

Số mol e dùng để điện phân H2CrO4.

moln 4,40600,52

3501 =×=

=> Số mol e dùng để điện phân nước tạo H2:

moln 6,5564,405972 =−=

=> 3807,66807013,1

29808314,02

6,556

2mlit

BarVH ==

××=

Anot: 2 H2O → O2 + 4 H+ + 4 e

365,33650013,1

29808314,04

597

2mlit

BarVO ==

××=

i. %03,231009,9

626326

1232,00157,0

)(% 223318 =××××××

=OHCCu )

4. Hợp chất CrO5

Cr O

O O-1

+6

-1

O O

H2CrO4 + 2 H2O2 H2SO4 CrO5 (ete) + 3 H2O

xanh

Câu 4.1. Để phân tích định tích crom người ta cho dung dịch dicromat trong môi trường axit tác dụng với H2O2 để tạo ra hợp chất CrO5 kém bền, mầu xanh. Hợp chất này được bền hóa bằng cách chiết vào ete hoặc thêm phối tử một càng như pyridin (py) để tạo ra phức tháp ngũ giác CrO5(py).

154

Page 149: 1ygm3hwjpgu090312060334 tài liệu bồi dưỡng giáo viên trường thpt chuyên   mon hóa học - phan 1- năm 2012

(a) Hãy viết phương trình phản ứng tạo thành CrO5.

(b) Hãy cho biết số oxi hóa của crom trong CrO5 và trong CrO5(py)? Hãy vẽ cấu trúc không gian của CrO5(py).

(a. Cr2O72 – + 4 H2O2 + 2 H+ H2SO4 2 CrO5 (ete) + 5 H2O

b. +6 và +6;

)

Câu 4.2. (a) Dự đoán hiện tượng, viết phương trình ion của các phản ứng xảy ra trong các thí nghiệm sau:

i/ Thêm ít K2Cr2O7 lần lượt vào mỗi dung dịch: Ba(CH3COO)2; (NH4)2CO3.

ii/ Thêm ít KNO2 lần lượt vào mỗi dung dịch: KMnO4 + H2SO4; KI + H2SO4.

Nêu vai trò của K2Cr2O7 và KNO2 trong các phản ứng trên.

(b) Thêm H2O2 vào dung dịch K2Cr2O7 + H2SO4 sinh ra sản phẩm trung gian Cr3(O2)3+ ít bền màu xanh ve dễ bị phân huỷ cho ion Cr3+. Viết phương trình ion của các phản ứng.

(c) Tính Eo của cặp Cr2O72 –/Cr(OH)3.

Cho: EoCr2O72 –/ 2 Cr3+ = 1,33V; Eo

MnO4 –/ Mn2+ = 1,51 V;

EoHNO2/ NO = 1,0 V; Eo

NO3 –/ HNO2 = 0,94 V; EoI3 –/ 3I – = 0,55 V.

CH3COOH có pKa = 4,76; H2CO3 có pKa1 = 6,35; pKa2 = 10,33;

H2CrO4 có pKa1 = 0,61, pKa2 = 6,50.

Cr2O72 – + H2O 2 HCrO4

– K = 10 –1,36

Tích số tan của Cr(OH)3 bằng 10 –30,8; của BaCrO4 bằng 10 –9,9.

Câu 4.3. Cho giản đồ quá trình khử – thế khử: quá trình khử diễn ra theo chiều mũi tên, thế khử chuẩn được ghi trên các mũi tên và đo ở pH = 0.

155

Page 150: 1ygm3hwjpgu090312060334 tài liệu bồi dưỡng giáo viên trường thpt chuyên   mon hóa học - phan 1- năm 2012

Cr2O72- Cr(V) Cr(IV) Cr

+0,293

+0,55 +1,34 E0x -0,408 E0

y

-0,744

Cr3+ Cr2+

(a) Tính E0x và E0

y.

(b) Dựa vào tính toán, cho biết Cr(IV) có thể dị phân thành Cr3+ và Cr(VI) được không?

(c) Viết quá trình xảy ra với hệ oxi hóa – khử Cr2O72 –/ Cr3+ và tính độ biến

thiên thế của hệ ở nhiệt độ 298 K, khi pH tăng 1 đơn vị.

(d) Phản ứng giữa K2Cr2O7 với H2O2 trong môi trường axit (loãng) được dùng để nhận biết crom vì sản phẩm tạo thành có màu xanh. Viết phương trình ion của phản ứng xảy ra và cho biết phản ứng này có thuộc loại phản ứng oxi hóa – khử hay không? Ghi số oxi hóa tương ứng trên mỗi nguyên tố.

Cho: VEo

CrOCr33,132

72 /=+− ; R = 8,3145 J.K–1.mol–1; F = 96485 C.mol–1.

(a. Có: 6 × 0,293 = 0,55 + 1,34 + E0x + 3 × (–0,744)

=> E0x = 2,1 V

Có: 3 × (–0,744) = –0,408 + 2E0y

=> E0y = –0,912 V;

b. 3 Cr(IV) → 2 Cr(III) + Cr(VI)

Có: VEIVCrOCr

945,02

34,155,00

)(/272

=+=−

Cách 1: 390592,0

)945,01,2(2

10.05,110 ==−×

K

Cách 2: ∆G0 = –nFE0pin = – 2 × 96485 × (2,1 – 0,945) = –222,88 kJ/mol < 0.

Vậy Cr(IV) tự phân hủy dị ly tạo Cr(III) và Cr(VI).

c. Cr2O72 – + 14 H+ + 6 e → 2 Cr3+ + 7 H2O

d. pH = 0: 33,10

// 3272

3272

== +−+− CrOCrCrOCrEE

pH = 1:

156

Page 151: 1ygm3hwjpgu090312060334 tài liệu bồi dưỡng giáo viên trường thpt chuyên   mon hóa học - phan 1- năm 2012

VCr

HOCrEE

CrOCrCrOCr19,1)10lg(

6

0592,033,1

][

]][[lg

6

0592,0 14123

142720

// 3272

3272

=+=+= −+

+−

+−+−

Vậy khi pH tăng 1 đơn vị thì thế khử của hệ giảm 0,14V;

d. Cr2O72 – + 4 H2O2 + 2 H+ → 2 CrO5 + 5 H2O

Không phải phản ứng oxi hóa khử do không có nguyên tố nào thay đổi số oxi hóa

Cr O

O O-1

+6

-1

O O )

Câu 4.4. Cặp dicromat /crom(III) ( VECrOCr

380,10

/ 3272

+=+− ) thường được sử

dụng trong hóa phân tích.

(a) Hãy viết bán phản ứng oxi hóa – khử của cặp trên và cho biết dung dịch crom(III)/ dicromat (c(Cr3+)=c(Cr2O7

2 –)= 1 mol/lit) có oxi hóa được iođua thành iot từ dung dịch trung hòa có E(I2/I–) = E°(I2/I–) = +0,54 V không.

Mầu vàng đậm của ion cromat và mầu da cam đậm của ion dicromat thường

được sử dụng để phát hiện ra crom. Ví dụ crom(III) oxit được đun nóng chảy với

kali nitrat và natri cacbonat, dung dịch Cr(III) phản ứng với hydro peoxit, natri

peoxo đisunphat, brom là những thí nghiệm được dùng để phát hiện Cr(III).

(b) Hãy viết phương trình của bốn phản ứng trên. Hãy cho biết vai trò của

natri nitrat và natri cacbonat trong phản ứng đầu tiên.

Căn cứ vào thế khử chuẩn của cặp brom/ bromua (E° (Br2/Br–) = +1,065 V)

có thể thấy phản ứng giữa Cr3+ với brom là không xảy ra ở điều kiện chuẩn.

(c) Hãy tính giá trị pH tối thiểu cần thiết lập để phản ứng xảy ra. Biết nồng độ

của các cấu tử khác đều bằng 1 M.

Một số cấu tử có mặt trong dung dịch có thể gây cản trở đến phép phân tích

định tính Cr(III). Ví dụ nếu sử dụng hidro peoxit thì các ion bromua và iođua sẽ

gây cản trở, còn nếu sử dụng brom thì Mn2+ sẽ gây cản trở.

(d) Hãy giải thích tại sao các cấu tử trên lại gây cản trở đến phép phân tích.

Một phản ứng khác để phát hiện crom là phản ứng của dicromat với hidro

peoxit. Sản phẩm tạo thành có mầu xanh thẫm:

157

Page 152: 1ygm3hwjpgu090312060334 tài liệu bồi dưỡng giáo viên trường thpt chuyên   mon hóa học - phan 1- năm 2012

Cr2O72– + 4 H2O2 + 2 H+ → 2 CrO5 + 5 H2O

(e) Hãy cho biết chất oxi hóa và chất khử trong phản ứng trên.

(a. Cr2O72 – + 14 H+ + 6 e → 2 Cr3+ + 7 H2O

Có:

VCr

HOCrEE

CrOCrCrOCr41,0)10lg(

6

0592,038,1

][

]][[lg

6

0592,0 14723

142720

// 3272

3272

=+=+= −+

+−

+−+−

=> −+− <IICrOCr

EE// 2

3272

, tức phản ứng oxi hóa iođua không xảy ra.

b. Cr2O3 + 3 KNO3 + 2 K2CO3 → 2 K2CrO4 + 3 KNO2 + 2 CO2

KNO3: chất oxi hóa

K2CO3: làm giảm nhiệt độ nóng chảy của hỗn hợp

2 Cr(OH)3 + 3 H2O2 + 4 OH – → 2 CrO42 – + 8 H2O

Hoặc: 2 Cr3+ + 3 H2O2 + H2O → Cr2O72 – + 8 H+

2 Cr3+ + 3 S2O82 – + 7 H2O → Cr2O7

2 – + 6 SO42 – + 14 H+

Hoặc: 2 Cr(OH)3 + 3 S2O82 – + 10 OH – → 2 CrO4

2 – + 6 SO42 – + 8 H2O

2 Cr(OH)3 + 3 Br2 + 10 OH – → 2 CrO42 – + 6 Br – + 8 H2O

Hoặc: 2 Cr3+ + 3 Br2 + 7 H2O → Cr2O72 – + 6 Br – + 14 H+

c. Để phản ứng xảy ra được thì −+− <BrBrCrOCr

EE// 2

3272

=> VHECrOCr

065,1]lg[6

0592,038,1 14

/ 3272

<+= ++−

=> [H+] < 5,24.10 –3 (pH > 2,28)

Vậy ở pH > 2,28 thì Br2 oxi hóa được Cr3+.

d. H2O2 + 2 Br – → Br2 + 2 OH –

H2O2 + 2 I – → I2 + 2 OH –

Br2 và I2 sinh ra có mầu làm cản trở sự quan sát mầu của cromat.

5 Br2 + 2 Mn(OH)2 + 12 OH – → 2 MnO4 – + 10 Br – + 8 H2O

Mầu tím của pemanganat làm cản trở sự quan sát mầu của cromat.

158

Page 153: 1ygm3hwjpgu090312060334 tài liệu bồi dưỡng giáo viên trường thpt chuyên   mon hóa học - phan 1- năm 2012

e. Không có chất oxi hóa và không có chất khử do không có nguyên tố nào thay đổi số oxi hóa

Cr O

O O-1

+6

-1

O O )

5. Ứng dụng của crom

Việc đưa crom vào thép làm tăng cao độ cứng, độ bền nhiệt, độ bền ăn mòn và độ bền hóa chất của các loại thép. Ví dụ thép không gỉ (inox) chứa 18 – 25% Cr.

Crom kim loại được sử dụng rộng rãi để mạ lên bề mặt kim loại làm lớp bảo vệ chống ăn mòn cho các dụng cụ, máy móc, thiết bị, đồ gia dụng...

Nhiều hợp chất của crom được sử dụng làm chất tạo mầu, ví dụ chì cromat PbCrO4 được sử dụng làm chất tạo mầu vàng cho sơn, chì cromat bazơ (PbCrO4·Pb(OH)2) được sử dụng làm chất tạo mầu đỏ. Cr2O3 được sử dụng làm chất tạo mầu xanh trong ngành công nghiệp kính và gốm. Cr(III) được sử dụng làm chất tạo mầu đỏ cho đá ruby nhân tạo.

Muối cromat được sử dụng để bảo quản gỗ khỏi mối, mọt. Muối crom(III) được sử dụng trong ngày thuộc da. Cromit và crom(III) oxit được sử dụng làm vật liệu chịu lửa. Một vài hợp chất của crom được sử dụng làm chất xúc tác. Axit cromic là chất oxi hóa mạnh thường được sử dụng làm chất tẩy rửa dụng cụ thủy tinh trong phòng thí nghiệm.

Câu 5.1. Quá trình mạ crom được thực hiện bằng cách điện phân dung dịch axit cromic. Vật cần mạ đóng vai trò catot và anot là điện cực trơ. Một bình điện phân chứa 100,0 lit dung dịch CrO3 có nồng độ 0,230 kg/ l.

Điện phân dung dịch với cường độ dòng 1500 A trong khoảng thời gian 10,0 giờ. Sau quá trình điện phân khối lượng catot tăng 0,679 kg.

(a) Hãy cho biết hiệu suất của quá trình điện phân điều chế crom.

(b) Hãy tính tỉ lệ thể tích khí thoát ra ở catot và ở anot (ở cùng điều kiện).

Thực nghiệm cho thấy tỉ lệ thể tích khí thoát ra ở catot và anot (ở cùng điều kiện) là:

159

Page 154: 1ygm3hwjpgu090312060334 tài liệu bồi dưỡng giáo viên trường thpt chuyên   mon hóa học - phan 1- năm 2012

603,1=A

C

V

V

(c) Hãy giải thích sự khác nhau giữa giá trị tính được ở (b) và giá trị thực nghiệm.

(d) Hãy tính thể tích khí thực tế sinh ra ở anot và ở catot (ở đktc).

(a. Có: molF

Itne 7,559

96485

3600101500 =××==

CrO42 – + 8 H+ + 6 e → Cr + 4 H2O

Số mol electron do cromat nhận là:

molne 35,78600,52

679' =×=

Hiệu suất của quá trình điện phân điều chế crom:

%0,141007,559

35,78'

=×==e

e

n

nH

b. Giả thiết rằng chỉ có khí hidro thoát ra ở catot và oxi thoát ra ở anot.

Anot (+): 2 H2O → O2↑ + 4 H+ + 4e

Catot (–): 2 H+ + 2e → H2↑

Có: molnn

n eeH 7,240

2

35,787,559

2

'

2=−=−=

moln

n eO 9,139

4

7,559

42===

=> 721,12

2 ==O

H

A

C

n

n

V

V

c. Thực tế có những quá trình phụ khác xảy ra, ví dụ:

CrO42 – + 8 H+ + 3e → Cr3+ + 4 H2O

d. CrO42 – + 8 H+ + 6 e → Cr + 4 H2O

78,35 mol

2 H+ + 2e → H2↑

2x x

160

Page 155: 1ygm3hwjpgu090312060334 tài liệu bồi dưỡng giáo viên trường thpt chuyên   mon hóa học - phan 1- năm 2012

2 H2O → O2↑ + 4 H+ + 4e

y 4y

Có: 78,35 + 2x = 4y

603,12

2 ===y

x

n

n

V

V

O

H

A

C

=> x = 158,2; y = 98,7

=> VH2 = 158,2× 22,4 = 3,54.103 lit

VO2 = 98,7× 22,4 = 2,21.103 lit)

Câu 5.2. Chì cromat được sử dụng rộng rãi làm chất mầu, tuy nhiên cả hai thành phần có mặt trong chất này đều có độc tính đối với người.

(a) Một mẫu nước ngầm được bão hòa PbCrO4 (r) và có pH = 6,00. Hãy tính nồng độ cân bằng của Pb2+, CrO4

2–, HCrO4– và Cr2O7

2–. Cho các hằng số cân bằng:132

42 10.82,2]][[ −−+ == CrOPbKsp

7

4

24

2 10.34,3][

]][[ −−

−+

==HCrO

CrOHKa

1422

42

272 10.13,3

][][

][ == −+

CrOH

OCrKD

1410.00,1]][[ −−+ == OHHKW

(b) Biết trong dạ dày của một người bị nhiễm độc crom có nồng độ cân bằng của HCrO4

– và Cr2O72– bằng nhau. Giả thiết dịch dạ dày có pH = 3,0. Hãy tính

nồng độ tổng cộng của crom hòa tan có trong dạ dày của người này.

(a. Các cân bằng xảy ra:

PbCrO4 (r) Pb2+ + CrO42 – 132

42 10.82,2]][[ −−+ == CrOPbKsp

(1)

CrO42 – + H+ HCrO4

– 7

4

24

2 10.34,3][

]][[ −−

−+

==HCrO

CrOHKa

(2)

161

Page 156: 1ygm3hwjpgu090312060334 tài liệu bồi dưỡng giáo viên trường thpt chuyên   mon hóa học - phan 1- năm 2012

2 CrO42 – + 2 H+ Cr2O7

2 – + H2O 1422

42

272 10.13,3

][][

][ == −+

CrOH

OCrKD

(3)

Có: S = [Pb2+] = [CrO42 –] + [HCrO4

–] + 2 [Cr2O72 –] (4)

(2) => ][994,210.34,3

][10]][[][ 2

47

24

6

2

24

4−

−−−+− === CrO

CrO

K

CrOHHCrO

a

(3) =>22

422

4261422

422

72 ][313][)10(10.13,3][][][ −−−−+− =×== CrOCrOCrOHKOCr D

(4) => 2-24

-24

-24-2

4

13

][CrO1332 ]2,994[CrO ][CrO ][CrO

10.82,2 ×++=−

=> 010.82,2][CrO994,3][CrO626 132-24

3-24 =−+ −

=> [CrO42 –] = 2,66.10 –7 M

MPb 67-

132 10.06,1

2,66.10

10.82,2][ −

−+ ==

MHCrO 774 10.96,710.66,2994,2][ −−− =×=

MOCr 1127272 10.21,2)10.66,2(313][ −−− =×=

b. Có: ][299410.34,3

][10]][[][ 2

47

24

3

2

24

4−

−−−+− === CrO

CrO

K

CrOHHCrO

a

224

8224

2314224

2272 ][10.13,3][)10(10.13,3][][][ −−−−+− =×== CrOCrOCrOHKOCr D

=> 224

824 ][10.13,3][2994 −− = CrOCrO

=> [CrO42 –] = 9,57.10 –6 M

MHCrOOCr 02864,010.57,92994][][ 64

272 =×== −−−

=> CCr = [CrO42 –] + [HCrO4

–] + 2 [Cr2O72 –]

= 9,57.10 –6 + 3× 0,02864 = 0,0860 M)

Câu 5.3. Các hợp chất của crom (VI), đặc biệt là các hidroxit có tính oxi hóa rất mạnh. Trong phòng thí nghiệm người ta lợi dụng tính oxi hóa của crom(VI) để loại trừ khí hidro sunfua. Trong một thí nghiệm người ta sục khí CO2 có lẫn H2S

162

Page 157: 1ygm3hwjpgu090312060334 tài liệu bồi dưỡng giáo viên trường thpt chuyên   mon hóa học - phan 1- năm 2012

vào dung dịch kali dicromat có mặt axit sunfuric, chất không tan mầu vàng được tạo thành và dung dịch chuyển sang mầu xanh.

(a) Hãy viết công thức của các hidroxit crom(VI) mà bạn biết.

(b) Hãy viết phản ứng xảy ra trong quá trình tinh chế khí CO2.

(c) Hãy cho biết có thể dùng phương pháp trên để loại khí SO2 lẫn trong khí CO2 được không? Nếu được hãy viết phương trình phản ứng xảy ra.

(a. H2CrO4; H2Cr2O7; H2Cr3O10; H2Cr4O13;... nCrO3.CrO2(OH)2;

b. 3 H2S + K2Cr2O7 + 4 H2SO4 → Cr2(SO4)3 + K2SO4 + 3 S + 7 H2O

c. 3 SO2 + K2Cr2O7 + H2SO4 → Cr2(SO4)3 + K2SO4 + H2O

Câu 5.4. (a) Trong môi trường axit, ion cromat màu vàng (CrO42 –) chuyển

hóa thành ion dicromat màu da cam (Cr2O72 –). Hãy viết phương trình phản ứng.

(b) Số oxi hóa của nguyên tử kim loại trong ion cromat và ion dicromat bằng bao nhiêu?

(c) Phản ứng viết ở (a) có phải là phản ứng oxi hóa khử không? Giải thích.

(d) Hãy cho biết yếu tố chính ảnh hưởng đến vị trí của cân bằng trên.

(e) Hãy vẽ cấu trúc hình học của CrO42 – và Cr2O7

2 –.

Vật cần mạ crom được nhúng vào dung dịch Cr2O72 – trong môi trường axit,

vật này đóng vai trò là catot trong bình điện phân. (Cr = 51,996; 1 F = 96485 C)

(f) Biết rằng tại anot xảy ra quá trình oxi hóa nước, hãy viết các bán phản ứng xảy ra ở mỗi điện cực và phản ứng tổng cộng xảy ra trong quá trình điện phân.

(g) Có bao nhiêu mol khí oxi thoát ra khi có 52,0 g crom kết tủa tại điện cực?

(h) Nếu cường độ dòng điện là 10,0 A thì phải mất bao lâu để thu được 52,0 g crom?

(i) Xét về mặt hóa học thì tại sao crom lại thường được sử dụng để mạ trên bề mặt các kim loại?

(a. 2 CrO42 – + 2 H+ Cr2O7

2 – + H2O

b. +6, +6;

c. không, không có nguyên tố nào thay đổi số oxi hóa.

d. pH;

163

Page 158: 1ygm3hwjpgu090312060334 tài liệu bồi dưỡng giáo viên trường thpt chuyên   mon hóa học - phan 1- năm 2012

e.

O

CrO

OO

O

CrO

OO

O

CrO

O

f. Anot (+) H2O → 1/2 O2 + 2H+ + 2e

Catot (–) Cr2O72 – + 14 H+ + 12e → 2 Cr + 7 H2O

Cr2O72 – + 2 H+ → 2 Cr + H2O + 3 O2

1 1,5

g. 1,5 mol;

h. 6==F

Itne

=> hsI

Ft 1,1657891

10

9648566 ==×==

i. Crom phản ứng với oxi tạo ra lớp màng oxit Cr2O3 rất mỏng, bền và trong suốt, giúp bảo vệ bề mặt kim loại khỏi bị ăn mòn)

Câu 5.5. Cho quy trình điều chế 4 chất mầu vô cơ như sau:

1) Đun nóng hỗn hợp dạng bột mịn chứa 5,0 g natri dicromat và 0,62 g lưu huỳnh ở 800°C. Hòa tan sản phẩm vào nước.

2) Đun nóng hỗn hợp dạng bột mịn chứa 3,0 g chì (II) oxit và 1,0 g thiếc (IV) oxit ở 650°C.

3) Đun nóng chì (II) oxit trong không khí.

4) Đun nóng hỗn hợp ở dạng bột mịn chứa 2,0 g silic dioxit, 0,66 g đồng (II) oxit, 0,83 g canxi cacbonat và 0,75 g borat ở 900°C trong vài ngày.

(a) Hãy viết các phương trình phản ứng xảy ra.

(b) Hãy cho biết vai trò của borat trong thí nghiệm 4?

(c) Hãy nêu thêm một chất mầu lục (chứa sắt), một chất mầu vàng (chứa chì), một chất mầu xanh (chứa đồng) và một chất mầu đỏ (chứa thủy ngân) và nêu phương pháp điều chế các chất mầu trên.

164

Page 159: 1ygm3hwjpgu090312060334 tài liệu bồi dưỡng giáo viên trường thpt chuyên   mon hóa học - phan 1- năm 2012

Trong một xưởng vẽ tranh có một cái can mất nhãn chứa chất bột mầu xanh. Để xác định thành phần của chất bột có trong can, người ta cân 1,818 g mẫu, hòa tan vào nước và định mức đến vạch trong bình định mức 100,00 ml bằng axit sunfuric. Lấy 10 ml dung dịch rồi thêm vào đó lượng dư dung dịch KI, chuẩn độ hỗn hợp thu được bằng dung dịch natri thiosunfat 0,100 M thấy vừa hết 16,45 ml.

(d) Hãy cho biết kim loại nào được phân tích? Hãy viết các phản ứng xảy ra và tính thành phần % theo khối lượng của kim loại trong mẫu.

(e) Hãy cho biết chất bột mầu xanh này có thể là chất mầu được điều chế ở phần (a) không? Giải thích.

Phép phân tích khác sẽ cho biết thành phần chính xác của chất bột mầu xanh. Lấy 2,000 g mẫu và đun nóng trong điều kiện không có không khí. Thu được 1,439 g chất rắn mầu đen, hơi nước và chất khí làm đục nước vôi trong.

(f) Hãy xác định thành phần của chất bột mầu xanh.

(a. Na2Cr2O7 + S → Na2SO4 + Cr2O3

2 PbO + SnO2 → Pb2SnO4

6 PbO + O2 → 2 Pb3O4

4 SiO2 + CuO + CaCO3 → CaCuSi4O10 + CO2

b. Vai trò của Na2B4O7.10H2O là làm chất chảy, làm giảm nhiệt độ nóng chảy của hỗn hợp.

c. Chất mầu lục chứa sắt: K[FeIIIFeII(CN)6] hoặc Fe4III[FeII(CN)6]3

FeCl3 + K4[Fe(CN)6] → K[FeIIIFeII(CN)6] + 3 KCl

4 FeCl3 + 3 K4[Fe(CN)6] → Fe4III[FeII(CN)6]3 + 12 KCl

Chất mầu vàng chứa chì: PbCrO4.

Pb(NO3)2 + K2CrO4 → PbCrO4 + 2 KNO3

Chất mầu xanh chứa đồng: Cu(CH3COO)2 · 3Cu(OH)2.

CuO + 2 CH3COOH + 4 H2O → Cu(CH3COO)2 · 3Cu(OH)2 · 2H2O

Chất mầu đỏ chứa thủy ngân: HgS (xinabar dạng mầu đỏ)

Hg(NO3)2 + H2S → HgS + 2 HNO3

165

Page 160: 1ygm3hwjpgu090312060334 tài liệu bồi dưỡng giáo viên trường thpt chuyên   mon hóa học - phan 1- năm 2012

d. Đồng

2 Cu2+ + 4 I – → 2 CuI + I2

I2 + 2 S2O32 – → 2 I – + S4O6

2 –

%5,57100818,1

54,6310221

10.45,161,0%

3

=××××××

=−

Cu

e. %Cu (trong CaCuSi4O10) = 16,9%, không phải chất mầu được điều chế ở phần (a);

f. CuCO3·Cu(OH)2)

Câu 5.6. Nguyên tử crom có bán kính là 126 pm và khối lượng riêng của crom là 7,14 g/ml. Biết crom kết tinh ở dạng tinh thể lập phương.

(a) Hãy xác định kiểu tinh thể của crom dựa vào các số liệu cho ở trên. NA = 6,022·1023 mol–1.

Phép phân tích định tính Cl – được tiến hành như sau: Hỗn hợp khan gồm chất chưa biết và kali dicromat được đun nóng với dung dịch H2SO4 đặc. Khí thoát ra được dẫn qua dung dịch NaOH, nếu dung dịch có mầu vàng chứng tỏ có chất chưa biết có mặt clo.

(b) Hãy cho biết hợp chất bay hơi nào của crom được tạo thành trong phản ứng trên? Hãy vẽ cấu trúc của hợp chất này. Biết rằng số oxi hóa của Cr và Cl không thay đổi trong phản ứng.

Axit hóa dung dịch kali cromat cho ion dicromat mầu da cam, tiếp đó là tạo thành các ion tri – và tetracromat mầu đỏ đậm. Nếu sử dụng dung dịch axit sunfuric đặc sẽ thu được kết tủa mầu đỏ không chứa kali.

(c) Hãy viết các phản ứng xảy ra và vẽ cấu trúc của các ion. Hãy cho biết cấu trúc của chất kết tủa mầu đỏ.

Giản đồ Latimer của crom trong môi trường axit (pH=0) và trong môi trường kiềm (pH=14) như sau:

166

Page 161: 1ygm3hwjpgu090312060334 tài liệu bồi dưỡng giáo viên trường thpt chuyên   mon hóa học - phan 1- năm 2012

(d) Hãy xác định các giá trị thế khử còn thiếu.

(e) Hãy cho biết Cr(V) và Cr(IV) có tự phân hủy dị ly không? Hãy cho biết hằng số cân bằng của phản ứng phân hủy dị ly Cr2+.

(f) Hãy tính tích số tan của crom (III) hidroxit và hằng số bền tổng cộng của anion phức tetrahidroxo cromat (III).

Giản đồ Latimer của oxi trong môi trường axit (pH=0) và môi trường kiềm (pH=14) như sau:

(g) Hãy cho biết có phản ứng nào xảy ra khi pH của dung dịch chứa cromat, Cr(III) và hidro peoxit có giá trị bằng 0 và bằng 14? Tính suất điện động chuẩn của các phản ứng đó.

167

Page 162: 1ygm3hwjpgu090312060334 tài liệu bồi dưỡng giáo viên trường thpt chuyên   mon hóa học - phan 1- năm 2012

(a. TH1: mạng lập phương đơn giản:

cmpmRa 810.52,225212622 −==×==

=>3

3823/40,5

)10.52,2(10.022,6

0,521 cmgamd =

××= −

=> không hợp lý

TH2: mạng lập phương tâm mặt:

cmpmR

a 810.564,34,3562

1264

2

4 −==×==

=>3

3823/63,7

)10.564,3(10.022,6

0,524 cmgamd =

××= −

=> không hợp lý

TH3: mạng lập phương tâm khối:

cmpmR

a 810.91,22913

1264

3

4 −==×==

=>3

3823/01,7

)10.91,2(10.022,6

0,522 cmgamd =

××= −

=> hợp lý;

Vậy crom kết tinh theo kiểu lập phương tâm khối.

b. CrO2Cl2, cromyl clorua;

c. 2 CrO42 – + 2 H+ → Cr2O7

2 – + H2O

3 CrO42 – + 4 H+ → Cr3O10

2 – + 2 H2O

4 CrO42 – + 6 H+ → Cr4O13

2 – + 3 H2O

CrO42 – + 2 H+ → CrO3 + H2O

Cấu trúc của các ion:

168

Page 163: 1ygm3hwjpgu090312060334 tài liệu bồi dưỡng giáo viên trường thpt chuyên   mon hóa học - phan 1- năm 2012

d. Cr2+ → Cr Eo = –0,9V;

Cr(V) → Cr(IV) Eo = 1,35V;

Cr2O72 – → Cr(III) Eo = 1,33V;

e.

Điều kiện: E2 > E1.

Cả Cr(V) và Cr(IV) đều tự phân hủy dị ly.

3 Cr2+ 2 Cr3+ + Cr

170592,0

)42,09,0(2

10.08,610 −+−

==K

f. Cr(OH)3 + 3e Cr + 3 OH – Eo = –1,33V

Cr3+ + 3e Cr Eo = –0,74V

Cr(OH)3 Cr3+ + 3 OH – T = 1,26.10 –30

Cr(OH)4 – + 3e Cr + 4 OH – Eo = –1,33V

Cr3+ + 3e Cr Eo = –0,74V

Cr3+ + 4 OH – Cr(OH)4 – β = 7,92.10 –29

g. pH = 0:

Cr2O72 – + 3 H2O2 + 8 H+ → 2 Cr3+ + 3 O2 + 7 H2O

169

Page 164: 1ygm3hwjpgu090312060334 tài liệu bồi dưỡng giáo viên trường thpt chuyên   mon hóa học - phan 1- năm 2012

Eo =1,33 – 0,695 = 0,635 V

pH = 14:

2 Cr(OH)4 – + 3 HO2

– → 2 CrO42 – + 5 H2O + OH –

Eo = 0,87 – (–0,72) = 1,59 V)

Câu 5.7. Người ta tiến hành tách ion bari và srontri ra khỏi nhau dựa vào độ tan khác nhau của muối cromat. Nồng độ cromat trong dung dịch có thể điểu chỉnh bằng cách thay đổi pH.

2 H+ + 2 CrO42 – Cr2O7

2 – + H2O K = 1,50 ×1015

Cho các giá trị tích số tan: TBaCrO4 = 8,5 ×10 –11; TSrCrO4 = 3,6 ×10 –5.

(a) Hãy tính độ tan của BaCrO4 và SrCrO4 trong môi trường bazơ mạnh.

(b) pH của dung dịch K2Cr2O7 0,10 mol/lit được điều chỉnh đến 3,00 bằng cách sử dụng đệm axetat. Hãy tính nồng độ Cr2O7

2 – và CrO42 – trong dung dịch thu

được.

Thêm vào đó hãy tính giá trị nồng độ nhỏ nhất của Ba2+ và Sr2+ trong dung dịch này để kết tủa cromat tương ứng bắt đầu xuất hiện.

(c) Để điều chế dung dịch đệm người ta sử dụng dung dịch axit axetic 0,10 mol/lit. Hãy tính khối lượng natri axetat cần thêm vào 1 lít dung dịch trên để thu được dung dịch đệm có pH = 3,00. Biết hằng số phân li của axit axetic Ka = 1,78 ×10 –5.

Dưới đây là giản đồ Latimer của crom trong môi trường axit (pH = 0).

Cr2O72- Cr(V) Cr(IV) [Cr(H2O)6]

3+ [Cr(H2O)6]2+ Cr

+0,293

+0,55 +1,34 x -0,408 y

da cam ®á lôctÝmxanh-0,744

(d) Hãy xác định các giá trị thế x và y.

(e) Hãy cho biết Cr(IV) có bị phân hủy dị ly để tạo ra Cr(III) và Cr(VI) không? Giải thích.

(f) Hãy viết bán phản ứng khử của cặp Cr2O72 –/Cr3+. Hãy tính sự giảm thế khử

của cặp oxi hóa – khử này khi pH tăng thêm 1 đơn vị.

Xét phức mầu đỏ tím [CrCl2(ox)2]3 –.

170

Page 165: 1ygm3hwjpgu090312060334 tài liệu bồi dưỡng giáo viên trường thpt chuyên   mon hóa học - phan 1- năm 2012

(g) Hãy cho biết số phối trí của phức trên.

(h) Hãy cho biết hình học phân tử của phức.

(i) Hãy gọi tên của ion phức.

(k) Hãy viết các đồng phân lập thể của phức trên.

(a. Trong môi trường kiềm thì CrO42 – bị thủy phân không đáng kể:

BaCrO4 Ba2+ + CrO42 –

S S

=> MTS BaCrO611 10.22,910.5,8

4

−− ===

=> MTS SrCrO35 10.610.6,3'

4

−− ===

b. 2 H+ + 2 CrO42 – Cr2O7

2 – + H2O K = 1,50 ×1015

Có: [Cr2O72 –] + 0,5 [CrO4

2 –]= 0,1

1522

42

272 10.50,1

][][

][ == −+

CrOH

OCrK

=> [Cr2O72 –] = 0,1 M

[CrO42 –] = 8,16.10 –6 M

Để kết tủa BaCrO4 bắt đầu xuất hiện:

MCrO

TBa BaCrO 5

6

11

24

2 10.04,110.16,8

10.5,8

][][ 4 −

−+ ===

Để kết tủa SrCrO4 bắt đầu xuất hiện:

MCrO

TBa BaCrO 41,4

10.16,8

10.6,3

][][

6

5

24

2 4 === −

−+

c.][][

][][][ −+

−++

−++−

=OHHC

OHHCKH

B

Aa

Có: [OH –] << [H+] << CA

=>][

][ ++

+=

HC

CKH

B

Aa

171

Page 166: 1ygm3hwjpgu090312060334 tài liệu bồi dưỡng giáo viên trường thpt chuyên   mon hóa học - phan 1- năm 2012

hay 353

10

1,010.78,110 −

−−

+=

BC

=> CB = 7,8.10 –4 M

=> mCH3COONa = 7,8.10 –4×82 = 0,064 gam

d. x = 2,1 V

y = –0,912V

e. Có: VEIVCrOCr

945,00

)(/272

=−

=>0

)(/)(0

)(/272

IIICrIVCrIVCrOCrEE <−

=> phản ứng phân hủy dị li tự xảy ra.

f. Cr2O72 – + 6e + 14 H+ + 5 H2O → 2 Cr(H2O)6

3+

VECrOCr

33,10

/ 3272

=+−

Có:][

]][[lg

6

0592,03

142720

// 3272

3272 +

+−

+= +−+−Cr

HOCrEE

CrOCrCrOCr

Xét: [Cr2O72 –] = [Cr3+] = 1M; [H+] = 0,1 M

=> VECrOCr

19,11,0lg6

0592,033,1 14

/ 3272

=+=+−

=> ∆Epin = 1,19 – 1,33 = –0,14 V

Vậy khi pH tăng thêm 1 đơn vị thì thế của cặp giảm 0,14V;

g. 6;

h. bát diện;

i. Đicloro bisoxalato cromat(III)

k.

O

O O

Cl

Cl

O

CrO

O O

O

Cl

Cl

Cr

3- 3-

O

OO

Cl

Cl

O

Cr

3-

)

172

Page 167: 1ygm3hwjpgu090312060334 tài liệu bồi dưỡng giáo viên trường thpt chuyên   mon hóa học - phan 1- năm 2012

173

Page 168: 1ygm3hwjpgu090312060334 tài liệu bồi dưỡng giáo viên trường thpt chuyên   mon hóa học - phan 1- năm 2012

TÀI LIỆU THAM KHẢO

1. Hoàng Nhâm, Hóa học vô cơ, tập ba, Nhà xuất bản Giáo Dục, 2003.

2. Vũ Đăng Độ, Triệu Thị Nguyệt, Hóa học vô cơ, quyển 2 Các nguyên tố d và f, Nhà xuất bản Giáo dục, 2009.

3. Lâm Ngọc Thụ, Cơ sở hóa học phân tích, Nhà xuất bản Đại học Quốc Gia Hà Nội, 2006.

4. Nguyễn Tinh Dung, Hóa học phân tích, phần III Các phương pháp định lượng hóa học, Nhà xuất bản Giáo Dục, 2003.

5. Đào Đình Thức, Cấu tạo nguyên tử và liên kết hóa học, tập 2, Nhà xuất bản Giáo Dục, 2005.

6. Douglas A. Skoog, Fundamentals of Analytical Chemistry, NewYork, 2003.

7. F. Albert Cotton, Advanced Inorganic Chemistry, 6th Edition, Wiley –Interscience, 1999.

8. Bài tập chuẩn bị IChO lần thứ 29, 31, 34, 37, 40, 42, 43, 44 (2012).

9. Đề thi học sinh giỏi Quốc Gia các năm 2010, 2011, 2012.

10. Đề thi học sinh giỏi Quốc Gia các nước Đức (2006, 2007, 2009), Áo (2006, 2008, 2009, 2010), Ấn Độ (2011).

174

Page 169: 1ygm3hwjpgu090312060334 tài liệu bồi dưỡng giáo viên trường thpt chuyên   mon hóa học - phan 1- năm 2012

Ph n 4ÇHãA HäC H÷U C¥

CẤU TRÚC CÁC HỢP CHẤT GLUXIT

Biên soạn: GS. TS. Nguyễn Hữu Đĩnh

Mở đầu

Gluxit còn được gọi là cacbohiđrat hoặc saccarit. Gluxit là loại hợp chất tinh vi về mặt cấu trúc, đa dạng và phức tạp về phản ứng hóa học. Vì vậy nó là phần khó dạy, khó học không những ở trung học mà ngay cả ở đại học. Thế nhưng nó lại thường được dùng để thử thách trình độ của học sinh giỏi môn hóa học trong các đề thi quốc gia và quốc tế.

Trong SGK hóa học 12 nâng cao chỉ giới thiệu một số kiến thức sơ đẳng về gluxit. Hóa học về loại hợp chất này được viết chi tiết hơn ở các giáo trình và sách tham khảo bậc đại học. Tuy nhiên trong các tài liệu đó có một số khái niệm cơ bản về cấu trúc được viết không giống nhau hoặc không rõ ràng.

Mục đích, yêu cầu

Cung cấp những khái niệm cơ bản, chuẩn xác về cấu trúc của các chất gluxit.

Cung cấp một số tư liệu phong phú và cập nhật về các chất gluxit.

Giới thiệu một số bài tập nhằm vận dụng những khái niệm cơ bản để giải quyết các vấn đề về cấu trúc và về tính chất các gluxit.

Nội dung

I. Những khái niệm cơ bản về cấu trúc

II. Bài tập

Phương pháp dạy và học

Nghiên cứu tài liệu, làm bài tập, đề xuất thắc mắc, thảo luận và giải đáp.

175

Page 170: 1ygm3hwjpgu090312060334 tài liệu bồi dưỡng giáo viên trường thpt chuyên   mon hóa học - phan 1- năm 2012

Tài liệu tham khảo

1. IUPAC-IUBMB Joint Commission on Biochemical Nomenclature (JCBN). Nomenclature of Carbohydrates. Recommendations 1996.

2. Robert V. Stick. Carbohydrates. Academic Press. 2001.

3. Helen Osborn. Carbohydrates. Academic Press. 2003.

4. Nguyễn Hữu Đĩnh (Chủ biên). Bài tập hóa học hữu cơ. NXB Giáo dục, 2008.

5. Nguyễn Hữu Đĩnh (Chủ biên). Phương pháp giải các dạng bài tập hóa học, tập 2. NXB Giáo dục, 2011.

I. NHỮNG KHÁI NIỆM CƠ BẢN

I.1. Cacbohiđrat (Carbohydrates)

Cacbohiđrat bao gồm monosaccarit, oligosaccarit, polisaccarit và các dẫn chất từ monosaccarit như thay nhóm CHO bằng nhóm CH2OH, thay nhóm đầu mạch bằng nhóm COOH, thay một hay nhiều nhóm hyđroxy bằng nguyên tử H (deoxi), nhóm amino, nhóm thiol hoặc nhóm chứa dị tố tương tự. Thuật ngữ ‘đường’ dùng để chỉ các monosaccarit và oligosaccarit thấp.

I.2. Monosaccarit (Monosaccharide)

Monosaccarit nền là những polyhiđroxyanđehit kiểu H-[CHOH]n-CHO (anđozơ) hoặc những polyhiđroxyxeton kiểu H-[CHOH]n-CO-[CHOH]m-H (xetozơ) chứa từ 3C trở lên.

Monosaccarit theo nghĩa rộng bao gồm anđozơ, xetozơ, dianđozơ, dixetozơ, anđoxetozơ và các dẫn chất amino, thiol, deoxi của chúng ở dạng cacbonyl hoặc hemiaxetan.

I.3. Anđozơ (Aldose) và xetozơ (ketose)

Anđozơ là những monosaccarit có nhóm cacbonyl anđehit hoặc nhóm cacbonyl anđehit tiềm năng (nhóm hemiaxetal vòng, khi mở vòng thì thành nhóm cacbonyl anđehit).

Xetozơ là những monosaccarit có nhóm cacbonyl xeton hoặc nhóm cacbonyl xeton tiềm năng (nhóm hemixetal vòng, khi mở vòng thì thành nhóm cacbonyl xeton).

176

Page 171: 1ygm3hwjpgu090312060334 tài liệu bồi dưỡng giáo viên trường thpt chuyên   mon hóa học - phan 1- năm 2012

Ví dụ:

Anđozơ 2n đồng phân cấu hình (Dãy D: 2n/2; Dãy L: 2n/2;)

Anđotriozơ 2 đồng phân cấu hình (D-Glixerandehit và L-Glixerandehit)

Anđotetrozơ 4 đồng phân cấu hình (D-Erithrozơ, D-Threozơ ;

L-Erithrozơ, L-Threozơ)

Anđopentozơ 8 đồng phân cấu hình (Dãy D: 4; Dãy L: 4)

Anđohexozơ 16 đồng phân cấu hình (Dãy D: 8; Dãy L: 8)

Xetozơ H-[CHOH]n-CO-[CHOH]m-H n và m đều ≥ 1

I.4. Dianđozơ (Dialdose):

Dianđozơ là những monosaccarit có 2 nhóm cacbonyl anđehit (hoặc cacbonyl anđehit tiềm năng).

Ví dụ:

L-threo-Tetrodialdozơ

galacto-Hexodialdozơ

α-D-gluco-Hexodialdo-

1,5-pyranozơ

(6R)-D-gluco-Hexodialdo-6,2-

pyranozơ

I.5. Dixetozơ (Diketose): Monosaccarit có 2 nhóm cacbonyl xeton (hoặc cacbonyl xeton tiềm năng).

Ví dụ:

177

Page 172: 1ygm3hwjpgu090312060334 tài liệu bồi dưỡng giáo viên trường thpt chuyên   mon hóa học - phan 1- năm 2012

D-threo-Hexo-2,4-diulozơ

L-altro-Octo-4,5-diulozơ

α-D-threo-Hexo-2,4-diulo-2,5-furanozơ

Methyl β-D-xylo-hexopyranosid-4-ulozơ

I.6. Anđoxetozơ (Aldoketose), Xetoanđozơ (Ketoaldose): Monosaccarit vừa có nhóm cacbonyl anđehit (hoặc cacbonyl anđehit tiềm năng) vừa có nhóm cacbonyl xeton (hoặc cacbonyl xeton tiềm năng).

Ví dụ:

D-arabino-Hexos-3-

ulose

Metyl β-D-xylo-

hexopyranosid-4-ulose

Metyl α-L-xylo-

hexos-2-ulo-2,5-furanoside

I.7. Công thức chiếu Fisơ (Fischer projection)

Ở công thức chiếu Fisơ (thường được gọi đơn giản là công thức Fisơ) nguyên tử C bất đối (đính với 4 nhóm thế khác nhau) chính là giao điểm giữa đường thẳng đứng nối 2 nhóm thế nằm ở phía dưới trang giấy với đường nằm ngang nối hai nhóm thế nằm ở phía trên trang giấy. Đối với saccarit người ta có thể dùng công thức Fisơ ở dạng đơn giản bằng cách viết C thay cho giao điểm giữa đường thẳng

178

Page 173: 1ygm3hwjpgu090312060334 tài liệu bồi dưỡng giáo viên trường thpt chuyên   mon hóa học - phan 1- năm 2012

đứng và đường nằm ngang và bỏ 2 đoạn nối 2 nhóm thế trên đường nằm ngang như ở ví dụ dưới đây.

Glucozơ Fructozơ

I.8. Cacbon cấu hình (Configurational carbon)

Glyxeranđehit là monosaccarit đơn giản nhất, nó có 2 đối quang khác nhau về cấu hình của C bất đối, cấu hình dưới đây mà nhóm OH ở bên phải của C bất đối được kí hiệu là D, nhóm OH ở bên trái của C bất đối được kí hiệu là L. Ở các monosaccarit có nhiều C bất đối thì C bất đối nào có số thứ tự lớn nhất sẽ được chọn làm cacbon cấu hình dùng để so sánh với C bất đối của glixeranđehit mà xếp monosaccarit vào dãy D hay dãy L. Trong bảng dưới đây C cấu hình được ghi số thứ tự.

Dãy D Dãy L

D-Glixeranđeit L-Glixeranđeit

D-Erithrozơ D-Threozơ L-Erithrozơ L-Threozơ

179

Page 174: 1ygm3hwjpgu090312060334 tài liệu bồi dưỡng giáo viên trường thpt chuyên   mon hóa học - phan 1- năm 2012

D-Glucozơ D-Fructozơ L-Glucozơ L-Fructozơ

3-C-(Hydroxymetyl)-D-

glycero-tetrozơ

(D-Apiozơ)

4-C-(Hydroxymetyl)-

D-erythro-pentozơ

3-C-Metyl-

D-glucozơ

3-Deoxy-3,3-dimetyl-

D-ribo-hexozơ

I.9. Hemiaxetan (Hemiacetal) và hemixetan (hemiketal)

Sản phẩm cộng 1 phân tử ancol vào nhóm cacbonyl của anđehit gọi là hemiaxetan, nhóm OH tạo ra khi đó được gọi là OH hemiaxetan. Sản phẩm cộng 1 phân tử ancol vào nhóm cacbonyl của xeton gọi là hemixetan, nhóm OH tạo ra khi đó được gọi là OH hemixetan. Nhóm OH hemiaxetan và OH hemixetan thường được gọi chung là hemiaxetan

Khi thay nhóm OH hemiaxetan hoặc hemixetan (in đậm) bằng nhóm ankoxy thì được axetan hoặc xetan, tương ứng.

Thí dụ:

Hemiaxetan Axetan Hemixetan Xetan

I.10. Hemiaxetan và hemixetan vòng của cacbohyđrat

180

Page 175: 1ygm3hwjpgu090312060334 tài liệu bồi dưỡng giáo viên trường thpt chuyên   mon hóa học - phan 1- năm 2012

Đại đa số monosaccarit tồn tại ở dạng hemiaxetan vòng nội phân tử hoặc hemixetan vòng nội phân tử và thường được gọi chung là hemiaxetan. Hemiaxetan vòng 3 cạnh được gọi là oxirozơ, 4 cạnh được gọi là oxetozơ, 5 cạnh được gọi là furanozơ, 6 cạnh được gọi là pyranozơ, 7 cạnh được gọi là septanozơ, 8 cạnh được gọi là octanozơ,...

Thí dụ:

α-D-Glucooxirozơ α-D-Glucooxetozơ α-D-Glucofuranozơ α-D-Glucopyranozơ α-D-Glucoseptanozơ

I.11. Công thức Havooc (Haworth representation)

Thông thường thì ở công thức Havooc vòng hemiaxetan của saccarit được coi là một đa giác phẳng và được đặt vuông góc với mặt phẳng trang giấy, các cạnh của đa giác ở phía trước mặt phẳng trang giấy được tô đậm, các cạnh ở phía sau không tô đậm. Nguyên tử C anome (Mục I.15) được đặt ở đỉnh phía phải của đa diện. Nguyên tử O thành viên của vòng được đặt ở đỉnh ngay sau C anome. Các nhóm thế ở bên phải trong công thức Fisơ thì đặt ở phía dưới đa diện, các nhóm thế ở bên trái trong công thức Fisơ thì đặt ở phía trên đa diện. Các liên kết C-H có thể không cần biểu diễn. Thí dụ:

Metyl β-D-allooxirozit Metyl α-L-altrooxetozit β-D-Ribofuranozơ-5-phosphat

Khi quay công thức Havooc quanh trục vuông góc với mặt phẳng vòng thì không phải thay đổi trật tự không gian của các nhóm thế ở C bất đối. Thí dụ :

181

Page 176: 1ygm3hwjpgu090312060334 tài liệu bồi dưỡng giáo viên trường thpt chuyên   mon hóa học - phan 1- năm 2012

α-D-Glucofuranozơ

Ngược lại, nếu quay công thức Havooc quanh trục nằm trên mặt phẳng vòng thì phải thay đổi trật tự không gian của các nhóm thế ở C bất đối cho phù hợp. Ví dụ:

α-D-Glucofuranozơ

I.12. Cách chuyển công thức Fisơ dạng vòng thành công thức Havooc

Nếu ở công thức Fisơ mà 1 trong 2 nguyên tử C liên kết với nguyên tử O tạo vòng không phải là C bất đối thì chỉ việc chuyển sang công thức Havooc theo quy định ở mục 1.11. Ví dụ:

α-D-Fructopyranozơ Methyl α-D-glucoseptanozit

Nếu ở công thức Fisơ mà cả 2 nguyên tử C liên kết với nguyên tử O tạo vòng đều là C bất đối thì trước hết phải “xoay vần” sao cho nguyên tử O tạo vòng nằm trên cùng đường thẳng đứng với các nguyên tử C để được công thức Fisơ tương đương (tức là lần lượt đổi chỗ 3 nhóm thế như chỉ bởi 3 mũi tên xung quang C trong thí dụ dưới) sau đó mới chuyển sang công thức Havooc theo quy định ở mục 1.11. Cách xoay vần như vậy sẽ không làm thay đổi cấu hình của C bất đối đang xét.

Ví dụ:

182

Page 177: 1ygm3hwjpgu090312060334 tài liệu bồi dưỡng giáo viên trường thpt chuyên   mon hóa học - phan 1- năm 2012

α-D-glucopyranozơ

β-D-Fructofuranozơ

β-L-Arabinofuranozơ

I.13. Công thức cấu dạng (Depiction of conformation)

Trong thực tế các vòng lớn hơn 3 cạnh đều không phẳng như biểu diễn bởi công thức Havooc. Các monosaccarit vòng 6 cạnh thường tồn tại ở dạng ghế, công thức cấu dạng của chúng được biểu diễn xuất phát từ công thức Havooc bằng cách thay vòng 6 cạnh phẳng bằng cấu dạng ghế và giữ đúng quan hệ không gian của các nhóm thế quang C bất đối. Ví dụ:

α-D-Glucopyranozơ

183

Page 178: 1ygm3hwjpgu090312060334 tài liệu bồi dưỡng giáo viên trường thpt chuyên   mon hóa học - phan 1- năm 2012

β-D-Galactopyranozơ

I.14. Công thức Mills (Mills depiction)

Trong nhiều trường hợp, như khi có thêm các vòng giáp cạnh với vòng hemiaxetan của saccarit, người ta vẽ vòng này như một đa giác phẳng trên mặt trang giấy và dùng đường nét đứt để chỉ các liên kết hướng ra sau mặt phẳng trang giấy, đường nét liền tô đậm để chỉ các liên kết hướng ra trước mặt phẳng trang giấy. Cách biểu diễn như vậy được gọi là công thức Mills. Thí dụ:

1,2:3,4-Di-O-isopropyliden-α-D-galactopyranozơ D-Glucaro-1,4:6,3-dilacton

I.15. Cacbon anome (Anomeric carbon)

Khi tạo thành hemiaxetan vòng thì sinh ra một nguyên tử C bất đối mới từ nguyên tử C cacbonyl ở dạng mạch hở, nguyên tử C bất đối mới đó được gọi là cacbon anome, hoặc tâm anome.

Để viết công thức Havooc của hemiaxetan từ công thức Fisơ dạng mạch hở mà không qua công thức Fisơ dạng mạch vòng (Mục I.12) thì đầu tiên gập mạch C thành hình gấp khúc nằm trên mặt phẳng vuông góc với mặt phẳng trang giấy sao cho các nhóm thế ở bên phải trong công thức Fisơ chuyển thành các nhóm nằm phía dưới đường gấp khúc, các nhóm bên trái thì nằm phía trên đường gấp khúc. Tiếp theo, lần lượt đổi chỗ 3 nhóm thế (ở nguyên tử C lựa chọn) cho nhau theo kiểu “xoay vần” để nhóm OH lại gần nhóm C=O thích hợp cho việc tạo vòng theo số cạnh mong nuốn. Cách xoay vần các nhóm thế như vậy là được phép vì nó không làm thay đổi cấu hình của C lựa chọn.

Ví dụ 1:

Ba nhóm thế ở C-5 lần lượt đổi chỗ cho nhau theo kiểu xoay vần nên không

làm thay đổi cấu hình của C-5. Ở dạng mạch hở (I, II, III) C-1 không phải là C

184

Page 179: 1ygm3hwjpgu090312060334 tài liệu bồi dưỡng giáo viên trường thpt chuyên   mon hóa học - phan 1- năm 2012

bất đối, nhưng khi tạo thành hemiaxetan vòng (IV hoặc V, D-glucopyranozơ) thì

C-1 trở thành C bất đối và được gọi là C anome.

Ví dụ 2:

Ba nhóm thế ở C-5 lần lượt đổi chỗ cho nhau theo kiểu xoay vần để nhóm

OH gần nhóm C=O thích hợp cho việc tạo vòng 5 cạnh. Ở dạng mạch hở C-2

không phải là C bất đối, nhưng khi tạo thành hemiaxetan vòng (D-fructofuranozơ)

thì C-2 trở thành C bất đối và là C anome.

I.16. Cặp anome (Anomers)

Hai đồng phân lập thể vốn là hai dạng hemiaxetan vòng của một saccarit khác

nhau chỉ ở cấu hình của cacbon anome gọi là hai đồng phân anome (có thể gọi

đơn giản là hai anome hoặc cặp anome). Chúng được kí hiệu là α hay β tùy thuộc

vào quan hệ về cấu hình giữa C anome với C cấu hình (là C quyết định cấu hình

D hoặc L của monosaccarit nền, mục I.2 và I.8).

Trong công thức Fisơ nếu nguyên tử O không thuộc vòng đính với C anome

(chỉ bởi mũi tên lớn) ở cùng một phía với nguyên tử O đính với C cấu hình (mũi

tên nhỏ) thì dùng kí hiệu α, nếu 2 nguyên tử O đó mà ở khác phía nhau thì dùng

kí hiệu β.

185

Page 180: 1ygm3hwjpgu090312060334 tài liệu bồi dưỡng giáo viên trường thpt chuyên   mon hóa học - phan 1- năm 2012

Thí dụ:

α-D-Glucofuranozơ β-D-Glucofuranozơ

Metyl α-D-glucopyranozit Metyl β-D-glucopyranozit

I.17. Anome hóa (Anomerization)

Anome hóa là quá trình chuyển đổi một anome này thành một anome kia. Đối với các saccarit còn nhóm OH hemiaxetan sự chuyển đổi đó xảy ra dễ dàng trong dung dịch và là một quá trình thuận nghịch dẫn tới một hỗn hợp của một cặp anome.

Cơ chế của sự anome hóa được minh họa bởi sự chuyển đổi giữa α-D-glucopyranozơ và β -D-glucopyranozơ như sau.

186

Page 181: 1ygm3hwjpgu090312060334 tài liệu bồi dưỡng giáo viên trường thpt chuyên   mon hóa học - phan 1- năm 2012

Tỉ lệ hai anome trong hỗn hợp khi đạt tới cân bằng phụ thuộc vào từng saccarit, vào dung môi và vào nhiệt độ. Với dung môi là nước, ở 40oC, trong dung dịch D-glucozơ thì α-D-glucopyranozơ chiếm 36%, β-D-glucopyranozơ chiếm 64%, trong dung dịch D-mannozơ thì α-D-mannopyranozơ chiếm 68%, β-D-mannopyranozơ chiếm 32%, trong dung dịch D-fructozơ thì α-D-fructopyranozơ chiếm 0-3%, β-D-fructopyranozơ chiếm 57-75%, α-D-fructofuranozơ chiếm 4-9%, β-D-fructofuyranozơ chiếm 21-31%.

I.18. Sự quay hỗ biến (Mutarotation)

Sự thay đổi độ quay cực riêng của một hợp chất quang hoạt theo thời gian dẫn tới một giá trị cân bằng gọi là sự quay hỗ biến. Các monosaccarit và disaccarit còn nhóm OH hemiaxetan đều có sự quy hỗ biến.

Ví dụ: Dung dịch α-D-(+)-glucopyranozơ (nóng chảy ở 146oC) khi vừa hòa tan có góc quay cực riêng là +112o nhưng sau đó giảm dần và đạt tới giá trị không đổi là +52,7o; Dung dịch β-D-(+)-glucopyranozơ (nóng chảy ở 150oC) khi vừa hòa tan có góc quay cực riêng là +18,7o nhưng sau đó tăng dần và đạt tới giá trị không đổi là +52,7o. Nguyên nhân là do chúng bị anome hóa như trình bày ở mục 1.17 và có thể được giải thích ngắn gọn là do chúng chuyển đổi cho nhau qua dạng mạch hở trong một quá trình thuận nghịch:

tonc:146 oC; [α]D25 : 112o tonc:150 oC; [α]D

25 : 18,7o

I.19. Cặp epime (Epimers)

Hai đồng phân lập thể có nhiều tâm bất đối nhưng khác nhau về cấu hình chỉ ở 1 trong số các tâm bất đối đó gọi là hai epime hoặc cặp epime. Cặp anome là trường hợp đặc biệt của cặp epime. Ví dụ:

187

Page 182: 1ygm3hwjpgu090312060334 tài liệu bồi dưỡng giáo viên trường thpt chuyên   mon hóa học - phan 1- năm 2012

Cặp epime 1: α-D-glucopyranozơ (I) và α-D-mannopyranozơ (II) khác nhau chỉ ở cấu hình C-2.

Cặp epime 2: β-D-glucopyranozơ (III) và β-D-mannopyranozơ (IV) khác nhau chỉ ở cấu hình C-2.

Cặp 3: α-D-glucopyranozơ (I) và β-D-glucopyranozơ (III) khác nhau chỉ ở cấu hình C anome (C-1) nên được gọi là cặp anome. Đó cũng là một cặp epime.

Cặp 4: α-D-mannopyranozơ (II) và β-D-mannopyranozơ (IV) khác nhau chỉ ở cấu hình C anome (C-1) nên được gọi là cặp anome. Đó cũng là một cặp epime.

I.20. Glycozit (Glycozide)

Glycozit là loại hợp chất mà phân tử gồm hợp phần saccarit liên kết ở nguyên tử C anome với hợp phần không phải saccarit, khi đó hợp phần saccarit được gọi là glycon hoặc gốc glycozyl, hợp phần không phải saccarit được gọi là aglycon. Ví dụ:

Axit 4-(α-D-Ribofuranozylthio) benzoic (IV)

(V) 2-β-D-Glucopyranozyl-1,3,6,7-tetrahydroxyxanthen-9-

one (Mangiferin)

(VI) Tetra-O-acetyl-α-D-mannopyranozyl

bromua

Liên kết giữa cacbon anome với hợp phần aglycon được gọi là liên kết glycozit. Các hợp chất I, II thuộc loại O-glycozit, hợp chất III thuộc loại N-

188

Page 183: 1ygm3hwjpgu090312060334 tài liệu bồi dưỡng giáo viên trường thpt chuyên   mon hóa học - phan 1- năm 2012

glycozit còn được gọi là glycozylamin, hợp chất IV thuộc loại S-glycozit, hợp chất V thuộc loại C-glycozit (tuy nhiên ít được chấp nhận), hợp chất VI thuộc loại glycozyl halogenua.

Glycozit đóng vai trò vô cùng quan trọng trong các cơ thể sống. Thực vật giữ các hoạt chất ở dạng glycozit không hoạt động, khi cần giải phóng chúng thì thủy phân nhờ xúc tác enzim. Nhiều glycozit thực vật được dùng làm thuốc chữa bệnh.

I.21. Glycozidaza (Glycozidase)

Glycozidaza (còn gọi là glycozit hydrolaza hoặc glycozyl hydrolaza) là những enzim xúc tác cho sự thủy phân liên kết glycozit giải phóng ra hemiaxetan saccarit và aglycon. Glycozidaza có thể xúc tác cho sự thủy phân các liên kết O-, N- và S-glycozit.

Các glycozidaza có tính chất xúc tác rất đặc hiệu. Ví dụ, mantaza xúc tác cho sự thủy phân matozơ, lactaza xúc tác cho sự thủy phân lactozơ, loại α-glycozidaza thì xúc tác cho sự thủy phân chỉ liên kết α-glycosit, loại β-glycozidaza thì xúc tác cho sự thủy phân chỉ liên kết β-glycosit.

I.22. Anđitol (Alditol)

Ancol hình thành từ anđozơ bằng cách thay nhóm CHO bằng nhóm CH2OH được gọi là aditol. Tên gọi cụ thể lấy từ tên anđozơ đổi đuôi ozơ thành itol.

Ví dụ:

D-GlucitolKhông nên gọi: Socbitol hay

Sorbitol.

meso-Galactiol D-ArabinitolKhông gọi:D-Lyxitol

I.23. Deoxy saccarit (deoxy sugar)

Khi thay nhóm OH ancol của monosaccarit bằng nguyên tử H thì được deoxy saccarit. Ví dụ:

189

Page 184: 1ygm3hwjpgu090312060334 tài liệu bồi dưỡng giáo viên trường thpt chuyên   mon hóa học - phan 1- năm 2012

6-Deoxy-α-L-galactopyranozơ

(α-L-Fucopyranozơ)

6-Deoxy-L-mannopyranozơ

(L-Rhamnopyranozơ)

2,6-Dideoxy-β-D-ribo-hexopyranozơ

(β-Digitoxopyranozơ)

2-Deoxyribozơ 5-phosphat

I.24. Axit anđonic (Aldonic acid)

Axit monocacboxylic hình thành từ anđozơ bằng cách thay nhóm CHO bằng nhóm COOH được gọi là axit anđonic. Tên riêng lấy từ tên anđozơ đổi đuôi ozơ thành onic và thêm từ axit. Ví dụ:

Axit D-Gluconic Metyl D-Gluconat D-Glucono-1,4-lacton

I.25. Axit uronic (Uronic acid)

Axit monocacboxylic hình thành từ anđozơ khi thay nhóm CH2OH bằng nhóm COOH được gọi là axit uronic. Tên riêng: nếu xuất phát từ anđozơ thì đổi đuôi ozơ thành uronic, nếu xuất phát từ glycozit thì đổi đuôi ozit thành osiduronic, nếu xuất phát từ glycozit thì đổi đuôi ozit thành osyluronic và thêm từ axit (Nhóm CHO vẫn mang số 1). Ví dụ:

Axit

D–glucuronic

Axit

α–D–mannopyranuronic

Axit phenyl β–D–glucopyranosiduronic

Axit methyl 2,3,4–tri–O–acetyl–α–D–

glucopyranosyluronatbromua

190

Page 185: 1ygm3hwjpgu090312060334 tài liệu bồi dưỡng giáo viên trường thpt chuyên   mon hóa học - phan 1- năm 2012

I.26. Axit anđaric (Aldaric acid)

Axit đicacboxylic hình thành từ anđozơ bằng cách thay 2 nhóm đầu mạch (CHO và CH2COOH) bằng hai nhóm cacboxy được gọi là axit anđaric. Tên gọi cụ thể lấy từ tên anđozơ đổi đuôi ozơ thành aric và thêm từ axit.

Ví dụ:

Axit L–threaric

Axit (2R,3R)–Tactaric

Axit D–threaric

Axit (2S,3S)–Tactaric

Axit erithraric

Axit meso–Tactaric

Axit D–Glucaric

Không gọi: Axit L–Glucaric

Axit meso–galactaric D–Mannaro–1,4:6,3–dilacton

I.27. Amino saccarit (amino sugar)

Monosaccarit mà nhóm OH ancol được thay bằng nhóm amino thì gọi là amino saccarit, nếu nhóm OH hemiaxetan được thay bằng nhóm amino thì gọi là glycozylamin. Ví dụ:

2–Amino–2–deoxy–D–glucopyranose

(D–glucosamine).

4,6–Dideoxy–4–formamido–2,3–di–O–methyl–

D–mannopyranose

2–Acetamido–1,3,4–tri–O–acetyl–2,6–dideoxy–α–

L–galactopyranose

I.28. Disaccarit (Disaccharide)

Disaccarit là hợp chất gồm hai mắt xích monosaccarit liên kết với nhau bằng liên kết glycozit. Ví dụ:

191

Page 186: 1ygm3hwjpgu090312060334 tài liệu bồi dưỡng giáo viên trường thpt chuyên   mon hóa học - phan 1- năm 2012

4–O–α–D–glucopyranozyl–D–glucopyranozơ

(Mantozơ)

4–O– β–D–glucopyranozyl–D–glucopyranozơ

(Xenlobiozơ)

β–D–Fructofuranozyl–α–D–glucopyranozit

(Sacarozơ)

4–O–β–D–galactopyranozyl–α–D–glucopyranozơ

(α–Lactozơ)

I.29. Oligosaccarit (Oligosaccharide)

Oligosaccarit là hợp chất gồm các mắt xích monosaccarit liên kết với nhau bằng liên kết glycozit. Tùy theo số mắt xích monosaccarit mà gọi là disaccarit, trisaccarit, tetrasaccarit, pentasaccarit...

Ví dụ:

192

Page 187: 1ygm3hwjpgu090312060334 tài liệu bồi dưỡng giáo viên trường thpt chuyên   mon hóa học - phan 1- năm 2012

α–D–Glucopyranosyl–

α–D–glucopyranosit

β–D–Fructofuranosyl–

α–D–galactopyranosyl–(16)–α–D–glucopyranosit

Cyclomaltohexaozơ

(α–cyclodextrin, α–CD)

I.30. Polisaccarit (Polysaccharide)

Polisaccarit (còn được gọi là glycan) dùng để chỉ các đại phân tử gồm một số lớn các mắt xích monosaccarit nối với nhau bằng liên kết glycozit. Polisaccarit gồm các mắt xích chỉ của một monosaccarit gọi là homosaccarit. Polisaccarit gồm các mắt xích của hơn một monosaccarit gọi là heterosaccarit. Tên chung của homosaccarit được gọi theo tên của monosaccarit nhưng đổi đuôi ozơ thành đuôi an, chẳng hạn như tinh bột và xenlulozơ đều thuộc loại glucan.

Ví dụ:

(2 1)–β–D–Fructofuranan

Amylozơ: (1 4)–α–D–Glucopyranan

193

Page 188: 1ygm3hwjpgu090312060334 tài liệu bồi dưỡng giáo viên trường thpt chuyên   mon hóa học - phan 1- năm 2012

Amilopectin: (1 4)– và (1 6)–α–D–Glucopyranan, (C6H10O5)n, [C6H7O2 (OH)3]n

Xenlulozơ: (1 4)– β–D–Glucopyranan, (C6H10O5)n , [C6H7O2 (OH)3]n

Gellan (một polysaccarit nguồn gốc vi khuẩn)

II. BÀI TẬP

II.1. Thế nào là cacbohiđrat, monosaccarit, disaccarit, oligosaccarit, polisaccarit ?

II.2. Thế nào là anđozơ, xetozơ, dianđozơ, dixetozơ, andoxetozơ?

II.3. Hãy viết công thức cấu tạo dạng mạch hở (có ghi số chỉ vị trí C) của glucozơ, fructozơ và cho biết tên, cấu tạo, số lượng và sự khác biệt của các nhóm chức có trong mỗi chất.

II.4. Khi oxi hóa glixerol người ta thu được 1,2–đihiđroxypropanal và 1,3–đihiđroxypropanon. Hãy viết công thức cấu tạo và cho biết chúng có thuộc loại monosaccarit hay không, vì sao ?

II.5. a) Hãy viết công thức cấu tạo dạng vòng phẳng 6 cạnh (có ghi số chỉ vị trí C) của α–D–glucozơ, β–D–glucozơ, và cho biết tên, cấu tạo, số lượng và sự khác biệt của mỗi nhóm chức có trong 2 dạng đó.

b) Hãy đưa ra bằng chứng thực nghiệm chứng tỏ rằng glucozơ có 2 dạng là α–glucozơ và β–glucozơ.

194

Page 189: 1ygm3hwjpgu090312060334 tài liệu bồi dưỡng giáo viên trường thpt chuyên   mon hóa học - phan 1- năm 2012

II.6. Trong một cuốn sách luyện thi người ta viết rằng từ fomalđehit với xúc tác Ca(OH)2 điều chế được glucozơ theo phản ứng sau:

6 CH2=O → HOCH2(CHOH)4CH=O

Sản phẩm thu được trong phản ứng nêu trên có phải là glucozơ hay không, vì sao?

II.7. Hãy viết công thức cấu tạo dạng vòng phẳng 5 cạnh (có ghi số chỉ vị trí C) của α–fructozơ, β–fructozơ, và cho biết tên, cấu tạo, số lượng và sự khác biệt của mỗi nhóm chức có trong 2 dạng đó.

II.8. Hãy viết công thức phân tử, công thức phân tử thu gọn và công thức cấu tạo dạng vòng phẳng của saccarozơ, mantozơ (có ghi số chỉ vị trí C cho từng gốc monosaccarit) và nhận xét về đặc điểm cấu tạo của mỗi chất.

II.9. Hãy viết công thức phân tử, công thức phân tử thu gọn và công thức cấu tạo dạng vòng phẳng của amilozơ, amilopectin, xenlulozơ (có ghi số chỉ vị trí C cho từng gốc monosaccarit) và nhận xét về đặc điểm cấu tạo của mỗi chất: cách thức liên kết giữa các gốc glucozơ, số lượng các gốc glucozơ, khối lượng và hình dạng phân tử. Hãy nêu rõ sự giống nhau và khác nhau về đặc điểm cấu tạo của xenlulozơ so với tinh bột.

II.10. a. Thế nào là cacbon cấu hình?

b. Hãy sắp xếp các hợp chất sau đây vào dãy D hoặc dãy L.CHO

H OH

HO H

H OH

H CH2OH

OH

OH

OHC H

HO H

H OH

HO CH2OH

H

OH

H CHO

H OH

HO H

HO H

CH2OH CHO

HHO

HHO

OHH

HHOH2C

OH

(I) (II) (III) (IV)

II.11. a. Thế nào là cặp epime, thế nào là cặp enantiomer (đối quang) ?

b. Trong các chất cho ở bài tập II.10, cặp chất nào là epime (epimer); cặp chất nào là enantiomer (đối quang) ?

II.12. Hai hợp chất A và B đều có cấu tạo 2,3,4–trihiđroxibutanal, đều quang hoạt. Khi bị oxi hoá bởi axit nitric loãng thì A chuyển thành axit D–tactric (axit (D)–2,3–đihiđroxibutanđioic) còn B chuyển thành axit mesotactric.

195

Page 190: 1ygm3hwjpgu090312060334 tài liệu bồi dưỡng giáo viên trường thpt chuyên   mon hóa học - phan 1- năm 2012

a. A và B có thể có cấu trúc như thế nào? Gọi tên và xác định cấu hình tương đối của chúng.

b. Viết công thức Fisơ, xác định cấu hình tuyệt đối của các nguyên tử C* ở axit D–tactric, axit mesotactric, A và B.

c. Trong các phản ứng oxi hoá ở trên, cấu hình của các nguyên tử C* có bị thay đổi hay không? giải thích?

d. Trong các chất cho ở trên có cặp epime nào không?

II.13. Có các hợp chất sau:X

H OH

HO H

H OH

H OH

Y

Z

I II III IV V VI VII

X CHO COOH COOH CH2OH CHO CH=NOH CH=NNHC6H5

Y CH2OH CH2OH COOH CH2OH CH3 CH2OH CH2OH

a. Gọi tên hệ thống các hợp chất trên theo 2 cách.

b. Viết sơ đồ phản ứng có ghi rõ điều kiện để chuyển hoá I lần lượt thành II, III, IV, V, VI và VII.

II.14. a. Thuốc thử Tollens được điều chế thế nào? Viết sơ đồ phản ứng (dùng công thức cấu trúc).

b. Thuốc thử Felinh (Fehling) được điều chế thế nào? Viết sơ đồ phản ứng (dùng công thức cấu trúc).

c. Có thể dùng 2 thuốc thử kể trên để phân biệt anđohexozơ và xetohexzơ được không, viết phương trình phản ứng để giải thích.

d. Nên dùng thuốc thử nào để phân biệt anđohexozơ và xetohexozơ. Viết phương trình phản ứng.

II.15. Dùng công thức cấu tạo, hãy giải thích vì sao xetozơ có phản ứng với thuốc thử Tolens, với thuốc thử Felinh.

II.16. a. Hãy viết phương trình các phản ứng xảy ra khi cho D–threozơ tác dụng với phenylhiđrazin lấy dư, phân loại chất trung gian và sản phẩm phản ứng.

b. Vì sao phản ứng dừng lại ở C2 mà không tiếp tục tới C3, …?

II.17. Có thể có tối đa mấy monosaccarit mà khi cho tác dụng với phenylhidrazin dư thì cho ra cùng một ozazon? Lấy thí dụ cụ thể để chứng minh.

196

Page 191: 1ygm3hwjpgu090312060334 tài liệu bồi dưỡng giáo viên trường thpt chuyên   mon hóa học - phan 1- năm 2012

II.18. a. Nêu các tác nhân thường dùng để khử nhóm cacbonyl của monosaccarit thành nhóm CH – OH.

b. Có gì khác nhau về hoá lập thể trong phản ứng khử nhóm C=O của D–glucozơ và D–fructozơ thành nhóm CH – OH.

II.19. a. Oxi hoá D–Glucozơ bằng dung dịch HNO3 loãng ở 1000C thì thu được hỗn hợp sản phẩm từ đó tách ra được 4 chất A, B, C, D. Cho biết MA = 210 đv C; MB = MC = 192 đv C, MD = 174 đvC. Hãy viết công thức của Fisơ của chúng.

b. Cũng trong điều kiện như trên, từ L–Gulozơ sẽ tạo ra những chất nào, công thức Fisơ của chúng?

II.20. Hãy viết công thức Fisơ những cặp anđohexozơ mà khi bị oxi hoá thì cho cùng một axit anđaric dạng meso và khi bị khử thì cho một anđitol dạng meso.

II.21. Hãy viết công thức Fisơ các cặp anđohexozơ mà khi bị oxi hoá thì cho cùng một axit anđaric và khi bị khử thì cho cùng một anđitol.

II.22. a. Hãy viết sơ đồ phản ứng tổng hợp 2 epime anđopentozơ xuất phát từ D–erythro theo phương pháp Kaliani – Fisơ (dùng công thức Fisơ).

b. Hai epime trên được tạo ra với lượng bằng nhau hay khác.

c. Khi oxi hoá bằng axit nitric thì epime thứ nhất tạo thành axit anđaric không quang hoạt, còn đồng phân thứ hai thì tạo thành axit anđaric quang hoạt. Hãy xác định cấu trúc của 2 đồng phân đó.

d. Ngày nay phương pháp Kiliani – Fisơ đã được cải tiến như thế nào?

II.23. a. Thế nào là giáng vị Ruff (giảm mạch cacbon theo Ruff)?

b. Thuỷ phân đisaccarit lactozơ (tách từ sữa) người ta thu được anđohexozơ D(+)–galactozơ. Khi chế hoá D(+)–galactozơ bằng HNO3 thì thu được axit galactaric không quang hoạt. Khi dùng phương pháp giáng vị Ruff thì thu được D(–)–lyxozơ. Oxi hoá D(–)–lyxozơ bằng axit nitric thì thu được axit lyxaric quang hoạt. Dựa vào các dữ kiện đã cho, hãy xác định công thức Fisơ của D(+)–galactozơ.

II.24. a. Hãy viết phản ứng của anđehit (RCH=O) và của anđohexozơ với metanol dư có mặt hiđroclorua. Gọi tên chung của sản phẩm.

b. Nêu rõ sự khác nhau và giải thích.

197

Page 192: 1ygm3hwjpgu090312060334 tài liệu bồi dưỡng giáo viên trường thpt chuyên   mon hóa học - phan 1- năm 2012

II.25. a. Hãy phân biệt khái niệm đồng phân anome và đồng phân epime.

b. Fructozơ tạo thành những anomer nào giải thích sự tạo thành và gọi tên chúng.

c. Thế nào là α – anome và β – anome?

II.26. a. Thế nào là sự quay hỗ biến.

b. α–D–glucopiranozơ có góc quay cực riêng là 1120, anomer với nó có góc

quay cực riêng là 190. Khi pha riêng từng chất vào nước rồi đo thì thấy sau một thời gian góc quay cực đều đạt tới giá trị 52,70. Hãy giải thích và tính hàm lượng % của các anomer khi góc quay đạt tới 52,7%. Biết rằng hàm lượng các dạng khác của D–glucozơ không đáng kể.

c. Có thể áp dụng bài toán trên cho D–Fructozơ được không? vì sao?

II.27. Viết công thức Fisơ và công thức Havoc của các hợp chất sau:

a. α–D–glucopiranozơ b. α–L–glucopiranozơ.

c. α–D–fructopiranozơ. d. β–D–fructofuranozơ.

e. β–D–manopiranozơ f. β–D–galactopiranozơ.

Hãy rút ra cách thức chuyển từ công thức Fisơ sang công thức Havoc.

II.28. Cách làm nào trong những cách dưới đây là được phép và không được phép, vì sao?

a. Quay công thức Havoc 900 trong mặt phẳng chứa vòng.

b. Quay công thức Havoc 1800 trong mặt phẳng chứa vòng.

c. Quay công thức Havoc 900 ra khỏi mặt phẳng chứa vòng.

d. Quay công thức Hacvoc 1800 ra khỏi mặt phẳng chứa vòng.

e. Chuyển các nhóm ở phía dưới lên phía trên, nhóm phía trên xuống phía dưới (đổi chỗ nhóm phía dưới và phía trên cho nhau).

II.29. a. Làm thế nào để chuyển công thức Fisơ của các piranozơ thành công thức cấu dạng?

b. Hãy chuyển công thức Fisơ của α–D–glucozơ thành công thức Havoc rồi thành 2 công thức dạng ghế.

c. Dạng ghế nào ở câu b bền hơn vì sao?

II.30. a. Hãy viết công thức 2 dạng ghế của β–D–glucopiranozơ và cho biết thông thường người ta dùng dạng nào vì sao?

198

Page 193: 1ygm3hwjpgu090312060334 tài liệu bồi dưỡng giáo viên trường thpt chuyên   mon hóa học - phan 1- năm 2012

b. Hãy viết công thức Havoc và công thức cấu dạng của hỗn hợp 2 anomer của D–glucopiranozơ.

c. Anome nào của D–glucopiranozơ bền hơn, vì sao?

II. 31. Gọi tên các monosaccarit cho sau đây.

O

OH

OH

OHOH

OH

OH

HOH2C

HO OH

O

CH2OH

«HH

H

HOH2C

HO

OHO

a. b. c.

g.H

HOH2C

CH2OH

OHO

OH

OH

OH

OH

O

OH

CH2OH

HO H

OH

OH

O

OH

HOH2C

OHH

d. e.

II.32. Biết rằng trong dung dịch, mannozơ tồn tại ở dạng piranozơ. Hãy viết sơ đồ phản ứng của D–mannozơ với các tác nhân dưới đây, gọi tên các sản phẩm tạo thành:

a. [Ag(NH3)2]+ b. Dung dịch HCl loãng

c. Dung dịch NaOH loãng. d. anhiđrit axetic

e. CH3OH / HCl f. Me2SO4/OH–

II.33. a. Hãy dùng công thức Fisơ hoàn thành sơ đồ chuỗi phản ứng theo phương pháp giáng vị Wolal.

D-Glucoz¬ oxim C18H25O12N C16H21O10N C5H10O5NH2OH Ac2O d- AcO -

- AcOHMeONaNaOH

(A) (B) (C) (D)

b. Giải thích sự tạo thành D từ C.

II.34. Hãy viết phương trình phản ứng của axit periodic dư với các chất sau. Cho nhận xét.

a) Etilenglycol b) Glixerol

c) Glixerandehit. d) Đihiđroxiaxeton.

e) HOCH2CH(OH) – COOH f) HOOC–CH(OH)CH = O.

199

Page 194: 1ygm3hwjpgu090312060334 tài liệu bồi dưỡng giáo viên trường thpt chuyên   mon hóa học - phan 1- năm 2012

II.35. L–Ramonozơ là 6–Đeoxi–L–mannozơ. Khi cho metyl α–L–ramonopiranozit

phản ứng với HIO4 dư thì thu được hợp chất A (C6H12O5). Trên phổ hồng ngoại của A không thấy có vân hấp thụ của nhóm cacbonyl. Cho A tác dụng với CH3I/Ag2O thì được dẫn xuất B (C8H16O5). Chế hoá A với H2/Ni hoặc NaBH4 đều cho hợp chất C (C6H14O4).

a. Viết công thức Fisơ, công thức Havoc của α–L–ramonozơ và của metyl α–

L–ramonopiranozit.

b. Giải thích sự hình thành A và cho biết vì sao trên phổ IR của nó không thấy có dấu hiệu của nhóm cacbonyl.

c. Giải thích sự tạo thành B từ A, viết cấu tạo của B.

d. Giải thích phản ứng của A với H2/Ni tạo thành C. Viết công thức Fisơ của C.

II.36. Dùng công thức Fisơ hoàn thành các sơ đồ phản ứng sau.

a. Metyl α–D–glucopiranozơ → 4HIO → OH / Br 22 →+OH3 A (hỗn hợp)

b. Metyl α–D–glucofuranozơ → 4HIO → OH / Br 22 →+OH3 B (hỗn hợp)

II.37. a. Vì sao monosaccarrit và đisaccarrit đều tan tốt trong nước, còn polisaccarrit thì rất ít tan trong nước.

b. Vì sao đisaccarrit có nhiệt độ nóng chảy cao hơn các monosaccarrit hợp thành.

II.38. Nguyên liệu để sản xuất vitamin C là D–glucozơ. Sản phẩm trung gian là L–sorbozơ được điều chế như sau:

D-Glucoz¬ D-GlucitolAxetobactersuboxydaza

CH2OH

O

HHO

OHH

HHO

CH2OH

H2 / Ni

(1)

O2 (2)

L-Sorboz¬

Ở phản ứng 2, vi khuẩn Axetobacter suboxyđaza đã giúp chuyển với hiệu suất tới 90%. Kết quả là đã chuyển được D–anđohexozơ (D–glucozơ) thành L–xetohexozơ.

L–Sorbozơ được chuyển hoá thành vitamin C theo sơ đồ sau:

200

Page 195: 1ygm3hwjpgu090312060334 tài liệu bồi dưỡng giáo viên trường thpt chuyên   mon hóa học - phan 1- năm 2012

L-Sorboz¬

CH2OH

HO

OH

HO

H2C O

CH3

CH3

OH3C

H3C

(3)

KMnO4 / OH-

BH3O+, to O

O

OHHO

HOH

CH2OH

(C)

(4) (5)

a. Hãy nhận xét về tác dụng của sự oxi hoá vi sinh ở phản ứng (2).

b. Viết công thức Fisơ của D–glucitol, B, C và công thức của tác nhân X.

c. Sự chuyển từ cấu hình D sang cấu hình L xảy ra ở giai đoạn phản ứng nào và bằng cách nào?

d. Vitamin C có tên là axit L – ascobic có pKa = 4,21.

Viết phương trình phân li của axit ascobic và giải thích vì sao lực axit của nó lớn hơn của axit axetic.

II.39. a. Thế nào là glicozit? Phân loại và cho ví dụ minh hoạ cho mỗi loại.

b, Thế nào là glicozyl? phân loại và cho ví dụ minh hoạ cho mỗi loại.

II.40. Hãy gọi tên và nêu đặc điểm của liên kết C – O giữa 2 mắt xích monosaccarit trong phân tử đisaccarit, cho thí dụ minh hoạ.

II.41. a. Hãy viết công thức cấu dạng gọi tên theo IUPAC của mantozơ biết

rằng nó được cấu tạo từ 2 mắt xích glucozơ liên kết với nhau bởi liên kết (1→4)–

α–glicozit.

b. Hãy viết công thức cấu tạo và gọi tên theo IUPAC của xenlulozơ biết rằng nó gồm 2 mắt xích glucozơ liên kết với nhau theo kiểu

(1→4) β– glicozit.

II.42. a. Thế nào gọi là đisaccarit có tính khử và đisaccarit không có tính khử.

b. Trong số các đisaccarit cho ở bài tập II.40 và II.41, đisaccarit nào thuộc loại có tính khử, không có tính khử, vì sao?

II.43. Hãy xác định cấu trúc (công thức cấu dạng) của đisaccarit melibiozơ dựa trên các dữ kiện sau:

1. Melibiơzơ có sự quay hỗ biến.

2. Thuỷ phân melibiozơ với xúc tác axit họăc α–galactoziđaza (enzim chỉ

phân cắt liên kết α–galactozit) đều cho D–galactozơ và D–glucozơ.

201

Page 196: 1ygm3hwjpgu090312060334 tài liệu bồi dưỡng giáo viên trường thpt chuyên   mon hóa học - phan 1- năm 2012

3. Cho tác dụng với nước brom thì thu được axit melibionic. Metyl hoá axit melibionic rồi thuỷ phân thì thu được 2,3,4,6–tetra–O–metyl–D–galactozơ và axit 2,3,4,5–tetra–O–metyl–D–gluconic.

II.44. Trehalozơ và isotrehalozơ có công thức C12H22O11 đều không có tính khử, khi bị thuỷ phân xúc tác axit đều tạo thành glucozơ. Isotrehalozơ bị thuỷ phân bởi cả maltaza và emulssin còn trehalozơ thì bị thuỷ phân chỉ bởi maltaza.

Biết rằng malataza thuộc loại α–glucoziđaza, còn emunsin thì thuộc loại β–

glucozidaza. Khi tác dụng với CH3I/OH– rồi thuỷ phân thì cả hai đều cho sản phẩm duy nhất là 2,3,4,6–tetra–O–metyl glucozơ.

Hãy xác định cấu trúc của trehalozơ và isotrehalozơ.

II.45. Khuấy trộn tinh bột trong nước lọc lấy phần không tan gọi là amilopectin; phần tan trong nước chính là amilozơ.

Amilozơ bị thuỷ phân bởi mantaza bài II.44 cho mantozơ bài II.41. Cho amilozơ phản ứng với CH3I dư trong môi trường kiềm rồi thuỷ phân sản phẩm thì thu được 2,3,6–tri–O–metyl–D–glucopinanozơ và 0,2 – 0,4% 2,3,4,6–tetra–O–metyl–D–glucopinanozơ.

a) Hãy suy ra cấu trúc của amilozơ.

b) Hãy tính khối lượng mol phân tử của amilozơ.

II.46. Amilopectin bị thuỷ phân bởi mantaza tạo thành mantozơ.

Cho Amilopectin phản ứng với (CH3)2SO4 dư trong môi trường kiềm, rồi thuỷ phân sản phẩm thì thu được 2,3,4,6–tetra–O–metyl glucozơ (A) 2,3–đi–O–metylglucozơ (B) còn lại là 2,3,6–tri–O–metyl glucozơ (C).

a. Hãy viết công thức Havoc của 3 sản phẩm trên và cho biết xuất xứ của chúng.

b. Hãy suy ra cấu trúc của amilopeđin.

c. Trong 3 sản phẩm trên, sản phẩm nào là chủ yếu, sản phẩm nào chiếm hàm lượng nhỏ nhất, vì sao?

II.47. Khi thuỷ phân không hoàn toàn xenlulozơ thì thu được xenlobiozơ (C12H22O11). Xenlobiozơ là đisaccarrit có tính khử, có hiện tượng quay hỗ biến, tạo được phenyl osazon. Metyl hoá hoàn toàn xenlobiozơ rồi thuỷ phân bởi axit loãng thì thu được hỗn hợp đẳng phân tử 2,3,4,6–tetra–O–metyl–D–glucozơ và 2,3,6–

202

Page 197: 1ygm3hwjpgu090312060334 tài liệu bồi dưỡng giáo viên trường thpt chuyên   mon hóa học - phan 1- năm 2012

tri–O–metyl–D–glucozơ. Xenlobiozơ không bị thuỷ phân bởi các α–glucozidaza

mà bị thuỷ phân bởi các β–glucozidaza.

a) Xác định công thức cấu trúc của xenlobiozơ.

b) Suy ra cấu trúc của xenlulozơ.

c) Vì sao xenlulozơ dễ kéo thành sợi còn tinh bột thì không?

II.48. Hãy so sánh amilozơ, amilopectin, glicogen, xenlulozơ về công thức phân tử, cấu trúc phân tử, tính chất vật lí, tính chất hoá học và vai trò sinh học.

II.49. a. Viết sơ đồ phản ứng của xenlulozơ với axit nitric đặc xúc tác bởi axit sunfuric đặc.

b. Nêu ứng dụng của các sản phẩm thu được.

II.50. a. Viết sơ đồ phản ứng của xenlulozơ với anhiđrit axetic.

b. Vì sao để chế tạo tơ axetat xenlulozơ (tơ axetat) người ta dùng xenlulozơ điaxetat hỗn hợp với xenlulozơ triaxetat mà không dùng riêng từng thứ.

II.51. a. Viết các phản ứng khi chế hoá xenlulozơ với NaOH đặc và CS2.

b. Nếu ý nghĩa của các phản ứng này.

II.52. a. Dung dịch Svayde (nước Svayde) là gì? được sản xuất như thế nào?

b. Vì sao nước Svayde hoà tan được xenlulozơ thành dung dịch nhớt.

c. Tơ đồng amoniac được chế tạo như thế nào?

II.53. Chỉ dùng một hoá chất để nhận biết từng chất trong mỗi nhóm dung dịch sau:

a. Glucozơ, etanol, saccarozơ.

b. Mantozơ, fomalin, saccarozơ.

c. Fructozơ, glixerin, axetanđehit.

d. Mannozơ, axetanđehit, etilen glicol và propanol.

e. Các dung dịch keo của amylozơ, amylopectin, glicogen.

II.54. Phần lớn glucozơ do cây xanh tổng hợp ra trong quá trình quang hợp là để tạo ra xenlulozơ. Biết rằng một cây bạch đàn 5 tuổi có khối lượng gỗ trung bình là 100kg chứa 50% xenlulozơ.

a. Tính xem 1 ha rừng bạch đàn 5 tuổi mật độ 1 cây/20m2 đã hấp thụ được bao nhiêu m3 CO2 và giải phóng ra bao nhiêu m3 O2 để tạo ra xenlulozơ.

203

Page 198: 1ygm3hwjpgu090312060334 tài liệu bồi dưỡng giáo viên trường thpt chuyên   mon hóa học - phan 1- năm 2012

b. Nếu dùng toàn bộ gỗ từ 1 ha bạch đàn nói trên để sản xuất giấy (giả sử chứa 95% xenlulozơ và 5% phụ gia) thì sẽ thu được bao nhiêu tấn giấy, biết rằng hiệu suất chung của quá trình là 80% tính theo lượng xenlulozơ ban đầu.

II.55. Etanol sản xuất từ tinh bột dùng làm nhiên liệu được gọi là "nhiên liệu xanh".

a. Dùng phương trình phản ứng để chứng tỏ rằng nếu dùng tinh bột để sản xuất etanol nhiên liệu thì sẽ giảm được sự phát thải CO2 (chất gây hiệu ứng nhà kính).

b. Giả sử rằng trong động cơ khi đốt cháy 4 mol etanol thì thay được cho 1 mol isooctan. Hãy giải thích xem có phải cứ sản xuất ra được 4 mol etanol nhiên liệu từ tinh bột thì đã giảm được một lượng CO2 đúng bằng lượng CO2 khi đốt cháy hoàn toàn 1mol isooctan hay không?

II.56. Hãy chọn định nghĩa đúng:

A. Monoaccarit nền là những chất có công thức phân tử Cn(H2O)m

B. Monoaccarit nền là loại hợp chất polihiđroxicacbonyl.

C. Monoaccarit nền là loại hợp chất polihiđroxicacbonyl mạch hở.

D. Monoaccarit nền là những hợp chất polyhiđroxyanđehit kiểu H–[CHOH]n–CHO và polyhiđroxyxeton kiểu H–[CHOH]n–CO–[CHOH]m–H.

II.57. Hãy điền các từ thích hợp vào chỗ trống trong các câu sau:

a. Glixeranđehit là một...

b. Threozơ là một...

c. Ribozơ là một...

d. Mannozơ là một...

A: hexozơ, B: pentozơ, C: tetrozơ, D: triozơ

II.58. Hãy ghép các từ ngữ thích hợp để hoàn chỉnh các câu sau:

a. Galactozơ thuộc loại...

b. Lactozơ thuộc loại...

c. Mantozơ thuộc loại...

d. Fructozơ thuộc loại...

A. đisaccarit có tính khử; B. đisaccarit không có tính khử

204

Page 199: 1ygm3hwjpgu090312060334 tài liệu bồi dưỡng giáo viên trường thpt chuyên   mon hóa học - phan 1- năm 2012

C. anđohexozơ, D. xetohexozơ

II.59. Trong dung dịch, glucozơ tồn tại chủ yếu ở dạng...

A. furanozơ.

B. Anđehit

C. Xeton.

D. piranozơ.

II.60. Hãy chọn câu đúng:

A. Những hợp chất khác nhau về cấu hình của chỉ 1 nguyên tử C* là những epime.

B. Những đồng phân đia khác nhau về cấu hình của chỉ 1 nguyên tử C* là những epime.

C. Những đồng phân đia của mono saccarit khác nhau về cấu hình của chỉ 1 nguyên tử C* thứ nhất gọi là những epime.

D. Những đồng phân đia của mono saccarit khác nhau về cấu hình của chỉ 1 nguyên tử C* (không kể C anome) gọi là những epime.

II.61. Hãy điền số từ thích hợp vào chỗ trống trong câu sau:

"Một anđohexozơ có thể có... epime"

A. 1; B. 2; C. 4; D. 5.

II.62. Hãy điền chữ Đ (đúng) hoặc S (sai) vào ô trống sau mỗi câu:

a. Hai anome khác nhau về cấu hình chỉ ở một C*.

b. Hai anome khác nhau về cấu hình chỉ ở C hemiaxetal.

c. Anome có thể có cấu dạng mạch hở.

d. Anome phải có cấu tạo dạng vòng.

II.63. Hãy điền chữ Đ (đúng) hoặc S (sai) vào ô trống sau mỗi câu:

a. Anđozơ có sự quay hỗ biến.

b. Xetozơ không có sự quay hỗ biến.

c. Glicozit có sự quay hỗ biến.

d. Đisaccarit không có sự quay hỗ biến.

II.64. Trong dung dịch, fructozơ tồn tại dưới dạng....

205

Page 200: 1ygm3hwjpgu090312060334 tài liệu bồi dưỡng giáo viên trường thpt chuyên   mon hóa học - phan 1- năm 2012

A. 2 cặp epime. B. 4 đồng phân đia.

C. 4 đồng phân cấu tạo. D. 2 cặp anome.

II.65. Hãy điền chữ Đ (đúng) hoặc S (sai) vào ô trống sau mỗi câu:

a. Nếu tên của saccarit tận cùng bằng ozơ thì nó có tính khử.

b. Nếu tên của saccarit tận cùng bằng ozơ thì nó có sự quay hỗ biến.

c. Nếu tên của saccarit tận cùng bằng ozit thì nó không có tính khử.

d. Nếu tên thay thế theo IUPAC của cacbohiđrat tận cùng bằng ozơ thì nó có tính khử.

II.66. Hãy ghi chữ Đ (đúng) hoặc S (sai) vào sau mỗi câu:

a. Ở công thức Havoc của saccarit, nguyên tử O tạo vòng luôn viết ở góc bên phải phía trên.

b. Quay công thức Havoc 90o trong mặt phẳng chứa vòng sẽ thu được công thức của đối quang.

c. Quay công thức Fisơ 90o trong mặt phẳng trang giấy sẽ thu được công thức của đối quang.

II.67. a. α–D–glucopiranozơ và β– D–glucopiranozơ là 2....

b. β– D–glucopiranozơ và β– D–mannopiranozơ là 2....

c. β– D–mannopiranozơ và β– L–mannopiranozơ là 2....

d. β– D–mannopiranozơ và α– L–mannopiranozơ là 2....

A. epime; B. anome; C. enantiome; D. điastereome (đồng phân đia).

II.68. Hãy ghi chữ Đ (đúng) hoặc S (sai) vào ô trống:

a. Glicozit là ete ở C1 của monosaccarit.

b. Glicozit là axetal của furanozơ hoặc piranozơ.

c. Nhóm glicozyl là monosaccarit bị loại đi 1 nhóm OH.

d. Nhóm glicozyl là monosaccarit bị loại đi nhóm OH hemiaxetal.

II.69. Hai phân tử D–glucopiranozơ có thể tạo thành... đisaccarit.

A. 2; B. 4; C. 8; D. 11.

II.70. Hai phân tử D–glucopiranozơ có thể tạo thành... đisaccarit có tính khử.

A. 2; B. 4; C. 8; D. 10.

206

Page 201: 1ygm3hwjpgu090312060334 tài liệu bồi dưỡng giáo viên trường thpt chuyên   mon hóa học - phan 1- năm 2012

HƯỚNG DẪN GIẢI

II.1. Xem các mục I.1, I.2, I.28 – I.20.

II.2. Xem các mục I.3 – I.6.

Ghi chú: Công thức của cacbohiđrat khá phức tạp, muốn ghi nhớ cần triệt để vận dụng phương pháp so sánh – liên tưởng 2 hoặc 3 chất tương tự trong từng tốp. Ví dụ, sau khi so sánh cấu tạo dạng mạch hở của glucozơ và fructozơ (Bài 2.3) ta chỉ cần nhớ cấu tạo của glucozơ sẽ liên tưởng ra được cấu tạo của fructozơ vì ”Glucozơ: C1 là nhóm anđehit CH=O, C2 – C6 đều có 1 nhóm OH; Fructozơ: C1 là nhóm CH2OH, C2 là nhóm xeton >C=O, còn lại thì giống với glucozơ; Amilozơ chứa khoảng 1000–4000 gốc glucozơ nối với nhau bởi liên kết α–1,4–glicozit: amilopectin có thêm liên kết α–1,6–glicozit và số gốc glucozơ thì lên tới 2000–200 000; xenlulozơ chỉ khác amilozơ là các gốc glucozơ nối với nhau bởi liên kết liên kết β–1,4–glicozit và số gốc glucozơ thì lên tới 70 000–200 000; Như vậy chỉ cần nhớ công thức của amilozơ thì liên tưởng ra được công thức của amilopectin và xenlulozơ. Các bài II.3 – II.9 sẽ giúp rèn luyện phương pháp so sánh – liên tưởng.

II.3.

Glucozơ Fructozơ

1 nhóm cacbonyl anđehit, –CH=O 1 nhóm cacbonyl xeton, >C=O

1 nhóm hiđroxy ancol bậc một, HO–CH2–

2 nhóm hiđroxy ancol bậc một, HO–CH2–

4 nhóm hiđroxy ancol bậc hai, HO–CH– 3 nhóm hiđroxy ancol bậc hai, HO–CH–

II.4. HOCH2CHOHCH=O (1,2–đihiđroxypropanal), HOCH2COCH2OH (1,3–đihiđroxypropanon) đều thuộc loại hợp chất polihiđroxycacbonyl phù hợp định nghĩa nêu ở mục I.1, I.2 nên đều thuộc cacbohiđrat loại monosaccarit.

II.5. a)

α–Glucozơ β–Glucozơ

207

Page 202: 1ygm3hwjpgu090312060334 tài liệu bồi dưỡng giáo viên trường thpt chuyên   mon hóa học - phan 1- năm 2012

Nhóm OH đính với nhóm C1–O gọi là nhóm OH hemiaxetal, nó nằm cùng phía (phía dưới của vòng) so với HO–C2, HO–C4 tức là cùng phía với liên kết O–C5 nên gọi là dạng α.

Nhóm OH đính với nhóm C1–O gọi là nhóm OH hemiaxetal, nó nằm khác phía (phía trên của vòng) so với HO–C2, HO–C4 tức là khác phía với liên kết O–C5 nên gọi là dạng β.

Nhóm C1–O– C5 là nhóm ete đặc biệt vì C1

còn đính với nhóm OH hemiaxetal, nên nhóm C1–O–C5 H–OH có thể tự chuyển thành nhóm C1H=O và nhóm C5–OH làm cho dạng mạch vòng chuyển thành dạng mạch hở.

Nhóm C1–O– C5 là nhóm ete đặc biệt vì C1

còn đính với nhóm OH hemiaxetal, nên nhóm C1–O–C5 H–OH có thể tự chuyển thành nhóm C1H=O và nhóm C5–OH làm cho dạng mạch vòng chuyển thành dạng mạch hở.

1 nhóm hiđroxy ancol bậc một, HO–C6 1 nhóm hiđroxy ancol bậc một, HO–C6

3 nhóm hiđroxy ancol bậc hai: HO–C 2 và HO–C4 ở phía dưới vòng, HO–C3 ở phía trên vòng

3 nhóm hiđroxy ancol bậc hai: HO–C 2 và HO–C4 ở phía dưới vòng, HO–C3 ở phía trên vòng

b) Glucozơ có thể kết tinh thành 2 dạng tinh thể có nhiệt độ nóng chảy khác nhau. Khi glucozơ tác dụng với metanol có xúc tác axit thì thu được 2 đồng phân có cùng công thức cấu tạo gọi là metyl–α–glucozit và metyl–β–glucozit.

II.6. Như ta đã biết, một công thức cấu tạo có thể ứng với nhiều cấu trúc không gian khác nhau, vì vậy một công thức cấu tạo có thể ứng với nhiều hợp chất khác nhau. Với công thức cấu tạo HOCH2(CHOH)4CH=O ứng với 16 cấu trúc không gian khác nhau về vị trí tương đối của 4 nhóm OH ancol bậc hai. Từ bài tập II.5 ta thấy: ở glucozơ nhóm HO–C2 và nhóm HO–C4 ở phía dưới vòng, còn nhóm HO–C3 ở phía trên vòng. Nếu nhóm HO–C2 ở phía trên vòng thì không còn là glucozơ nữa mà là mannozơ. Do đó thực tế thì phản ứng đã cho sẽ tạo ra một hỗn hợp 16 chất khác nhau có cùng cấu tạo, trong đó chỉ có 1/16 là glucozơ. Như vậy thì sản phẩm tạo ra không gọi là glucozơ được.

II.7.

α–Fructozơ (vòng 5 cạnh) β–Fructozơ (vòng 5 cạnh)

208

Page 203: 1ygm3hwjpgu090312060334 tài liệu bồi dưỡng giáo viên trường thpt chuyên   mon hóa học - phan 1- năm 2012

Nhóm OH đính với nhóm C2–O gọi là nhóm OH hemiaxetal, nó nằm cùng phía (phía dưới của vòng) với HO–C4 tức là cùng phía với liên kết O–C5 nên gọi là dạng α.

Nhóm OH đính với nhóm C2–O gọi là nhóm OH hemiaxetal, nó nằm khác phía (phía trên của vòng) với HO–C4 tức là cùng phía với liên kết O–C5 nên gọi là dạng β.

Nhóm C2–O– C5 là nhóm ete đặc biệt vì C2

còn đính với nhóm OH hemiaxetal, nên nhóm C2–O–C5 H–OH có thể tự chuyển thành nhóm >C2=O và nhóm C5–OH, khi đó dạng mạch vòng chuyển thành dạng mạch hở.

Nhóm C2–O–C5 là nhóm ete đặc biệt vì C2

còn đính với nhóm OH hemiaxetal, nên nhóm C2–O–C5 H–OH có thể tự chuyển thành nhóm >C2=O và nhóm C5–OH, khi đó dạng mạch vòng chuyển thành dạng mạch hở.

2 nhóm hiđroxy ancol bậc một, HO–C1 và HO–C6

2 nhóm hiđroxy ancol bậc một, HO–C1 và HO–C6

2 nhóm hiđroxy ancol bậc hai: HO–C3 ở phía trên vòng, HO–C4 ở phía dưới vòng

2 nhóm hiđroxy ancol bậc hai: HO–C3 ở phía trên vòng, HO–C4 ở phía dưới vòng

II.8.

Saccarozơ (kết tinh và trong dung dịch), C12H22O11

Mantozơ (kết tinh)

C12H22O11

Gốc α–glucozơ và gốc β–fructozơ liên kết với nhau qua nguyên tử O giữa C1 của gốc

α–glucozơ và C2 của gốc β–fructozơ (αC1–O– βC2 gọi tắt là liên kết α,β–1,2–glicozit).

Hai gốc α–glucozơ liên kết với nhau qua nguyên tử O giữa C1 của gốc α–glucozơ

này và C4 của gốc α–glucozơ kia (ỏ C1–O– C4 gọi tắt là liên kết α–1,4–glicozit).

Ở gốc α–glucozơ nhóm OH hemiaxetal (HO–C1) không còn, ở gốc β–fructozơ nhóm OH hemiaxetal (HO–C2) cũng không còn, vì vậy không thể mở vòng tạo ra nhóm C=O được.

Ở gốc α–glucozơ thứ nhất nhóm OH hemiaxetal (HO–C1) không còn, ở gốc α–glucozơ thứ hai nhóm OH hemiaxetal (HO–C1) vẫn còn, vì vậy trong dung dịch có thể mở vòng tạo ra nhóm CH=O.

II.9.

209

Page 204: 1ygm3hwjpgu090312060334 tài liệu bồi dưỡng giáo viên trường thpt chuyên   mon hóa học - phan 1- năm 2012

Amilozơ, (C6H10O5)n, [C6H7O2 (OH)3]n Amilopectin, (C6H10O5)n, [C6H7O2 (OH)3]n

Các gốc glucozơ nối với nhau bằng liên kết α–1,4–glicozit, n = 1000–4000, M = 150 000– 600 000 au; phân tử xoắn thành dạng lò xo.

Cả phân tử chỉ có 1 nhóm OH hemiaxetal.

Cứ khoảng 20–30 gốc glucozơ nối với nhau bằng liên kết α–1,4–glicozit hợp thành 1 chuỗi, gốc glucozơ cuối cùng của chuỗi tạo liên kết α–1,6–glicozit với 1 gốc glucozơ của chuỗi khác hình thành sự phân nhánh, n = 2000–200 000, M = 300 000 – 3 000 000 au; phân tử xoắn thành dạng búi rễ. Cả phân tử chỉ có 1 nhóm OH hemiaxetal.

Xenlulozơ, (C6H10O5)n , [C6H7O2 (OH)3]n Tinh bột, (C6H10O5)n, [C6H7O2 (OH)3]n

Tinh bột là hỗn hợp của amilozơ (khoảng 20–30 %) và amilopectin

(khoảng 70–80 %). Công thức cấu tạo xem ở bài II.8.

Các gốc glucozơ nối với nhau bằng liên kết β–1,4–glicozit, n = 70 000–200 000, M = 1 000 000 – 2400 000 au; phân tử duỗi thẳng.

Cả phân tử chỉ có 1 nhóm OH hemiaxetal.

Các gốc glucozơ nối với nhau bằng liên kết α–1,4–glicozit và α–1,6–glicozit, n = 1000–200000, M = 150 000– 3 000 000 au; phân tử xoắn thành dạng lò xo và dạng búi rễ.

Cả phân tử chỉ có 1 nhóm OH hemiaxetal.

II.10. a. Xem mục I.8.

b. Dãy D: II, dãy L: I, III và IV. Để giải bài tập này cần tự thể nghiệm quy tắc sau đây: Đối với C* đầu mạch, khi đổi chỗ 3 nhóm thế tuần tự theo kiểu xoay vần (xem bài II.27) thì cấu hình của nó không bị thay đổi.

II.11. a) Xem mục I.19.

b) Epimer: I và II; III và IV, Enantiomer: II và III.

210

Page 205: 1ygm3hwjpgu090312060334 tài liệu bồi dưỡng giáo viên trường thpt chuyên   mon hóa học - phan 1- năm 2012

II.12. a) A là D–Threozơ; B có thể là D–Erithrozơ hoặc L–Erithorozơ (B').

CHO

HO H

CH2OH

H OH

COOH

HO H

COOH

H OH

HNO3

A (2S,3R) Axit D-tactric (2S,3S)

b.

HNO3

CHO

H OH

H OH

CH2OH

COOH

H OH

H OH

COOH COOH

HHO

HHO

COOH

CH2OH

HHO

HHO

CHO

HNO3

B (2R, 3R) Axit mesotactric (2R,3S) B' (2S, 3S)

c. Phản ứng oxi hoá bằng HNO3 trong các trường hợp trên không làm thay đổi trật tự của 4 liên kết xung quanh các nguyên tử C* vì vậy về thực chất là không làm thay đổi cấu hình. Tuy nhiên vì sau phản ứng trật tự về độ hơn cấp của 4 nhóm thế quanh nguyên tử C* bị thay đổi, do đó kí hiệu cấu hình (R, S, D, L) có thể bị thay đổi (Xem kỹ câu b).

d. D–Threozơ (A) và D–Erithrozơ (B).

II.13. a. I: D – Glucozơ, (2R, 3S, 4R, 5R)–2,3,4,5,6–pentahiđroxihexanal.

b. HOCH2ZCHO (I) → O2/H2Br HOCH2ZCOOH (II), axit D–gluconic.

axit (2R, 3S, 4R, 5R)–2,3,4,5,6–pentahiđroxi hexanoic.

II.14. a. AgNO3 + 3NH3 + H2O → [H3N – Ag – NH3]OH + NH4NO3.

b. Cho dung dịch CuSO4 vào dung dịch Natri kali tactrat và NaOH trong

nước: CuSO4 + 2NaOH → Cu(OH)2 + Na2SO4

COOK

HC

HC OH

COONa COONa

CH

CH

HO

HO

COOK

+ Cu(OH)2 +

COOK

HC

HC

O

O

COONa COONa

CH

CH

O

O

COOK

Cu

H

H+ 2H2O

(Fel.)

OH

c. Vì: HOCH2[CHOH]3–CO–CH2OH →−OH HOCH2[CHOH]4CHO

211

Page 206: 1ygm3hwjpgu090312060334 tài liệu bồi dưỡng giáo viên trường thpt chuyên   mon hóa học - phan 1- năm 2012

Nên cả anđohexozơ và xetohexozơ đều có phản ứng tráng bạc và phản ứng với thuốc thử Feling. Do đó, không phân biệt được anđohexozơ và xetohexozơ bằng hai thuốc thử trên.

d. Anđohexozơ làm mất màu nước brom còn xetohexozơ thì không.

II.15.

CH2OH

C O

HC OH

Xetoz¬

CHOH

C OH

HC OH

D¹ng enol

CH=O

HC OH

HC OH

An®oz¬

OH -

II.16. a.

CH=O

HO H

CH2OH

H OH

CH=N-NH-Ph

HO H

CH2OH

H OH

CH=N-NH-Ph

C O

CH2OH

H OH

CH=N-NH-Ph

C N-NH-Ph

CH2OH

H OH

PhNHNH2

-H2O

PhNHNH2

- PhNH2 , - NH3

PhNHNH2

-H2O

thuéc lo¹ i hi®razon thuéc lo¹ i ozazonthuéc lo¹ i hi®razon

b. Do tạo liên kết hiđrô nội phân tử dạng vòng 6 cạnh bền (hãy vẽ ra).

II.17. Như đã biết ở bài II.16, phản ứng tạo ozazon xảy ra chỉ ở C1 và C2. Sau phản ứng C1 và C2 không còn khác nhau về cấu tạo và cấu hình nữa. Vì thế, tối đa có 3 monosaccarit: 2 epime ở C2 của anđozơ và 1 xetozơ tương ứng (cùng số nguyên tử C, cùng cấu hình từ C3 trở đi). Ví dụ, trong số 16 hexozơ, có các bộ ba như : D–Glucozơ, D– Mannozơ và D–Fructozơ; L–Glucozơ, L–Mannozơ và L–Fructozơ.

II.18. a. H2/Ni; Na/Hg; NaBH4.

b. D–Glucozơ → /NiH2 D–Gluxitol (xem bài II.13).

D–Fructozơ → /NiH2 D–Gluxitol + (2R, 3R, 4R, 5R)–hexan–1,2,3,4,5,6–hexaol.

II.19. a.

212

Page 207: 1ygm3hwjpgu090312060334 tài liệu bồi dưỡng giáo viên trường thpt chuyên   mon hóa học - phan 1- năm 2012

COOH

H OH

HO H

H OH

H OH

COOH

C

H OH

HO H

H O

H OH

COOH

O COOH

H OH

O H

H OH

H OH

C O

C

H OH

O H

H O

H OH

C O

O

(A) (B) (C) (D)

b. Tương tự như câu a.

II.20. Đó là các cặp anđohezozơ mà ở công thức Fisơ của chúng 4C* (C2, C3, C4, C5) nhận mặt phẳng vuông góc với liên kết C3 – C4 (tại trung điểm) làm mặt phẳng đối xứng: D–anlozơ và L–anlozơ; D–galactozơ và L–galactozơ.

II.21. Đó là cặp anđohexozơ mà ở công thức Fisơ 4C* (C2, C3, C4, C5) của chất này khi quay 1800 trong mặt phẳng giấy thì chuyển thành 4C* của chất kia. Ví dụ:

H OH

H OH

H OH

H OH

(A) (B) (C) (D)

HHO

HHO

HHO

HHO

180o

H OH

HO H

H OH

H OH HHO

OHH

HHO

HHO

180o

D–allozơ và L–allozơ; D–galactozơ và L–galactozơ; D–glucozơ và L–gulozơ, D–gulozơ và L–glucozơ, D–talozơ và L–altrozơ; D–altrozơ và L–talozơ.

II. 22. a.

213

Page 208: 1ygm3hwjpgu090312060334 tài liệu bồi dưỡng giáo viên trường thpt chuyên   mon hóa học - phan 1- năm 2012

CH=O

H OH

CH2OH

H OH

CN

CHOH

H OH

H OH

CH2OH

COOH

CHOH

H OH

H OH

CH2OH

C

CHOH

H OH

H O

CH2OH

OCH=O

H OH

H OH

CH2OH

OHH

CH=O

H OH

H OH

CH2OH

HHO

+Na/Hg

NaCN / HCN 1. Ba(OH)2

2. H3O+

(II) (I)

b. Không bằng nhau vì phản ứng cộng vào nhóm C=O theo quy tắc Cram.

d. Ngày nay người ta khử trực tiếp xianohiđrin bằng hiđro với xúc tác thành anđozơ.

II.23. b.

CHO

H OH

HO H

HO H

H OH

CH2OH

COOH

H OH

HO H

HO H

H OH

COOH

CHO

HO H

HO H

H OH

CH2OH

COOH

HO H

HO H

H OH

COOH

HNO3 HNO3Br2

H2OH2O2

Fe2(SO4)3

2

4

5

3

4

5

3

2

3

4

5 5

4

3

Axit D-Galactaric D(+)-Galactoz¬ D(-)-Lyxoz¬ Axit D-Lyxaric

Axit D–Galactaric không quang hoạt suy ra OH (2) cùng phía phải OH (5) và OH (3) cùng phía với OH (4).

Axit D–Lyxaric quang hoạt mà đã biết OH (3) cùng phía với OH (4) vậy cả hai đều phải ở bên trái.

214

Page 209: 1ygm3hwjpgu090312060334 tài liệu bồi dưỡng giáo viên trường thpt chuyên   mon hóa học - phan 1- năm 2012

II.24. a.

RCH=O R - CHOCH3

OHR - CH

OCH3

OCH3

CH3OH / HCl CH3OH / HCl

- H2O

Hemiaxetal Axetal

CH3OH / HCl

- H2OCH2 - CH - CH - CH - CH - CH - OH

OH OH OH OH

O* CH2 - CH - CH - CH - CH - CH - OCH3

OH OH OH OH

O

Hemiaxetal néi OH hemiaxetal Glicozit (Glycoside)

Cacbon hemiaxetal (cacbon anomer)

b. Anđehit phản ứng với 2 mol ancol tạo thành hemiaxetal rồi axetal. Anđohexozơ thường đã tồn tại ở dạng hemiaxetal nội phân tử nên chỉ phản ứng với 1 mol ancol.

II.25. a. Xem mục I.16, I.19.

b. Hãy tự viết cân bằng giữa dạng mạch hở và 4 dạng mạch vòng.

c. So với C cấu hình (mục I.16, thường là C* cuối cùng trong phân tử).

II.26. b. 36,2% và 63,8%.

c. Trong dung dịch D–fructozơ tồn tại ở 2 cặp epimer:

α–D–Fructo piranozơ 3% ; β–D–fructopiranozơ 57%

α–D–Fructofuranozơ 9%; β–D–fructofuranozơ 31%

II.27. Chú ý rằng đối với công thức Fisơ, có thể "xoay vần" 3 nhóm thế ở C*

cuối mạch mà không làm thay đổi cấu hình của nó (được phép) cách thức chuyển từ công thức Fisơ sang công thức Havoc.

α-L-Glucopiranoz¬

HO H

H OH

HO H

H CH2OH

O

HO H

OOH OH

OH

OH

H

CH2OH"Xoay vÇn" cho O xuèng d­ í i

ChuyÓn phÝa bªn ph¶i xuèng phÝa d­ í i lôc gi¸ c

b.

HO H

H OH

HO H

O H

CH2OH

HO H

1

2

3

4

5

1

2

3

4

5

6

6

215

Page 210: 1ygm3hwjpgu090312060334 tài liệu bồi dưỡng giáo viên trường thpt chuyên   mon hóa học - phan 1- năm 2012

CH2OH

HO

HO H

H OH

H O

CH2OH

1

2

3

4

56

d. CH2OH

HO

HO H

H OH

HOH2C H

O

CH2OH

OH

H

CH2OH

OH H

H OHO

β-D-Fructofuranoz¬

1

2

34

5

6

II.28. Công thức Fisơ biểu diễn được quan hệ không gian của 4 nhóm thế ở C* nhưng không phải là công thức lập thể mà là công thức chiếu theo quy ước. Vì vậy không được quay nó một cách tuỳ ý. Ngược lại, công thức Havoc là công thức lập thể (nó giống như một vật thể xác định), vì vậy có thể xoay nó theo mọi cách mà không làm thay đổi cấu hình của C*. Đối với C*, nếu ta đổi chỗ 2 nhóm bất kì cho nhau thì đều làm thay đổi cấu hình của nó. Điều đó là không được phép.

II.29.

a. Nên chuyển qua công thức Havoc (xem bài III.27).

OHO

HO

OH

OH

HOH2C

O

OH

HO OH

HO

CH2OH

O

OH OH

OH

OH

CH2OHb vµ c.

II.30. a.

OHO

HOOH

OH

OH

O

OH

HO

OH

HO

CH2OH

216

Page 211: 1ygm3hwjpgu090312060334 tài liệu bồi dưỡng giáo viên trường thpt chuyên   mon hóa học - phan 1- năm 2012

O

OH

OH

OH

CH2OH

OH

OHO

HOOH

HOH2C

OH

b.

c. Trong dung dịch, β–D–glucopiranozơ chiếm 64%, α–D–glucopiranozơ

chiếm 36%.

II.31. Hãy chỉ ra C anome, chuyển về công thức Fisơ rồi so với công thức trong các bảng hệ thống ở chương cacbohiđrat trong các giáo trình hoá hữu cơ.

c. và d. β–D–Fructozơ (quay công thức khỏi mặt phẳng vòng 1800 được công

thức d) thao tác đó là được phép, xem bài tập II. 28).

e. α–D–Glucofuranozơ.

d. Epime ở C4 của α–D–glucofuranozơ.

II.32.

b. và c. H+ và OH– đều xúc tác cho các phản ứng sau:

andoz¬ en®iol xetoz¬

Mannoz¬ en®iol Fructoz¬

d. 1,2,3,4,6–penta–O–axetyl–D–mannopiranozơ

e. Metyl D–mannopiranozit

f. Metyl 2,3,4,6–tetra–O–metyl–D–mannopiranozit

O

H

AcO

H

AcO

OAc

HH

OAc

OAc

O

H

HO

H

HO

OH

HH

OH

OMe

O

H

MeO

H

MeO

OMe

HH

OMe

OMe

II.33. a. A: HOCH2[CHOH]4CH=NOH ; B: AcOCH2[CHOAc]4CH=NOAc

C: AcOCH2[CHOAc]4CH≡N ; D: Arabinozơ

b) MeO– tấn công cacbon cacboxyl của các gốc CH3–CO tạo ra metyl este (este hoá lại), đồng thời ngắt proton của OH ở C2 làm cho CN– tách ra (phản ứng

217

Page 212: 1ygm3hwjpgu090312060334 tài liệu bồi dưỡng giáo viên trường thpt chuyên   mon hóa học - phan 1- năm 2012

nghịch của phản ứng cộng CN– vào nhóm cacbonyl). Hãy viết rõ sơ đồ phản ứng và dùng mũi tên cong để biểu diễn sự giải thích trên.

II.34.

CH = O

HC OH

H2C OH

HCOOH

HCOOH

HCH = O

2HIO4

- 2HIO3

+

+

H2C

C O

H2C OH

OH

2HIO4

- 2HIO3, -H2O

HCH = O

HCH = O

CO2

+

+

c. d.

COOH

HC OH

H2C OH

CH = O

HCH = O

HIO4

- HIO3+

COOH

HC OH

CH = O

HIO4

e. g.

- HIO3, -H2O

COOH

CH = O+

COOH

HCOOH

Axit periođic phân cắt được liên kết C – C nếu 2C đó là cacbon ancol, cacbon anđehit và cacbonxeton; cứ 1 mol HIO4 phân cắt được 1mol liên kết C – C và hình thành 1 mol HIO3, nhóm –CH2OH (bậc I) chuyển thành HCHO; nhóm –CHOH (bậc 2) và nhóm –CH = O chuyển thành HCOOH; nhóm C = O (xeton của xetozơ) chuyển thành CO2.

II.35.a.

CH3

O H

HO H

H OH

H OH

HHO

OOH OH

OHOH

CH3

OOH OCH3

OHOH

CH3

CH2OH

H3C H

O

H OCH3

CH2OH

(C)

b. Phản ứng với HIO4 xảy ra bình thường. Nhóm CH=O hình thành từ C4

chuyển thành nhóm enol (lấy H ở C5). Nhóm CH = O hình thành từ C2 cộng với 1 phân tử H2O tạo ra gemđiol hiđrat bền do 2 nhóm OR hút electron).

c. 2 nhóm OH ancol phản ứng được với CH3I/Ag2O tạo ra metyl ete nhóm OH enol thì không.

d. Nhóm enol bị khử thành ancol, nhóm –CH=O sinh ra từ gemđiol cũng bị khử thành nhóm –CH2OH.

218

Page 213: 1ygm3hwjpgu090312060334 tài liệu bồi dưỡng giáo viên trường thpt chuyên   mon hóa học - phan 1- năm 2012

II.36. a. A: HCOOH + O=HC–COOH + HOCH2CH(OH)COOH

b. B: HCOOH + O=HC–COOH + HOOC–CH(OH)–COOH

II.37. a. Phân tử mono và đisaccarit có kích thước tương đối nhỏ lại có nhiều liên kết phân cực và nhiều nhóm có khả năng tạo liên kết hiđro với nước. Kích thước của các phân tử polisaccarit rất lớn (gồm hàng ngàn mắt xích) bản thân chúng liên kết với nhau bằng lực Vanđeran bằng liên kết hiđro tạo ra các hạt, các sợi kích cỡ lớn, giảm tương tác với nước.

b. Khối lượng phân tử lớn gần gấp đôi, số nhóm phân cực và liên kết hiđro cũng vậy.

II.38. a. Rất đặc hiệu có tính đặc thù lập thể và chỉ tấn công vào một C* duy nhất.

b. X: axeton để bảo vệ các nhóm OH.

c. Ở phản ứng (2) bằng cách chỉ tấn công vào một C* duy nhất, hãy chỉ ra C* đó.

d. Điện tích âm ở ion enolat hình thành từ OH ở C3 được giải toả trong hệ liên hợp.

O C C C O O C C C O

II.39. a. Xem mục I.20

b. Phần còn lại khi bỏ nhóm OH – hemiaxetal (nhóm OH anome) khỏi phân tử monoaccarit được gọi là glicozyl, đó không phải là một loại hợp chất như glicozit mà chỉ là tên gọi của một loại nhóm thế đặc biệt. Chúng được phân loại tương tự như glicozit (xem mục I.20).

II.40. Trong 2 liên kết C – O giữa 2 mắt xích monosaccarit, nhất thiết phải có 1 liên kết C – O thuộc loại liên kết gilcozit.

219

Page 214: 1ygm3hwjpgu090312060334 tài liệu bồi dưỡng giáo viên trường thpt chuyên   mon hóa học - phan 1- năm 2012

II.41. a. α–D–Glucopiranozyl–(1→4)–D–glucopiranozơ.

b. β–D–Glucopiranozyl–(1→4)–D–glucopiranozơ.

II.42. b. Saccarozơ (Sucrose) là đường không có tính khử vì cả hai nhóm OH hemiaxetal ở glucozơ và ở fructozơ đều đã tạo thành liên kết glicozit không cho phép quay về dạng anđose hoặc dạng xetozơ nên không có tính khử.

II.43. Melibiozơ bị thuỷ phân bởi α–galactozidaza chứng tỏ mắt xích D–

galactozơ ở dạng α–D–galactozit.

220

Page 215: 1ygm3hwjpgu090312060334 tài liệu bồi dưỡng giáo viên trường thpt chuyên   mon hóa học - phan 1- năm 2012

II.44. Dữ kiện thuỷ phân bởi enzim cho biết cả hai liên kết glicozit ở trehalozơ

đều là α–glicozit, còn ở isotrehalo thì có 1 liên kết là α–glicozit, còn liên kết kia thì

là β–glicozit. Sản phẩm duy nhất 2,3,4,6–tetra–O–metyl glucozơ cho biết cả 2 mắt

xích D–glucozơ đều ở dạng vòng 6 cạnh (piranozit). Vậy, Trehalozơ là α–D–

glucopiranozyl–(1→1)–α–D–glucopiranozit, Isotrehalozơ là α–D–glucopiranozyl–

(1→1)–β–D–glucopiranozit.

221

Page 216: 1ygm3hwjpgu090312060334 tài liệu bồi dưỡng giáo viên trường thpt chuyên   mon hóa học - phan 1- năm 2012

II.45.

O

HOOH

OH

OO

HO

O

OH

OH

n

n = 250 - 500

II.46. a.

O

OMe

OMe

OMe

CH2OMe

OH

O

OH

OMe

OMe

CH2OH

OH

O

OH

OMe

OMe

CH2OMe

OH

(A) (B) (C)

A: Mắt xích glucozơ đầu mạch (vì có 4 nhóm OH đều bị metyl hoá).

B: Mắt xích glucozơ ở chỗ phân nhánh (vì chỉ có 2 nhóm OH bị metyl hoá).

C: Mắt xích glucozơ ở giữa mạch (giống như giữa mạch amylozơ).

b. Bị thuỷ phân bởi mantaza cho biết amilopeđin chỉ chứa liên kết α–glicozit.

Từ dữ kiện ở câu (a) cho thấy các mắt xích glucozơ liên kết với nhau theo hai

cách (1→4)–α–glucozit và (1→6)–α–glucozit.

c. Amilopedin có cấu tạo phân nhánh, thường cứ khoảng 20 – 25 mắt xích glucozơ thì lại phân nhánh. Vì vậy C >> B > A.

II.47. a. Xem bài II.41.

b. Các mắt xích glucozơ ở xenlulozơ liên kết với nhau bởi liên kết (1→4)–β–

glicozit.

c. Các liên kết (1→4)–α–glicozit và liên kết (1→4)–β–glicozit của

amilopectin làm cho mạch phân tử tinh bột bị xoắn lại, bị phân nhánh chằng chịt khó kéo thành đoạn thẳng. Xenlulozơ không phân nhánh nên có dạng thẳng, dễ

tạo thành sợi lại chỉ chứa các liên kết (1→4)–β–glicozit.

222

Page 217: 1ygm3hwjpgu090312060334 tài liệu bồi dưỡng giáo viên trường thpt chuyên   mon hóa học - phan 1- năm 2012

II.48. Hãy lập bảng so sánh sau.

Amilozơ Amilopectin Glicozen Xenlulozo

Công thức

phân tử

(C6H10O5)n

n= 250– 500

(C6H10O5)n

n =….

(C6H10O5)n

n =….

(C6H10O5)n

n =….

Cấu trúc

phân tử

Liên kết

(1→4)–α–

glicozit

Cứ 20 – 25 liên kết

(1→4)–α–glicozit lại có

1 liên kết (1→6)–α–

glucozit

Cứ 8 – 12 liên kết

(1→4)–α–glicozit lại

có 1 liên kết (1→6)–

α–glucozit

Liên kết

(1→4)–β–

glicozit

Tính chất

vật lí

………

…………. …………. …………. ………….

II.49. a. Tuỳ theo điều kiện phản ứng để tạo thành xenlulozơ mono, đi nitrat hoặc trinitrat.

II. 50. a. Phản ứng thường tạo ra hỗn hợp xenlulozơ mono–, đi– và triaxetat.

b. Xenlulozơ trinitrat không có hiđro linh động đủ để tạo liên kết hiđro giữa các chuỗi polime với nhau nên sợi sẽ kém bền. Xenlulozơ điaxetat còn nhiều liên kết hiđro giữa chuỗi polime làm cho sợi kém mảnh và kém mềm mại.

II.51. b. Hoà tan được xenlulozơ thành dung dịch nhớt (visco) để chế tơ visco.

II.52. a. Cho vụn đồng tác dụng với dung dịch amoniac và oxi không khí thì thu được dung dịch đồng ammiacat gọi là dung dịch Savayde.

Cu + 5NH3 + 1/2 O2 → [Cu(NH3)4] (OH)2.

b. Trong môi trường kiềm, 2 nhóm OH cạnh nhau ở một mắt xích glucozơ của xenlulozơ có thể tạo phức với Cu++ thay cho 2 phân tử NH3.

II.53. a. Glucozơ tạo với Cu(OH)2 phức chất màu xanh da trời, khi đun sôi thì bị khử thành Cu2O màu đỏ gạch. Saccarozơ tạo với Cu(OH)2 phức chất màu xanh, khi đun sôi không xuất hiện kết tủa đỏ gạch. Etanol không tạo phức màu xanh với Cu(OH)2, khi đun sôi cũng không tạo ra kết tủa đỏ gạch.

b, c, d tương tự như câu a.

e. Dung dịch I2 và KI trong nước có màu hơi vàng. Amilozơ tạo màu xanh thẫm, amilopectin tạo màu nâu nhạt hoặc tím hung, glicogen hầu như không có phản ứng.

II.54. a. 6CO2 + 6H2O → s¸ng ¸nh lôc, DiÖp C6H12O6 + 6O2 (1)

223

Page 218: 1ygm3hwjpgu090312060334 tài liệu bồi dưỡng giáo viên trường thpt chuyên   mon hóa học - phan 1- năm 2012

nC6H12O6 → Enzim (C6H10O5)n + nH2O (2)

2COV = 2OV = 20741 m3

b. mgiấy = 21,053 tấn.

II.55. a. Tinh bột tạo ra từ CO2 của khí quyển, khi chuyển thành etanol rồi đốt cháy lại giải phóng CO2 vào khí quyển đúng bằng lượng đã tạo ra nó.

6CO2 + 5H2O → s¸ng ¸nh lôc, DiÖp (C6H10O5)n + 6O2

(C6H10O5)n + nH2O → n C6H12O6

nC6H12O6 → Zimaza 2nC2H5OH + 2nCO2

2nC2H5OH + 6nO2 → 4nCO2 + 6nH2O

6nCO2 → 6nCO2

b. 4C2H5OH + 12O2 → 8CO2 + 12H2O

C8H8 + 12,5O2 → 8CO2 + 9H2O

Lượng CO2 ở 2 phản ứng như nhau. Tuy nhiên để sản xuất ra tinh bột rồi ra etanol phải cần có năng lượng cho máy móc hoạt động và cho nhân công làm việc. Nếu dùng nhiên liệu hoá thạch để đảm bảo nhu cầu năng lượng đó thì sẽ phát thải ra CO2, nếu dùng etanol để đảm bảo nhu cầu đó thì không còn đủ 4mol etanol thay cho 1mol isooctan. Vậy?….

II.56. D

II.57. a) D; b) C; c) B; d) A.

II.58. a) C; b) B; c) A; d) D.

II.59. D

II.60. D

II.61. C

II.62. a. Đ; b. Đ; c. S; d. Đ

II.63. a. Đ; b. S; c. S; d. S

II.64. D

II.65. a. S; b. S; c.Đ; d. Đ.

II.66. a. S; b. S; c. Đ.

II.67. a. B; b. A; c. C; d. D.

II.68. a. S; b. Đ; c. S; d. Đ.

II.69. D.

II.70. C.

224

Page 219: 1ygm3hwjpgu090312060334 tài liệu bồi dưỡng giáo viên trường thpt chuyên   mon hóa học - phan 1- năm 2012

Ho¸ häc cacbonyl

TS. Đoàn Duy Tiên, PGS.TS. Nguyễn Văn Đậu,

Bộ môn Hóa hữu cơ, Khoa Hóa học, ĐHKHTN, ĐHQGHN

Email: [email protected]

1. MỞ ĐẦU

Hoá học cacbonyl là nền tảng của tổng hợp hữu cơ do khả năng tạo ra các liên kết cacbon–cacbon và cacbon với các nguyên tố khác. Các phản ứng quan trọng nhất là phản ứng cộng hợp được gây ra bởi axit hay bazơ vào liên kết đôi

cacbon–oxy và các phản ứng của Hα.

2. CẤU TRÚC CỦA LIÊN KẾT ĐÔI CACBON–OXY

Giống liên kết đôi cacbon–cacbon, nguyên tử cacbon lai hoá sp2. Các obital

2p trên cacbon và oxy tạo thành liên kết π, trên nguyên tử oxi còn lại hai cặp

electron không phân chia. Do sự phân cực trong liên kết đôi cacbonyl nên nguyên tử cacbon trở thành trung tâm electrophil gây ra các phản ứng cộng hợp thuận nghịch và không thuận nghịch. Cũng do nguyên tử cacbon dương điện trong nhóm cacbonyl nên nguyên tử Hα trở nên linh động nhờ hiệu ứng siêu liên hợp làm cho Hα bị tách ra dưới tác dụng của các tác nhân kiềm yếu gây ra các phản ứng ngưng tụ. Cả hai phản ứng này đều tạo ra các liên kết mới, đặc biệt là liên kết C–C, đó là nền tảng của tổng hợp hữu cơ, ngoài ra nó cũng tạo ra các liên kết C–N là các phương pháp tổng hợp amin và aminoaxit.

C O

H

H

L i ª n k Õ t π ® ­ î c t ¹ o r a t õ h a i o b i t a l 2 p x e n p h ñ

L i ª n k Õ t C - H ® ­ î c t ¹ o r a t õ s ù x e n p h ñ 1 s - s p 2

C ¸ c e l e c t r o n c Æ p ® ¬ n ® é c

L i ª n k Õ t σ

Hình 1. Cấu trúc obital của focmandehit.

225

Page 220: 1ygm3hwjpgu090312060334 tài liệu bồi dưỡng giáo viên trường thpt chuyên   mon hóa học - phan 1- năm 2012

C O

C O

O 2 p

C 2 p

π

π ∗

N¨n

g l­

îng

π

Hình 2. Giản đồ obital của hợp chất cacbonyl.

3. DANH PHÁP CỦA CÁC HỢP CHẤT CACBONYL

Hợp chất cacbonyl tổng quát có công thức R2C=O, các hợp chất đơn giản

nhất, trong đó cả hai R là các hydrocacbon, như ankyl, ankenyl, hay aryl, được gọi

là các xeton. Nếu một nhóm R là hydro, hợp chất là andehit, và phân tử trong đó

cả hai nhóm R là hydro thì được gọi là formandehit

C O

H

H

C O

H

R

C O

R

R

F o rm a n d e h i t A n d e h i t X e t o n

Hình 3. Các hợp chất cacbonyl.

Tên gọi thông thường được sử dụng cho các andehit chứa không quá 4 cacbon

ví dụ như andehit focmic, andehit axetic.

Gọi tên theo IUPAC: sử dụng hậu tố “al” vào cuối tên gọi của ankan. Trong

hệ thống ưu tiên, nhóm andehit luôn được đánh số 1, các số cao hơn được gán cho

các nhóm thế bất kì.

Khi các andehit gắn vào các vòng, thuật ngữ “cacbandehit” hay “cacboxandehit”

được sử dụng, như trong “xyclopropancacboxandehit”.

226

Page 221: 1ygm3hwjpgu090312060334 tài liệu bồi dưỡng giáo viên trường thpt chuyên   mon hóa học - phan 1- năm 2012

C O

H

CH3

C O

H

CH2CH3

C O

H

CH2CH3CH2

C O

H

CH2(CH3)2

C O

H

CH2CH3CH2CH2

C O

H

CH2CH2CH2CH2CH3

C O

H

CHCH3CH2CH2

CH3

C O

H

CH2CH2

Cl

COH

CH

A x e t a n d e h i t( e t a n a l )

P r o p io n a n d e h i t( p r o p a n a l )

B u t y ra n d e h i t( b u t a n a l )

Is o b u t y a n d e h i t( 2 - m e t y l p r o p a n a l )

P e n t a n a l

H e x a n a l 2 - M e t y lp e n t a n a l 3 - C lo p r o p a n a l X y c lo h e x a n c a c b o x a n d e h i t

Hình 4. Tên gọi một số hợp chất andehit.

Các hợp chất chứa hai nhóm andehit là các dial. Trong trường hợp này, hậu tố “dial” được thêm vào cuối tên gọi của ankan tương ứng.

O

C

O

CHH

O

CH C

H

OPropandial B u t a n d ia l

Hình 5. Tên gọi một số hợp chất diandehit.

Các xeton cũng có các tên gọi thông thường, nhưng phần lớn bị bỏ qua. Một ngoại lệ là tên gọi của hợp chất đơn giản nhất dimetyl xeton vẫn còn được gọi là axeton. Các xeton được gọi tên theo nhiều cách. Chẳng hạn, axeton cũng được gọi là 2–oxopropan, propanon, hay dimetyl xeton.

Danh pháp “oxo” nếu coi nguyên tử oxy là một nhóm thế, được nhận số thứ tự giống như một nhóm bất kì.

Nếu sử dụng hệ thống “on”, hậu tố “on” được viết thêm vào tên gọi của ankan.

CH3

O

CCH3 CH3

O

CCH2CH3 CH

O

CCH3

CH3

CH2CH3

A x e t o nd i m e t y l x e t o n

p r o p a n o n2 - o x o p r o p a n

E t y l m e t y l x e t o nb u t a n o n

2 - o x o b u t a n

3 - M e t y l - 2 - p e n t a n o ns e c - b u t y l m e t y l x e t o n3 - m e t y l - 2 - o x o p e n t a n

Hình 6. Tên gọi một số hợp chất xeton.

227

Page 222: 1ygm3hwjpgu090312060334 tài liệu bồi dưỡng giáo viên trường thpt chuyên   mon hóa học - phan 1- năm 2012

Các hợp chất chứa hai nhóm xeton là các dion. Giống như các dial, tên gọi của các dion được bắt đầu với các số chỉ vị trí của hai nhóm cacbonyl, sau đó là tên gọi của khung phân tử chính và hậu tố “dion”. Các xetoandehit được gọi tên như là các andehit, mà không là các xeton, vì nhóm andehit có độ ưu tiên cao hơn so với nhóm xeton.

CCH3

O

CH3C

O

C

C

O

O

CCH2

O

HC

O

CH2CH3 CCH3

HC

O

O

2 , 4 - P e n t a n d i o n 1 , 4 - X y c l o h e x a n d i o n 3 - O x o h e x a n a l 2 - O x o p r o p a n a l( a n d e h i t p y r u v i c )

Hình 7. Tên gọi một số hợp chất dion.

Tên gọi thông thường được sử dụng cho các andehit và xeton thơm. Chẳng hạn, andehit thơm đơn giản nhất không gọi là “benzencacboxandehit” mà là benzandehit.

COH

H

H

H

O

C

CH3

OHC

OCH3

OH

B e n z a n d e h i t 4 - M e t o x y b e n z a n d e h i t( a n i s a n d e h i t )

4 - M e t y l b e n z a n d e h i t( p - t o l u a n d e h i t )

t r a n s - X i n a m a n d e h i t

Hình 8. Tên gọi một số hợp chất xeton.

Các xeton thơm thường được gọi tên bằng cách xem vòng benzen như là nhóm phenyl, như trong isopropyl phenyl xeton. Một số ít có thể được gọi tên thông qua một hệ thống, trong đó khung Ph–C=O được chỉ ra bằng thuật ngữ ophenon và phần còn lại của phân tử được chỉ ra bằng một trong các tên gọi luân phiên trong. Do đó, diphenyl xeton cũng là benzophenon, metyl phenyl xeton là axetophenon, etyl phenyl xeton là propiophenon, và phenyl propyl xeton là butyrophenon.

228

Page 223: 1ygm3hwjpgu090312060334 tài liệu bồi dưỡng giáo viên trường thpt chuyên   mon hóa học - phan 1- năm 2012

C

O

C

O

CH3

CH3

C

O

C

O

CH3

X y c lo p r o p y l p h e n y l x e t o n Is o p r o p y l p h e n y l x e t o n D ip h e n y l x e t o n( b e n z o p h e n o n )

M e t y l p h e n y l x e t o n( a x e t o p h e n o n )

Hình 9. Tên gọi một số hợp chất xeton thơm.

4. TÍNH CHẤT VẬT LÍ CỦA CÁC HỢP CHẤT CACBONYL

Tất cả các hợp chất cacbonyl đều có cực, các phân tử nhỏ đều tan trong nước. Chẳng hạn, axeton và axetandehit có thể trộn lẫn với nước. Nhiệt độ sôi của chúng cao hơn đáng kể so với điểm sôi của các hydrocacbon tương ứng. Chẳng hạn,

propan và propen là các chất khí với điểm sôi tương ứng là –42,1 và –47,4°C, còn

axetandehit sôi ở +20,8°C, cao hơn khoảng 63°C. Axeton sôi cao hơn 60°C so với

isobuten.

5. PHẢN ỨNG CỘNG HỢP THUẬN NGHỊCH

Trong các phản ứng cộng hợp thuận nghịch đơn giản, các hợp chất cacbonyl có cả đặc tính axit Lewis và bazơ Lewis.

5.1. Cộng hợp nước

Nước hoạt động như là một nucleophil và cộng hợp vào nhóm cacbonyl có tính axit Lewis.

CO

+

+

H O H

H O H

O HH

O O HH

A n k e n A n c o l

C a c b o n y l H y d r a t

Hình 10. Phản ứng cộng hợp nước vào anken và xeton

229

Page 224: 1ygm3hwjpgu090312060334 tài liệu bồi dưỡng giáo viên trường thpt chuyên   mon hóa học - phan 1- năm 2012

C O

σ

σ ∗

C O

C O

n

π ∗

C O

H2O

O

H

H

O b i t a l π * t r è n g

L i ª n k Õ t O - C m í i N

¨ng

l­în

g

Hình 11. Giản đồ obital của quá trình cộng hợp H2O vào hợp chất cacbonyl.

5.2. Cộng hợp xianua và bisulfit

Cộng hợp xianua: Ion xyanua là một nucleophil tốt, nó tấn công hợp chất cacbonyl để tạo ra một ankoxit. Ankoxit này bị proton hoá bởi dung môi, thường là nước, để cho sản phẩm cuối là xyanohydrin.

C OK

NCOH

OCNC

K

O

H

H

OCNC HK

X y a n o h y d r i n

Hình 12. Cộng hợp xianua vào hợp chất cacbonyl.

Cộng hợp bisulfit: nguyên tử lưu huỳnh trong bisulfit đóng là tác nhân nucleophil tấn công vào nguyên tử cacbon trong cacbonyl để tạo liên kết C–S, tuy nhiên đây là phản ứng thuận nghịch.

T R ¦ ê N G H î P C H U N G

V Ý D ô R I £ N G

R

CO

H

OS

O

OH

SC

R

H

O O

O

OH

Na

Na

SC

R

H

O O

O

O

H

Na

C

O

N a H S O 3

H 2 O

C

OH SO3Na

( 9 8 % )

Hình 13. Sự cộng hợp bisunfit.

230

Page 225: 1ygm3hwjpgu090312060334 tài liệu bồi dưỡng giáo viên trường thpt chuyên   mon hóa học - phan 1- năm 2012

5.3. Phản ứng cộng hợp tạo thành axetal và ketal

Các ancol hoạt động như là một nucleophil và cộng hợp vào nhóm cacbonyl có tính axit Lewis. Nếu sử dụng elilenglycol thì đây là phương pháp bảo vệ nhóm chức cacbonyl.

ROH

H

C O

R

+ OHCRO

R

H

ROH

R

C O

R

+ OHCRO

R

R

A n d e h i t X e t o nH e m i a x e t a l( t h ­ ê n g ® ­ î c ­ u t i ª n ë c © n b » n g )

H e m i a x e t a l( t h ­ ê n g k h « n g ® ­ î c

­ u t i ª n ë c © n b » n g )

Hình 14. Sự cộng hợp ancol.

5.4. Phản ứng cộng hợp của bazơ nitơ: sự tạo thành imin và enamin

Nguyên tử nitơ của một amin (RNH2) là một nucleophil có thể cộng hợp vào các hợp chất cacbonyl. Phản ứng được xúc tác bằng axit giữa một hợp chất cacbonyl và một amin để cho một cacbinolamin hoàn toàn tương tự với các phản ứng của nước và các ancol.

Trong số các phản ứng đó, nếu khử hóa nhóm imin sẽ thu được amin, vì vậy đây là phương pháp tạo liên kêt C–N có hiệu quả bằng phản ứng ngưng tụ được sử dụng để tổng hợp amin.

Các phản ứng cụ thể được chỉ ra ở hình 15.

231

Page 226: 1ygm3hwjpgu090312060334 tài liệu bồi dưỡng giáo viên trường thpt chuyên   mon hóa học - phan 1- năm 2012

T R ¦ ê N G H î P C H U N G

V Ý D ô R I £ N G

C O + R NH2 C N

R

I m i n

C O + R NH2

C NH

C N

Ph

P h e n y l i m i n

C N

NH2

H y d r a z o n

C N

NHPh

P h e n y l h y d r a z o n

C N

NH NO2

O2N

2 , 4 - D i n i t r o p h e n y l h y d r a z o n

NO2

NH

O2N

NH2

O

CNH2 NH NH2

OH NH2

C N

NH C

O

NH2

C N

OH

S e m i c a c b a z o n

O x i m

NH2 NH2

NH NH2Ph

H NH2

Ph NH2

Hình 15. Sự cộng hợp các bazơ nitơ vào hợp chất cacbonyl.

232

Page 227: 1ygm3hwjpgu090312060334 tài liệu bồi dưỡng giáo viên trường thpt chuyên   mon hóa học - phan 1- năm 2012

6. CỘNG HỢP KHÔNG THUẬN NGHỊCH

6.1. Tổng hợp ancol

Phản ứng với các hợp chất cơ kim: các tác nhân cơ kim có khả năng tạo thành các liên kết cacbon–cacbon do phản ứng cộng hợp vào các hợp chất cacbonyl.

Các phản ứng cụ thể được chỉ ra ở hình 16.

T R ¦ ê N G H î P C H U N G

R L i +R

C O

R

OCR

R

R

LiHOH

OHCR

R

R

( 1 6 . 3 )( h a y R M g X )

R L i +R

C O

R

OCR

R

R

LiHOH

OHCR

R

R

( 1 6 . 4 )

R L i +R

C O

R'

OCR

R'

R

LiHOH

OHCR

R'

R

( 1 6 . 5 )

R L i +R"

C O

R'

OCR

R'

R"

LiHOH

OHCR

R'

R"

( 1 6 . 6 )

V Ý D ô R I £ N G

C

O

1 . C H 3 M g B r

e t e2 . 0 o C , H 2 O

C

OH

CH3

( 9 5 % )

Hình 16. Sự cộng hợp các hợp chất cơ kim vào hợp chất cacbonyl.

Các phản ứng khử hóa hợp chất cacbonyl

233

Page 228: 1ygm3hwjpgu090312060334 tài liệu bồi dưỡng giáo viên trường thpt chuyên   mon hóa học - phan 1- năm 2012

T R ¦ ê N G H î P C H U N G

V Ý D ô R I £ N G

+H

C O

H

OCH

H

H

LiHOH

OHCH

H

H

M e t y l a n c o l

H

C O

R

+ OCH

R

H

HOH

OHCH

R

H

H

C O

R

+ OCR

R

H

HOH

OHCR

R

H

Li

C ¸ c a n c o l b Ë c h a i

( 9 7 % )

LiAlH4

(hay NaBH4)

F o r m a n d e h i t

LiAlH4 Li

C ¸ c a n c o l b Ë c m é t

C

H

C

C

H

H

O

1 . N a B H 4

C H 3 O H , 2 0 - 3 0 o C 2 . H 2 O

C

H

C

CH2

H

OH

LiAlH4

Hình 17. Khử hóa các hợp chất cacbonyl.

C O C

H

HZ n / H g

H C lC O C

H

HN H 2 N H 2

K O H , ∆H O C H 2 C H 2 O H

S ù k h ö h o ¸ C l e m m e n s e n S ù k h ö h o ¸ W o l f f - K i s h n e r

Hình 18. Khử hóa các hợp chất cacbonyl theo Clemensen và Wolff–Kishner.

234

Page 229: 1ygm3hwjpgu090312060334 tài liệu bồi dưỡng giáo viên trường thpt chuyên   mon hóa học - phan 1- năm 2012

H

C O

R

OHCH2R

s ù o x i h o ¸

s ù k h ö h o ¸

R

C O

R

OHCH

R

Rs ù o x i h o ¸

s ù k h ö h o ¸

C ¸ c ® i Ò u k i Ö n k h ö h o ¸ : 1 . L i A l H 4 ( h a y N a B H 4 ) 2 . H 2 O

Hình 19. Quan hệ giữa khử hóa các hợp chất cacbonyl và oxi hóa ancol.

6.2. Phản ứng wittig

Quá trình bắt đầu với phản ứng giữa photphin và ankyl halogenua để cho photphoni halogenua qua phản ứng thế SN2

(Ph)3P CH3 IS N 2

CH3(Ph)3P + IT r ip h e n y l p h o t p h in I o n p h o t p h o n i

Cacbon trong ylit là một nucleophil cộng hợp vào liên kết đôi trong cacbonyl tạo ra vòng oxaphotphetan. Các oxaphotphetan không phân hủy thành photphin oxit và anken.

CH2(Ph)3P

CO

R

R

R

CH2(Ph)3P

CO R

R

CH2P

CO R

(Ph)3

O(Ph)3P C

CH2

RR+

O x a p h o t p h e t a nT r i p h e n y l p h o t p h i n

o x i tS ¶ n p h È m

a n k e n

235

Page 230: 1ygm3hwjpgu090312060334 tài liệu bồi dưỡng giáo viên trường thpt chuyên   mon hóa học - phan 1- năm 2012

s ù c h u y Ó n h o ¸ t ­ ¬ n g h ç C H U N G

V Ý D ô R I£ N G

H

O

H

CH2

( P h ) 3 P = C H 2

1 2 h , 6 5 o Ce t e ( 6 7 % )

O CCOOC2H5H

( 4 3 % )

( P h ) 3 P = C H C O O C 2 H 5

1 0 h , 1 7 0 o C

C O

R

R

W i t t i g

O z o n p h © nC CH2

R

R

Hình 20. Phản ứng Wittig và ozon phân

7. PHẢN ỨNG CỦA Hα

7.1. Sự tạo thành anion enolat

CCH

O

Ha

Ha

B

C a c b o n α

CCH

O

HaCCH

O

Ha + HaB

E n o l ® ­ î c b Ò n h o ¸ b » n g c é n g h ­ ë n g

Hình 21. Quá trình tách hydro α tạo thành anion enolat

236

Page 231: 1ygm3hwjpgu090312060334 tài liệu bồi dưỡng giáo viên trường thpt chuyên   mon hóa học - phan 1- năm 2012

7.2. Phản ứng thế ở Hα

O

HCH3

CH2CH3

*

O

DCH3

CH2CH3

O

BrCH3

CH2CH3

O

HCH3

CH2CH3

D 2 O / N a O D

d i o x a n3 5 o C

B r 2

N a O H / H 2 O

H 2 O / N a O H

3 5 o C

* N g h Üa l µ h o ¹ t ® é n g q u a n g h ä c

AS ù t r a o ® æ i d e u t e r i

Bα - B r o m h o ¸

CR a x e m i c h o ¸

Hình 22. Các phản ứng thế hydro α

Sự raxemic hoá. Việc xử lí một andehit hay xeton hoạt động quang học với axit hay bazơ dẫn tạo thành hỗn hợp raxemic.

CC

CH3

CH2

O

HCH3

C H 3 C O O H

H N O 3

3 7 o C

CC

CH3

CH2

O

HCH3

CC CH3

CH2

O

H

CH3

( S ) ( S ) ( R )

+

Hình 23. Sự raxemic hoá ở hydro α

7.3. Phản ứng ankyl hoá

Các anion enolat hoạt động như các nucleophil trong các phản ứng thế SN2 để

ankyl hoá vị trí α của các hợp chất cacbonyl tạo ra liên kết cacbon–cacbon.

237

Page 232: 1ygm3hwjpgu090312060334 tài liệu bồi dưỡng giáo viên trường thpt chuyên   mon hóa học - phan 1- năm 2012

CH2

CR

O

CH2

CR

O CH3 I

E n o l a t

S N 2 CH2

CR

O

CH3 + I

E n o l a t ® ­ î c m e t y l h o ¸ , s ¶ n p h È m x e t o n

Hình 24. Cơ chế phản ứng alkyl hoá ở hydro α

Ví dụ: quá trình metyl hóa 2–metylxiclohexanon tạo thành 4 sản phẩm ankyl hóa

C

O

CH3 1 . ( P h ) 3 C K

2 . C H 3 I( 9 % ) ( 4 1 % ) ( 6 % ) ( 2 1 % )

C

O

CH3 CH3 C

O

CH3

CH3

C

O

CH3

CH3

CH3

CH3

C

O

CH3

CH3

CH3

+ + +

7.4. Các phản ứng ngưng tụ của các hợp chất cacbonyl

7.4.1. Ngưng tụ andol

Các phản ứng ngưng tụ andol được xúc tác bằng bazơ và axit tạo ra hợp chất

β–hydroxy cacbonyl, được gọi là andol.

CHCH3

CCH2H

O OH

αβ

H 3 O

H 2 OCH

CH3

CCHH

O OH2

HH 2 O

+H 3 O CCH3

CCH

O H

H

αβ

A n d e h i t α , β - k h ô n g n oA n d o l , m é t β - h y d r o x y a n d e h i t

Cơ chế phản ứng được chỉ ra ở hình 25.

H OH2

CH2

CH

OH

CH3

CH

OH OH2

CH2

CH

O

H + H3O

S ù p r o t o n h o ¸ / d e p r o t o n h o ¸ c ñ a e n o l ® Ó t ¸ i s i n h a x e t a n d e h i t

P h ¶ n ø n g c ñ a e n o l v í i c a c b o n y l b Þ p r o t o n h o ¸ ® Ó c h o a n d o l

CH2

CH

OH CH3

CH

OH

CCH3

CCH2H

O OH

H

H

OH2

CCH3

CCH2H

O OH

H

+ H3O

A n d o l

238

Page 233: 1ygm3hwjpgu090312060334 tài liệu bồi dưỡng giáo viên trường thpt chuyên   mon hóa học - phan 1- năm 2012

Hình 25. Cơ chế phản ứng andol

7.4.2. Ngưng tụ Andol nội phân tử

T R ¦ ê N G H î P C H U N G § ¦ î C X ó C T ¸ C B » N G A X I T

C O

R

C CH3

O

BB H +

C O

R

C CH2

O( )

C

C

CH2

R O

O

B H

C

C

CH2

R OH

O

BC

C

CH

R OH

O

B H +C

C

CH

R

O

+OH

C ¸ C V Ý D ô § ¦ î C X ó C T ¸ C B » N G B A Z ¥ V µ A X I T R I £ N G

CC

O

CH3

CH3

O

K O H

H 2 O

C

C

O

O

H 2 S O 4

H 2 O

CH3

CCH3

O

C

O

( 9 6 % )

( 8 3 % )

Hình 26. Ngưng tụ andol nội phân tử.

7.4.3. Ngưng tụ Andol chéo

CH3C

O

NLi

− 7 8 o C

CH2C

O ( )

HC

O

C

O

C

O

OH2

C

O

C

OH

( 6 5 % )

239

Page 234: 1ygm3hwjpgu090312060334 tài liệu bồi dưỡng giáo viên trường thpt chuyên   mon hóa học - phan 1- năm 2012

Hình 27. Phản ứng andol chéo

7.4.4. Ngưng tụ Knoevenagel

Các phản ứng ngưng tụ không bị giới hạn cho các andehit và xeton. Hợp chất bất kì, có thể cho một anion, thì có thể tấn công nhóm cacbonyl để cho sản phẩm.

T R ¦ ê N G H î P C H U N G

V Ý D ô R I £ N G

CH2

R

R'

B

B H

+CH

R

R'

R" R'"C

O

CH

R

R'

C

R"

O

R'"

CH

R

R'

C R'"

R"

OH

B H

+ BB

C

R

R'

C

R"

OH

R'"C

R

R'

C

R"

R'"

+HO

( R , R ' l µ c ¸ c n h ã m l µ m b Ò n h o ¸ a n i o n )

CH2

C

C

CH2O

O

CH3

CH2OCH3

O

CH

O

b e n z e n , p y r o l i d i nC

C

C

CH2O

O

CH3

CH2OCH3

O

C

H

( 8 9 % )

C

H

O

(CH3)2N + CH3NO2

a m y l a m i n

1 0 0 o C , 1 p h ó t

2 5 o C , 2 4 h

C

H

CHNO2

(CH3)2N

( 8 3 % )

Hình 28. Phản ứng ngưng tụ Knoevenagel

Nhiều phản ứng khác, một vài phản ứng có tên gọi, nhiều phản ứng không có tên gọi, trong đó chúng bao gồm sự cộng hợp của một anion phức vào nhóm cacbonyl. Nguồn anion nucleophil không phải luôn là andehit hoặc xeton. Chẳng hạn, xyclopentadien (pKa = 15) là một axit đủ mạnh để cho một anion khi được xử lí với bazơ, và ion xyclopentadienyl có thể cộng hợp vào các andehit hoặc xeton. Kết quả cuối cùng tạo thành chất có màu là các funven.

240

Page 235: 1ygm3hwjpgu090312060334 tài liệu bồi dưỡng giáo viên trường thpt chuyên   mon hóa học - phan 1- năm 2012

H

H

CH2OH CH3

CH2O CH3

H+

CH3 CH3

C

O

H

COCH3

CH3

CH2O CH3H

H

COCH3

CH3

H

CH2O CH3

C OH

CH3

CH3

CH2O CH3H+CH3

CH3

+ OH

6 , 6 - D i m e t y l f u n v e n( 7 5 % )

Hình 29. Phản ứng tạo thành funven

7.4.5. Phản ứng Michael

Phản ứng hoá học đặc biệt quan trọng liên quan đến hợp chất cacbonyl α,β–

không no là phản ứng Michael.

Các liên kết đôi cacbon–cacbon bình thường không có khả năng phản ứng với các nucleophil. Ngược lại, nhiều nucleophil cộng hợp dễ dàng vào các liên kết đôi

của các hợp chất cacbonyl α,β–không no theo phản ứng Michael.

C

R

R

C

R

CR

O

Nu

O

Nu C C

R

R

R

CR

O

Nu C C

R

R

R

CR

H î p c h Ê t c a c b o n y l α , β - k h « n g n o

E n o l a t( ® ­ î c b Ò n h o ¸ b » n g c é n g h ­ ë n g )

Hình 30. Cơ chế cộng hợp Michael vào các hợp chất cacbonyl α,β–không no

241

Page 236: 1ygm3hwjpgu090312060334 tài liệu bồi dưỡng giáo viên trường thpt chuyên   mon hóa học - phan 1- năm 2012

C H

H

O

1 . L i A l H 4 , e t e

2 . H 2 O

C

H

O

H

C H

H

OH H

+

CH3

CH3

O 1 . C H 3 L i

2 . H 2 O

CH3

CH3

OH

CH3

( 8 1 % )K h « n g c ã s ù c é n g

h î p v µ o p h Ç n a n k e n

( 9 4 % ) ( 2 % )

Hình 31. Cộng hợp Michael vào các hợp chất cacbonyl α,β–không no

7.4.6. Sự kết vòng Robinson

Một trong các trình tự thường gặp là một tổ hợp của các ngưng tụ Michael và andol nội phân tử

T R ¦ ê N G H î P C H U N G

CO

b a z ¬

O HC

OC

CH3 O

+ OH2

OC

OC

CH3

M i c h a e l

L ­ u ý s ù t ¹ o t h µ n h e n o l a t

OC

OC

CH3

OH2

+ O H

S ù t ¹ o t h µ n h

e n o l a t m í i

OC

OC

H2CC

OO

a n d o l

OH2

C

OHO

O H

s ù t ¹ o t h µ n h e n o l a t m é t

l Ç n n ÷ a

C

OHO

d e h y d r a t h o ¸

CO

+ O H

( t i Õ p t ô c )

p r o t o n h o ¸

Hình 32. Cơ chế phản ứng Robinson

242

Page 237: 1ygm3hwjpgu090312060334 tài liệu bồi dưỡng giáo viên trường thpt chuyên   mon hóa học - phan 1- năm 2012

7.4.7. Phản ứng mannic H

Andehit hay xeton được đun nóng với một chất xúc tác axit khi có mặt của formandehit và một amin. Sau đó, ion amoni được tạo thành ban đầu được xử lí với bazơ để giải phóng sản phẩm cuối cùng là amin tự do.

T R ¦ ê N G H î P C H U N G

RC

CH2

O

R

H C l / E t O H

H 2 C = O , ( C H 3 ) 2 N H RC

CH

O

NCH2

(CH3)2

R H

C l N a O H

H 2 O RC

CH

O

NCH2

(CH3)2

R + +ClH NaCl

( t i Õ p t ô c )

HC

H

O( C H 3 ) 2 N H / H C l

HC

H

N(CH3)2

CH3

CCH3

O

CH2

CCH3

OH

E n o l

I o n i m o n i

CH3

CCH2

OH

CH2

N(CH3)2 ClH

CH3

CCH2

O

CH2

N(CH3)2 H

H 2 ON a O H

C l

++ClH NaClCH3

CCH2

O

CH2

N(CH3)2

Hình 33. Cơ chế phản ứng Mannich.

7.5. Các phản ưng khác

7.5.1. Phản ứng Cannizzaro

Phản ứng của các hợp chất cacbonyl không có Hα nên không bị enol hoá. Đó là phản ứng tự oxi hoá–khử xảy ra do sự chuyên ion hydrua. Ví dụ benzandehit bị oxi hoá thành benzoat bị khử hoá đến benzyl ancol.

C

O

H

B e n z a n d e h i t

K O H

H 2 O

C

O

O K+

CH2OH

K a l i b e n z o a t( b Þ o x i h o ¸ )

B e n z y l a n c o l( b Þ k h ö h o ¸ )

243

Page 238: 1ygm3hwjpgu090312060334 tài liệu bồi dưỡng giáo viên trường thpt chuyên   mon hóa học - phan 1- năm 2012

7.5.2. Cân bằng Meerwein–Ponndorf–Verley–Oppenauer.

Trong phản ứng Meerwwein–Ponndorf–Verley–Oppenauer (MPVO), nguyên tử nhôm được sử dụng để giữ hai nửa của phản ứng Cannizzzaro với nhau. Nhôm ankoxit, điển hình là nhôm isopropoxit, được sử dụng như là nguồn hydrua để khử hoá hợp chất cacbonyl thứ hai.

C

O

H

1 . A l [ O C H ( C H 3 ) 2 ] 3

H O C H ( C H 3 ) 2

2 . H 2 O

CH2

OH

( 8 9 % )

PHẦN BÀI TẬP

Bài 1: Gọi tên các chất sau

Bài 2: Viết cấu tạo của sản phẩm phản ứng của C6H5CH2CHO với các tác nhân sau:

244

Page 239: 1ygm3hwjpgu090312060334 tài liệu bồi dưỡng giáo viên trường thpt chuyên   mon hóa học - phan 1- năm 2012

Bài 3: Viết cấu tạo của sản phẩm của các phản ứng sau:

Bài 4: Viết cấu tạo của sản phẩm của các phản ứng sau

Bài 5: Viết cấu tạo của các chất A, B, C, D, E, F, G trong các chuyển hóa sau

Bài 6: Hoàn thành các phản ứng sau

245

Page 240: 1ygm3hwjpgu090312060334 tài liệu bồi dưỡng giáo viên trường thpt chuyên   mon hóa học - phan 1- năm 2012

Bài 7: Viết cấu tạo của các chất A, B, C, D, E, F trong các chuyển hóa sau:

Bài 8: Hoàn thành các phản ứng

Bài 9: Etoposide là hợp chất chữa ung thư phổi.

a) Xác định các trung tâm axetal

b) Sản phẩm tạo thành khi thủy phân axetal trong axit loãng

246

Page 241: 1ygm3hwjpgu090312060334 tài liệu bồi dưỡng giáo viên trường thpt chuyên   mon hóa học - phan 1- năm 2012

Bài 10: Điều chế các anken sau bằng phản ứng Wittig

Bài 11: Điều chế các chất sau từ Benzandehit

Bài 12: Điều chế 2–butanon từ các chất sau

Bài 13: Hoàn thành các phản ứng sau

Bài 14: Điều chế các chất sau từ xiclohexen

Bài 15: Điều chế các chất sau từ benzen và phenol

247

Page 242: 1ygm3hwjpgu090312060334 tài liệu bồi dưỡng giáo viên trường thpt chuyên   mon hóa học - phan 1- năm 2012

Bài 16: Điều chế thuốc trị hen xuyễn albuterol từ X

Từ chất đầu

Bài 17: Hoàn thành phản ứng (Kèm theo cơ chế)

Bài 18: Hoàn thành phản ứng (Kèm theo cơ chế)

248

Page 243: 1ygm3hwjpgu090312060334 tài liệu bồi dưỡng giáo viên trường thpt chuyên   mon hóa học - phan 1- năm 2012

Bài 19: Hoàn thành cơ chế phản ứng

Bài 19: Hoàn thành cơ chế phản ứng

Bài 20: Salsolinol là hợp chất có nhiều trong chuối, socola. Chất này được điều chề từ dopamin (chất dẫn truyền xung thần kinh) và axetaldehit. Hoàn thành cơ chế phản ứng

Bài 21: Hoàn thành cơ chế phản ứng

Bài 22: Hoàn thành các phản ứng và cho biết cấu trúc của brevicomin (Pheromon có trong bọ cánh cứng).

249

Page 244: 1ygm3hwjpgu090312060334 tài liệu bồi dưỡng giáo viên trường thpt chuyên   mon hóa học - phan 1- năm 2012

Bài 23: Hoàn thành các phản ứng của B và các tác nhân dưới đây

Bài 24: Tổng hợp các chất sau từ este hoặc clorua axit

Bài 25: Khử hóa các chất sau bằng LiAlH4

Bài 26: Tổng hợp các chất sau từ amit tương ứng bằng khử hóa với LiAlH4

Bài 27: Khử hóa các chất sau bằng LiAlH4 và NaBH4

Bài 28: Hoàn thành phản ứng

250

Page 245: 1ygm3hwjpgu090312060334 tài liệu bồi dưỡng giáo viên trường thpt chuyên   mon hóa học - phan 1- năm 2012

Bài 29: Hoàn thành phản ứng của CH3CH2CH2CH2CHO và CH3COCH2CH2CH3

với các tác nhân sau (Nếu có)

Bài 30: Hoàn thành phản ứng của CH3CH2CH2MgBr với các tác nhân sau

Bài 31: Hoàn thành phản ứng của CH3CH2CH2CH2)2CuLi với các chất sau (Nếu có)

Bài 32: Hoàn thành phản ứng của A với các tác nhân sau (Nếu có)

251

Page 246: 1ygm3hwjpgu090312060334 tài liệu bồi dưỡng giáo viên trường thpt chuyên   mon hóa học - phan 1- năm 2012

Bài 33: Hoàn thành các phản ứng sau

Bài 34: Hoàn thành các phản ứng sau

Bài 35: Hoàn thành các phản ứng sau

252

Page 247: 1ygm3hwjpgu090312060334 tài liệu bồi dưỡng giáo viên trường thpt chuyên   mon hóa học - phan 1- năm 2012

Bài 36: Giải thích sự chọn lọc lập thể của các phản ứng sau:

Bài 37: Tìm ra chỗ sai của phản ứng sau. Đề xuất phương pháp tổng hợp chất A

Bài 38: Hoàn thành các phản ứng sau

Bài 39: Hoàn thành các phản ứng sau

253

Page 248: 1ygm3hwjpgu090312060334 tài liệu bồi dưỡng giáo viên trường thpt chuyên   mon hóa học - phan 1- năm 2012

Bài 40: Cơ chế của các phản ứng sau

Bài 41: Cơ chế của phản ứng sau

Bài 42: Hoàn thành phản ứng sau

Bài 43: Điều chế các chất sau từ hợp chất andehit hoặc xeton

Bài 44: Điều chế các chất sau từ hợp chất andehit hoặc xeton

254

Page 249: 1ygm3hwjpgu090312060334 tài liệu bồi dưỡng giáo viên trường thpt chuyên   mon hóa học - phan 1- năm 2012

Bài 45: Điều chế các chất sau từ hợp chất andehit hoac xeton khác nhau

Bài 46: Điều chế các chất sau từ hợp chất epoxi

Bài 47: Điều chế các chất sau từ xiclohexanon

Bài 48: Điều chế các chất sau từ 2–propanol

Bài 49: Tổng hợp mestranol từ estradiol (Hormon nữ)

255

Page 250: 1ygm3hwjpgu090312060334 tài liệu bồi dưỡng giáo viên trường thpt chuyên   mon hóa học - phan 1- năm 2012

Bài 50: Tổng hợp các chất sau từ benzen

Bài 51: Tổng hợp các chất sau từ ancol có ít hơn 4 nguyên tử các bon

Bài 52: Tổng hợp salmeterol từ các hợp chất cho bên cạnh

Bài 53: Hoàn thành sơ đồ tổng hợp venlafaxine

Bài 54: Hoàn thành sơ đồ tổng hợp Juvenile

256

Page 251: 1ygm3hwjpgu090312060334 tài liệu bồi dưỡng giáo viên trường thpt chuyên   mon hóa học - phan 1- năm 2012

Bài 55: Hoàn thành sơ đồ tổng hợp prozac

Bài 56: Sản phẩm lập thể của các phản ứng sau

Bài 57: Tổng hợp các chất sau từ các hợp chất Cơ magie

Bài 58: Sản phẩm lập thể của các phản ứng sau

257

Page 252: 1ygm3hwjpgu090312060334 tài liệu bồi dưỡng giáo viên trường thpt chuyên   mon hóa học - phan 1- năm 2012

Bài 59: Hoàn thành phản ứng hai giai đoạn đầu trong quá trình tổng hợp hoormon Periplanone (Hoormon in female cockroach)

Bài 60: Hoàn thành phản ứng tổng hợp chất trung gian X bằng phản ứng ngưng tụ giữa hai phân tử và hoàn thành phản ứng tổng hợp Donepezil từ X.

Bài 61: Hoàn thành phản ứng tổng hợp Progesterone

258

Page 253: 1ygm3hwjpgu090312060334 tài liệu bồi dưỡng giáo viên trường thpt chuyên   mon hóa học - phan 1- năm 2012

Bài 62: Hoàn thành phản ứng

Bài 63: Hoàn thành phản ứng tổng hợp Avobenzen là chất dùng trong kem chống nắng qua phản ứng ngưng tụ

Bài 64: Hoàn thành phản ứng tổng hợp thuốc kháng viêm ibuprofen

Bài 65: Hoàn thành các phản ứng sau

259

Page 254: 1ygm3hwjpgu090312060334 tài liệu bồi dưỡng giáo viên trường thpt chuyên   mon hóa học - phan 1- năm 2012

Bài 66: Hoàn thành các phản ứng

Bài 67: Hoàn thành phản ứng tổng hợp chất X

Bài 68: Điều chế các chất đầu của quá trình bằng phản ứng cộng Michael và đề xuất cơ chế giai đoạn đề CO2

Bài 69: Hoàn thành phản ứng sau (NaOEt/EtOH làm xúc tác và dung môi)

260

Page 255: 1ygm3hwjpgu090312060334 tài liệu bồi dưỡng giáo viên trường thpt chuyên   mon hóa học - phan 1- năm 2012

Bài 70: Hoàn thành phản ứng cộng Michael

Bài 71: Điều chế các chất sau bằng phản ứng cộng Michael

Bài 72: Hoàn thành phản ứng Robinson

Bài 73: Hoàn thành phản ứng Robinson

261

Page 256: 1ygm3hwjpgu090312060334 tài liệu bồi dưỡng giáo viên trường thpt chuyên   mon hóa học - phan 1- năm 2012

Bài 74: Hoàn thành phản ứng Robinson

Bài 75: Sử dụng phương pháp phân cắt dưới đây để hoàn thành các phản ứng Robinson

Bài 76: Hoàn thành các phản ứng (Kèm theo cơ chế)

Bài 77: Điều chế các chất sau bằng phản ứng ngưng tụ

262

Page 257: 1ygm3hwjpgu090312060334 tài liệu bồi dưỡng giáo viên trường thpt chuyên   mon hóa học - phan 1- năm 2012

Bài 78: Hoàn thành phản ứng tổng hợp thuốc kháng viêm Nabumetone

Bài 79: Điều chế các chất sau bằng phản ứng ngưng tụ nội phân tử

Bài 80: Hoàn thành các phản ứng sau

Bài 81: Điều chế các chất sau bằng phản ứng ngưng tụ

263

Page 258: 1ygm3hwjpgu090312060334 tài liệu bồi dưỡng giáo viên trường thpt chuyên   mon hóa học - phan 1- năm 2012

Bài 82: Hoàn thành các phản ứng của B nếu cho B phản ứng với NaOMe/MeOH và giải thích sự tạo thành một sản phẩm duy nhất

Bài 83: Hoàn thành phản ứng sau (Sử dụng xúc tác bazơ yếu)

Bài 84: Điều chế các chất sau bằng phản ứng cộng Michael

264

Page 259: 1ygm3hwjpgu090312060334 tài liệu bồi dưỡng giáo viên trường thpt chuyên   mon hóa học - phan 1- năm 2012

Bài 85: Hoàn thành phản ứng tổng hợp Vetivone

Bài 86: Viết các sản phẩm tạo thành

Bài 87: Hoàn thành phản ứng

Bài 88: Hoàn thành phản ứng

265

Page 260: 1ygm3hwjpgu090312060334 tài liệu bồi dưỡng giáo viên trường thpt chuyên   mon hóa học - phan 1- năm 2012

Bài 89: Hoàn thành phản ứng tổng hợp A và B trong chuỗi các phản ứng tổng hợp hoormon sinh trưởng thực vật gibberellic tìm thấy ở cây lúa von do vi khuẩn Fujikuroi tạo ra.

Bài 90: Hoàn thành phản ứng và giải thích sự tạo thành sản phẩm chính (sản phẩm thứ nhất)

Bài 91: Hoàn thành phản ứng

Bài 92: Cortison là hoormon thuộc tuyến thượng thận được Woodward tổng hợp năm 1951 (Nobel năm 1965). Hoàn thành phản ứng Robinson trong giai đoạn đầu.

Bài 93: Hoàn thành phản ứng tổng hợp polime polytulipalin thân thiện môi trường

266

Page 261: 1ygm3hwjpgu090312060334 tài liệu bồi dưỡng giáo viên trường thpt chuyên   mon hóa học - phan 1- năm 2012

Bài 94: Hoàn thành phản ứng

Bài 95: Hoàn thành phản ứng tổng hợp các chất sau từ C6H5COCH3

Bài 96: Hoàn thành phản ứng

Bài 97: Hoàn thành tổng hợp các chất qua một phản ứng của hợp chất cacbonyl

Bài 98: Hoàn thành tổng hợp Octinoxate (Sunscreens) từ phenol và ancol có số nguyên tử cacbon bé hơn 5

267

Page 262: 1ygm3hwjpgu090312060334 tài liệu bồi dưỡng giáo viên trường thpt chuyên   mon hóa học - phan 1- năm 2012

Bài 99: Hoàn thành các phản ứng tổng hợp

Bài 100: Đề xuất cơ chế phản ứng

Bài 101: Đề xuất cơ chế phản ứng

Bài 102: Đề xuất cơ chế phản ứng

268

Page 263: 1ygm3hwjpgu090312060334 tài liệu bồi dưỡng giáo viên trường thpt chuyên   mon hóa học - phan 1- năm 2012

Ph n 5ÇTHùC nghiÖm HãA HäC chuyªn

5.1. BÀI THỰC HÀNH PHỤC VỤ TẬP HUẤN NĂM 2012

Bài 1: THỰC HÀNH HÓA VÔ CƠ – PHÂN TÍCHTỔNG HỢP VÀ PHÂN TÍCH PHỨC CHẤT SẮT (III) OXALAT

I. MỤC ĐÍCH CỦA THÍ NGHIỆM:

- Tổng hợp phức chất sắt(III) oxalat

- Xác định hàm lượng oxalat trong phức chất sản phẩm

II. CƠ SỞ LÝ THUYẾT:

Sắt là một trong những kim loại có vai trò quan trọng trong đời sống và

trong công nghiệp. Phức chất của sắt với phối tử hữu cơ có nhiều ứng dụng trong

hóa dược, hóa sinh và quang hóa. Trong hầu hết các phức chất nguyên tử trung

tâm Fe có số phối trí là 6. Phức chất Fe(III) oxalat là tinh thể đơn tà màu lục và là

một chất nhạy sáng. Dưới tác dụng của ánh sáng tinh thể phức Fe(III) oxalat sẽ

chuyển từ màu xanh sang màu vàng cam. Chất này thường được dùng trong các

thiết bị quang hóa hoặc lợi dụng tính chất quang hóa của nó để in, scan các bản

vẽ.

Trong bài thí nghiệm này phức chất Fe(III) oxalat được tổng hợp qua 2

giai đoạn và đi từ chất đầu là muối Fe(II). Giai đoạn đầu cho muối Fe(II) tác dụng

với gốc oxalat, sau đó tác dụng với chất oxi hóa, trong điều kiện có dư oxalat. Sản

phẩm thu được có thể là một trong 3 phức chất sau:

269

Page 264: 1ygm3hwjpgu090312060334 tài liệu bồi dưỡng giáo viên trường thpt chuyên   mon hóa học - phan 1- năm 2012

Số lượng phối tử oxalat trong phức chất Fe(III) oxalat tổng hợp được sẽ được xác định bằng phép chuẩn độ với dung dịch KMnO4.

III. DỤNG CỤ, HÓA CHẤT:

1- Dụng cụ: (dùng cho một nhóm)

STT Tên dụng cụ Số lượng Ghi chú

1 Eclen loại 125ml 2 chiếc

2 Cốc thủy tinh (hoặc eclen) chịu nhiệt 50ml

2 chiếc

3 Cốc thủy tinh 250ml 1 chiếc

4 Cốc thủy tinh thường (100ml) 3 chiếc

5 Pipét 10ml 1 chiếc

6 Quả bóp cao su 1 chiếc

7 Đũa thủy tinh 1 chiếc

8 Ống đong 10ml 1 chiếc

9 Phễu thủy tinh 1 chiếc (cuống phễu vừa Buret)

10 Phễu lọc thường 1 chiếc

11 Buret 1 chiếc

11 Bếp điện 1 chiếc

12 Máy khuấy từ (có gia nhiệt) 1 chiếc

14 Nhiệt kế 1 chiếc

15 Bộ lọc hút chân không

(phễu lọc, bình Busne, máy hút chân không....)

1 bộ

16 Giấy lọc 1 hộp

17 Lưới amiăng 1 chiếc

18 Kéo loại nhỏ 1 chiếc để cắt giấy lọc

270

Page 265: 1ygm3hwjpgu090312060334 tài liệu bồi dưỡng giáo viên trường thpt chuyên   mon hóa học - phan 1- năm 2012

19 Cân phân tích 1 chiếc

20 Lọ thủy tinh nhỏ 1 chiếc để đựng tinh thể phức

Ghi chú:

– Một số dụng cụ học viên có thể dùng chung nhau như: bộ lọc hút chân không, cân phân tích.

– Nước đá để làm lạnh.

2. Hóa chất:

1 – Chất rắn

– Muối Mohr (NH4)2(SO4). FeSO4.6H2O

– H2C2O4.2H2O hoặc Na2C2O4

2 – Dung dịch

– H2SO4 6M

– K2C2O4 2M

– H2O2 6%

– KMnO4 ≈ 0,02M (do phòng thí nghiệm pha, bí mật nồng độ chính xác)

– C2H5OH (cồn tuyệt đối)

IV. CÁC BƯỚC TIẾN HÀNH THÍ NGHIỆM:

Phần A: Tổng hợp phức chất

Bước 1

1. Hòa tan khoảng 2,0 gam (NH4)2[Fe(H2O)2(SO4)2]·6H2O trong bình tam giác (hoặc cốc thủy tinh) 50 ml bằng 6 ml nước đã được cho thêm 6 giọt dung dịch H2SO4 6 M.

2. Vừa khuấy vừa cho từ từ 10 ml dung dịch axit oxalic 1,0 M vào bình tam giác rồi đun sôi dung dịch (khuấy liên tục trong quá trình đun nóng).

Lấy bình ra khỏi bếp điện và để cho chất rắn lắng xuống đáy bình.

3. Gạn tách chất rắn ra khỏi dung dịch: chú ý không làm xáo trộn chất rắn ở đáy bình. (Chuyển phần dung dịch vừa gạn ra vào một bình tam giác khác và dán

nhãn DUNG DỊCH THẢI). Để rửa chất rắn vừa thu được thêm khoảng 3÷4 ml

nước nóng vào bình tam giác (đun nước bằng bình tam giác trên bếp điện đến nhiệt độ khoảng 80oC), lắc hỗn hợp rồi để hỗn hợp cho lắng xuống và dùng pipet

271

Page 266: 1ygm3hwjpgu090312060334 tài liệu bồi dưỡng giáo viên trường thpt chuyên   mon hóa học - phan 1- năm 2012

để hút phần nước lọc ra khỏi bình (chú ý để không làm xáo trộn đến chất rắn ở đáy bình) và chuyển dung dịch vừa hút vào bình chứa dung dịch thải.

Rửa lại chất rắn thêm một lần nữa.

Bước 2

1. Cho thêm 4 ml dung dịch K2C2O4 2M vào chất rắn thu được (khi chưa sấy

khô).

2. Để bình tam giác trong bể điều nhiệt ở 40oC (cách thủy ở 40÷50oC), rót từ

từ 4 ml dung dịch H2O2 6% vào dung dịch (vừa cho vừa lắc bình tam giác).

3. Đặt bình tam giác lên bếp điện, thêm 3 ml dung dịch axit oxalic 1M và

đun sôi hỗn hợp thu được. Để hỗn hợp sôi trong khoảng 1 phút.

4. Lấy bình tam giác ra khỏi bếp điện và để nguội đến nhiệt độ phòng.

5. Tách phần nước lọc và chất rắn bằng cách lọc thông thường (thu phần

nước lọc vào trong cốc thủy tinh 100 ml sạch).

6. Làm lạnh phần nước lọc trong nước đá. Để kết tinh phức chất tạo thành từ

dung dịch, thêm 15 ml etanol vào nước lọc và khuấy nhẹ.

7. Tách lấy phần chất rắn thu được bằng lọc hút chân không.

8. Làm khô tinh thể thu được bằng không khí (có thể làm khô bằng cách ép

các tinh thể giữa 2 tờ giấy lọc).

9. Xác định khối lượng phức sắt (III) oxalat thu được.

10. Tính hiệu suất của quá trình tổng hợp.

11. Quan sát hình dạng tinh thể trên kính hiển vi.

Phần B. Phân tích thành phần phức sắt (III) oxalat

(Tham khảo, tùy thuộc thời gian cho phép)

Bước 1: Chuẩn hóa dung dịch KMnO4 0,02 M.

1. Cho chính xác 10 ml dung dịch KMnO4 0,02 M vào burret. Cho vào bình tam giác chính xác khoảng 0,02 gam natri oxalate vào bình. Thêm vào bình khoảng 20 ml nước cất và 5 ml dung dịch H2SO4 6 M. Làm nóng dung dịch trong

bể điều nhiệt (hoặc đun cách thủy) đến khoảng 80°.

272

Page 267: 1ygm3hwjpgu090312060334 tài liệu bồi dưỡng giáo viên trường thpt chuyên   mon hóa học - phan 1- năm 2012

2. Chuẩn độ lượng natri oxalate trong bình bằng dung dịch KMnO4 chuẩn cho đến khi dung dịch xuất hiện màu hồng nhạt (màu của KMnO4, bền trong khoảng 1 phút) thì dừng lại. Ghi thể tích dung dịch KMnO4 dùng trong thí nghiệm này và tính nồng độ mol/lít của dung dịch KMnO4.

Bước 2

1. Cho vào bình định mức 125 mL một lượng chính xác khoảng 0,02 gam phức sắt (III) oxalate thu được ở phần trên. Cho vào bình 20 ml nước cất và 5 ml dung dịch H2SO4 6 M. Làm nóng dung dịch trong bể điều nhiệt (hoặc đun cách

thủy) đến khoảng 80°.

2. Chuẩn độ dung dịch thu được (khi dung dịch đang nóng) bằng dung dịch KMnO4 đã biết trước nồng độ (tính được từ thí nghiệm trên) đến khi xuất hiện màu hồng nhạt (bền trong 30s) thì dừng lại. Ghi thể tích dung dịch KMnO4 đã sử dụng cho phép chuẩn độ.

Chú ý: Học viên xem kĩ bản báo cáo kết quả thí nghiệm, yêu cầu CBHD kí tên xác nhận kết quả cân, thể tích chuẩn độ.

V. MỘT SỐ LƯU Ý ĐỂ THÍ NGHIỆM THÀNH CÔNG:

Tuân thủ các bước tiến hành như đã nêu trong phần trên.

VI. PHÂN TÍCH KẾT QUẢ THÍ NGHIỆM VÀ BÁO CÁO:

Từng nhóm viết báo cáo theo các nội dung dưới đây và trả lời các câu hỏi sau:

Câu 1- Tính nồng độ của dung dịch KMnO4?

Câu 2- Tính hàm lượng oxalat trong phức chất Fe(III) oxalat tổng hợp được. Từ

kết quả này hãy cho biết công thức phân tử, tính hiệu suất của phản ứng tổng hợp

phức chất Fe(III) oxalat? Nêu các nguyên nhân có thể dẫn đến sai số?

273

Page 268: 1ygm3hwjpgu090312060334 tài liệu bồi dưỡng giáo viên trường thpt chuyên   mon hóa học - phan 1- năm 2012

BẢN BÁO CÁO KẾT QUẢ THÍ NGHIỆM

Thành viên nhóm:

Ngày, giờ làm thí nghiệm................................ ...Vị trí thí nghiệm:..............

CÁC SỐ LIỆU THỰC NGHIỆM VÀ KẾT QUẢ

1. Tổng hợp phức chất Fe (III) oxalat

Khối lượngmuối Mohr

Chữ kí xác nhậncủa CBHD

Khối lượngphức chất

Fe (III) oxalat

Chữ kí xác nhậncủa CBHD

Lần 1

Lần 2

2. Chuẩn độ, xác định nồng độ dung dịch KMnO4

Khối lượng

Na2C2O4 hoặc H2C2O4.2H2O

Chữ kí xác nhận

của CBHD

Thể tích

KMnO4 đã dùng

Chữ kí xác nhận

của CBHD

Lần 1

Lần 2

3. Phân tích phức chất Fe (III) oxalat

Khối lượngphức chất

Fe(III) oxalat

Chữ kí xác nhậncủa CBHD

Thể tíchKMnO4

Chữ kí xác nhậncủa CBHD

Lần 1

Lần 2

VII. CÂU HỎI KIỂM TRA VÀ MỞ RỘNG:

Câu 1 – Tại sao khi tổng hợp phức chất sắt (III) oxalat lại đi từ chất đầu là muối Mohr (NH4)2(SO4). FeSO4.6H2O mà không dùng các loại muối Fe(II) khác như FeCl2, FeSO4...?

274

Page 269: 1ygm3hwjpgu090312060334 tài liệu bồi dưỡng giáo viên trường thpt chuyên   mon hóa học - phan 1- năm 2012

Câu 2 – Cho biết vai trò của H2SO4 trong giai đoạn đầu tổng hợp phức chất sắt (III) oxalat và trong các phép chuẩn độ với KMnO4.

Câu 3 – Nêu cách pha chế, bảo quản dung dịch KMnO4 từ tinh thể KMnO4. Tại sao sau khi pha phải chuẩn độ để xác định lại nồng độ dung dịch KMnO4?

Câu 4 – Hãy giải thích tại sao trong phép chuẩn độ pemanganat người ta luôn để KMnO4 ở trên buret? Có thể làm ngược lại để KMnO4 ở trong bình Eclen được không? Tại sao?

Câu 5 – Phức chất Fe(III) oxalat có tính chất quang hóa và tính chất này được dùng để in, scan các bản vẽ. Người ta làm thí nghiệm như sau: Hòa tan khoảng 0,1g phức chất Fe(III) oxalat vào 5ml nước cất, thêm 5ml dung dịch K3[Fe(CN)6]. Quét đều hỗn hợp lên một tờ giấy trắng, sau đó đặt một tờ giấy can đã có sẵn hình vẽ lên trên và chiếu sáng bằng một bóng đèn có công suất lớn (200W). Nhúng tờ giấy trắng vào nước. Cho biết hiện tượng xảy ra và giải thích?

Câu 6 – Cho biết vai trò của ancol etylic khi cho vào dung dịch nước lọc ở giai đoạn cuối của phần 1 (tổng hợp phức chất). Có thể thay C2H5OH bằng chất khác được không? Giải thích?

Câu 7 – Tại sao trong phép chuẩn độ chỉ cần đun nóng dung dịch (H2C2O4 + H2SO4) lúc đầu, sau đó trong quá trình chuẩn độ lại không cần đun nóng? Đề xuất

cơ chế của phản ứng giữa 4MnO− và 2

2 4C O − ?

Câu 8 – Có hiện tượng gì xảy ra khi để lâu dung dịch muối Fe(II) và muối Fe(III) trong không khí. Nêu cách bảo quản hai dung dịch này ?

Câu 9 – Bằng tính toán hãy chứng minh rằng trong môi trường kiềm Fe(II) có tính khử mạnh hơn trong môi trường axit.

Cho biết: 3+ 2+e / e

o

F FE = 0,77V,

2e(OH)FT = 10–15; 3e(OH)FT = 10–37.

Ghi chú: Phần (II) và phần (III) học viên viết ra giấy và nộp cùng với bản báo cáo kết quả thí nghiệm.

275

Page 270: 1ygm3hwjpgu090312060334 tài liệu bồi dưỡng giáo viên trường thpt chuyên   mon hóa học - phan 1- năm 2012

Bài 2:

PHÂN TÍCH DUNG DỊCH HỖN HỢP MUỐI Cu(NO3)2 VÀ Ni(NO3)2

I. Nguyên tắc chung

– Định lượng riêng hàm lượng Cu(II) bằng phương pháp iot.

– Định lượng tổng hàm lượng Cu(II) và Ni(II) bằng phương pháp Complexon.

1. Định lượng Cu(II) bằng phương pháp iốt

* Nguyên tắc

Cho KI dư vào dung dịch Cu(II) và chuẩn độ lượng Iôt giải phóng ra bằng dung dịch chuẩn Na2S2O3 (Không có phản ứng oxi hóa khử của Ni(II) với I–).

* Phản ứng trước chuẩn độ

Phản ứng xảy ra giữa Cu2+ và I– có kèm theo sự tạo thành kết tủa CuI.

2Cu2+ + 2I− + 2e ⇌ 2CuI 0,85VE0

/CuICu2 =+

3I– ⇌ I3− + 2e 0,5355VE0

/3II --3

=

2Cu2+ + 5I– → 2CuI + I3– K=1010,66

* Phản ứng chuẩn độ

Khi nhỏ Na2S2O3 vào hỗn hợp dung dịch chuẩn độ trên sẽ xảy ra phản ứng:

I3− + 2e ⇌ 3I− 0,5355VE0

/3II --3

=

2S2O32− ⇌ S4O6

2− + 2e 0,08VE0

O3/2SOS 22

264

=−−

I3− + 2S2O3

2– ⇌ S4O62– + 3I– K=1015,39

* Trước điểm tương đương. Trong hỗn hợp có kết tủa CuI và (I2 + I–) có màu vàng rơm.

* Chọn chỉ thị: Hồ tinh bột.

* Tại điểm kết thúc chuẩn độ. Tại thời điểm cuối chuẩn độ, khi mà I2 đã gần

hết, lượng I– nhiều: hỗn hợp (I2+I–) có màu vàng rơm, thêm lượng nhỏ dung dịch hồ tinh vào hỗn hợp có màu xanh tím, thêm từng giọt Na2S2O3 xuống dung dịch (I2+Hồ tinh bột) dung dịch sẽ mất màu.

2. Định lượng hỗn hợp Cu2+ và Ni2+ bằng phương pháp Complexon

276

Page 271: 1ygm3hwjpgu090312060334 tài liệu bồi dưỡng giáo viên trường thpt chuyên   mon hóa học - phan 1- năm 2012

* Nguyên tắc

– Dùng phản ứng chuẩn độ bằng EDTA để xác định tổng hàm lượng Ni2+ và Cu2+.

– Điều kiện tiến hành: Chuẩn độ bằng EDTA, môi trường bazơ NH3 (pH >10), dùng chỉ thị là Murexit (trong môi trường NH3 chỉ thị tồn tại chủ yếu dạng H2In3– có màu tím).

* Phản ứng trước chuẩn độ

– Khi cho NH3 vào dung dịch Ni2+, Cu2+:

Cu2+ + 2NH3 + 2H2O → Cu(OH)2 + 2NH4+

Ni2+ + 2NH3 + 2H2O → Ni(OH)2 + 2NH4+

Cu(OH)2 + 4NH3 → Cu(NH3)42+

Ni(OH)2 + 4NH3 → Ni(NH3)42+

(do CNH3>> CCu2+ nên các phức khác không đáng kể).

– Khi cho chỉ thị Murexit vào:

Cu(NH3)42+ + H2In3– CuIn3– + 2NH4

+ + 2NH3

Ni(NH3)42+ + H2In3– NiIn3– + 2NH4

+ + 2NH3

Dung dịch trước chuẩn độ có màu vàng (của phức CuIn3– và NiIn3–)

* Phản ứng chuẩn độ

Cu(NH3)42+ + Y4– → CuY2– + 4NH3

Ni(NH3)42+ + Y4– → NiY2– + 4NH3

* Phản ứng kết thúc chuẩn độ

CuIn3– + Y4– → CuY2– + In5–

NiIn3– + Y4– → NiY2– + In5–

Tại điểm kết thúc chuẩn độ, dung dịch chuyển từ màu vàng sang màu tím của chỉ thị tự do.

277

Page 272: 1ygm3hwjpgu090312060334 tài liệu bồi dưỡng giáo viên trường thpt chuyên   mon hóa học - phan 1- năm 2012

II. Cách tiến hành

1. Định lượng Cu(II) bằng phương pháp iốt

– Tráng buret bằng chính Na2S2O3, sau đó cho đầy buret và hiệu chỉnh đến vạch 0.

– Dùng pipet hút chính xác 10,0 ml dung dịch cho vào bình chuẩn độ, thêm 3 ml axit axetic 4M và thêm tiếp 3 ml Na2CO3 0,1 M (Để tạo môi trường CO2 bao phủ trên bề mặt của dung dịch chuẩn độ ngăn oxy không khí chui vào dung dịch trong quá trình chuẩn độ) và 10 ml KI 6%, lắc kỹ. Đậy kín bằng mặt kính đồng hồ hoặc giấy, để yên trong bóng tối 5 phút. Mang hỗn hợp dung dịch đi chuẩn độ bằng Na2S2O3 cho đến màu vàng rơm thì thêm 4 giọt hồ tinh bột (xuất hiện màu xanh tím), chuẩn độ tiếp tục dung dịch cho đến mất màu. Thêm 2 ml KSCN 20% và chuẩn độ tiếp đến mất màu xanh thì dừng. Ghi lại thể tích Na2S2O3 tiêu thụ.

Tiến hành thêm 1, 2 lần nữa cho đến khi nào Vn+1 ≠ Vn ≤ 0,20 ml, rồi tính thể tích

trung bình cộng Na2S2O3 tiêu thụ.

– Làm thêm 2, 3 lần nữa, lấy giá trị trung bình.

2. Định lượng hỗn hợp Cu2+ và Ni2+ bằng phương pháp Complexon

– Tráng buret bằng chính EDTA, sau đó cho đầy buret và hiệu chỉnh đến vạch 0.

– Dùng pipet hút chính xác 10,0 ml dung dịch cho vào bình chuẩn độ, thêm dung dịch NH3 vào bình chuẩn độ cho đến khi toàn bộ Ni2+ và Cu2+ chuyển sang phức. Thêm 1 ít chỉ thị Murexit (bằng hạt ngô), lắc đều (dung dịch có màu vàng xỉn).

– Chuẩn độ từ từ đến khi dung dịch chuyển sang màu tím thì dừng, ghi lại giá trị VEDTA.

– Làm thêm 2, 3 lần nữa, lấy giá trị trung bình.

III. Tính kết quả

Yêu cầu: Tính nồng độ mol của Cu2+ và Ni2+ trong dung dịch

278

Page 273: 1ygm3hwjpgu090312060334 tài liệu bồi dưỡng giáo viên trường thpt chuyên   mon hóa học - phan 1- năm 2012

Bài 3:

TỔNG HỢP IMIN TỪ ANILIN VÀ P–CLOBENZANĐEHIT

I. Mục đích thí nghiệm

- Tổng hợp imin theo cơ chế cộng nucleophin đi từ anđehit và amin thơm.

- Rèn luyện một số thao tác thí nghiệm cơ bản trong phòng thí nghiệm Hóa hữu

cơ: Tháo lắp dụng cụ, đun nóng, lọc, kết tinh, đo điểm chảy...

II. Cơ sở lý thuyết

Imin là một hợp chất hữu cơ có nhiều ứng dụng trong công nghiệp và thực

tiễn, đặc biệt là trong công nghiệp dược phẩm. Phản ứng tạo ra imin đi từ anđehit

và amin thường tiến hành trong môi trường trung tính hoặc trong môi trường axit

yếu. Phản ứng xảy ra theo hai giai đoạn:

- Giai đoạn 1: Cộng nucleophin (AN) của nguyên tử nitơ trong nhóm amino vào

nguyên tử cacbon mang điện dương của nhóm cacbonyl. Nhóm cacbonyl có thể

được hoạt hóa bởi chất xúc tác axit:

- Giai đoạn 2: Tách nước từ sản phẩm cộng ở giai đoạn 1

III. Dụng cụ, hóa chất thí nghiệm

Thí nghiệm minh họa phản ứng tổng hợp imin sẽ đi từ p-clobenzanđehit và

anilin theo phương trình phản ứng sau:

279

Page 274: 1ygm3hwjpgu090312060334 tài liệu bồi dưỡng giáo viên trường thpt chuyên   mon hóa học - phan 1- năm 2012

Cụng cụ và hóa chất:

Hoá chất Dụng cụAnilin Bình cầu 1 cổ 100 mlp-Clobenzanđehit Sinh hàn hồi lưuAncol etylic tuyệt đối Phễu lọc sứ BucneCH3COOH Bình Bunsen

Giấy lọcBếp điện

IV. Các bước tiến hành thí nghiệm

Tiến hành phản ứng

Cân 0,01 mol p-clobenzanđehit (1,40 g) vào bình cầu 1 cổ

dung tích 100 ml. Hoà tan lượng anđehit đó bằng 5 ml ancol etylic

tuyệt đối. Cho tiếp vào bình phản ứng 0,1 mol anilin (0,93 g) mới

chưng cất lại và 1 giọt CH3COOH làm xúc tác. Thêm hạt sôi, lắp

bình cầu vào hệ thống sinh hàn hồi lưu như hình vẽ (chú ý bôi mỡ

vazơlin vào các khớp nối), đun hồi lưu hỗn hợp trong vòng 15

phút để phản ứng xảy hoàn toàn.

Lọc, rửa sản phẩm thô

Làm lạnh hỗn hợp phản ứng, lọc lấy sản phẩm ở dạng chất rắn kết tinh trên

phễu lọc Bucne, rửa sản phẩm thô bằng một lượng nhỏ ancol etylic lạnh.

Tinh chế sản phẩm

Hòa tan sản phẩm thô trong một tối thiểu etanol ở nhiệt độ sôi của etanol. Lọc

nóng dung dịch để loại tạp chất rắn không tan (nếu cần). Làm lạnh dung dịch để

sản phẩm kết tinh hoàn toàn. Lọc, rửa sản phẩm tinh khiết bằng một lượng nhỏ

ancol lạnh. Sấy khô sản phẩm khoảng 15 phút ở 40oC, đo nhiệt độ nóng chảy của

sản phẩm và tính hiệu suất của phản ứng.

V. Một số lưu ý để thực hiện thí nghiệm thành công

- Chưng cất lại anilin và kết tinh lại p-clobenzanđehit trước khi tiến hành phản

ứng.

- Trước khi cho anilin vào bình phản ứng phải hòa tan hoàn toàn anđehit.

280

Page 275: 1ygm3hwjpgu090312060334 tài liệu bồi dưỡng giáo viên trường thpt chuyên   mon hóa học - phan 1- năm 2012

- Kết tinh lại từ từ sản phẩm để thu được tinh thể sạch.

VI. Phân tích kết quả thí nghiệm và báo cáo

- Quan sát sự thay đổi màu sắc của hỗn hợp phản ứng trong suốt quá trình làm thí

nghiệm.

- Quan sát màu sắc, hình dạng của imin tạo thành.

- Kiểm tra độ tinh khiết bằng cách xác định điểm chảy của sản phẩm.

- Tính hiệu suất của phản ứng.

VII. Câu hỏi kiểm tra và mở rộng

1. Nêu các yếu tố (cấu trúc chất đầu, điều kiện phản ứng) ảnh hưởng đến khả năng

phản ứng và hiệu suất của phản ứng tổng hợp imin từ anđehit và amin thơm.

2. Vì sao phải tinh chế các chất đầu ngay trước khi tiến hành thí nghiệm?

3. Nêu một số ứng dụng thực tiễn của imin.

4. Vì sao imin tạo thành từ anđehit và amin thơm thường bền hơn imin từ anđehit

và amin béo?

5. Giải thích tại sao phản ứng tổng hợp imin thường được thực hiện trong môi

trường axit yếu?

281